NSG 203 CH 78 NERVOUS SYSTEM DISORDERS, Neurology NCLEX questions, NCLEX style neuro Exam 3, neuro 2, Neurologic NCLEX Questions, Neurologic 2, MS exam#4 neuro stuff, Cerebrovascular Disorders (PrepU Ch 67), prepu chapter 38, Neurology NCLEX question...

Ace your homework & exams now with Quizwiz!

what patient subjective complaint complaint is indicative of subarachnoid hemorrhage?

"worst headache of my life"

(SELECT ALL THAT APPLY) A client with tonic-clonic seizure disorder is being discharged with a prescription for phenytoin (Dilantin). Which instructions about phenytoin should the nurse give this client?

(1) Monitor for skin rash., (3) Perform good oral hygiene, including daily brushing and flossing., (4) Periodic follow-up blood work is necessary., (5) Report to the physician problems with walking and coordination, slurred speech, or nausea.

neuron

(basic cell of the nervous system) is made up of a dendrite, a cell body, and an axon

16. Which of the following would lead the nurse to suspect that a child with meningitis has developed disseminated intravascular coagulation? a. Hemorrhagic skin rash b. Edema c. Cyanosis d. Dyspnea on exertion

16. A. DIC is characterized by skin petechiae and a purpuric skin rash caused by spontaneous bleeding into the tissues. An abnormal coagulation phenomenon causes the condition.

4. Which of the following values is considered normal for ICP? a. 0 to 15 mm Hg b. 25 mm Hg c. 35 to 45 mm Hg d. 120/80 mm Hg

4. A. Normal ICP is 0-15 mm Hg.

A 154-pound woman has been prescribed tPA (0.9 mg/kg) for an ischemic stroke. The nurse knows to give how many mg initially?

6.3mg A person who weighs 154 lbs weighs 70 kg. To calculate dosage, multiply 70 × 0.9 mg/kg = 63 mg. The nurse gives 10% (6.3 mg) over 1 minute.

Neurotransmitter

A _____________ is necessary for transmission of an impulse across the synapse.

A nurse is caring for a client with a cerebral aneurysm who is on aneurysm precautions and is monitoring the client for signs of aneurysm rupture. The nurse understands that an early sign of rupture is which of the following?

A decline in the level of consciousness

1 hour

A patient who has had a MRA (Magnetic Resonance Angiography) should be instructed to lie completely still for how long?

A client with quadriplegia is in spinal shock. What should the nurse expect?

Absence of reflexes along with flaccid extremities

Cranial nerve VIII

Acoustic nerve

The nurse educator is preparing a lecture on dementia. The educator will include that which is the most most common cause of dementia in an elderly client?

Alzheimer's disease

_________ often relieves symptoms, ____________ are often recurring problems. There is a need for health-promotion teachings

Bed rest Bladder and respiratory infections

What is the most common side effect of tissue plasminogen activator (tPA)?

Bleeding

What is the expected outcome of thrombolytic drug therapy? A Increased vascular permeability. B Vasoconstriction. C Dissolved emboli. D Prevention of hemorrhage

C Dissolved emboli. Thrombolytic therapy is use to dissolve emboli and reestablish cerebral perfusion.

NEURON

CELL THAT MAKES UP NERVE TISSUE

CNS

Central Nervous System infection

Peripheral Nervous System

Cranial nerve III to XII Spinal nerves peripheral parts of the ANS

The nurse is discharging home a client who had a stroke. The client has a flaccid right arm and leg and is experiencing urinary incontinence. The nurse makes a referral to a home health nurse because of an awareness of what common client response to a change in body image?

Depression

EEG

ELECTROEPHALOGRAM

The nurse assesses normal pupils in a client who had a craniotomy, and then writes "PERRLA" in the nurse's notes, along with other findings. What does the "E" stand for in this acronym?

Equal

A client with a neurological impairment experiences urinary incontinence. Which nursing action should help the client adapt to this alteration?

Establishing a toileting schedule

The nurse is caring for a client with L1-L2 paraplegia who is undergoing rehabilitation. Which goal is appropriate?

Establishing an intermittent catheterization routine every 4 hours

Hydrocephalus

Excessive buildup of cerebrospinal fluid

A nurse is assisting the health care provider in performing a lumbar puncture. The nurse prepares for the procedure by placing the client in which position?

Fetal position

Circle of Willis

Formed by communicating arteries that join basilar and internal carotid arteries

Which is a nonmodifiable risk factor for ischemic stroke?

Gender

Cranial nerve IX

Glossopharyngeal

The nurse is caring for a client with stroke in evolution. Which nursing intervention is priority?

Have tracheal suction available at all times.

WEAKNESS, NUMBNESS, OR DIMISHED MOVEMENT

INJURY TO A PLEXUS(A GROUP OF SPINAL NERVES) WOULD RESULT IN?

Expansion

In _________, the person communicating with the patient completes the patient's sentences when he/she is able to verbalize but cannot yet speak in complete sentences.

Which nursing diagnosis takes highest priority for a client with Parkinson's crisis?

Ineffective airway clearance

halo sign

Inspect the wet area after a few minutes for a halo sign: if a yellow ring encircles a central ring that is red, the red ring indicates blood, and the yellow ring suggest CSF.

MID BRAIN

KNOWN AS THE "REFLEX CENTER"

A nurse is caring for a client diagnosed with a hemorrhagic stroke. When creating this client's plan of care, what goal should be prioritized?

Maintain and improve cerebral tissue perfusion.

Thalamus

Major relay center for afferent inputs to cerebral cortex

A nurse is suctioning an unconscious client who has a tracheostomy. The nurse should avoid which action during this procedure?

Making sure not to suction for longer than 30 seconds

A nurse is caring for a client with a diagnosis of right (nondominant) hemispheric brain attack (stroke). The nurse notes that the client is alert and oriented to time and place. Based on these findings, the nurse interprets that the client:

May likely have perceptual and spatial disabilities

CSF

Measure pressure of cerebrospinal fluid

A client is admitted to the emergency department with a C4 spinal cord injury. The nurse performs which intervention first when collecting data on the client?

Monitoring the respiratory rate

Bladder dysfunction (spinal cord) Hypotonic

More ability than atonic bladder but less than normal Interruption of afferent pathways from bladder

AXON

OUTGROWTH OF THE BODY OF A NERVE CELL THAT CONDUCTS IMPULSES AWAY FROM THE CELL BODY

PNS

PERIPHERAL NERVOUS SYSTEM

photophobia

Photphobia (intolerance to light) and pain when the eyes move from side to side occur. The affected person may have seizures. Aptechial purpuric rash is also possible.

Neuron

Primary FUNCTIONAL unit of the NS

Glial cells

Provide support, nourishment, and protection to neurons

A client with spinal cord injury suddenly experiences an episode of autonomic dysreflexia. After checking vital signs, the nurse immediately:

Raises the head of the bed and removes the noxious stimulus

A nurse is collecting data on a client suspected of having Alzheimer's disease. The priority data would focus on which of the following characteristic of this disease?

Recent memory loss

An adult client had a cerebrospinal fluid (CSF) analysis after lumbar puncture. The nurse interprets that a negative value of which of the following is consistent with normal findings?

Red blood cells

left-Hemisphere CVa

Right hemiplegia Right visual field deficits Aphasia Aware of deficits Impaired intellectual ability Slow, cautious behavior High level of frustration over losses

The nurse practitioner is able to correlate a patient's neurologic deficits with the location in the brain affected by ischemia or hemorrhage. For a patient with a left hemispheric stroke, the nurse would expect to see:

Right-sided paralysis.

The nurse practitioner is able to correlate a patient's neurologic deficits with the location in the brain affected by ischemia or hemorrhage. For a patient with a left hemispheric stroke, the nurse would expect to see:

Right-sided weakness or paralysis

Which nursing diagnosis takes highest priority for a client admitted for evaluation for Ménière's disease?

Risk for injury related to vertigo

RECEPTORS

SENSORY NERVE ENDING THAT RESPOND TO STIMULI

AFFERRENT

SENSORY NEURONS THAT RECEIVE MESSAGES FROM ALL PARTS OF THE BODY & TRANSMITS THEM BY WAY OF SENSORY NERVES TO THE CENTRAL NERVOUS SYSTEM

VENTRICLE

SMALL CAVITY OR CHAMBER

A nurse has applied a hypothermia blanket to a client with a fever. The nurse should inspect the skin frequently to detect which complication of hypothermia blanket use?

Skin breakdown

I Olfactory

Smell

The nurse is participating in a health fair for stroke prevention. Which will the nurse say is a modifiable risk factor for ischemic stroke?

Smoking

Oligodendrocytes

Specialized cells that produce myelin sheath of nerve fibers in CNS

Which data collection finding supports the possible diagnosis of Bell's palsy?

Speech or chewing difficulties accompanied by facial droop

TRUE

T/F SPINAL CORD INJURIES CAN BE FATAL

HEMISPHERES (R & L)

THE CEREBRUM IS DIVIDED INTO 2 _________________________?

A nurse is teaching the client with myasthenia gravis about prevention of myasthenic and cholinergic crises. The nurse tells the client that this is most effectively done by:

Taking medications on time to maintain therapeutic blood levels

A nurse is told in report that a client has a positive Chvostek's sign. What other data would the nurse expect to find on data collection? Select all that apply.

Tetany Diarrhea Possible seizure activity Positive Trousseau's sign

GCS

The Glasgow coma scale (GCS) is commonly used as a broad indicator of the severity of brain injury.p

Afferent and Efferent

The PNS is subdivided into which 2 divsions?

chorea

The disease progresses at different rates. HD usually starts with abnormal involuntary movement, called chorea, as fidgeting, jerking, and spasms.

Dermatomes

The distribution pathways of the spinal nerves are called what?

neuralgia

The term neuralgia literally means "pain in a nerve." Neuralgia often applies to fleeting pain in the shoulder and upper arm or pain caused by a herniated intervertebral disk.

Dura mater

Thick; forms outermost layer of meninges

A patient diagnosed with an ischemic stroke should be treated within the first 3 hours of symptom onset with which of the following?

Tissue Plasminogen activator(tPA) within first 3 hours

A client accidentally splashes chemicals into his eye. The nurse knows that eye irrigation with plain tap water should begin immediately and continue for 15 to 20 minutes. What is the primary purpose of this first-aid treatment?

To prevent vision loss

A client diagnosed with transient ischemic attacks (TIAs) is scheduled for a carotid endarterectomy. The nurse explains that this procedure will be done for what purpose?

To remove atherosclerotic plaques blocking cerebral flow

XII Hypoglossal

Tongue movement

Pyramidal

Tracts that begin in the cerebral cortex, end in the spinal cord, and control skeletal muscle movement are what type of tracts?

transection

Transection (severing) of the cord can be incomplete (partial) or complete. If the transection is complete, all sensation and voluntary movements below the site of injury are lost.

Symptoms involving motor function usually begin in the _______________ with *weakness progressing to spastic paralysis*. ______________occurs in 90% of the cases. MS is more common in women and progression is not prderly

Upper extremities Bowel and bladder dysfunction

A nurse is administering mouth care to an unconscious client. The nurse should avoid doing which of the following?

Using products with lemon or alcohol

II Optic

Vision

Cranial nerve II

Visual fields and acuity

3

When using the GCS (Glasgow Coma Scale), a score of _____ indicates a totally comatose patient.

Medulla oblongata

Which part of the brainstem contains reticular formation that regulates heartbeat, respiration, and blood pressure?

Medulla oblongata

Which part of the brainstem is the control center for swallowing, coughing, sneezing, and vomiting?

Which client would be most at risk for secondary Parkinson disease caused by pharmacotherapy?

a 30-year-old client with schizophrenia taking chlorpromazine

Global aphasia:

a combination of expressive and receptive aphasia.

myelin sheath

a white fatty substance

Cranial nerve VI

abducens; helps move the eye

aneurysm

an excessive localized enlargement of an artery caused by a weakening of the artery wall.

Myelin sheath

an insulator for the conduction of impulses

what term refers to inability to perform previously learned purposeful motor acts on a voluntary basis

apraxia

What deficit would you expect from damage to the temporal lobe?

auditory agnosia

what term refers to the failure to recognize significance of sounds

auditory agnosia

axon

carries nerves impulses to other neurons or to end organs

cerebell/o

cerebellum

dysphagia

difficulty in swallowing

Autonomic nervous system

divided into SNS and PNS

ptosis

drooping of one or botheyelids

left hemisphere

e is responsible for speech, problem solving, reasoning, and calculations.

nonmodifiable stroke risk factors

family hx age gender race--once again african ameri more at risk`

Brudzinski's Sign

finding indicative of meningitis (see picture)

cerebral thrombosis

formation or development of a blood clot withing the cerebral arteries or their branches

A client with a spinal cord injury has a neurogenic bladder. When planning for discharge, the nurse anticipates that the client will need which procedure or program?

intermittent catheterization

low systemic perfusion and ischemic stroke

low systemic perfusion from cardiac failure or significant blood loss (with resulting hypotension) can also cause ischemic stroke systemic failure=global, bilateral neurological dfecits

F-A-S-T

mnemonic for rapid CVA recognition (Face, Arm, Speech, Time)

neur/o

nerve

What is the initial diagnostic test for a stroke?

noncontrast CT Scan

-algia

pain

what term refers to the continued and automatic repetition of an activity, word, or phrase that is no longer appropriate

perseveration

what can cause variance in cerebral blood flow?

progressive narrowing.....secondary to athersclerosis

A physician orders aspirin, 325 mg P.O. daily for a client who has experienced a transient ischemic attack (TIA). The nurse should teach the client that the physician has ordered this medication to:

reduce the chance of blood clot formation.

-lepsy

seizure, attack

myel/o

spinal cord or bone marrow

The nurse is providing information about strokes to a community group. What would the nurse identify as the primary initial symptoms of an ischemic stroke? (3)

weakness on one side of the body, difficulty with speech, change in mental status

Multiple sclerosis, Nursing assessment

❈ Optic neuritis (loss of vision or blind spot) ❈ Visual or swallowing difficulties ❈ Gait disturbances; intention tremors ❈ Unusual fatigue,weakness, and clumsiness ❈ Numbness, particularly on one side of face ❈ Impaired bladder and bowel control ❈ Speech disturbances ❈ Scotomas (white spots in visual field, diplopia)

SYMPATHETIC

"FIGHT OR FLIGHT" RESPONDS TO EMERGENCY

A client with Guillain-Barré syndrome has paralysis affecting the respiratory muscles and requires mechanical ventilation. The client has questions about the paralysis. What information will the nurse tell the client about the paralysis?

"The paralysis caused by this disease is temporary."

A client with Guillain-Barré syndrome has paralysis affecting the respiratory muscles and requires mechanical ventilation. What should the nurse tell the client about the paralysis?

"The paralysis caused by this disease is temporary."

(SELECT ALL THAT APPLY) A client with a history of epilepsy is admitted to the medical-surgical unit. While assisting the client from the bathroom, the nurse observes the start of a tonic-clonic seizure. Which nursing interventions are appropriate for this client?

(1) Assist the client to the floor., (2) Turn the client to his side., (3) Place a pillow under the client's head.

(SELECT ALL THAT APPLY) The nurse is planning care for a client with multiple sclerosis. Which problems should the nurse expect the client to experience?

(1) Visual disturbances, (3) Balance problems, (5) Mood disorders

esthesi/o

-esthesia, sensation or feeling

A client has an exacerbation of multiple sclerosis accompanied by leg spasticity. The physician prescribes dantrolene sodium (Dantrium), 25 mg by mouth daily. How soon after administration can the nurse expect to see a significant reduction in spasticity?

1 to 2 weeks

The client with a cervical fracture is being discharged in a halo device. Which teaching instruction should the nurse discuss with the client? 1. Discuss how to correctly remove the insertion pins. 2. Instruct the client to report reddened or irritated skin areas. 3. Inform the client that the vest liner cannot be changed. 4. Encourage the client to remain in the recliner as much as possible.

2 Reddened areas, especially under thebrace, must be reported to the HCPbecause pressure ulcers can occurwhen wearing this appliance for an extended period.

Thrombolytic therapy should be initiated within what time frame of an ischemic stroke to achieve the best functional outcome?

3 hours

Which of the following clients on the rehab unit is most likely to develop autonomic dysreflexia? 1. A client with a brain injury 2. A client with a stroke 3. A client with a herniated nucleus pulposus 4. A client with a high cervical spine injury

4

PLEXUS

A NETWORK OR TANGLE; AS NERVE OR VEINS

Once a CVA has occurred, immediate medical attention is needed.

At first, the medical team concentrates on diagnosing the type of CVA and preserv- ing life. Drugs are ordered to reduce IICP and prevent neu- rologic deficits. After the acute phase, care focuses on the patient's rehabilitation.

Descending tracts

Carry impulses that are responsible for muscle movement

A patient admitted with a stroke is coming to the unit from the emergency department. The nurse assigned to care for the new patient knows that what assessment finding is indicative of a stroke?

Difficulty speaking

Myasthenia Gravis

Disorder affecting the neuromuscular transmission of impulses in the *voluntary muscles* ❈ Considered *autoimmune* disease characterized by the presence of AChR antibodies, interfere with neuronal transmission

A patient suffering a stroke is having a difficult time swallowing. What would the nurse document this finding as?

Dysphagia

What are some nursing interventions for pts with hearing problems?

Early detection Safety measures (due to hearing loss) Assistive devices Hearing aids Implants (e.g., cochlear devices) Surgical

A nurse is assisting in gathering data on cranial nerve XII of a client who sustained a brain attack (stroke). The nurse understands that the client should be asked to:

Extend the tongue.

CAT or CT Scan

If you wanted to examine the brain from many different angles or obtain a three dimensional view of the brain, which diagnostic test would you use?

Brain

Includes cerebrum, brainstem, and cerebellum

In a client with amyotrophic lateral sclerosis and respiratory distress, which finding is the earliest sign of reduced oxygenation?

Increased restlessness

If temperature elevated, take quick measures to decrease it, because fever _________________________________

Increases cerebral metabolism and can increase cerebral edema

A client with weakness and tingling in both legs is admitted to the medical-surgical unit with a tentative diagnosis of Guillain-Barré syndrome. In this syndrome, polyneuritis leads to progressive motor, sensory, and cranial nerve dysfunction. On admission, which assessment is most important for this client?

Lung auscultation and measurement of vital capacity and tidal volume

A 75-year-old client who was admitted to the hospital with a stroke informs the nurse that he doesn't want to be kept alive with machines. He wants to make sure that everyone knows his wishes. Which action should the nurse take?

Make arrangements for the client to receive information about advance directives.

A client who experienced a stroke has left-sided facial droop. During mouth care, the client begins to cough violently. What should the nurse do?

Make sure a tonsil suction device is readily available while providing mouth care.

A client who was trapped inside a car for hours after a head-on collision is rushed to the emergency department with multiple injuries. During the neurologic examination, the client responds to painful stimuli with decerebrate posturing. This finding indicates damage to which part of the brain?

Midbrain

A nurse is reviewing the medical record of a client diagnosed with amyotrophic lateral sclerosis (ALS). Which initial clinical manifestation of this disorder supports this diagnosis?

Mild clumsiness

A client was seen and treated in the emergency department for treatment of a concussion. The nurse determines that the family needs further discharge instructions if they say to bring the client back to the emergency department if which of the following occurs?

Minor headache

What's an expected finding for a pt with Chronic Kidney Disease?

Oliguria. CKD results in the inability of the kidneys to excrete waste, and fluid retention, acidosis, azotemia and oliguria are all common.

Hyperkalemia EKG findings...

Prolonged PR intervals and widened QRS complex.

A 70-year-old client with a diagnosis of left-sided stroke is admitted to the facility. To prevent the development of disuse osteoporosis, which of the following objectives is appropriate?

Promoting weight-bearing exercises

A client with Alzheimer's disease is admitted for hip surgery after falling and fracturing the right hip. The spouse tells the nurse of feeling guilty for letting the accident happen and reports not sleeping well because the spouse has been getting up at night and doing odd things. Which nursing diagnosis is most appropriate for the client's spouse?

Risk for caregiver role strain related to increased client care needs

A client is admitted in a disoriented and restless state after sustaining a concussion from a car accident. Which nursing diagnosis takes highest priority in this client's plan of care?

Risk for injury

The nurse is administering neostigmine to a client with myasthenia gravis. Which nursing intervention should the nurse implement?

Schedule the medication before meals.

The nurse caring for an older adult client understands that which of the following can increase disorientation in this client? Select all that apply.

Sedatives Anesthesia Physical restraints Analgesics

A patient is exhibiting classic signs of a hemorrhagic stroke. What complaint from the patient would be an indicator of this type of stroke?

Severe headache

TRUE

T/F THE NERVE FIBERS OF THE SPINAL CORD CANNOT REGENERATE AFTER AN INJURY

Damage to which area of the brain results in receptive aphasia?

Temporal lobe

When caring for a client with head trauma, the nurse notes a small amount of clear, watery fluid oozing from the client's nose. What should the nurse do?

Test the nasal drainage for glucose.

rhinorrhea

The dye may appear to bulge, and other symptoms of vasodilation are seen, such as edema, lacrimation (tear formation), rhinorrhea (runny nose), diaphoresis (sweating), and flushing of the affected side.

Cranial nerve X

Vagus

A client with hypertension comes to the clinic for a routine checkup. Because hypertension is a risk factor for cerebral hemorrhage, the nurse questions the client closely about warning signs and symptoms of hemorrhage. Which complaint is a possible indicator of cerebral hemorrhage in this client?

Vertigo

A nurse is reviewing the record of a client with a suspected diagnosis of Huntington's disease. Which documented early symptom supports this diagnosis?

Vertigo

Autonomic

Which system of the PNS is active in maintaining homeostasis?

Neuron conductivity

ability to transmit an impulse

Encephalitis

also called "Sleeping sickness," inflammation of the brain commonly caused by exposure to viruses spread by insects or from the herpes simplex virus

Trigeminal Neuralgia

also called "Tic Douloureux" degeneration or pressure on the 5th cranial nerve resulting in excruciating muscle spasms of the face brought on by minimal stimulus

Deep tendon reflexes

are assessed at the wrists, elbows, knees, and Achilles ten- don with a reflex hammer.

Subarachnoid space

area between arachnoid layer and pia mater

pelvic postural deviations associated with stroke

asymmetrical WB with majority of weight of stronger side fear of shifting weight toward more affected sdie when sitting, sacral sitting (posterior tilt) when standing, unilateral retraction and elevation on more affected side

cerebr/o

cerebrum

Limbic system

concerned with emotional aggression, feeding behavior, and sexual response

dysphasia

difficulty in speaking

A client is suspected of having amyotrophic lateral sclerosis (ALS). To help confirm this disorder, the nurse prepares the client for various diagnostic tests. The nurse expects the physician to order:

electromyography (EMG).

To evaluate a client's cranial nerve function, the nurse should assess:

gag reflex.

Tonic-Clonic Seizures

generalized seizure involving a clonic phase, a tonic phase and a postictal phase, usually preceded by an aura

chronic phase stroke rehab

generally defined to be more than 6 months post stroke typically delivered in an outpatient rehav facility, in a communicty setting, or at home (pt can enter and exit home with ease)

Generalized Seizures

includes: absence, myoclonic, atonic, clonic, tonic and clonic-tonic

Intracranial Bleeds

includes: epidural, subdural and subarachnoid hemorrhages and hematomas

reflexes and stroke

intially stroke results in hyporeflexia with flaccidity when spasticity and synergies develop--hyperrelfexia is seen they will have hyperactive DTRs may demonstrate clonus, clasp-knife, +babinski ATNR re-emergence

reflex i

involuntary motor response to a stimulus.

Acetycholine

neurotransmitter associated with voluntary movement, sleep and wakefulness. Too little is associated with Alzheimer's

Terminal Stage of Alzheimer's

no link to past or present, oblivious to surrounding, mute or speaks few incoherent words, dysphagia, completely passive, may have seizures, totally dependent for care

subarachnoid hemorrhage

occurs from bleeding into subarachnoid space typically from a saccular or berry aneurysm (?) affecting primarily large blood vessels

hemorrhagic stroke

occurs when blood vessels rupture causes leakage of blood in or around the brain

-lexia

reading

what is the term for when patient is unable to comprehend the spoken word

receptive aphasia

Synapse

structural and functional junction between two neurons

stroke basic definition

sudden loss of neurological function caused by an interrruption of blood flow to the brain

A client has a 12-year history of migraine headaches and is frustrated over how these headaches impact lifestyle. What would the nurse indicate to the client is the origin of migraines?

vascular

A nurse has administered timolol to a client with glaucoma. What symptom would be of greatest concern to the nurse?

wheezing

unilateral neglect expounded upon

will limit movement and use of more involved extremities (usually nondominant left side of body - RHemisphere) pt typically does not react to sensory stim presented on more involved side

A client with a mild concussion reports a headache. When offered acetaminophen, the client asks for a stronger pain medication. Which response by the nurse is appropriate?

• "Opioids are avoided after a head injury because they may hide a worsening condition."

The nurse is caring for the client in the ER following a head injury. The client momentarily lost consciousness at the time of the injury and then regained it. The client now has lost consciousness again. The nurse takes quick action, knowing this is compatible with: 1. Epidural hematoma 2. Subdural hematoma 3. Concussion 4. Skull fracture

1

The home health nurse is caring for a 28-year-old client with a T10 SCI who says, "I can't do anything. Why am I so worthless?" Which statement by the nurse wouldbe the most therapeutic? 1. "This must be very hard for you. You're feeling worthless?" 2. "You shouldn't feel worthless—you are still alive." 3. "Why do you feel worthless? You still have the use of your arms." 4. "If you attended a work rehab program you wouldn't feel worthless."

1 Therapeutic communication addresses the client's feelings and attempts to allow the client to verbalize feelings;the nurse should be a therapeutic listener

When developing a plan of care for a patient with dysfunction of the cerebellum, the nurse will include interventions to a. prevent falls. b. stabilize mood. c. enhance swallowing ability. d. improve short-term memory.

ANS: A Because functions of the cerebellum include coordination and balance, the patient with dysfunction is at risk for falls. The cerebellum does not affect memory, mood, or swallowing ability. DIF: Cognitive Level: Application REF: 1410

Visual field testing

A semicircular bowl-like instrument shows light in different parts of the bowl to map field of vision. Used to evaluate the progression of glaucoma. *Nursing Implications* Procedure does not cause pain but can tire the patient.

A client is at risk for increased ICP. Which of the following would be a priority for the nurse to monitor? A Unequal pupil size B Decreasing systolic blood pressure C Tachycardia D Decreasing body temperature

A Unequal pupil size

2 hour

A bladder training program usually begins with a ________ schedule for toileting.

Occipital lobe

A region of the cerebral cortex that processes visual information. Processing pf sight takes place here

A client with a cerebral aneurysm exhibits signs and symptoms of an increase in intracranial pressure (ICP). What nursing intervention would be most appropriate for this client?

Absolute bed rest in a quiet, nonstimulating environment

For a male client with suspected increased intracranial pressure (ICP), a most appropriate respiratory goal is to: A. prevent respiratory alkalosis. B. lower arterial pH. C. promote carbon dioxide elimination. D. maintain partial pressure of arterial oxygen (PaÓ) above 80 mm Hg

C. Promote Carbon dioxide elimination The goal of treatment is to prevent acidemia by eliminating carbon dioxide. That is because an acid environment in the brain causes cerebral vessels to dilate and therefore increases ICP.

A stroke victim is experiencing memory loss and impaired learning capacity. The nurse knows that brain damage has most likely occurred in which lobe?

Frontal

1- CELL BODY 2- PROCESSES (AXON) (DENDRITES)

NAME THE 3 PARTS OF A NUERON

NERVE IMPULSES

"ALL OR NONE" LAW WITHOUT EXCEPTION

ptosis

(drooping eyelids)

Central nervous system

Brain Spinal cord cranial nerves I and II

Ventricles

Four interconnected fluid-filled cavities

APROX 3 LBS

HOW MUCH DOES THE BRAIN WIEGHT

Which of the following terms refers to blindness in the right or left halves of the visual fields of both eyes?

Homonymous hemianopsia

Parietal lobe

Interprets spatial information and contains the sensory cortex

rhythmic, involuntary movements or oscillations of the eyes

Nystagmus

A patient diagnosed with an ischemic stroke should be treated within the first 3 hours of symptom onset with which of the following?

Tissue plasminogen activator (tPA)

70-125 cm H2O

What is the normal CSF pressure?

Aphasia

_________ may be receptive, expressive, or global.

encephal/o

brain

MRI

detects areas of infarction earlier than a CT scan.

Mr. Cooper

enjoys grilling out and barbecue

areas of function looked at by glasgow coma scale

eyeopening, best motor response, verbal responses

Ictus

seizure

ton/o

tension, tone

Encephalon

the brain

Paresthesia (sensory system)

alteration in sensation lesions in the posterior column or sensory cortex

Dermatomes

area of skin innervated by sensory fibers of a single dorsal root of a spinal nerve

stats for pts with ABI

atherothrombotic brain infarction 70% have HTN 30% have Heart disease 15% have CHF 30% peripheral artery disease 15% have diabetes mellitus

Parkinson's Disease

chronic degenerative condition due to a lack of dopamine and excess acetylcholine

Cranial nerve VII

facial nerve; innervates muscle of facial expression

Receptive Aphasia

inability to comprehend spoken or written language

causes of stroke in kids

perinatal arterial ischemic stroke sickle cell congenital heart diesease thrombophlbeitis trauma

VIII Vestibulocochlear

Sense of hearing and equilibrium

Diagnostic tests - Arterial blood gases

used to rule out low oxygen or high carbon dioxide levels, another cause of altered LOC.

Age-related Changes in the Eye and vision

presbyopia. ectropion entropion Tears are decreased, so the eyes may feel dry and scratchy.

Defect in vision in a specific area in one or both eyes

scotoma

Dendrites

short process extending from the cell body

Which nursing task would be most appropriate for the nurse to delegate to the unlicensed assistive personnel? 1. Teach Credé's maneuver to the client needing to void. 2. Administer the tube feeding to the client who is quadriplegic. 3. Assist with bowel training by placing the client on the bedside commode. 4. Observe the client demonstrating self-catheterization technique.

3 The assistant can place the client on the bedside commode as part of bowel training; the nurse is responsible for the training but can delegate this task

The nurse is caring for a patient with late-stage cirrhosis. The nurse considers which factor when participating in a patient care conference? A Late-stage cirrhosis is irreversible. B Late-stage cirrhosis can be managed with lifestyle changes. C Late-stage cirrhosis can be cured with lactulose and spironolactone. D Late-stage cirrhosis is characterized by periods of remission alternating with flare-ups.

A Late-stage cirrhosis is irreversible.

concussion

A concussion is the result of any blow to the head. The concussion may not damage any brain structures, but temporary unconsciousness is possible.

30 minutes

A patient with dysphagia should be sitting upright or in a high Fowler's position when eating, and should maintain this position for at least __________ after the meal.

An increase in ICP

A rising SBP and a widening pulse pressure might indicate what?

A patient is being educated on how to take their anti-thyroid medication. Which of the following statements are INCORRECT?* A. "I will continue taking aspirin daily." B. "I will take this medication at the same time every day." C. "It may take a while before I notice that the medication is helping my condition." D. "I will avoid foods containing high levels of iodine."

A. "I will continue taking aspirin daily."

The nurse is teaching family members of a patient with a concussion about the early signs of increased ICP. Which of the following would she cite as an early sign of increased ICP? A. Headache and vomiting (This is an EARLY sign all the other signs come later if ICP continues for a longer period of time) B. Dilated pupils C. Inability to wake patient with noxious stimuli D. Decreased systolic BP

A. Headache and vomiting Rationale: This is the EARLIEST sign of increased ICP. Although C is the most *reliable* indicator of increased ICP, it is not the earliest sign. A and B occur after prolonged increased ICP

The nurse is caring for a client admitted with spinal cord injury. The nurse minimizes the risk of compounding the injury most effectively by: 1. Placing the client on a Stryker frame 2. Logrolling the client on a soft mattress 3. Logrolling the client on a firm mattress 4. Keeping the client on a stretcher

1

The charge nurse is observing a new staff nurse who is assessing a patient with a traumatic spinal cord injury for sensation. Which action indicates a need for further teaching of the new nurse about neurologic assessment? a. The new nurse tests for light touch before testing for pain. b. The new nurse has the patient close the eyes during testing. c. The new nurse asks the patient if the instrument feels sharp. d. The new nurse uses an irregular pattern to test for intact touch.

ANS: C When performing a sensory assessment, the nurse should not provide verbal clues. The other actions by the new nurse are appropriate.

During the neurologic assessment, the patient is unable to respond verbally to the nurse but cooperates with the nurse's directions to move his hands and feet. The nurse will suspect a. cerebellar injury. b. a brainstem lesion. c. frontal lobe damage. d. a temporal lobe lesion.

ANS: C Expressive speech is controlled by Broca's area in the frontal lobe. The temporal lobe contains Wernicke's area, which is responsible for receptive speech. The cerebellum and brainstem do not affect higher cognitive functions such as speech.

Propranolol (Inderal), a β-adrenergic blocker that inhibits sympathetic nervous system activity, is prescribed for a patient. The nurse monitors the patient for a. dry mouth. b. constipation. c. slowed pulse. d. urinary retention.

ANS: C Inhibition of the fight or flight response leads to decreased heart rate. Dry mouth, constipation, and urinary retention are associated with peripheral nervous system blockade. DIF: Cognitive Level: Comprehension REF: 1407

A client with C7 quadriplegia is flushed and anxious and complains of a pounding headache. Which of the following symptoms would also be anticipated? A Decreased urine output or oliguria B Hypertension and bradycardia C Respiratory depression D Symptoms of shock

B Hypertension, bradycardia, anxiety, blurred vision, and flushing above the lesion occur with autonomic dysreflexia due to uninhibited sympathetic nervous system discharge. The other options are incorrect

A female client with cancer is scheduled for radiation therapy. The nurse knows that radiation at any treatment site may cause a certain adverse effect. Therefore, the nurse should prepare the client to expect: A hair loss. B stomatitis. C fatigue. D vomiting.

C fatigue.

Nurse April is teaching a client who suspects that she has a lump in her breast. The nurse instructs the client that a diagnosis of breast cancer is confirmed by: A breast self-examination. B mammography. C fine needle aspiration. D chest X-ray.

C fine needle aspiration.

Nurse Kristine is trying to communicate with a client with a stroke who has aphasia. Which of the following actions by the nurse would be least helpful to the client? A. Speaking to the client at a slower rate with a normal tone B. Allowing plenty of time for the client to respond C. Completing the sentences that the client cannot finish D. Looking directly at the client during attempts at speech

C. Completing the sentences that the client cannot finish Rationale: This will not allow the patient to improve their communication skills

A female client has clear fluid leaking from the nose following a skull fracture. The nurse assess that this is cerebrospinal fluid if the fluid: A. Is clear and tests negative for glucose B. Clumps together on a dressing and has a pH of 7.0 C. Separates into rings and tests positive for glucose D. Tests negative for Beta 2 Transferrin

C. Separates into rings and tests positive for glucose Rationale: A Halo ring and a positive test for glucose will indicate CSF. Beta 2 Transferrin is only present in CSF, so it will be positive.

The patient with advanced cirrhosis asks why his abdomen is so swollen. The nurse's response is based on the knowledge that A. a lack of clotting factors promotes the collection of blood in the abdominal cavity. B. portal hypertension and hypoalbuminemia cause a fluid shift into the peritoneal space. C. decreased peristalsis in the GI tract contributes to gas formation and distention of the bowel. D. bile salts in the blood irritate the peritoneal membranes, causing edema and pocketing of fluid.

B. portal hypertension and hypoalbuminemia cause a fluid shift into the peritoneal space.

A client with a subdural hematoma becomes restless and confused with dilation of the pupil. The physician orders Mannitol for which of the following reasons? A. To reduce the intraocular pressure B. To prevent acute tubular necrosis C. To promote osmotic diuresis to decrease ICP D. To draw water into the vascular system to increase blood pressure

C. To promote osmotic diuresis to decrease ICP Rationale: Mannitol causes a rapid decrease in fluid by drawing the fluid into the vascular space by osmosis for excretion.

All of the following are part of the philosophy behind hospice, EXCEPT: A Medical care should be limited to that which is palliative, not curative. B Death should be viewed as normal, to be faced and accepted, not avoided. C Control over the patient's care should be in the hands of the patient and his or her family. D Life should be prolonged as much as possible provided that the patient is conscious.

D Life should be prolonged as much as possible provided that the patient is conscious.

A client presents to the Emergency Department with a head injury received in a fall at home. On admission, the client's Glasgow Coma Scale (GCS) score is 12. Within 20 minutes of arrival, the GCS is 8. What should the nurse do? A Turn up the client's IV. B Lower the head of the bed to 30 degrees. C Repeat the client's blood pressure reading. D Prepare the client for intubation.

D Prepare the client for intubation

A client is suspected of having systemic lupus erythematous. The nurse monitors the client, knowing that which of the following is one of the initial characteristic sign of systemic lupus erythematous? A Weight gain B Subnormal temperature C Elevated red blood cell count D Rash on the face across the bridge of the nose

D Rash on the face across the bridge of the nose.

Nurse Ruffa is providing dietary instructions to the client with systemic lupus erythematosus. Which of the following dietary items would the nurse instruct the client to avoid? A Cantaloupe B Turkey C Broccoli D Steak

D Steak

When planning care for a pt with cirrhosis, the nurse will give highest priority to which nursing diagnosis? A impaired skin integrity related to edema, ascites, and pruritis B imbalanced nutrition: less than body requirements related to anorexia C excess fluid volume related to portal hypertension and hyperaldosteronism D ineffective breathing pattern related to pressure on diaphragm and reduced lung volume

D ineffective breathing pattern related to pressure on the diaphragm and reduced lung volume

When reviewing the chart for a patient with cervical cancer, the nurse notes that the cancer is staged as Tis, N0, M0. The nurse will teach the patient that A the cancer cells are well-differentiated. B it is difficult to determine the original site of the cervical cancer. C further testing is needed to determine the spread of the cancer. D the cancer is localized to the cervix.

D the cancer is localized to the cervix.

Of the following patients, the nurse recognizes that the one with the highest risk for a stroke is a(n) A. Obese 45-year-old Native American B. 35-year-old Asian American woman who smokes C. 32-year-old white woman taking oral contraceptives D. 65-year-old African-American man with HTN

D. 65-year-old African-American man with HTN

A client with a SCI at level C3-C4 is being cared for in the emergency department (ED). What is the priority assessment? A. Determine the level at which the client has intact sensation B. Assess the level at which the client has retained mobility C. Check the blood pressure and pulse for signs of spinal shock D. Monitor respiratory effort and oxygen saturation level

D. Monitor respiratory effort and oxygen saturation level Rationale: An SCI above C4 results in complete paralysis of respiratory function. Therefore, this patient needs to be vented and monitored for respiratory function. Remember your ABCs!!

A nurse is trying to help the family of an unconscious client cope with the situation. Which intervention should the nurse plan to incorporate into the care routine for the client?

Explaining equipment and procedures on an ongoing basis

Which clinical manifestation is observed in the clonic phase of a seizure?

Extension spasms of the body

From which direction should a nurse approach a client who is blind in the right eye?

From the left side of the client

The nurse is providing diet-related advice to a male patient following a cerebrovascular accident (CVA). The patient wants to minimize the volume of food and yet meet all nutritional elements. Which of the following suggestions should the nurse give to the patient about controlling the volume of food intake?

Provide thickened commercial beverages and fortified cooked cereals.

A client is scheduled for a digital subtraction angiography. The nurse supports the client's understanding that the test is directed toward which outcome?

Providing information about the blood vessels

A nurse is caring for a client following craniotomy who has a supratentorial incision. The nurse reviews the client's plan of care, expecting to note that the client should be maintained in which of the following positions?

Semi-Fowler's position

shingles

Shingles, or herpes zoster, is an acute viral inflammation of a nerve caused by the varicella-zoster virus (the same virus that causes chickenpox).

A client has been hit on the head with a baseball bat. The nurse notes clear fluid draining from the ears and nose. Which nursing intervention is appropriate?

checking the fluid for glucose with a dipstick

Neurotransmitters

chemicals that affect the transfer of impulses across the synaptic cleft

Pulmonary embolism signs/sx

chest pain, tachypnea, tachycardia, anxiety, restlesness, apprehension together with persistent cough tx: supp 02, thrombolytic drugs about half of pts diagnosed with a DVT have already had a PE

Multiple Sclerosis

chronic demyelinating disease which present in vague individualized symptoms and is characterized by exacerbations and remissions leaving muscles weakened, spastic and eventually paralyzed

Middle Stage of Alzheimer's

chronic loss of recent memory, moderate aphasia, gets lost in familiar places, engages in repetitive actions, agnosia, apraxia, mood and behavioral disturbances, need reminders with their ADL's

Normal CSF

clear, is colorless, and contains no RBCs, a few WBCs, very little protein, and glucose of 50 to 70 mg/dL.

ateriovenous malformation

cogenital defect that can result in stroke tangle of arteries and veins with agenesis of an interposing capillary network abnormal vessels undergo progressive dilation w/ age, 50% eventually bleed out

bowel and bladder issues and stroke

common during acute phase----29% of stroke pts can result from hyper/hyporeflexia, disturbances of sphnicter control (pelvic floor?), sensory loss recommendation-tolieting sched for prompted voiding prevent UTIs, prevent skin breakdown

seizures and stroke

common right after stroke during the acute phase tend to be of the partial motor type more common, slightly, in occlusive carotid disease than MCA meds- (phenytoin)dilantin, (carbamazepine)tegretol, (penobarbital) solfoton

Cerebrum

composed of right and left cerebral hemispheres

Late Stage of Alzheimer's

confusion over past and present, expressive and receptive aphasia, doesn't recognize familiar places of people, move slowly and falls easily, behavioral disturbances, need help with all ADL's

The nurse is teaching a client who has facial muscle weakness and has recently been diagnosed with myasthenia gravis. The nurse should teach the client that myasthenia gravis is caused by:

destruction of acetylcholine receptors.

A nurse collecting data on a post-craniotomy client finds the urinary catheter bag with 1,500 mL the first hour and the same amount for the second hour. Which complication should the nurse suspect as a cause of this amount of output?

diabetes insipidus

Expressive Aphasia

difficulty speaking or writing

The nurse is monitoring a client for increasing intracranial pressure (ICP). Early signs of increased ICP include:

diminished responsiveness.

most common source of cerebral embolus

disease of the cardiovascular system

Kernig's Sign

finding indicative of meningitis (see picture)

Early Stage of Alzheimer's

frequent short term memory loss, mild aphasia, need reminder of ADL's, may get lost in familiar places, may have difficulty writing of using objects

ESRD, chronic kidney disease and stroke

guess what they do increase risk of stroke

CT scan

identifies the size and location of the CVA. It is useful to differentiate between an infarction and hemorrhage.

Cerebral arteriogram

identifies vessel abnormalities such as an aneurysm.

atrial fib and stroke

if you have this, 5x at risk than others for stroke

Apraxia

impairment in the ability to perform purposeful tasks of manipulate objects

medical management post stroke: goals

improve cerebral perfusion-reestablish circulation, oxygenation. assist in stopping progression of the lesion to limit defecits-02 mask or up nose maintain adequate BP maintain sufficient CO restore electrolyte balance maintain blood glucose in normal ranges control edema, ICP, herniation-use antiedema agents maintain bowel and bladder function-may include urinary catheter turning schedule decrease risk of all the other shit-DVT, aspriation, etc

Apraxia (motor system)

inability to perform learned movements despite having desire and physical ability to perform them Cerebral cortex lesion

ideational apraxia

inability to produce movement either on command or automatically complete breakdown in conceptualization of the task pt has no idea how to do a given movement

Anosognosia (mental status)

inability to recognize bodily defect or disease Lesions in right parietal cortex

Agnosia

inability to recognize familiar objects or people

A graduate nurse, working in a long-term facility, is caring for a client who has hearing loss. When observing the graduate, the nurse mentor would intervene if which action is taken by the graduate?

moves around and multitasks when speaking

Benign tumors continue to grow and take up space in the confined area of cranium, causing _______________________ in the brain, increased ICP, and necrosis of brain tissue. Even benign tumors must be treated because they may have malignant effects

neural and vascular compromise

A client recovering from a stroke has right-sided hemiplegia and telegraphic speech and often seems frustrated and agitated, especially when trying to communicate. However, the chart indicates that the client's auditory and reading comprehension are intact. The nurse suspects that the client has:

nonfluent aphasia.

A white female client is admitted to an acute care facility with a diagnosis of stroke. Her history reveals bronchial asthma, exogenous obesity, and iron-deficiency anemia. Which history finding is a risk factor for stroke?

obese

A white female client is admitted to an acute care facility with a diagnosis of stroke. Her history reveals bronchial asthma, exogenous obesity, and iron deficiency anemia. Which history finding is a risk factor for stroke?

obesity

cranial nerve I

olfactory nerve

skull

protects brain from external trauma

subacute phase stroke rehab

pts are referred to inpt rehab if they can tolerate an intensity of services consisting of two or more rehab discipline, 6 days a week, for a min of 3 hrs of active rehab a day

what types of pts demonstrate impairments in executive function?

pts with lesions of the prefrontal cortex they are imulsive, inflexible in thinking, lack abstract thinking ability, bad organizers, bad sequencers, decreased insight, cant plan, bad judgement unable to realistically appriase their environ and react appropriately safety risk when yout executive functions are impaired

Hypothalamus

regulates the autonomic nervous system and endocrine system

A client has tension headaches. The nurse recommends massage as a treatment for tension headaches. How does massage help clients with tension headaches?

relaxes muscles causes local dilation of blood vessels, and relieves HA, does not likely help client with migraine of cluster HA

A patient is exhibiting classic signs of a hemorrhagic stroke. What complaint from the patient would be an indicator of this type of stroke?

severe headache

thec/o

sheath

Common reflexes

stretch, deep tendon, with- drawal, and superficial.

young stroke-definition

stroke affecting a person 45 or younger

stroke death stats

stroke deaths # greater than 143k annually strokes account for 1 out of ever 18 deaths in the USA hemorrhagic stroke has highest death rate-mortaility rate is 37-38% at 1 month ischemic death rate-8-12% at one month

The nurse is providing diet-related advice to a client who experienced a cerebrovascular accident (CVA). The client wants to minimize his volume of food and yet meet all nutritional requirements. To control the volume of food intake, the nurse should suggest that the client consume:

thickened commercial beverages and fortified cooked cereals.

Expressive aphasia:

an inability to speak or write due to damage of Broca area. The patient usually can under- stand what is being said.

Receptive aphasia:

an inability to understand the spoken word due to damage of Wernicke area. The patient can speak, but the words do not make sense.

Clinical manifestations of a stroke depend on: (4)

area of the cerebral cortex, the affected hemisphere, the degree of blockage, and the presence or absence of collateral circulation.

intracerebral hemorrhage

caused by rupture of cerebral vessel with subsequent bleeding into the brain

ateriography

xray of carotid artery with special dye injected into an artery in the leg or arm considered invasive and carries small risk of causing a stroke in and of itself

Myasthenia Gravis, Nursing assessment

❈ Diplopia (double vision), ptosis ❈ Mask-like affect: sleepy appearance due to facial muscle involvement ❈ Weakness of laryngeal and pharyngeal muscles: dysphagia, chocking, food aspiration, difficulty speaking ❈ Muscle weakness improved by rest, worsened by activity ❈ Advanced cases: Resp. failure, bladder and bowel incontinence ❈ *Myasthenia crisis symptoms: asso. with under-medication; more difficulty swallowing, diplopia, ptosis, dyspnea* ❈ Cholinergic crisis (anticholinesterase OD): diaphoresis, diarrhea, fasciculations, cramps, marked worsening of symptoms

A nurse has instructed the client with myasthenia gravis about ways to manage his or her own health at home. The nurse determines that the client needs more information if the client makes which of the following statements?

"Going to the beach will be a nice, relaxing form of activity."

The nurse is evaluating neurological signs of the male client in spinal shock following spinal cord injury. Which of the following observations by the nurse indicates that spinal shock persists? 1. Inability to elicit a Babinski's reflex 2. Positive reflexes 3. Hyperreflexia 4. Reflex emptying of the bladder

1

Which of the following nursing interventions is appropriate for a client with an ICP of 20 mm Hg? 1. Encourage the client to hyperventilate 2. Give the client a warming blanket 3. Administer low-dose barbiturate 4. Restrict fluids

1

A client with head trauma develops a urine output of 300 ml/hr, dry skin, and dry mucous membranes. Which of the following nursing interventions is the most appropriate to perform initially? A Evaluate urine specific gravity B Anticipate treatment for renal failure C Provide emollients to the skin to prevent breakdown D Slow down the IV fluids and notify the physician

A Evaluate urine specific gravity

What should a male client over age 52 do to help ensure early identification of prostate cancer? A Have a digital rectal examination and prostate-specific antigen (PSA) test done yearly. B Have a transrectal ultrasound every 5 years. C Perform monthly testicular self-examinations, especially after age 50. D Have a complete blood count (CBC) and blood urea nitrogen (BUN) and creatinine levels checked yearly.

A Have a digital rectal examination and prostate-specific antigen (PSA) test done yearly.

INTERNEURON

A NEURON BETWEEN THE FIRST AFFERENT NEURON & THE LST MOTOR NEURON; NEURONS WHOLE PROCESSES ARE ALL IN A SPECIFIC AREA, SUCH AS OLFACTORY LOBE

A female patient with systemic lupus erythematosus (SLE) complains of photosensitivity. The LPN/LVN should instruct the patient on which self-care measure? A Wear sunblock whenever she is outdoors. B Include foods high in beta-carotene in her diet. C Manage joint pain with prescribed medications. D Use a tanning bed once per week to help with rashes.

A Wear sunblock whenever she is outdoors.

A nurse is preparing a plan of care for a client with a brain attack (stroke) who has global aphasia. The nurse incorporates communication strategies in the plan of care, knowing that the client's speech will be:

Associated with poor comprehension

The nurse is caring for a client who suffered a spinal cord injury 48 hours ago. The nurse monitors for GI complications by assessing for: A A flattened abdomen B Hematest positive nasogastric tube drainage C Hyperactive bowel sounds D A history of diarrhea

B After spinal cord injury, the client can develop paralytic ileus, which is characterized by the absence of bowel sounds and abdominal distention. Development of a stress ulcer can be detected by hematest positive NG tube aspirate or stool. A history of diarrhea is irrelevant.

A client has been pronounced brain dead. Which findings would the nurse assess? Check all that apply. A Decerebrate posturing B Dilated nonreactive pupils C Deep tendon reflexes D Absent corneal reflex

B,C,D

Which of the following are common cardiac signs with hyperthyroidism? A Atrial stenosis and tachycardia B Diastolic murmurs and low cardiac output C High cardiac output, tachycardia, and atrial fibrillation D Premature ventricular contractions and low cardiac output

C High cardiac output, tachycardia, and atrial fibrillation

DECUSSATION

CROSSING

Diagnostic tests - complete blood count (CBC) with differential and cultures from blood, urine, throat, and nose

To identify infectious diseases such as meningitis or encephalitis

The nurse is assessing a client with a suspected stroke. What assessment finding is most suggestive of a stroke?

Facial droop

Which of the following is the chief cause of intracerebral hemorrhage (ICH)?

Uncontrolled hypertension

cholesterol and stroke

elevated total blood cholsterol aka hypercholesterolemia 240mg/dl or greater increases risk for stroke too evelated LDL aka bad cholesterol increases risk borderline high LDL-130-159 high LDL-160-189 very high LDL 190 and up low levels HDL aka good cholesterol also increases risk 40 and below for men 50 and below for chicks fasting tryglyceride level greater than 150mg/dl is elevated...also risk factor for stroke

Risk factors for stroke(13)

obesity history of ischemic episodes cardiovascular disease diabetes mellitus atherosclerosis of the cranial vessels hypertension polycythemia smoking hypercholesterolemia hormonal contraceptive use emotional stress family history of stroke advancing age.

A client is admitted to an acute care facility with a diagnosis of stroke. The nursing student is reviewing the client's history. Which history findings noted will the student report as risk factors for stroke? Select all that apply.

obesity smoking ½ pack of cigarettes a day hypertension

chemical regulation of cerebral blood flow

occurs is response to changes in blood concentrations of carbon dioxide or oxygen vasodilation, increased CBF---in repsonse to increase PaCO2-partial pressure of co2 in blood..OR decrease in PaO2 ...dont know dont care fall in pH (more acid) produces vasodilation rise in pH (more alkaline) produces decrease in blood flow changes in BP produce minor alteration of CBF-as pressure rises, artery is stretched, dec patency of vessel, decrease in CBF. ...............takeway is....these are all autoregulatory mechanisms, the question will prob say its an autoregulatory mechanism or something along those lines, following a stroke, autoregulatory mechanisms may be impaired

stuff that increases risk of embolic stroke

rheumatic heart valvular disease endocarditis surgery like CABG

narc/o

sleep

-phasia

speech

-sthenia

strength

A nurse working on a surgical floor observes an unlicensed assistive personnel (UAP) completing assigned tasks. Which situation requires the nurse to intervene?

turning a client who is 24 hours post-op laminectomy

A nurse is providing discharge instructions to a client who has undergone transsphenoidal surgery for a pituitary adenoma. Which statement by the client indicates the client understands the discharge instructions?

"I need to call the doctor if I develop frequent swallowing or postnasal drip."

A halo vest is applied to a client following a cervical spine fracture. The nurse provides instructions to the client regarding safety measures related to the vest. Which statement by the client indicates a need for further instructions?

"I will bend at the waist, keeping the halo vest straight to pick up items."

A nurse reinforces home care instructions to the postcraniotomy client. Which statement by the client indicates the need for further instruction?

"I will not hear sounds clearly unless they are loud."

A nurse has given suggestions to the client with trigeminal neuralgia about strategies to minimize episodes of pain. The nurse determines that the client needs additional information if the client made which of the following statements?

"I will try to eat my food either very warm or very cold."

A client recently experienced a stroke with accompanying left-sided paralysis. His family voices concerns about how to best interact with him. They report the client doesn't seem aware of their presence when they approach him on his left side. What advice should the nurse give the family?

"The client is unaware of his left side. You should approach him on the right side."

A client is receiving pilocarpine eye drops. Which statement made by the client shows correct understanding of the medication?

"The medication will help decrease pressure in my eyes."

kinesi/o

-kinesia, movement

Neurons characteristics

1. excitability 2. conductibility 3. Influence

severity and symptoms of stroke are depedent on what factors?

1. location of ischemic process 2. size of ischemic area 3.nature and structure of functions involved 4. availibility of collateral circulation

What interventions are used to slow the progress of Chronic Kidney Disease?

1.Control blood pressure 2. Make fluid or dietary adjustments

The client with a C6 SCI is admitted to the emergency department complaining of asevere pounding headache and has a BP of 180/110. Which intervention should theemergency department nurse implement? 1. Keep the client flat in bed. 2. Dim the lights in the room .3. Assess for bladder distention. 4. Administer a narcotic analgesic.

3 This is an acute emergency caused by exaggerated autonomic responses to stimuli and only occurs after spinal shock has resolved in the client with a spinal cord injury above T6. The most common cause is a full bladder.

Which assessment finding indicates that lactulose is effective in decreasing the ammonia level in the client with hepatic encephalopathy? A Passage of two or three soft stools daily B Evidence of watery diarrhea C Daily deterioration in the client's handwriting D Appearance of frothy, foul-smelling stools

A Passage of two or three soft stools daily

After undergoing a subtotal thyroidectomy, a female client develops hypothyroidism. Dr. Smith prescribes levothyroxine (Levothroid), 25 mcg P.O. daily. For which condition is levothyroxine the preferred agent? A Primary hypothyroidism B Graves' disease C Thyrotoxicosis D Euthyroidism

A Primary hypothyroidism

3. A male client is having a tonic-clonic seizures. What should the nurse do first? A. Elevate the head of the bed. B. Restrain the client's arms and legs. C. Place a tongue blade in the client's mouth. D. Take measures to prevent injury.

Answer D. Protecting the client from injury is the immediate priority during a seizure. Elevating the head of the bed would have no effect on the client's condition or safety. Restraining the client's arms and legs could cause injury. Placing a tongue blade or other object in the client's mouth could damage the teeth.

A male client is having a tonic-clonic seizures. What should the nurse do first? a. Elevate the head of the bed. b. Restrain the client's arms and legs. c. Place a tongue blade in the client's mouth. d. Take measures to prevent injury.

Answer D. Protecting the client from injury is the immediate priority during a seizure. Elevating the head of the bed would have no effect on the client's condition or safety. Restraining the client's arms and legs could cause injury. Placing a tongue blade or other object in the client's mouth could damage the teeth.

Propranolol (Inderal), a β-adrenergic blocker that inhibits sympathetic nervous system activity, is prescribed for a patient who has extreme anxiety about public speaking. The nurse monitors the patient for a. dry mouth. b. bradycardia. c. constipation. d. urinary retention

ANS: B Inhibition of the fight or flight response leads to a decreased heart rate. Dry mouth, constipation, and urinary retention are associated with peripheral nervous system blockade.

A client with a cervical spine injury has Gardner-Wells tongs inserted for which of the following reasons? A To hasten wound healing B To immobilize the cervical spine C To prevent autonomic dysreflexia D To hold bony fragments of the skull together

B Gardner-Wells, Vinke, and Crutchfield tongs immobilize the spine until surgical stabilization is accomplished.

A patient with a diagnosis of lung cancer is receiving chemotherapy and reports nausea and loss of appetite resulting in decreased food intake. What should the healthcare provider recommend to promote adequate nutrition? Advise the patient to: A Eat only favorite foods to increase appetite. B Eat small meals throughout the day. C Eat large meals but less frequently throughout the day. D Eat only when feeling hungry.

B Eat small meals throughout the day.

2. After electroencephalography (EEG), what nursing action should the nurse implement? a. Monitor for signs of bleeding at insertion site b. Encourage client to increase fluid intake c. Monitor for nausea and vomiting d. Assist client to shampoo hair

Assist client to shampoo hair. Rationale: Paste is used to attach electrodes to the scalp and should be washed out to increase client comfort. An EEG is noninvasive, so there is no insertion site. An EED does not cause nausea and vomiting. Increasing fluids is unnecessary after EEG, but cleaning paste out of the hair is appropriate.

A client diagnosed with a brain tumor experiences a generalized seizure while sitting in a chair. How should the nurse intervene first?

Assist the client to a side-lying position on the floor, and protect her with linens.

A client with a new diagnosis of ischemic stroke is deemed to be a candidate for treatment with tissue plasminogen activator (t-PA) and has been admitted to the ICU. In addition to closely monitoring the client's cardiac and neurologic status, the nurse monitors the client for signs of what complication?

Bleeding

A nurse is monitoring a client with a blunt head injury sustained from a motor vehicle crash. Which of the following would indicate a basal skull fracture as a result of the injury?

Bloody or clear drainage from the auditory canal

A nurse is preparing for the admission of a client with a suspected diagnosis of herpes simplex encephalitis. Which diagnostic test will be prescribed to confirm this diagnosis?

Brain biopsy

An incoherent female client with a history of hypothyroidism is brought to the emergency department by the rescue squad. Physical and laboratory findings reveal hypothermia, hypoventilation, respiratory acidosis, bradycardia, hypotension, and nonpitting edema of the face and pretibial area. Knowing that these findings suggest severe hypothyroidism, nurse Libby prepares to take emergency action to prevent the potential complication of: A Thyroid storm. B Cretinism. C Myxedema coma. D Hashimoto's thyroiditis.

C Myxedema coma.

The nurse is caring for a male client with cirrhosis. Which assessment findings indicate that the client has deficient vitamin K absorption caused by this hepatic disease? A Dyspnea and fatigue B Ascites and orthopnea C Purpura and petechiae D Gynecomastia and testicular atrophy

C Purpura and petechiae

A female client with hypothyroidism (myxedema) is receiving levothyroxine (Synthroid), 25 mcg P.O. daily. Which finding should nurse Hans recognize as an adverse drug A Dysuria B Leg cramps C Tachycardia D Blurred vision

C Tachycardia

___________ is an autoimmune disorder where the body attacks the thyroid gland that causes it to stop releasing T3 and T4. The patient is likely to have the typical signs/symptoms of hypothyroidism, however, they may present with what other sign as well?* A. Myxedema coma; joint pain B. Thyroid storm; memory loss C. Hashimoto's Thyroiditis; goiter D. Toxic nodular goiter (TNG); goiter

C. Hashimoto's Thyroiditis; goiter

CSF

CEREBRAL SPINAL FLUID

EFFERANT

CONDUCTING AWAY FROM THE CENTER, AS AN EFFERANT NERVE

PARIETAL LOBE

CONTAINS THE SENSORY AREA OF THE BRAIN. INTERPRETS TOUCH, TEMP AND PAIN SENSATIONS RECEIVED FROM THE BRAIN. (TOUCH, SPATIAL ABLITY, SPEECH AND COMMUNICATION), RECOGNIZE SIZE AND SHAPE

VAGUS NERVE

CRANIAL NERVE (X) AFFECTS MANY BODY FUNCTIONS BEYOND CONSCIOUS CONTROL

A client diagnosed with a cerebral aneurysm reports a severe headache to the nurse. What action is a priority for the nurse?

Call the health care provider immediately.

A patient diagnosed with a cerebral aneurysm reports a severe headache to the nurse. What action is a priority for the nurse?

Call the health care provider immediately.

Pyramidal tract

Carry voluntary impulses from cerebral cortex to cranial and peripheral nerves

cephalgia

Cephalgia (headache) is one of the most common symptoms of a neurologic disorder. It is also associated with many other diseases and disorders. Headache is not a disease in itself, but rather it is a symptom of an underlying disorder.

A patient presents to the emergency room with complaints of having an "exploding headache" for the last 2 hours. The patient is immediately seen by a triage nurse who suspects the patient is experiencing a stroke. Which of the following is a possible cause based on the characteristic symptom?

Cerebral aneurysm

A client experiences an episode of Bell's palsy and complains about increasing clumsiness. The nurse should prepare the client for which diagnostic study (studies) to determine the cause of the complaints? Select all that apply.

Cerebral angiography Lumbar puncture (LP) Computed tomography

The nurse is performing a mental status examination on a client diagnosed with a subdural hematoma. This test assesses which of the following functions?

Cerebral function

LOC

Change in behaviors, mentation, level of consciousness, alertness, and orientation could be subtle but significant.

A client complains of periorbital aching, tearing, blurred vision, and photophobia in her right eye. Ophthalmologic examination reveals a small, irregular, nonreactive pupil — a condition resulting from acute iris inflammation (iritis). As part of the client's therapeutic regimen, the physician prescribes atropine sulfate (Atropisol), two drops of 0.5% solution in the right eye twice daily. Atropine sulfate belongs to which drug classification?

Cholinergic blocker

Pyridostigmine bromide

Cholinergic causes mucles to contract, activates pain responses and regulates endocrine and REM sleep functions. Needs to be *life-long*, requires family teaching and support ❈ Inhibits the action of cholinesterase at the cholinergic nerve ending ❈ Promote accumulation of acetylcholine @ cholinergic receptor site Adverse reactions: cholinergic crisis can occur with overdose; symptoms include *fasciculations, abdominal cramps, diarrhea, incontinence of stool/urine; hypotension, bradycardia, respiratory depression, lacrimation, blurred vision* Nursing implications ❈ *Atropine* is the antidote for drug-induced bradycardia ❈ Take drug with food to decreased GI side effects

How can the nurse evaluate if a patient is successfully adapting to lifestyle changes and functionality following a stroke?

Clients are evaluated as coping successfully with lifestyle changes after a brain attack (stroke) if they make appropriate lifestyle alterations, use the assistance of others, and have appropriate social interactions.

Paralytic ileus is common in _____________. A gastric tube aids in gastric decompression

Comatose patients

Extrapyramidal

Conduction pathways that are ______________ tracts control muscle movements associated with posture and balance.

A client with Parkinson's disease "freezes" while ambulating, increasing the risk for falls. Which suggestion should the nurse include in the client's plan of care to alleviate this problem?

Consciously think about walking over imaginary lines on the floor.

A client sustained a closed head injury in a fall from a tree that happened 2 hours ago. There is MRI evidence of a contusion. The client has just begun to regain consciousness and has a current Glasgow Coma Scale (GCS) score of 11. The nurse should plan care for a client with which level of injury from this contusion? A Severe B Mild C Extreme D Moderate

D Moderate

Spinal Cord Injury

Disruption in nervous system function, lead to compete or incomplete loss of *motor and sensory* function ❈ Injury described by location of the spinal cord, most common in *5th, 6th, 7th cervical vertebrae* the twelfth thoracic, and the first lumbar ❈ Damages ranges from contusion to complete transection

EEG

Electroencephalography records the brain's electrical impulses as a graph. This test is used frequently in the diagnosis of seizure disorders, brain tumors, intracranial lesions (in Parkinson disease, Alzheimer disease, and narcolepsy), blood clots, infections (meningitis, encephalitis), and sleep disorders.

A nurse is caring for a client who sustained a spinal cord injury. While administering morning care, the client developed signs and symptoms of autonomic dysreflexia. The initial nursing action is to:

Elevate the head of the bed.

A client recovering from a head injury is arousable and participating in care. The nurse determines that the client understands measures to prevent elevations in intracranial pressure (ICP) if the nurse observes the client doing which of the following activities?

Exhaling during repositioning

When preparing to discharge a client home, the nurse has met with the family and warned them that the client may exhibit unexpected emotional responses. The nurse should teach the family that these responses are typically a result of what cause?

Frustration around changes in function and communication

Shortly after admission to an acute care facility, a client with a seizure disorder develops status epilepticus. The physician orders diazepam (Valium), 10 mg I.V. stat. How soon can the nurse administer a second dose of diazepam, if needed and prescribed?

In 10 to 15 minutes

Modeling

In ___________, the patient's sentences are completed, and new information is added.

otorrhea

In a basilar skull fracture, rhinorrhea, leakage of CSF from the nose (otorrhea), or leakage of CSF from the ear may occur.

A client with Parkinson's disease is experiencing a parkinsonian crisis. The nurse should immediately place the client:

In a quiet, dim room with respiratory and cardiac support available

The nurse is caring for a client after a supratentorial craniotomy in which a large tumor was removed from the left side. Choose the positions in which the nurse can safely place the client. Select all that apply.

In a semi-Fowler's position With the head in a midline position

Caloric testing

In caloric testing, cold or warm water is injected into the semicircular canals. The patient is observed for nystagmus, nausea, vomiting, falling, or vertigo, indicating labyrinth disease. *Nursing Implications* Ensure patient safety by observing for vomiting and assisting as necessary to prevent aspiration.

A client with myasthenia gravis is having difficulty speaking. The client's speech is dysarthric and has a nasal tone. The nurse should use which communication strategies when working with this client? Select all that apply.

Listening attentively Asking yes and no questions when able Using a communication board when necessary Repeating what the client said to verify the message

A nurse is reinforcing instructions to a client taking divalproex sodium (Depakote). The nurse tells the client to return to the clinic for follow-up laboratory studies related to which test?

Liver function studies

A nurse is assisting in caring for a client with a suspected diagnosis of meningitis. The nurse reinforces to the client information regarding which diagnostic test that is commonly used to confirm this diagnosis?

Lumbar puncture

A client is admitted to an acute care facility for treatment of a brain tumor. When reviewing the chart, the nurse notes that the client's extremity muscle strength is rated 1/5. What does this mean?

Muscle contraction is palpable and visible.

Which is the initial diagnostic test for a stroke?

Noncontrast computed tomography

A nurse working in a long-term care facility is approached by the son of a resident, who wants his 78-year-old father to have a heating pad, because "his feet are always cold at night." The nurse should incorporate which of the following concepts when formulating a response to the family member?

Older adults often have slower neurological response times and are therefore more at risk for burns.

Ophthalmoplegia (cranial nerve)

Paralysis of eye muscles Lesion in brainstem

A client with a brain attack (stroke) is experiencing residual dysphagia. The nurse would remove which of the following food items that arrived on the client's meal tray from the dietary department?

Peas

After having a stroke, a patient has cognitive deficits. What are the cognitive deficits the nurse recognizes the patient has as a result of the stroke? Select all that apply.

Poor abstract reasoning Decreased attention span Short- and long-term memory loss

Nursing care during the immediate recovery period from an ischemic stroke should include which of the following?

Positioning to avoid hypoxia

The nurse is planning care for a client who suffered a stroke in the right hemisphere of his brain. What should the nurse do?

Provide close supervision because of the client's impulsiveness and poor judgment.

A female client with Guillain-Barre syndrome has ascending paralysis and is intubated and receiving mechanical ventilation. Which of the following strategies would the nurse incorporate in the plan of care to help the client cope with this illness? A. Giving client full control over care decisions and restricting visitors B. Providing positive feedback and encouraging active range of motion C. Providing information, giving positive feedback, and encouraging relaxation D. Providing intravenously administered sedatives, reducing distractions and limiting visitors

Providing information, giving positive feedback, and encouraging relaxation. The client with Guillain-Barré syndrome experiences fear and anxiety from the ascending paralysis and sudden onset of the disorder. The nurse can alleviate these fears by providing accurate information about the client's condition, giving expert care and positive feedback to the client, and encouraging relaxation and distraction. The family can become involved with selected care activities and provide diversion for the client as well.

III Oculomotor

Pupil constriction Eyeball movement Raising of upper eyelid

Neurologic viral signs also include:

Pupil size, limb movement, and vital signs

A nurse is planning to institute seizure precautions for a client who is being admitted from the emergency department. Which of the following measures should the nurse avoid in planning for the client's safety?

Putting a padded tongue blade at the head of the bed

During an episode of autonomic dysreflexia in which the client becomes hypertensive, the nurse should perform which of the following interventions? 1.Elevate the client's legs 2.Put the client flat in bed 3.Put the client in the Trendelenburg's position 4. Put the client in the high-Fowler's position

Putting the client in the high-Fowler's position will decrease cerebral blood flow, decreasing hypertension. Elevating the client's legs, putting the client flat in bed, or putting the bed in the Trendelenburg's position places the client in positions that improve cerebral blood flow, worsening hypertension.

SENSORY (AFFERENT)

RECIEVES & TRANSMITS MESSAGES TO THE CNS FROM ALL BODY PARTS **CARRY IMPULSE TO CNS FROM RECEPTOR

THALMUS

RELAY STATION BETWEEN THE CUTANEOUS RECEPTORS & CEREBRAL CORTEX FOR ALL SENSORY IMPULSES EXCEPT SMELL

PARASYMPATHETIC

RESTORES EQUILIBRUIM, CONSERVES ENERGY, MAINTAINS HOMEOSTATIS, "REST AND DIGEST"

CEREBULLUM

RESULT IN LOSS OF EQUILIBRIUM, MOVEMENT, VOLUNTARY MUSCLES, WILL HAVE UNSTEADY COULD OR IS RELATED TO INJURY TO ???

RAS

RETICULAR ACTIVATING SYSTEM

GYRI

RIDGE OR FOLD ON THE CEREBRAL SURFACE IN THE BRAIN

An adult client has undergone lumbar puncture to obtain cerebrospinal fluid (CSF) for analysis. The nurse checks for which of the following negative values if the CSF is normal?

Red blood cells

A client with CVA is prescribed medication to treat the disorder. The client wants to know what other measures may help reduce CVA. Which is an accurate suggestion for the client?

Reduce hypertension and high blood cholesterol

1. Which of the following visual changes should the nurse expect the older adult client to report? a. Increased tear secretion b. Reduced ability to differentiate blue and green colors c. Reduced vision during daylight hours d. Difficulty focusing on objects in the distance

Reduced ability to differentiate blue and green colors. Rationale: Older adults report reduced ability to discriminate between blue and green colors. They experience less tear secretion. Their vision is reduced at night. They have difficulty focusing on near objects, not far objects.

A nurse is providing care to a client with increased intracranial pressure (ICP). Which approach(es) may be beneficial in controlling the client's ICP from an environmental viewpoint? Select all that apply.

Reducing environmental noise Maintaining a calm atmosphere Allowing the client uninterrupted time for sleep

A client with a cervical spine injury has Crutchfield tongs applied in the emergency department. The nurse should avoid which of the following when planning care for this client?

Removing the weights to reposition the client

V Trigeminal

Sensation of the scalp, nose, mouth, and cornea Chewing

A client has clear fluid leaking from the nose after a basilar skull fracture. The nurse determines that this is cerebrospinal fluid (CSF) if the fluid:

Separates into concentric rings and tests positive for glucose

A nurse is caring for the client who has suffered spinal cord injury. The nurse further monitors the client for signs of autonomic dysreflexia and suspects this complication if which of the following is noted?

Severe, throbbing headache

The parents of a client who sustained a closed head injury in a motor vehicle accident voice their concerns about the distance and additional cost of the rehabilitation center which will not be covered by health care chosen for their son. Which health care team member can help the parents with their questions and concerns?

Social worker

The parents of a client who sustained a closed head injury in a motor vehicle accident voice their concerns about the distance and cost of the rehabilitation center chosen for their son. Which health care team member can help the parents with their questions and concerns?

Social worker

Computed tomography (CT) scans

Specialized radiographic procedures that produce computer-generated images with significantly more detail than standard x-rays allow. May be done with or without contrast medium. Detects problems such as hemorrhage, edema, hematoma, infarction, tumor, brain abscess, aneurysm, as well as size and location of cerebrovascular accident (CVA). *Nursing Implications*

Neurogenic bladder

Spinal cord injuries and lesions produce what is known as a cord bladder or _________ _________.

OPTIC

WHAT IS CRANIAL NERVE II(2), RESPONSIBLE FOR VISION, SENSORY NERVE

presbyopia.

The lens becomes less elastic, affecting near vision. (Patients with presbyopia may feel that their arms have become too short to read the newspaper comfortably.)

LP

The lumbar puncture (LP, spinal tap) involves the insertion of a hollow needle with a stylet (guide) into the subarachnoid space of the lumbar region of the spinal canal.

neurology

The medical specialty related to the nervous system is neurology.

Pia mater, arachnoid, and dura mater

The meninges consist of what 3 layers?

An emergency department nurse is awaiting the arrival of a client with signs of an ischemic stroke that began 1 hour ago, as reported by emergency medical personnel. The treatment window for thrombolytic therapy is which of the following?

Three hours

Cranial nerve V

Trigeminal Nerve by which cranial nerve is Meniere's disease diagnosed?

Cranial nerve IV

Trochlear; helps move the eyes

A cluster headache occurs suddenly and severely, often affecting only one side and involving the eye, neck, and face on that side

True

SYMPATHETIC

A DIVISION OF THE AUTONOMIC NERVOUS SYSTEM RESPONDS TO EMERGENCIES "FIGHT OR FLIGHT"

A nurse is caring for a client with a spinal cord injury. The nurse prepares to place high-top sneakers on the client's feet to prevent the occurrence of:

Footdrop

Hashimoto's disease is: A Chronic inflammation of the thyroid gland B Diagnosed most frequently in Asian-Americans and Pacific Islanders C A form of hyperthyroidism D A rare form of hypothyroidism

A Chronic inflammation of the thyroid gland

ANS

AUTONOMIC NERVOUS SUYSTEM

A client with Bell's palsy exhibits facial asymmetry and cannot close the eye completely on one side. The client is also drooling and has loss of tearing in one eye. The nurse documents that the client displays symptoms of involvement of which of the following cranial nerves (CNs)?

CN VII

A nurse reviews the health care provider's treatment plan for a client with Guillain-Barré syndrome. Which prescription, if noted in the client's record, should the nurse question?

Clear liquid diet

Family members would like to bring in a birthday cake for a client with nerve damage. What cranial nerve needs to be functioning so the client can chew?

Cranial nerve V

fissures

Deep grooves separate the hemispheres and sepa- rate the cerebrum from the cerebellum.

Deep tendon reflex (sensory system)

Diminished or absent motor response lower motor neurons lesions

A client who is paraplegic after spinal cord injury has been taught muscle-strengthening exercises for the upper body. The nurse determines that the client will derive the least muscle-strengthening benefit from which activity?

Doing active range of motion to finger joints

major risk factors for stroke

HTN heart disease arrhytmia Diabetes

A physician orders several drugs for a client with hemorrhagic stroke. Which drug order should the nurse question?

Heparin sodium

A client with suspected Guillain-Barré syndrome has a lumbar puncture performed. The cerebrospinal fluid (CSF) protein is 750 mg/dL. The nurse analyzes these results as:

Higher than normal supporting the diagnosis of Guillain-Barré

Dyskinesia (motor system)

Impairment of voluntary movement, resulting in fragmentary or incomplete movements. disorder of basal ganglia, idiosyncratic reaction to psychotropic drugs

Brainstem

Includes mid-brain, pons, and medulla

A nurse is planning to put aneurysm precautions in place for the client with a cerebral aneurysm. Which item would be included as part of the precautions?

Maintaining the head of the bed at 15 degrees

Sympathetic nervous system

Preganglionic cell bodies are located in spinal segments T1 through L2

TROCHLEAR

WHAT IS CRANIAL NERVE IV(4), RESPONSIBLE FOR MOVEMENT OF EYE , SMALLEST CRANIAL NERVE. MOTOR FUNCTION. EASIEST INJURED. EX: MVA, PT COMPLAINTS OF DBL VISION (COULD BE OR IS DUE TO DAMAGE/INJURY TO THIS NERVE )

Ganglia

Spinal nerves and __________ link the sensory organs, muscles, and other body parts to the brain and spinal cord.

NEUROTRANSMITTERS

THESE ARE EXAMPLES OF ? DOPAMINE, NOEPINEPHRINE, ACETYLCHOLINE

A client is having a tonic-clonic seizure. What should the nurse do first?

Take measures to prevent injury.

entropion

The lid margin may turn inward causing the lashes to irritate the eye.

Voluntary

The somatic system is often called the ___________ system.

Synapse

The space between 2 neurons is termed what?

Which of the following risk factors in the client's history is most likely to increase the potential in developing a CVA? Select all that apply. a. Age 50 or older b. Use of oral contraceptives for the past 10 years c. Presence of atherosclerosis d. Consumption of one beer per day e. Overweight by 50 pounds f. Of Caucasian race

Use of oral contraceptives for the past 10 years. Presence of atherosclerosis. Overweight by 50 pounds. Rationale: Use of birth control pills, atherosclerosis, and obesity increase stroke risk. Persons older than 65, not 50, have increased risk for CVA. Those with excessive, not low, alcohol intake have higher risk for CVA. African Americans, not Caucasians, have higher risk for CVA.

Skull and spine x-rays

Used to identify fractures, bone erosion, and calcifications. *Nursing Implications* Noninvasive test. Explain if different positions are needed.

TRIGEMINAL

WHICH IS THE LARGEST CRANIAL NERVE?

Myelography

Which diagnostic test might be used to detect spinal lesions, intervertebral disk problems, tumors, or cysts?

Brain Scan (Radionuclide Imaging)

Which diagnostic test might be used to determine an intracranial mass?

A client is scheduled for an EEG after having a seizure for the first time. Client preparation for this test should include which instruction?

"Avoid stimulants and alcohol for 24 to 48 hours before the test."

REFLEX ARC

CIRCLE IN THE SPINAL CORD THAT SENDS AND RECIEVE MESSAGES THROUGH NERVE FIBERS

Anoxia

The brain's neurons cannot survive ________ for more than 4-6 minutes.

A client is scheduled for an electroencephalogram (EEG) after having a seizure for the first time. Which instruction does the nurse provide to the client as preparation for this test?

"Avoid stimulants and alcohol for 24 to 48 hours before the test."

Papilledema (cranial nerve)

"choked disc," swelling of optical nerve head Increased in intracranial pressure

CSF(CEREBRAL SPINAL FLUID)

ACTS AS SHOCK ABSORBER FOR BRAIN AND SPINAL CORD, CARRIES NUTRIENTS TO BRAIN, CARRIES WASTE AWAY FROM BRAIN, KEEPS BRAIN AND SPINAL CORD MOST * USED TO TEST FOR DISORDERS OR TO TRANSMIT MEDICATIONS

1. A client admitted to the hospital with a subarachnoid hemorrhage has complaints of severe headache, nuchal rigidity, and projectile vomiting. The nurse knows lumbar puncture (LP) would be contraindicated in this client in which of the following circumstances? a. Vomiting continues b. Intracranial pressure (ICP) is increased c. The client needs mechanical ventilation d. Blood is anticipated in the cerebralspinal fluid (CSF)

1. B. Sudden removal of CSF results in pressures lower in the lumbar area than the brain and favors herniation of the brain; therefore, LP is contraindicated with increased ICP. Vomiting may be caused by reasons other than increased ICP; therefore, LP isn't strictly contraindicated. An LP may be preformed on clients needing mechanical ventilation. Blood in the CSF is diagnostic for subarachnoid hemorrhage and was obtained before signs and symptoms of ICP.

18. The nurse is assessing a child diagnosed with a brain tumor. Which of the following signs and symptoms would the nurse expect the child to demonstrate? Select all that apply. a. Head tilt b. Vomiting c. Polydipsia d. Lethargy e. Increased appetite f. Increased pulse

18. A.B and D. Head tilt, vomiting, and lethargy are classic signs assessed in a child with a brain tumor. Clinical manifestations are the result of location and size of the tumor.

laceration

A laceration is tearing of the brain tissue caused by direct impact or penetrating injury. Lacerations are commonly associated with depressed skull fractures.

A client in the emergency department is diagnosed with Bell's palsy. The nurse collecting data on this client expects to note which of the following?

A lag in closing the bottom eyelid

Mr. Miller has been diagnosed with bone cancer. You know this type of cancer is classified as: A sarcoma. B lymphoma. C carcinoma. D melanoma

A sarcoma.

seizure

A seizure, also known as a convulsion, is an episode of abnormal motor, sensory, cognitive, and psychic activity caused by erratic and abnormal electrical discharges of brain cells.

A client with spinal cord injury becomes angry and belligerent whenever the nurse tries to administer care. The nurse should:

Acknowledge the client's anger and continue to encourage participation in care.

20. The client with a brain attack (stroke) has residual dysphagia. When a diet order is initiated, the nurse avoids doing which of the following? A. Giving the client thin liquids B. Thickening liquids to the consistency of oatmeal C. Placing food on the unaffected side of the mouth D. Allowing plenty of time for chewing and swallowing

Answer A. Before the client with dysphagia is started on a diet, the gag and swallow reflexes must have returned. The client is assisted with meals as needed and is given ample time to chew and swallow. Food is placed on the unaffected side of the mouth. Liquids are thickened to avoid aspiration.

PONS

BRAIN AREA THAT CONTAINS NERVE TRACTS THAT CARRIES MESSAGES BETWEEN CEREBRUM AND MEDULLA

The nurse is caring for a client with multiple sclerosis. Which one of these medications should the nurse expect to give if the client develops urinary frequency? a. Amantadine (Symmetrel) b. Dexamethasone (Decadron) c. Dantrolene (Dantrium) d. Propantheline (Pro-Banthine)

Bacterial meningitis. Rationale: Meningitis is an inflammation of the spinal cord and meninges. It is usually secondary to an upper respiratory infection. Encephalitis is caused by a virus in most cases, not by bacteria. Brain abscesses occur from middle ear infection. A subdural hematoma is a collection of blood underneath the dura mater, not an infection of the meninges.

KEY POINTS

Because the nervous system controls the body's movements, disorders in this system may cause unwanted movement or immobility. Seizure disorders have different manifestations, ranging from generalized tonic-clonic movements to uncontrolled movements without loss of consciousness. Spinal cord injuries can result in a range of physical and mental deficits, including paralysis. Degenerative disorders of the nervous system can cause difficulties in movement, sensory deficits, or varying degrees of alteration in mental status. Inflammatory disorders of the nervous system can quickly become life-threatening. Increased ICP has many causes. It is a significant sign of a brain disorder. One of the first and most important signs of ICP and other disorders of the brain is a change in LOC. Most brain tumors are nonmalignant. Benign tumors, however, cause pressure on the brain and can be fatal.

A patient who has had a previous stroke and is taking warfarin tells the nurse that he started taking garlic to help reduce his blood pressure. The nurse knows that garlic when taken together with warfarin will produce which type of interaction?

Can greatly increase the international normalization ratio (INR), increasing the risk of bleeding

A patient who just suffered a hemorrhagic stroke is brought to the emergency department by ambulance. What should be the nurse's primary assessment focus?

Cardiac and respiratory status

Spinothalamic tracts

Carry pain and temperature sensations

A patient with ICP monitoring has a pressure of 12 mm Hg. The nurse understands that this pressure reflects A. A severe decrease in cerebral perfusion pressure B. An alteration in the production of CSF C. The loss of autoregulatory control of ICP D. A normal balance between brain tissue, blood, and CSF

D. A normal balance between brain tissue, blood, and CSF

A patient has an extremely high T3 and T4 level. Which of the following signs and symptoms DO NOT present with this condition?* A. Weight loss B. Intolerance to heat C. Smooth skin D. Hair loss

D. Hair loss

double vision or the awareness of two images of the same object occurring in one or both eyes

Diplopia

A nurse is monitoring a client who sustained a head injury and suspects that the client has a skull fracture. This conclusion is based on which of the following findings? Select all that apply.

Drainage from ear Bruising around the eyes Pink-tinged drainage from the nose

The nurse is teaching a client and his family about baclofen (Lioresal) therapy. Baclofen is an analogue of which neurotransmitter?

Gamma aminobutyric acid (GABA)

The nurse is caring for a client in a coma who has suffered a closed head injury. What intervention should the nurse implement to prevent increases in intracranial pressure (ICP)?

Elevate the head of the bed to 30 degrees.

After a stroke, a 75-year-old client is admitted to a health care facility. The client has left-sided weakness and an absent gag reflex. He's incontinent and has a tarry stool. His blood pressure is 90/50 mm Hg, and his hemoglobin is 10 g. Which action is a priority for this client?

Elevating the head of the bed to 30 degrees

When caring for a client who has had a stroke, a priority is reduction of ICP. What client position is most consistent with this goal?

Elevation of the head of the bed

Cerebral Angiography

Which diagnostic test might be used to determine the presence of stricture, tumor, aneurysm, thrombus, or hematoma in the cerebral arteries?

A client with seizure disorder comes to the physician's office for a routine checkup. Knowing that the client takes phenytoin (Dilantin) to control seizures, the nurse assesses for which common adverse drug reaction?

Excessive gum tissue growth

Glasgow Coma Scale

Eye opening (4), Motor response (6), Verbal response (5) Score <7 or less indicates coma, low score indicates high morality and poor prognosis

IV Trochlear

Eyeball movement

ectropion

Eyelid muscles may lose tone, causing the lower lid to turn out

A client who sustained a closed head injury in a skating accident pulls out his feeding tube, I.V. catheter, and indwelling urinary catheter. To ensure this client's safety, a physician prescribes restraints. Which action should a nurse take when using restraints?

Fasten the restraint to the bed frame using a quick-release knot.

MYELIN SHEATH

FATTY COVERING OF NERVE FIBERS; ELECTRICALLY INSULATE ONE NERVE CALL FROM ANOTHER

A nurse is assigned to care for an adult client who had a brain attack (stroke) and is aphasic. Choose the appropriate interventions for communicating with the client. Select all that apply.

Face the client when talking. Speak slowly and maintain eye contact. Use gestures when talking to enhance words. Give the client directions using short phrases and simple terms.

A nurse determines that motor function of which cranial nerve is intact if the client can perform this action? Refer to figure.

Facial

Bell palsy is a temporary, partial one-sided facial paralysis and weakness caused by ischemia of inflammation of the 7th cranial nerve and is not a result of a brain lesion.

False

Fluorescein stain

Fluorescein dye is injected onto the cornea, and the cornea is viewed with a slit lamp. The green staining allows identification of corneal ulceration or abrasion. *Nursing Implications* Explain that the dye may sting slightly when inserted, and the staining will wash away with tears.

A nurse is preparing for the admission of a client with a diagnosis of early stage Alzheimer's disease. The nurse assists in developing a plan of care, knowing that which of the following is a characteristic of early Alzheimer's disease?

Forgetfulness

Hypothalamus

Which division of the brain controls body temperature, appetite, and water balance and links the nervous and endocrine systems?

Cerebrum

Which division of the brain is responsible for thinking, learning, language capacity, judgement, personality, and storing memories?

contusion

In contusion, the brain tissue is bruised.

Dura mater

Which layer of the meninges is a tough membrane protecting the brain and spinal cord?

right-Hemisphere CVa

Left hemiplegia Left visual field deficits Spatial-perceptual deficits Denies or unaware of deficits Easily distracted Poor judgment Impulsive

A nurse is assisting in caring for a client with a supratentorial lesion. The nurse monitors which of the following as the critical index of central nervous system (CNS) dysfunction?

Level of consciousness

A nurse is turning a postoperative client who had extensive back surgery yesterday. What turning intervention or position would be best for repositioning this client?

Logrolling

MG

Myasthenia gravis is a progressive disorder of weakness of the voluntary muscles. Generally considered an acquired autoimmune disorder, it also can involve genetic factors.

DURA MATER ARACHNOID PIA MATTER

NAME THE 3 MENEGIES

A client is recovering at home after suffering a brain attack (stroke) 2 weeks ago. A home caregiver tells the home health nurse that the client has some difficulty swallowing food and fluids. Which nursing action would be appropriate as a first action?

Observe the client feeding himself or herself.

BRAINSTEM

PART OF THE BRAIN CONNECTS THE CEREBRAL HEMISPHERES & THE SPINAL CORD

A nurse is assisting in checking for Tinel's sign in a client suspected of having carpal tunnel syndrome (CTS). Which technique would the nurse expect to be used to elicit this sign?

Percuss the medial nerve at the wrist as it enters the carpal tunnel, and monitor for tingling sensations.

A client is admitted to the emergency department with a suspected overdose of an unknown drug. Arterial blood gas values indicate respiratory acidosis. What should the nurse do first?

Prepare to assist with ventilation.

MOTOR(EFFERENT)

RCV & TRANS MESSAGES FROM CNS TO ALL BODY PARTS OF THE BODY **CARRIES IMPULSES AWAY FROM CNS VIA EFFECTORS

Which serum levels are expected after a lab was taken for a pt with a hx of kidney disease?

Serum potassium levels are increased in kidney disease.

Sympathetic and Parasympathetic

The autonomic system is divided into what 2 nerve groups?

Midbrain, pons, and medulla

The brainstem consists of what 3 things?

TRIGEMINAL

WHAT IS CRANIAL NERVE V(5)? *SENSORY&MOTOR NERVE (MIXED NERVE) *RESP FOR CHEWING, PAIN TEMP, TOUCH FROM SKIN, EYES TEETH, AND ANTERIOR 2/3 OF THE TONGUE(NOT TASTE) *IF NERVE WAS CUT OR SEVERED YOU WOULD NOT BE ABLE TO CHEW OR FEEL FACE. *DENTAL ANESTHESIA- NUMBS THIS NERVE AND 2/3 OF THE TONGUE

Neglect syndrome (or unilateral neglect):

The patient ignores the affected side of the body; tends to bump into walls, walk to one side, or fail to dress the affected side.

Audiometry

The patient wears earphones through which sounds are presented to determine the hearing range. Used to diagnose conductive hearing loss. *Nursing Implications* Explain that the test is done by an audiologist

A client is admitted to the hospital for observation with a probable minor head injury after an automobile crash. The nurse would plan on leaving the cervical collar in place until:

The result of spinal x-rays is known.

A client comes to the clinic for an ophthalmologic screening, which will include measurement of intraocular pressure with a tonometer. When teaching the client about the test, the nurse should cover which point?

The tonometer will register the force required to indent or flatten the corneal apex.

A nurse is teaching a community class that those experiencing symptoms of ischemic stroke need to enter the medical system early. The primary reason for this is which of the following?

Thrombolytic Therapy has a time window of 3 hours

A nurse is teaching a community class that those experiencing symptoms of ischemic stroke need to enter the medical system early. The primary reason for this is which of the following?

Thrombolytic therapy has a time window of only 3 hours.

PET scan

To assess for cell death or damage in brain tissue, which diagnostic test might be used?

VESTIBULOCOCHLEAR

WHAT IS CRANIAL NERVE VIII(8)? SENSORY NERVE, RESPONSIBLE FOR BALANCE AND HEARING

MID BRAIN

WHAT IS KNOW AS THE "REFLEX CENTER" VISUAL & AUDITORY REFLEX INTEGRATION ALSO KEEPS HEAD UPRIGHT AND BALANCE

SOMATIC

WHAT KIND OF EFFECTOR IS VOLUNTARY REACTION TO A STIMULI; A REACTION THAT CAN BE CONTROLLED (EX- MOVING ARM FROM SHARP OBJECT)

ANS (AUTONOMIC NERVOUS SYSTEM)

WHAT NERVOUS SYSTEM IS RESPONSIBLE FOR INVOLUNTARY ACTIONS(CARDIAC MUSCLE, VISCERAL MUSCLES AND GLANDS)

PONS

WHAT PART OF THE BRAIN STEM CONTAINS NERVE TRACT CARRIES MESSAGES BETWEEN CEREBRUM & MEDULLA. ALSO KNOWN AS THE RESPIRATORY CENTER

LIMBIC SYSTEM

WHAT SYSTEM IN THE BRAIN IS LARGELY RESPONSIBLE FOR MAINTAING A PERSONS LEVEL OF AWARENESS

LIMBIC SYSTEM

WHAT SYSTEM RESPONSIBLE FOR PERSONS LEVEL OR CONCIOUSNESS, WAKEFULNESS/SLEEP, LEARNING, AND LONG TERM MEMORY

RIGHT SIDE OF THE CEREBRAL HEMISPHERE

WHICH CEREBRAL HEMISPHERE PROCESSES SPATIAL PERCEPTION, PICTURES, ART MUSICAL ABILITIES?

ARACHNOID(MIDDLE LAYER)

WHICH MENINGES & LAYER IS MADE UP OF DELICATE WEB LIKE TISSUE

DURA MATTER(OUTER LAYER

WHICH MENINGES&LAYER ADHERES TO THE BONES IN THE SKULL?

lumbar puncture

done to obtain CSF, which is sent for a culture and sensitivity and Gram stain, in order to identify intracranial infections.

The nurse is caring for a client who underwent a lumbar laminectomy 2 days ago. Which finding should the nurse consider abnormal?

Urine retention or incontinence

The nurse is collecting data on a 38-year-old client diagnosed with multiple sclerosis. Which of the following symptoms would the nurse expect to find?

Vision changes

A 58-year-old client complaining of difficulty driving at night states that the "lights bother my eyes." The client wears corrective glasses. The nurse would suspect that the client is experiencing a deficiency in which of the following vitamins?

Vitamin A

SOMATIC(VOLUNTARY) VISCERAL (INVOLUNTARY)

WHAT ARE 2 CLASSIFICATIONS OF EFFECTORS?

CNS(CENTRAL NERVOUS SYSTEM) PNS (PERIPHERAL NERVOUS SYSTEM)

WHAT ARE THE 2 DIVISIONS OF THE NERVOUS SYSTEM?

CRANIAL NERVES/SPINAL NERVES

WHAT ARE THE 2 SETS OF NERVE GROUPS THAT MAKE UP THE PNS (PERIPHERIAL NEVEROUS SYSTEM)

BRAIN & SPINAL CORD

WHAT DOES CNS CONSIST OF? CENTRAL NERVOUS SYSTEM

Self-talk

____________ helps the aphasic person associate activities with specific words and phrases.

Health promotion efforts to decrease the risk for ischemic stroke involve encouraging a healthy lifestyle including

a low-fat, low-cholesterol diet and increased exercise.

A client has been having cluster headaches intermittently over the last year. In an effort to determine the trigger for the cluster headaches, the client has maintained a journal of all oral consumption. What on the list would the nurse suspect could be triggering headaches?

alcoholic beverages

Basal ganglia

are a group of structures located centrally in the cerebrum and mid-brain. Most of then are on both side of the thalamus. The function of it include - imitation - execution and completion of voluntary movement - learning - emotional response - autonomic movements associated with skeletal muscle activity (swinging the arms while walking, swallowing saliva, and blinking)

A nurse performs a neurologic data collection on a client reporting headache and dizziness. Which data collection technique helps determine the motor function of cranial nerve VII?

asking the client to frown, smile, and raise the eyebrows

cerebral embolus

bits of matter eg blood clot or plaque form somwhere and are released into the bloodstream they travel to the cerebral arteries and lodge producing occlusion and infarction

Epidural Hemorrhage

bleeding between the skull and the dura mater, commonly caused from trauma and treated with a burr hole in an emergent situation

If a client experienced a stroke that damaged the hypothalamus, the nurse would anticipate that the client has problems with:

body temperature control.

Gray matter

cell bodies of voluntary motor neuron, preganglionic autonomic motor neurons, and associated neurons

Immediate complications of hemorrhagic stroke

cerebral hypoxia, decreased cerebral blood flow, extension in the area of the injury

main tool to establish the dx of ischemic stroke, rule out hemorrhagic stroke or other CNS lesions like tumor or abcess

cerebrovascular imaging like CT scan

A client is newly diagnosed with myasthenia gravis. When reinforcing education what should the nurse indicate as the cause of this disease?

destruction of acetylcholine receptors, causing muscle weakness

Atonic Seizures

generalized seizures characterized by a sudden loss of muscle tone and usually causing a loss of consciousness, may last seconds of minutes

major stroke-definition

pt has stable, usually severe impairments

You are caring for a client with chronic migraines who is prescribed medication. What drug-related instructions should the nurse give the client?

take medication as soon as symptoms of the migraine begin; take before going to bed or in the morning

Cerebrum

the largest area of the brain and is divided into a right and left hemisphere.

A client has a cervical spinal cord injury at the level of C5. Which condition would the nurse anticipate during the acute phase?

the need for mechanical ventilation

An emergency department nurse is awaiting the arrival of a client with signs of an ischemic stroke that began 1 hour ago, as reported by emergency medical personnel. The treatment window for thrombolytic therapy is which of the following?

three hours

shoulder postural deviations assocaited with stroke

unequal height-more affected shoulder depressed gh subluxation with scapular downard rotation and lateral flexion of trunk may have scap instability leading to winging

impairments of body scheme or image associated with stroke

unilateral neglect anosognosia-Impaired awareness of illness (unaware OR in denial of disability) somatoagnosia-The inability to identify any part of the body, either one's own or another's body. finger agnosia-irst defined in 1924 by Josef Gerstmann, is the loss in the ability "to distinguish, name, or recognize the fingers", not only with the patient's own fingers, but also the fingers of others, and drawing and other representations of fingers rt-lt discrimination

A client has a 12-year history of cluster headaches. After the client describes the characteristics of the head pain, the nurse begins to discuss its potential causes. What would the nurse indicate that the origin of the headaches is:

unknown.

Extensor plantar response (reflex)

up going toes with plantar stimulation suprasegmental or upper motor neuron lesion

The nurse is observing pupillary responses from a client. Which method should the nurse use to evaluate pupil accommodation?

• Observe for pupil constriction and convergence while focusing on an object coming toward the client.

A nurse is positioning a client with flaccid left-sided hemiparesis in bed following a cerebral vascular accident (CVA). Which is the nurse's best intervention?

• Position the left arm supported on a pillow.

The parent of a child with a ventriculoperitoneal shunt calls the nurse saying that the child has a temperature of 101.2° F (38.4° C), a blood pressure of 108/68 mm Hg, and a pulse of 100 beats/minute. The child is lethargic and vomited the night before. Other children in the family have had similar symptoms. Which nursing intervention is most appropriate?

• Tell the parent to bring the child to the primary health care provider's office.

A client is admitted to the facility for investigation of balance and coordination problems, including possible Ménière's disease. When reviewing this client's chart, the nurse expects to find which signs and symptoms?

• Vertigo, tinnitus, and hearing loss

MRI scan and stroke

has evolved to first-line imaging in some stroke centers, but used in some places only when CT has failed to nail down evidence of lesion location more sensitive in the dx of acute strokes downside-cannot be performed of ppl with implants like a pacemaker

stroke incidence stats

incidence of stroke increases dramaticlly with age, doubles in decade after age 65 28% of strokes occur in ppl younger than 65 5-14% of ppl who survive initial stroke will experience another one within a yr...within 5 yrs, 24% of women will have recurrence, 42% of men

Partial Seizures

includes: simple-partial and complex-partial

Decerebrate Posturing

indicating damage to the brain stem or mid-brain

ischemic cascade

ischemia triggers a lot of damaging cellular events excess neurotransmitters like glutamate and aspartate produce progressive disturbance of energy metabolism and axonic depolarization... ....results in inability for brain cells to produce energy (ATP) ....followed by excess influx of of calcium ions....neuronal membrane pump failure the excess calcium reacts with intracellular phospholipids and forms free radicals...the excess calcium also stims release of nitric oxide and cytokines......both of these things also damage brain cells

The nurse is caring for a client with an acute bleeding cerebral aneurysm. The nurse should take all of the following steps except:

keep the client in one position to decrease bleeding.

dysphasia (speech)

language disorder marked by deficiency in the generation of speech, and sometimes also in its comprehension, due to brain disease or damage.

When obtaining the health history from a client with retinal detachment, the nurse expects the client to report:

light flashes and floaters in front of the eye.

mening/o

meninges

UE postural deviations associarted with stroke

more affected UE is typically held in flexed, adducted position..limb is NWB stronger UE is used for postural support

To encourage adequate nutritional intake for a client with Alzheimer's disease, the nurse should:

stay with the client and encourage him to eat.

stroke stats that for some reason may be on the test

stroke is 4th leading cause of death and leading cause of long term disability in the USA 7,000,000 amercians over 20 yrs age have had a stroke each year 795,000 ppl have stroke...610,00 of those are first time strokes, the other 185,000 are recurrent strokes

Doppler ultrasound

studies evaluate the flow of blood through the carotid arteries and identify if a vessel is partially or completely occluded.

Crede

The ________ maneuver, in which the open hand is pressed over the bladder area and directed toward the suprapubic area, can facilitate emptying a flaccid bladder.

FOUR (Full Outline of Unresponsiveness)

The ________ score is becoming the preferred scale to assess comatose or intubated patients who cannot speak.

Efferent

The _________ division of the PNS carries impulses away from the CNS.

Afferent

The _________ division of the PNS carries impulses to the CNS.

Sympathetic

The ___________ nerves work by mobilizing energy to initiate changes aimed at maintaining or restoring homeostasis.

A female client with myasthenia gravis comes to the health care provider's office for a scheduled office visit. The client is very concerned and tells the nurse that her husband seems to be avoiding her because she is very unattractive. The appropriate nursing response is:

"Have you thought about sharing your feelings with your husband?"

A client with a conductive hearing disorder caused by ankylosis of the stapes in the oval window undergoes a stapedectomy to remove the stapes and replace the impaired bone with a prosthesis. After the stapedectomy, the nurse should provide which client instruction?

"Don't fly in an airplane, climb to high altitudes, make sudden movements, or expose yourself to loud sounds for 30 days."

A nurse is educating the parents of a 1-year-old infant with otitis media. Which statement regarding predisposing factors for otitis media would be most accurate for the nurse to make?

"Eustachian tubes are short, wide, and straight and lie in a horizontal plane."

A client is somewhat nervous about having magnetic resonance imaging (MRI). Which statement by the nurse would provide reassurance to the client about the procedure?

"Even though you are alone in the scanner, you will be in voice communication with the technologist during the procedure."

A nurse is instructing the wife of a patient who suffered a stroke about the use of eating devices her husband will be using. During the teaching, the wife starts to cry and states "One minute my husband is laughing, and the next he's crying; I just don't understand what's wrong with him." The nurse's best response is which of the following?

"Following a stroke, emotional lability is common, and it usually improves with time."

A nurse has given medication instructions to the client receiving phenytoin (Dilantin). The nurse determines that the client understands the instructions if the client states:

"Good oral hygiene is needed, including brushing and flossing."

The nurse is preparing a client for a computed tomography (CT) scan, which requires infusion of radiopaque dye. Which question is important for the nurse to ask?

"Are you allergic to seafood or iodine?"

A nurse is caring for a client with a diagnosis of multiple sclerosis who has been prescribed oxybutynin (Ditropan). The nurse evaluates the effectiveness of the medication by asking the client which of the following questions?

"Are you getting up at night to urinate?"

When providing discharge teaching for a client with multiple sclerosis (MS), the nurse should include which instruction?

"Avoid hot baths and showers."

A quadriplegic client is prescribed baclofen, 5 mg by mouth three times daily. The client asks what is the indication for baclofen. How would the nurse respond?

"Baclofen is indicated when there are muscle spasms with paraplegia or quadriplegia from spinal cord lesions."

When teaching a client about levodopa and carbidopa (Sinemet) therapy for Parkinson's disease, the nurse should include which instruction?

"Be aware that your urine may appear darker than usual."

A client with Parkinson's disease quickly develops akinesia while ambulating, increasing the risk for falls. Which suggestion should the nurse provide to the client to alleviate this problem?

"Consciously think about walking over imaginary lines on the floor."

Which statement by the parent of a child with otitis media indicates an understanding of the nurse's discharge instructions about the use of antibiotics?

"I'll give my child the full course of antibiotics."

A resident in a long-term care facility prepares to walk out into a rainstorm after saying, "My father is waiting to take me for a ride." Which of the following would be an appropriate response by the nurse?

"I'm glad you told me that. Let's have a cup of coffee and you can tell me about your father."

A client with multiple sclerosis who is unable to bathe herself complains that other staff members haven't been bathing her. How should the nurse respond to this client's complaint?

"I'm sorry you haven't been bathed. I'm available to bathe you now."

A client who sustained a closed head injury in a motor vehicle accident is diagnosed as brain dead by a neurosurgeon. The physician has scheduled a meeting with the client's family about discontinuing life support. Before the meeting, a family member asks the nurse her opinion about life support. Which response by the nurse is appropriate?

"What has the physician explained about the client's prognosis?"

what percent of cardiac output does cerebral blood flow typically represent

17%

The physician orders measurement of the serum acetaminophen level of a client admitted with a suspected overdose of this drug. To ensure an accurate result, the nurse should wait how long after acetaminophen (Tylenol) ingestion before drawing the blood sample?

4 hours

The health care provider prescribed t-PA, a thrombolytic agent. The order is for 0.9 mg/kg over 1 hour. The client weighs 110 lb (50 kg). What is the total dose in milligrams the client will receive? Record your answer using a whole number.

45 Explanation: 0.9 mg/kg × 50 kg = 45 mg

NEUROTRANSMITTER

A CHEMICAL SUBSTANCE THAT AN AXON RELEASES TO ALLOW NERVE IMPULSES TO CROSS THE SYNAPSE & REACH DENDRITES

A client is transferred to the intensive care unit after evacuation of a subdural hematoma. Which nursing intervention would reduce the client's risk of increased intracranial pressure (ICP)?

Administering a stool softener as prescribed

41. A client is at risk for increased ICP. Which of the following would be a priority for the nurse to monitor? 1. Unequal pupil size 2. Decreasing systolic blood pressure 3. Tachycardia 4. Decreasing body temperature

Answer: 1. Increasing ICP causes unequal pupils as a result of pressure on the third cranial nerve. Increasing ICP causes an increase in the systolic pressure, which reflects the additional pressure needed to perfuse the brain. It increases the pressure on the vagus nerve, which produces bradycardia, and it causes an increase in body temperature from hypothalamic damage.

4. A female client with Guillain-Barré syndrome has paralysis affecting the respiratory muscles and requires mechanical ventilation. When the client asks the nurse about the paralysis, how should the nurse respond? A. "You may have difficulty believing this, but the paralysis caused by this disease is temporary." B. "You'll have to accept the fact that you're permanently paralyzeD. However, you won't have any sensory loss." C. "It must be hard to accept the permanency of your paralysis." D. "You'll first regain use of your legs and then your arms."

Answer: A. "You may have difficulty believing this, but the paralysis caused by this disease is temporary." The nurse should inform the client that the paralysis that accompanies Guillain-Barré syndrome is only temporary. Return of motor function begins proximally and extends distally in the legs.

10. Emergency medical technicians transport a 27-year-old iron worker to the emergency department. They tell the nurse, "He fell from a two-story building. He has a large contusion on his left chest and a hematoma in the left parietal area. He has a compound fracture of his left femur and he's comatose. We intubated him and he's maintaining an arterial oxygen saturation of 92% by pulse oximeter with a manual resuscitation bag." Which intervention by the nurse has the highest priority? A. Assessing the left leg B. Assessing the pupils C. Placing the client in Trendelenburg's position D. Assessing level of consciousness

Answer: A. Assessing the left leg In the scenario, airway and breathing are established so the nurse's next priority should be circulation. With a compound fracture of the femur, there is a high risk of profuse bleeding; therefore, the nurse should assess the site.

1. If a male client experienced a cerebrovascular accident (CVA) that damaged the hypothalamus, the nurse would anticipate that the client has problems with: A. Body temperature control. B. Balance and equilibrium. C. Visual acuity. D. Thinking and reasoning.

Answer: A. Body temperature control. The body's thermostat is located in the hypothalamus; therefore, injury to that area can cause problems of body temperature control.

5. The nurse is working on a surgical floor. The nurse must log roll a male client following a: A. Laminectomy. B. Thoracotomy. C. Hemorrhoidectomy. D. Cystectomy.

Answer: A. Laminectomy. The client who has had spinal surgery, such as laminectomy, must be logrolled to keep the spinal column straight when turning.

25. A male client with Bell's Palsy asks the nurse what has caused this problem. The nurse's response is based on an understanding that the cause is: A. Unknown, but possibly includes ischemia, viral infection, or an autoimmune problem B. Unknown, but possibly includes long-term tissue malnutrition and cellular hypoxia C. Primary genetic in origin, triggered by exposure to meningitis D. Primarily genetic in origin, triggered by exposure to neurotoxins

Answer: A. Unknown, but possibly includes ischemia, viral infection, or an autoimmune problem Bell's palsy is a one-sided facial paralysis from compression of the facial nerve. The exact cause is unknown but may include vascular ischemia, infection, exposure to viruses such as herpes zoster or herpes simplex, autoimmune disease, or a combination of these factors.

18. A female client is admitted to the facility for investigation of balance and coordination problems, including possible Ménière's disease. When assessing this client, the nurse expects to note: A. Vertigo, tinnitus, and hearing loss. B. Vertigo, vomiting, and nystagmus C. Vertigo, pain, and hearing impairment. D. Vertigo, blurred vision, and fever.

Answer: A. Vertigo, tinnitus, and hearing loss. Ménière's disease, an inner ear disease, is characterized by the symptom triad of vertigo, tinnitus, and hearing loss. The combination of vertigo, vomiting, and nystagmus suggests labyrinthitis. Ménière's disease rarely causes pain, blurred vision, or fever.

28. The nurse is assessing a 37-year-old client diagnosed with multiple sclerosis. Which of the following symptoms would the nurse expect to find? A. Vision changes B. Absent deep tendon reflexes C. Tremors at rest D. Flaccid muscles

Answer: A. Vision changes Vision changes, such as diplopia, nystagmus, and blurred vision, are symptoms of multiple sclerosis.

30. A female client has a neurological deficit involving the limbic system. Specific to this type of deficit, the nurse would document which of the following information related to the client's behavior. A. Is disoriented to person, place, and time B. Affect is flat, with periods of emotional lability C. Cannot recall what was eaten for breakfast today D. Demonstrate inability to add and subtract; does not know who is the president

Answer: B. Affect is flat, with periods of emotional lability The limbic system is responsible for feelings (affect) and emotions.

16. A client, age 22, is admitted with bacterial meningitis. Which hospital room would be the best choice for this client? A. A private room down the hall from the nurses' station B. An isolation room three doors from the nurses' station C. A semi private room with a 32-year-old client who has viral meningitis D. A two-bed room with a client who previously had bacterial meningitis

Answer: B. An isolation room three doors from the nurses' station A client with bacterial meningitis should be kept in isolation for at least 24 hours after admission.

26. A male client is color blind. The nurse understands that this client has a problem with: A. Rods. B. Cones. C. Lens. D. Aqueous humor.

Answer: B. Cones. Cones provide daylight color vision, and their stimulation is interpreted as color. If one or more types of cones are absent or defective, color blindness occurs.

13. After an eye examination, a male client is diagnosed with open-angle glaucoma. The physician prescribes Pilocarpine ophthalmic solution (Pilocar), 0.25% gtt i, OU q.i.D. Based on this prescription, the nurse should teach the client or a family member to administer the drug by: A. Instilling one drop of pilocarpine 0.25% into both eyes daily. B. Instilling one drop of pilocarpine 0.25% into both eyes four times daily. C. Instilling one drop of pilocarpine 0.25% into the right eye daily. D. Instilling one drop of pilocarpine 0.25% into the left eye four times daily.

Answer: B. Instilling one drop of pilocarpine 0.25% into both eyes four times daily. The abbreviation "gtt" stands for drop, "i" is the apothecary symbol for the number 1, OU signifies both eyes, and "q.i.d." means four times a day. Therefore, one drop of pilocarpine 0.25% should be instilled into both eyes four times daily.

9. A female client with amyotrophic lateral sclerosis (ALS) tells the nurse, "Sometimes I feel so frustrateD. I can't do anything without help!" This comment best supports which nursing diagnosis? A. Anxiety B. Powerlessness C. Ineffective denial D. Risk for disuse syndrome

Answer: B. Powerlessness This comment best supports a nursing diagnosis of Powerlessness because ALS may lead to locked-in syndrome, characterized by an active and functioning mind locked in a body that can't perform even simple daily tasks.

2. The nurse is teaching a female client with multiple sclerosis. When teaching the client how to reduce fatigue, the nurse should tell the client to: A. Take a hot bath. B. Rest in an air-conditioned room. C. Increase the dose of muscle relaxants. D. Avoid naps during the day.

Answer: B. Rest in an air-conditioned room. Fatigue is a common symptom in clients with multiple sclerosis. Lowering the body temperature by resting in an air-conditioned room may relieve fatigue; however, extreme cold should be avoided. Other measures to reduce fatigue in the client with multiple sclerosis include treating depression, using occupational therapy to learn energy conservation techniques, and reducing spasticity.

1. A white female client is admitted to an acute care facility with a diagnosis of cerebrovascular accident (CVA). Her history reveals bronchial asthma, exogenous obesity, and iron deficiency anemia. Which history finding is a risk factor for CVA? A. Caucasian race B. Female sex C. Obesity D. Bronchial asthma

Answer: C. Obesity Obesity is a risk factor for CVA. Other risk factors include a history of ischemic episodes, cardiovascular disease, diabetes mellitus, atherosclerosis of the cranial vessels, hypertension, polycythemia, smoking, hypercholesterolemia, oral contraceptive use, emotional stress, family history of CVA, and advancing age.

A patient diagnosed with a stroke is ordered to receive warfarin (Coumadin). Later, the nurse learns that the warfarin is contraindicated and the order is canceled. The nurse knows that the best alternative medication to give is which of the following?

Aspirin

7. The nurse is performing a mental status examination on a male client diagnosed with a subdural hematoma. This test assesses which of the following? A. Cerebellar function B. Intellectual function C. Cerebral function D. Sensory function

Answer: C. Cerebral function The mental status examination assesses functions governed by the cerebrum. Some of these are orientation, attention span, judgment, and abstract reasoning.

22. Nurse Kristine is trying to communicate with a client with brain attack (stroke) and aphasia. Which of the following actions by the nurse would be least helpful to the client? A. Speaking to the client at a slower rate B. Allowing plenty of time for the client to respond C. Completing the sentences that the client cannot finish D. Looking directly at the client during attempts at speech

Answer: C. Completing the sentences that the client cannot finish Clients with aphasia after brain attack (stroke) often fatigue easily and have a short attention span. The nurse would avoid shouting (because the client is not deaf), appearing rushed for a response, and letting family members provide all the responses for the client.

15. A male client in the emergency department has a suspected neurologic disorder. To assess gait, the nurse asks the client to take a few steps; with each step, the client's feet make a half circle. To document the client's gait, the nurse should use which term? A. Ataxic B. Dystrophic C. Helicopod D. Steppage

Answer: C. Helicopod A helicopod gait is an abnormal gait in which the client's feet make a half circle with each step.

17. A physician diagnoses a client with myasthenia gravis, prescribing pyridostigmine (Mestinon), 60 mg P.O. every 3 hours. Before administering this anticholinesterase agent, the nurse reviews the client's history. Which preexisting condition would contraindicate the use of pyridostigmine? A. Ulcerative colitis B. Blood dyscrasia C. Intestinal obstruction D. Spinal cord injury

Answer: C. Intestinal obstruction Anticholinesterase agents such as pyridostigmine are contraindicated in a client with a mechanical obstruction of the intestines or urinary tract, peritonitis, or hypersensitivity to anticholinesterase agents.

11. Nurse Mary witnesses a neighbor's husband sustain a fall from the roof of his house. The nurse rushes to the victim and determines the need to opens the airway in this victim by using which method? A. Flexed position B. Head tilt-chin lift C. Jaw-thrust maneuver D. Modified head tilt-chin lift

Answer: C. Jaw-thrust maneuver If a neck injury is suspected, the jaw thrust maneuver is used to open the airway.

23. A female client has experienced an episode of myasthenic crisis. The nurse would assess whether the client has precipitating factors such as: A. Getting too little exercise B. Taking excess medication C. Omitting doses of medication D. Increasing intake of fatty foods

Answer: C. Omitting doses of medication Myasthenic crisis often is caused by under medication and responds to the administration of cholinergic medications, such as neostigmine (Prostigmin) and pyridostigmine (Mestinon). Option B: Cholinergic crisis (the opposite problem) is caused by excess medication and responds to withholding of medications. Options A and D: Too little exercise and fatty food intake are incorrect. Overexertion and overeating possibly could trigger

3. After striking his head on a tree while falling from a ladder, a young man age 18 is admitted to the emergency department. He's unconscious and his pupils are nonreactive. Which intervention would be the most dangerous for the client? A. Give him a barbiturate. B. Place him on mechanical ventilation. C. Perform a lumbar puncture. D. Elevate the head of his bed.

Answer: C. Perform a lumbar puncture. The client's history and assessment suggest that he may have increased intracranial pressure (ICP). If this is the case, lumbar puncture shouldn't be done because it can quickly decompress the central nervous system and, thereby, cause additional damage.

10. For a male client with suspected increased intracranial pressure (ICP), a most appropriate respiratory goal is to: A. Prevent respiratory alkalosis. B. Lower arterial pH. C. Promote carbon dioxide elimination. D. Maintain partial pressure of arterial oxygen (PaO2) above 80 mm Hg

Answer: C. Promote carbon dioxide elimination. The goal of treatment is to prevent acidemia by eliminating carbon dioxide. That is because an acid environment in the brain causes cerebral vessels to dilate and therefore increases ICP.

19. A male client with a conductive hearing disorder caused by ankylosis of the stapes in the oval window undergoes a stapedectomy to remove the stapes and replace the impaired bone with a prosthesis. After the stapedectomy, the nurse should provide which client instruction? A. "Lie in bed with your head elevated, and refrain from blowing your nose for 24 hours." B. "Try to ambulate independently after about 24 hours." C. "Shampoo your hair every day for ten (10) days to help prevent ear infection." D. "Don't fly in an airplane, climb to high altitudes, make sudden movements, or expose yourself to loud sounds for 30 days."

Answer: D. "Don't fly in an airplane, climb to high altitudes, make sudden movements, or expose yourself to loud sounds for 30 days." For 30 days after a stapedectomy, the client should avoid air travel, sudden movements that may cause trauma, and exposure to loud sounds and pressure changes (such as from high altitudes).

DVT, pulmonary embolism and stroke

DVT, PE are potential complications for all immobilized pts DVT incidence for stroke pts-as high as 47% dangers particularly high during acute phase----venous stasis from immobility and prolonged bed rest, limb paralysis, hemi-neglect, reduced cognitive function enhance the risk 50% of cases dont present with clinically detectable symptoms-----gold standard for dx- a doppler duplex ultrasonography prompt dx of DVT is necessary to reduce risk of fatal PE treatment: heparin, coumadin, etc prevention-heparin, stuff like ankle pumps, TED hose, early mobilization Pt symptoms: calf pain and tenderness, or a tight feeling in the calf.

A nurse is monitoring a client with a head injury and notes that the client is assuming this posture. The nurse notifies the registered nurse immediately to report that the client is exhibiting: Refer to figure.

Decorticate posturing

A patient has been diagnosed as having global aphasia. The nurse recognizes that the patient will be unable to do which of the following actions?

Form understandable words and comprehend the spoken word global= expressive+ receptive aphasia

A client who is recovering from a brain attack (stroke) has residual dysphagia. The licensed practical nurse instructs the nursing assistant to avoid which of the following at mealtime?

Giving the client thin liquids

CNS

The ______ is made up of the brain and spinal cord.

HD

Huntington's disease, also known as Huntington's chorea, is a chronic, progressive, hereditary condition in which brain cells in the basal ganglia prematurely die.

Cranial nerve XII

Hypoglossal nerve

Astereognosis (sensory system)

Inability to recognize form of object by touch Lesions in parietal cortex

Bladder dysfunction (spinal cord) Hypertonic

Increase in muscle tone, diminished capacity, reflex emptying, dribbling, incontinence lesion in pyramidal tract (efferent pathways)

A nurse is caring for a client with a diagnosis of brain attack (stroke) with anosognosia. To meet the needs of the client with this deficit, the nurse plans activities that will:

Increase the client's awareness of the affected side.

A client is admitted with a cervical spine injury sustained during a diving accident. When planning this client's care, the nurse should assign highest priority to which nursing diagnosis?

Ineffective breathing pattern

Anisocoria (eyes)

Inequality of pupil size optic nerve injury

Schwann

The ________ cells form myelin sheaths that wrap around the axons in the PNS.

The family of an unconscious client with increased intracranial pressure is talking at the client's bedside. They are discussing the gravity of the client's condition and wondering if the client will ever recover. The nurse intervenes, based on the understanding that:

It is possible the client can hear the family.

X Vagus

Swallowing

ataxia

The person has difficulty maintaining balance and coordination (ataxia).

A client has a cerebellar lesion. The nurse determines that the client is adapting successfully to this problem if the client demonstrates proper use of which of the following items?

Walker

The physician prescribes mannitol (Osmitrol) I.V. stat for a client who develops increased intracranial pressure after a head injury. While preparing to administer mannitol, the nurse notices crystals in the solution. What should the nurse do?

Warm the solution in hot water to dissolve the crystals.

A stable and calm environment

What is extremely important for a confused patient to have?

Diencephalon

Which division of the brain consists of 2 parts?

Neuron excitability

ability to generate a nerve impulse

definition of motor praxis

ability to plan and execute coordinated movment

if you have a patient with a lesion involving the cortex of the dominant hemipshere (usually left) what is a really common type of impairment they are going to have?

speech and language impairments

A nurse is collecting data on a client with myasthenia gravis. The nurse determines that the client may be developing myasthenic crisis if the client states:

"I can't swallow very well today."

A client with Parkinson's disease visits the physician's office for a routine checkup. The nurse notes that the client takes benztropine (Cogentin), 0.5 mg by mouth daily, and asks when the client takes the drug each day. Which response indicates that the client understands when to take benztropine?

"I take the medication at bedtime."

A nurse is providing instructions to the client who has just been fitted for a halo vest. Which statement by the client indicates the need for further instructions?

"I will avoid driving at night because the vest limits the ability to turn the head."

A client who was diagnosed with multiple sclerosis 3 years ago now presents with lower extremity weakness and heaviness. During the admission process, the client presents her advance directive, which states that she doesn't want intubation, mechanical ventilation, or tube feedings should her condition deteriorate. How should the nurse respond?

"It's important for us to have this information. You should review the document with your physician at every admission."

The parent of a child with a history of closed-head injury asks the nurse why the child would begin having seizures without warning. Which response by the nurse is the most accurate?

"It's not unusual to develop seizures after a head injury because of brain trauma."

A client with idiopathic seizure disorder is being discharged with a prescription for phenytoin (Dilantin). Client teaching about this drug should include which instruction?

"Schedule follow-up visits with your physician for blood tests."

A family member brings the client to the clinic for a follow-up visit after a stroke. The family member asks the nurse what he can do to decrease his chance of having another stroke. What would be the nurse's best answer?

"Stop smoking as soon as possible."

A client who has experienced an initial transient ischemic attack (TIA) states: "I'm glad it wasn't anything serious." Which is the best nursing response to this statement?

"TIA is a warning sign. Let's talk about lowering your risks."

A nurse is caring for a child with spina bifida. The child's mother asks the nurse what she did to cause the birth defect. Which statement would be the nurse's best response?

"The cause is unknown, and there are many environmental factors that may contribute to it."

A client arrives in the emergency department after a bicycle crash. Which of the following assessment questions is most important for the nurse to ask? a. "Were you wearing a helmet?" b. "Have you had difficulty sleeping at night?" c. "Do you have a family history of diabetes?" d. "Do you take any herbal preparations?"

"Were you wearing a helmet?" Rationale: The nurse should ask if the client was wearing a helmet in order to assess the extent of injury. Difficulty sleeping does not relate to this case. It is important to know if the client has diabetes, but not the most important for this client. Taking herbal preparations is not relevant for this client's situation.

A client who sustained an L1 to L2 spinal cord injury in a construction accident asks a nurse if he'll ever be able to walk again. Which response by the nurse is appropriate?

"What has your physician told you about your ability to walk again?"

Subarachnoid Hemorrhage

"the worst headache of my life"

(SELECT ALL THAT APPLY) The nurse is teaching a client with trigeminal neuralgia how to minimize pain episodes. Which comments by the client indicate that he understands the instructions?

(2) "I'll try to chew my food on the unaffected side.", (4) "Drinking fluids at room temperature should reduce pain.", (5) "If brushing my teeth is too painful, I'll try to rinse my mouth instead."

(SELECT ALL THAT APPLY) A client is admitted to the medical-surgical unit after undergoing intracranial surgery to remove a tumor from the left cerebral hemisphere. Which nursing interventions are appropriate for the client's postoperative care?

(2) Turn the client on his right side., (5) Apply a soft collar to keep the client's neck in a neutral position.

(SELECT ALL THAT APPLY) A client who had a massive stroke exhibits decerebrate posture. What are the characteristics of this posture?

(2) Wrist pronation, (3) Stiff extension of the arms and legs, (4) Plantar flexion of the feet (5) Opisthotonos

(SELECT ALL THAT APPLY) The nurse is assigned to care for a client with early stage Alzheimer's disease. Which nursing interventions should be included in the client's care plan?

(3) Furnish the client's environment with familiar possessions., (4) Assist the client with activities of daily living (ADLs) as necessary., (5) Assign tasks in simple steps.

What should the nurse do when administering pilocarpine (Pilocar)?

Apply pressure on the inner canthus to prevent systemic absorption.

A nurse is teaching the daughter of a client who has Alzheimer's disease. The daughter asks, "Will the medication my mother is taking improve her dementia" How should the nurse respond "It will allow your mother to live independently for several more years." "It is used to halt the advancement of Alzheimer's disease but will not cure it." "It will not improve her dementia but can help control emotional responses." "It is used to improve short-term memory but will not improve problem solving."

* "It will not improve her dementia but can help control emotional responses."* Drug therapy is not effective for treating dementia or halting the advancement of Alzheimer's disease. However, certain drugs may help suppress emotional disturbances and psychiatric manifestations. Medication therapy may not allow the client to safely live independently.

A nurse delegates care for a client with Parkinson disease to an unlicensed assistive personnel (UAP). Which statement should the nurse include when delegating this client's care "Allow the client to be as independent as possible with activities." "Assist the client with frequent and meticulous oral care." "Assess the client's ability to eat and swallow before each meal." "Schedule appointments early in the morning to ensure rest in the afternoon."

*"Allow the client to be as independent as possible with activities."* Clients with Parkinson disease do not move as quickly and can have functional problems. The client should be encouraged to be as independent as possible and provided time to perform activities without rushing. Although oral care is important for all clients, instructing the UAP to provide frequent and meticulous oral is not a priority for this client. This statement would be a priority if the client was immune-compromised or NPO. The nurse should assess the client's ability to eat and swallow; this should not be delegated. Appointments and activities should not be scheduled early in the morning because this may cause the client to be rushed and discourage the client from wanting to participate in activities of daily living.

A nurse obtains a focused health history for a client who is suspected of having bacterial meningitis. Which question should the nurse ask "Do you live in a crowded residence" "When was your last tetanus vaccination" "Have you had any viral infections recently" "Have you traveled out of the country in the last month"

*"Do you live in a crowded residence"* Meningococcal meningitis tends to occur in multiple outbreaks. It is most likely to occur in areas of high-density population, such as college dormitories, prisons, and military barracks. A tetanus vaccination would not place the client at increased risk for meningitis or protect the client from meningitis. A viral infection would not lead to bacterial meningitis but could lead to viral meningitis. Simply knowing if the client traveled out of the country does not provide enough information. The nurse should ask about travel to specific countries in which the disease is common, for example, sub-Saharan Africa.

A nurse cares for a client with advanced Alzheimer's disease. The client's caregiver states, "She is always wandering off. What can I do to manage this restless behavior" How should the nurse respond "This is a sign of fatigue. The client would benefit from a daily nap." "Engage the client in scheduled activities throughout the day." "It sounds like this is difficult for you. I will consult the social worker." "The provider can prescribe a mild sedative for restlessness."

*"Engage the client in scheduled activities throughout the day."* Several strategies may be used to cope with restlessness and wandering. One strategy is to engage the client in structured activities. Another is to take the client for frequent walks. Daily naps and a mild sedative will not be as effective in the management of restless behavior. Consulting the social worker does not address the caregiver's concern.

After teaching a client who is diagnosed with new-onset status epilepticus and prescribed phenytoin (Dilantin), the nurse assesses the client's understanding. Which statement by the client indicates a correct understanding of the teaching "To prevent complications, I will drink at least 2 liters of water daily." "This medication will stop me from getting an aura before a seizure." "I will not drive a motor vehicle while taking this medication." "Even when my seizures stop, I will continue to take this drug."

*"Even when my seizures stop, I will continue to take this drug."* Discontinuing antiepileptic drugs can lead to the recurrence of seizures or status epilepticus. The client does not need to drink more water and can drive while taking this medication. The medication will not stop an aura before a seizure.

What should the nurse do when administering pilocarpine?

Apply pressure on the inner canthus to prevent systemic absorption.

After teaching the wife of a client who has Parkinson disease, the nurse assesses the wife's understanding. Which statement by the client's wife indicates she *correctly understands* changes associated with this disease "His masklike face makes it difficult to communicate, so I will use a white board." "He should not socialize outside of the house due to uncontrollable drooling." "This disease is associated with anxiety causing increased perspiration." "He may have trouble chewing, so I will offer bite-sized portions."

*"He may have trouble chewing, so I will offer bite-sized portions."* Because chewing and swallowing can be problematic, small frequent meals and a supplement are better for meeting the client's nutritional needs. A masklike face and drooling are common in clients with Parkinson disease. The client should be encouraged to continue to socialize and communicate as normally as possible. The wife should understand that the client's masklike face can be misinterpreted and additional time may be needed for the client to communicate with her or others. Excessive perspiration is also common in clients with Parkinson disease and is associated with the autonomic nervous system's response.

A nurse witnesses a client with late-stage Alzheimer's disease eat breakfast. Afterward the client states, "I am hungry and want breakfast." How should the nurse respond "I see you are still hungry. I will get you some toast." "You ate your breakfast 30 minutes ago." "It appears you are confused this morning." "Your family will be here soon. Let's get you dressed."

*"I see you are still hungry. I will get you some toast."* Use of validation therapy with clients who have Alzheimer's disease involves acknowledgment of the client's feelings and concerns. This technique has proved more effective in later stages of the disease, when using reality orientation only increases agitation. Telling the client that he or she already ate breakfast may agitate the client. The other statements do not validate the client's concerns.

After teaching a client newly diagnosed with epilepsy, the nurse assesses the client's understanding. Which statement by the client indicates a need for additional teaching "I will wear my medical alert bracelet at all times." "While taking my epilepsy medications, I will not drink any alcoholic beverages." "I will tell my doctor about my prescription and over-the-counter medications." "If I am nauseated, I will not take my epilepsy medication."

*"If I am nauseated, I will not take my epilepsy medication."* The nurse must emphasize that antiepileptic drugs must be taken even if the client is nauseous. Discontinuing the medication can predispose the client to seizure activity and status epilepticus. The client should not drink alcohol while taking seizure medications. The client should wear a medical alert bracelet and should make the doctor aware of all medications to prevent complications of polypharmacy.

A nurse is teaching a client who has chronic headaches. Which statements about headache triggers should the nurse include in this client's plan of care (Select all that apply.) "Increase your intake of caffeinated beverages." "Incorporate physical exercise into your daily routine." "Avoid all alcoholic beverages." "Participate in a smoking cessation program." "Increase your intake of fruits and vegetables."

*"Incorporate physical exercise into your daily routine."* *"Avoid all alcoholic beverages."* *"Participate in a smoking cessation program."* Triggers for headaches include caffeine, smoking, and ingestion of pickled foods, so these factors should be avoided. Clients are taught to eat a balanced diet and to get adequate exercise and rest. Alcohol does not trigger chronic headaches but can enhance headaches during the headache period.

A nurse prepares to discharge a client with Alzheimer's disease. Which statement should the nurse include in the discharge teaching for this client's caregiver "Allow the client to rest most of the day." "Place a padded throw rug at the bedside." "Install deadbolt locks on all outside doors." "Provide a high-calorie and high-protein diet."

*"Install deadbolt locks on all outside doors."* Clients with Alzheimer's disease have a tendency to wander, especially at night. If possible, alarms should be installed on all outside doors to alert family members if the client leaves. At a minimum, all outside doors should have deadbolt locks installed to prevent the client from going outdoors unsupervised. The client should be allowed to exercise within his or her limits. Throw rugs are a slip and fall hazard and should be removed. The client should eat a well-balanced diet. There is no need for a high-calorie or high-protein diet.

A nurse is teaching a client with chronic migraine headaches. Which statement related to complementary therapy should the nurse include in this client's teaching "Place a warm compress on your forehead at the onset of the headache." "Wear dark sunglasses when you are in brightly lit spaces." "Lie down in a darkened room when you experience a headache." "Set your alarm to ensure you do not sleep longer than 6 hours at one time."

*"Lie down in a darkened room when you experience a headache."* At the onset of a migraine attack, the client may be able to alleviate pain by lying down and darkening the room. He or she may want both eyes covered and a cool cloth on the forehead. If the client falls asleep, he or she should remain undisturbed until awakening. The other options are not recognized therapies for migraines.

A nurse delegates care for a client with early-stage Alzheimer's disease to an unlicensed assistive personnel (UAP). Which statement should the nurse include when delegating this client's care "If she is confused, play along and pretend that everything is okay." "Remove the clock from her room so that she doesn't get confused." "Reorient the client to the day, time, and environment with each contact." "Use validation therapy to recognize and acknowledge the client's concerns."

*"Reorient the client to the day, time, and environment with each contact."* Clients who have early-stage Alzheimer's disease should be reoriented frequently to person, place, and time. The UAP should reorient the client and not encourage the client's delusions. The room should have a clock and white board with the current date written on it. Validation therapy is used with late-stage Alzheimer's disease.

A nurse is teaching a client who experiences migraine headaches and is prescribed a beta blocker. Which statement should the nurse include in this client's teaching "Take this drug only when you have prodromal symptoms indicating the onset of a migraine headache." "Take this drug as ordered, even when feeling well, to prevent vascular changes associated with migraine headaches." "This drug will relieve the pain during the aura phase soon after a headache has started." "This medication will have no effect on your heart rate or blood pressure because you are taking it for migraines."

*"Take this drug as ordered, even when feeling well, to prevent vascular changes associated with migraine headaches."* Beta blockers are prescribed as prophylactic treatment to prevent the vascular changes that initiate migraine headaches. Heart rate and blood pressure will also be affected, and the client should monitor these side effects. The other responses do not discuss appropriate uses of the medication.

MCA symptoms basic

**what are all the areas of lesion if the stroke occurred at the level of middle cerebral artery?** the areas of lesion would be the motor cortex and that would have an effect on the upper limb and the face. the sensory cortex which would also have an effect on the upper limb and face. the temporal lobe which can to wernicke's area being affected the frontal lobe and that could lead to Broca's area being affected **what is a symptom that can occur due to motor cortex being affected in MCA stroke?** at the level of the middle cerebral artery due to the effects on the motor cortex this will lead to a contralateral paralysis of the upper limb and face **what is a symptom that can arise due to sensory cortex being affected?** stroke at the level of the middle cerebral artery leads to the sensorycortex being affected. in this there would be a contralateral loss of sensation and the upper limb and face. *** in what situation would aphasias happen?** in a middle cerebral artery stroke aphasias can occur if the lesion affects the dominant hemisphere which is usually the left hemisphere **what happens in a middle cerebral artery stroke that affects the non-dominant side** in a middle cerebral artery stroke that affects the non-dominant side which is usually the right side this can cause heavy neglect

ACA stroke effects basic

**what are the areas of lesion and a stroke at the level of anterior cerebral artery?** stroke at the level of anterior cerebral artery can cause motor cortex lesion and that would affect the lower limbs.... as well as sensory cortex and that would also affect the lower limbs **what are the symptoms that can arise due to motor cortex being involved in an anterior cerebral artery stroke?** due to the motor cortex being involved this can lead to contralateral paralysis of the lower limbs **what is a symptom that can arise due to sensory cortex being involved in an anterior cerebral artery stroke? ** the symptoms that would arise would be a contralateral loss of sensation in the lower limbs

A nurse assesses clients on a medical-surgical unit. Which clients should the nurse identify as at risk for secondary seizures (Select all that apply.) A 26-year-old woman with a left temporal brain tumor A 38-year-old male client in an alcohol withdrawal program A 42-year-old football player with a traumatic brain injury A 66-year-old female client with multiple sclerosis A 72-year-old man with chronic obstructive pulmonary disease

*A 26-year-old woman with a left temporal brain tumor* *A 38-year-old male client in an alcohol withdrawal program* *A 42-year-old football player with a traumatic brain injury* Clients at risk for secondary seizures include those with a brain lesion from a tumor or trauma, and those who are experiencing a metabolic disorder, acute alcohol withdrawal, electrolyte disturbances, and high fever. Clients with a history of stroke, heart disease, and substance abuse are also at risk. Clients with multiple sclerosis or chronic obstructive pulmonary disease are not at risk for secondary seizures.

A nurse evaluates the results of diagnostic tests on a client's cerebrospinal fluid (CSF). Which fluid results alerts the nurse to possible viral meningitis (Select all that apply.) Clear Cloudy Increased protein level Normal glucose level Bacterial organisms present Increased white blood cells

*Clear* *Increased protein level* *Normal glucose level* In viral meningitis, CSF fluid is clear, protein levels are slightly increased, and glucose levels are normal. Viral meningitis does not cause cloudiness or increased turbidity of CSF. In bacterial meningitis, the presence of bacteria and white blood cells causes the fluid to be cloudy.

Multiple scelerosis

*Demyelinating* disease resulting in the destruction of CNS myelin and consequent disruption in the transmission of impulse ❈ Insidious onset ❈ Diagnosis determined by: presenting symptoms, increased white matter density seen in CT scan; Presence of plaques, CSF electrophoresis shows presence of oligoclonal bands

A nurse assesses a client who has encephalitis. Which manifestations should the nurse recognize as signs of increased intracranial pressure (ICP), a complication of encephalitis (Select all that apply.) Photophobia Dilated pupils Headache Widened pulse pressure Bradycardia

*Dilated pupils* *Widened pulse pressure* *Bradycardia* ICP is a complication of encephalitis. The nurse should monitor for signs of increased ICP, including dilated pupils, widened pulse pressure, bradycardia, irregular respirations, and less responsive pupils. Photophobia and headache are not related to increased ICP.

The nurse on the neurologic unit must provide care for four clients who require different levels of care. Which client should the nurse assist first with morning care?

A client who requires minimal bathing assistance and ambulates with a walker independently

A nurse assesses a client with Alzheimer's disease who is recently admitted to the hospital. Which psychosocial assessment should the nurse complete Assess religious and spiritual needs while in the hospital. Identify the client's ability to perform self-care activities. Evaluate the client's reaction to a change of environment. Ask the client about relationships with family members.

*Evaluate the client's reaction to a change of environment.* As Alzheimer's disease progresses, the client experiences changes in emotional and behavioral affect. The nurse should be alert to the client's reaction to a change in environment, such as being hospitalized, because the client may exhibit an exaggerated response, such as aggression, to the event. The other assessments should be completed but are not as important as assessing the client's reaction to environmental change.

A nurse plans care for a client with epilepsy who is admitted to the hospital. Which interventions should the nurse include in this client's plan of care (Select all that apply.) Have suction equipment at the bedside. Place a padded tongue blade at the bedside. Permit only clear oral fluids. Keep bed rails up at all times. Maintain the client on strict bedrest. Ensure that the client has IV access.

*Have suction equipment at the bedside.* *Keep bed rails up at all times.* *Ensure that the client has IV access.* Oxygen and suctioning equipment with an airway must be readily available. The bed rails should be up at all times while the client is in the bed to prevent injury from a fall if the client has a seizure. If the client does not have an IV access, insert a saline lock, especially for those clients who are at significant risk for generalized tonic-clonic seizures. The saline lock provides ready access if IV drug therapy must be given to stop the seizure. Padded tongue blades may pose a danger to the client during a seizure and should not be used. Dietary restrictions and strict bedrest are not interventions associated with epilepsy. The client should be encouraged to eat a well-balanced diet and ambulate while in the hospital.

A nurse plans care for a client with Parkinson disease. Which intervention should the nurse include in this client's plan of care Ambulate the client in the hallway twice a day. Ensure a fluid intake of at least 3 liters per day. Teach the client pursed-lip breathing techniques. Keep the head of the bed at 30 degrees or greater.

*Keep the head of the bed at 30 degrees or greater.* Elevation of the head of the bed will help *prevent aspiration.* The other options will not prevent aspiration, which is the greatest respiratory complication of Parkinson disease, nor do these interventions address any of the complications of Parkinson disease. Ambulation in the hallway is usually implemented to prevent venous thrombosis. Increased fluid intake flushes out toxins from the client's blood. Pursed-lip breathing increases exhalation of carbon dioxide.

A nurse assesses a client who is experiencing an absence seizure. For which clinical manifestations should the nurse assess (Select all that apply.) Intermittent rigidity Lip smacking Sudden loss of muscle tone Brief jerking of the extremities Picking at clothing Patting of the hand on the leg

*Lip smacking* *Picking at clothing* *Patting of the hand on the leg* Automatisms are characteristic of absence seizures. These behaviors consist of lip smacking, picking at clothing, and patting. Rigidity of muscles is associated with the tonic phase of a seizure, and jerking of the extremities is associated with the clonic phase of a seizure. Loss of muscle tone occurs with atonic seizures.

A nurse cares for a client who is experiencing status epilepticus. Which prescribed medication should the nurse prepare to administer Atenolol (Tenormin) Lorazepam (Ativan) Phenytoin (Dilantin) Lisinopril (Prinivil)

*Lorazepam (Ativan)* Initially, intravenous lorazepam is administered to stop motor movements. This is followed by the administration of phenytoin. Atenolol, a beta blocker, and lisinopril, an angiotensin-converting enzyme inhibitor, are not administered for seizure activity. These medications are typically administered for hypertension and heart failure.

A client with a T1 spinal cord injury arrives at the emergency department with a BP of 82/40, pulse 34, dry skin, and flaccid paralysis of the lower extremities. Which of the following conditions would most likely be suspected? 1. Neurogenic shock 2. Autonomic dysreflexia 3. Sepsis 4. Hypervolemia

1

A nurse obtains a health history on a client prior to administering prescribed sumatriptan succinate (Imitrex) for migraine headaches. Which condition should alert the nurse to hold the medication and contact the health care provider Bronchial asthma Prinzmetal's angina Diabetes mellitus Chronic kidney disease

*Prinzmetal's angina* Sumatriptan succinate effectively reduces pain and other associated symptoms of migraine headache by binding to serotonin receptors and triggering cranial vasoconstriction. Vasoconstrictive effects are not confined to the cranium and can cause coronary vasospasm in clients with Prinzmetal's angina. The other conditions would not affect the client's treatment.

A nurse is caring for a client with meningitis. Which laboratory values should the nurse monitor to identify potential complications of this disorder (Select all that apply.) Sodium level Liver enzymes Clotting factors Cardiac enzymes Creatinine level

*Sodium level* *Clotting factors* Inflammation associated with meningitis can stimulate the hypothalamus and result in excessive production of antidiuretic hormone. The nurse should monitor sodium levels for early identification of syndrome of inappropriate antidiuretic hormone. A systemic inflammatory response (SIR) can also occur with meningitis. A SIR can result in a coagulopathy that leads to disseminated intravascular coagulation. The nurse should monitor clotting factors to identify this complication. The other laboratory values are not specific to complications of meningitis.

A nurse prepares to provide perineal care to a client with meningococcal meningitis. Which personal protective equipment should the nurse wear (Select all that apply.) Particulate respirator Isolation gown Shoe covers Surgical mask Gloves

*Surgical mask* *Gloves* Meningeal meningitis is spread via saliva and droplets, and *Droplet Precautions* are necessary. Caregivers should wear a surgical mask when within 6 feet of the client and should continue to use Standard Precautions, including gloves. A particulate respirator, an isolation gown, and shoe covers are not necessary for Droplet Precautions.

A nurse assesses a client with a history of epilepsy who experiences stiffening of the muscles of the arms and legs, followed by an immediate loss of consciousness and jerking of all extremities. How should the nurse document this activity Atonic seizure Tonic-clonic seizure Myoclonic seizure Absence seizure

*Tonic-clonic seizure* Seizure activity that begins with stiffening of the arms and legs, followed by loss of consciousness and jerking of all extremities, is characteristic of a tonic-clonic seizure. An atonic seizure presents as a sudden loss of muscle tone followed by postictal confusion. A myoclonic seizure presents with a brief jerking or stiffening of extremities that may occur singly or in groups. Absence seizures present with automatisms, and the client is unaware of his or her environment.

A nurse witnesses a client begin to experience a tonic-clonic seizure and loss of consciousness. Which action should the nurse take Start fluids via a large-bore catheter. Turn the client's head to the side. Administer IV push diazepam. Prepare to intubate the client.

*Turn the client's head to the side.* The nurse should turn the client's head to the side to prevent aspiration and allow drainage of secretions. Anticonvulsants are administered on a routine basis if a seizure is sustained. If the seizure is sustained (status epilepticus), the client must be intubated and should be administered oxygen, 0.9% sodium chloride, and IV push lorazepam or diazepam.

A nurse assesses a client who has a history of migraines. Which clinical manifestation should the nurse identify as an early sign of a migraine with aura Vertigo Lethargy Visual disturbances Numbness of the tongue

*Visual disturbances* Early warning of impending migraine with aura usually consists of visual changes, flashing lights, or diplopia. The other manifestations are not associated with an impending migraine with aura.

A nurse is caring for a client who has undergone craniotomy with a supratentorial incision. The nurse should plan to place the client in which position postoperatively?

Head of bed elevated 30 to 45 degrees, head and neck midline

alges/o

-algesia, sensitivity to pain

A hospitalized child is to receive 75 mg of acetaminophen for fever control. How much will the nurse administer if the acetaminophen concentration is 40 mg per 0.4 ml? Record your answer using two decimal places.

0.75 Use the following equations: Dose on hand/Quantity on hand = Dose desired/X 40 mg/0.4 ml = 75 mg/X X = 0.75 ml

A 23-year-old client has been hit on the head with a baseball bat. The nurse notes clear fluid draining from his ears and nose. Which of the following nursing interventions should be done first? 1. Check the fluid for dextrose with a dipstick 2. Position the client flat in bed 3. Insert nasal and ear packing with sterile gauze 4. Suction the nose to maintain airway patency

1

A client is admitted with a spinal cord injury at the level of T12. He has limited movement of his upper extremities. Which of the following medications would be used to control edema of the spinal cord? 1. Methylprednisolone (Solu-Medrol) 2. Acetazolamide (Diamox) 3. Furosemide (Lasix) 4. Sodium bicarbonate

1

A client is at risk for increased ICP. Which of the following would be a priority for the nurse to monitor? 1. Unequal pupil size 2. Decreasing systolic blood pressure 3. Decreasing body temperature 4. Tachycardia

1

A client with a C6 spinal injury would most likely have which of the following symptoms? 1. Tetraplegia 2. Aphasia 3. Paraplegia 4. Hemiparesis

1

A male client is admitted with a cervical spine injury sustained during a diving accident. When planning this client's care, the nurse should assign highest priority to which nursing diagnosis? 1. Ineffective breathing pattern 2. Self-care deficit: Dressing/grooming 3. Disturbed sensory perception (tactile) 4. Impaired physical mobility

1

Which of the following interventions describes an appropriate bladder program for a client in rehabilitation for spinal cord injury? 1. Schedule intermittent catheterization every 2 to 4 hours 2. Insert an indwelling urinary catheter to straight drainage 3. Perform a straight catheterization every 8 hours while awake 4. Perform Crede's maneuver to the lower abdomen before the client voids.

1

The nurse on the rehabilitation unit is caring for the following clients. Which client should the nurse assess first after receiving the change-of-shift report? 1. The client with a C6 SCI who is complaining of dyspnea and has crackles in the lungs. 2. The client with an L4 SCI who is crying and very upset about being discharged home. 3. The client with an L2 SCI who is complaining of a headache and feeling very hot. 4. The client with a T4 SCI who is unable to move the lower extremities.

1 This client has signs/symptoms of a respiratory complication and should be assessed first.

The nurse in the neurointensive care unit is caring for a client with a new C6 SCI who is breathing independently. Which nursing interventions should be implemented? Select all that apply. 1. Monitor the pulse oximetry reading. 2. Provide pureed foods six (6) times a day. 3. Encourage coughing and deep breathing. 4. Assess for autonomic dysreflexia. 5. Administer intravenous corticosteroids.

1 3 5 Oxygen is administered initially to prevent hypoxemia, which can worsen the spinal cord injury; therefore, thenurse should determine how muchoxygen is reaching the periphery. Breathing exercises are supervised by the nurse to increase the strengthand endurance of inspiratory muscles, especially those of the diaphragm Corticosteroids are administered to decrease inflammation, which will decrease edema, and help preventedema from ascending up the spinalcord, causing breathing difficulties

A client has an exacerbation of multiple sclerosis accompanied by leg spasticity. The physician prescribes dantrolene sodium, 25 mg by mouth daily. How soon after administration of dantrolene will the nurse anticipate the health care provider to prescribe an appointment to assess for a significant reduction in spasticity?

1 to 2 weeks

The nurse is caring for a client with a T5 complete spinal cord injury. Upon assessment, the nurse notes flushed skin, diaphoresis above the T5, and a blood pressure of 162/96. The client reports a severe, pounding headache. Which of the following nursing interventions would be appropriate for this client? Select all that apply. 1. Loosen constrictive clothing 2. Assess for bladder distention and bowel impaction 3. Elevate the HOB to 90 degrees 4. Use a fan to reduce diaphoresis 5. Administer antihypertensive medication

1,2,3,5

A nurse is positioning the client with increased intracranial pressure (ICP). Which position should the nurse avoid?

Head turned to the side

When evaluating an ABG from a client with a subdural hematoma, the nurse notes the PaCO2 is 30 mm Hg. Which of the following responses best describes this result? 1.Appropriate; lowering carbon dioxide (CO2) reduces intracranial pressure (ICP). 2.Emergent; the client is poorly oxygenated. 3. Normal 4. Significant; the client has alveolar hypoventilation.

1. A normal PaCO2 value is 35 to 45 mm Hg. CO2 has vasodilating properties; therefore, lowering PaCO2 through hyperventilation will lower ICP caused by dilated cerebral vessels. Oxygenation is evaluated through PaO2 and oxygen saturation. Alveolar hypoventilation would be reflected in an increased PaCO2.

In assessing a client with a T12 SCI, which clinical manifestations would the nurseexpect to find to support the diagnosis of spinal shock? 1. No reflex activity below the waist. 2. Inability to move upper extremities. 3. Complaints of a pounding headache. 4. Hypotension and bradycardia.

1. Spinal shock associated with SC Irepresents a sudden depression of reflex activity below the level of the injury. T12 is just above the waist;therefore, no reflex activity below the waist would be expected

What're some NURSING interventions for a pt with Chronic Kidney Disease?

1. Monitor fluid status 2. Encourage nutrition by decreasing N/V, stomatitis 3. Encourage a low protein, low potassium, low sodium diet 4. Manage constipation 5. Balance rest and activity 6. Encourage skin care and emphasize moisturizing

A nurse on the neurologic unit evaluates her client care assignment after receiving the shift report. Which client in her assignment should she attend to first?

A client who sustained a fall on the previous shift and is attempting to get out of bed

A client who's receiving phenytoin (Dilantin) to control seizures is admitted to the health care facility for observation. The physician orders measurement of the client's serum phenytoin level. Which serum phenytoin level is therapeutic?

10 to 20 mcg/ml

10. The client is having a lumbar puncture performed. The nurse would plan to place the client in which position for the procedure? a. Side-lying, with legs pulled up and head bent down onto the chest b. Side-lying, with a pillow under the hip c. Prone, in a slight Trendelenburg's position d. Prone, with a pillow under the abdomen.

10. A. The client undergoing lumbar puncture is positioned lying on the side, with the legs pulled up to the abdomen, and with the head bent down onto the chest. This position helps to open the spaces between the vertebrae.

11. A nurse is assisting with caloric testing of the oculovestibular reflex of an unconscious client. Cold water is injected into the left auditory canal. The client exhibits eye conjugate movements toward the left followed by a rapid nystagmus toward the right. The nurse understands that this indicates the client has: a. A cerebral lesion b. A temporal lesion c. An intact brainstem d. Brain death

11. C. Caloric testing provides information about differentiating between cerebellar and brainstem lesions. After determining patency of the ear canal, cold or warm water is injected in the auditory canal. A normal response that indicates intact function of cranial nerves III, IV, and VIII is conjugate eye movements toward the side being irrigated, followed by rapid nystagmus to the opposite side. Absent or dysconjugate eye movements indicate brainstem damage.

Cranial nerves

12 paired nerves composed pf cell bodies with fibers that exit from cranial cavity

It is imperative to reverse spinal shock as quickly as possible, Permanent paralysis can occur if a spinal cord is compressed for _______________

12-24 hours

12. The nurse is assessing the motor function of an unconscious male client. The nurse would plan to use which plan to use which of the following to test the client's peripheral response to pain? A. Sternal rub B. Nail bed pressure C. Pressure on the orbital rim D. Squeezing of the sternocleidomastoid muscle

12. Answer: B. Nail bed pressure Motor testing in the unconscious client can be done only by testing response to painful stimuli. Nail bed pressure tests a basic peripheral response. Options A, C, and D: Cerebral responses to pain are tested using

12. The nurse is caring for the client with increased intracranial pressure. The nurse would note which of the following trends in vital signs if the ICP is rising? a. Increasing temperature, increasing pulse, increasing respirations, decreasing blood pressure. b. Increasing temperature, decreasing pulse, decreasing respirations, increasing blood pressure. c. Decreasing temperature, decreasing pulse, increasing respirations, decreasing blood pressure. d. Decreasing temperature, increasing pulse, decreasing respirations, increasing blood pressure.

12. B. A change in vital signs may be a late sign of increased intracranial pressure. Trends include increasing temperature and blood pressure and decreasing pulse and respirations. Respiratory irregularities also may arise.

13. The nurse is evaluating the status of a client who had a craniotomy 3 days ago. The nurse would suspect the client is developing meningitis as a complication of surgery if the client exhibits: a. A positive Brudzinski's sign b. A negative Kernig's sign c. Absence of nuchal rigidity d. A Glascow Coma Scale score of 15

13. A. Signs of meningeal irritation compatible with meningitis include nuchal rigidity, positive Brudzinski's sign, and positive Kernig's sign. Nuchal rigidity is characterized by a stiff neck and soreness, which is especially noticeable when the neck is fixed. Kernig's sign is positive when the client feels pain and spasm of the hamstring muscles when the knee and thigh are extended from a flexed-right angle position. Brudzinski's sign is positive when the client flexes the hips and knees in response to the nurse gently flexing the head and neck onto the chest. A Glascow Coma Scale of 15 is a perfect score and indicates the client is awake and alert with no neurological deficits.

14. A client is arousing from a coma and keeps saying, "Just stop the pain." The nurse responds based on the knowledge that the human body typically and automatically responds to pain first with attempts to: a. Tolerate the pain b. Decrease the perception of pain c. Escape the source of pain d. Divert attention from the source of pain.

14. C. The client's innate responses to pain are directed initially toward escaping from the source of pain. Variations in individuals' tolerance and perception of pain are apparent only in conscious clients, and only conscious clients are able to employ distraction to help relieve pain.

A client with a tentative diagnosis of myasthenia gravis is admitted for a diagnostic workup. When reviewing the client's chart, the nurse expects to find which documentation that confirms the client has Myasthenia gravis?

A positive edrophonium (Tensilon) test

15. During the acute stage of meningitis, a 3-year-old child is restless and irritable. Which of the following would be most appropriate to institute? a. Limiting conversation with the child b. Keeping extraneous noise to a minimum c. Allowing the child to play in the bathtub d. Performing treatments quickly

15. B. A child in the acute stage of meningitis is irritable and hypersensitive to loud noise and light. Therefore, extraneous noise should be minimized and bright lights avoided as much as possible. There is no need to limit conversations with the child. However, the nurse should speak in a calm, gentle, reassuring voice. The child needs gentle and calm bathing. Because of the acuteness of the infection, sponge baths would be more appropriate than tub baths. Although treatments need to be completed as quickly as possible to prevent overstressing the child, any treatments should be performed carefully and at a pace that avoids sudden movements to prevent startling the child and subsequently increasing intracranial pressure.

17. When interviewing the parents of a 2-year-old child, a history of which of the following illnesses would lead the nurse to suspect pneumococcal meningitis? a. Bladder infection b. Middle ear infection c. Fractured clavicle d. Septic arthritis

17. B. Organisms that cause bacterial meningitis, such as pneumococci or meningococci, are commonly spread in the body by vascular dissemination from a middle ear infection. The meningitis may also be a direct extension from the paranasal and mastoid sinuses. The causative organism is a pneumonococcus. A chronically draining ear is frequently also found.

19. A lumbar puncture is performed on a child suspected of having bacterial meningitis. CSF is obtained for analysis. A nurse reviews the results of the CSF analysis and determines that which of the following results would verify the diagnosis? a. Cloudy CSF, decreased protein, and decreased glucose b. Cloudy CSF, elevated protein, and decreased glucose c. Clear CSF, elevated protein, and decreased glucose d. Clear CSF, decreased pressure, and elevated protein

19. B. A diagnosis of meningitis is made by testing CSF obtained by lumbar puncture. In the case of bacterial meningitis, findings usually include an elevated pressure, turbid or cloudy CSF, elevated leukocytes, elevated protein, and decreased glucose levels.

A 30-year-old was admitted to the progressive care unit with a C5 fracture from a motorcycle accident. Which of the following assessments would take priority? 1. Neurological deficit 2. Pulse ox readings 3. Bladder distension 4. The client's feelings about the injury

2

A client arrives at the ER after slipping on a patch of ice and hitting her head. A CT scan of the head shows a collection of blood between the skull and dura mater. Which type of head injury does this finding suggest? 1. Subdural hematoma 2. Epidural hematoma 3. Subarachnoid hemorrhage 4. Contusion

2

Ultrasonography

A probe is placed against the cornea to measure for lens implant after cataract removal and to diagnose retinal detachment. *Nursing Implications* Explain to the patient that cornea is anesthetized before the procedure is done.

A client comes into the ER after hitting his head in an MVA. He's alert and oriented. Which of the following nursing interventions should be done first? 1. Assess full ROM to determine extent of injuries 2. Immobilize the client's head and neck 3. Call for an immediate chest x-ray 4. Open the airway with the head-tilt chin-lift maneuver

2

A client is admitted to the ER for head trauma is diagnosed with an epidural hematoma. The underlying cause of epidural hematoma is usually related to which of the following conditions? 1. Rupture of the carotid artery 2. Laceration of the middle meningeal artery 3. Thromboembolism from a carotid artery 4. Venous bleeding from the arachnoid space

2

A client with a spinal cord injury suddenly experiences an episode of autonomic dysreflexia. After checking the client's vital signs, list in order of priority, the nurse's actions (Number 1 being the first priority and number 5 being the last priority). *A*. Check for bladder distention *B*. Raise the head of the bed *C*. Contact the physician *D*. Loosen tight clothing on the client *E*. Administer an antihypertensive medication. 1. A, B, C, D, E 2. B, D, A, C, E. 3. C, A, D, E, B 4. C, D, A, B, E

2

A female client who was trapped inside a car for hours after a head-on collision is rushed to the emergency department with multiple injuries. During the neurologic examination, the client responds to painful stimuli with decerebrate posturing. This finding indicates damage to which part of the brain? 1. Diencephalon 2. Midbrain 3. Cortex 4. Medulla

2

An 18-year-old client is admitted with a closed head injury sustained in a MVA. His intracranial pressure (ICP) shows an upward trend. Which intervention should the nurse perform first? 1. Administer 100 mg of pentobarbital IV as ordered. 2. Reposition the client to avoid neck flexion 3. Increase the ventilator's respiratory rate to 20 breaths/minute 4. Administer 1 g Mannitol IV as ordered

2

The nurse is caring for a client who suffered a spinal cord injury 48 hours ago. The nurse monitors for GI complications by assessing for: 1. A history of diarrhea 2. Hematest positive nasogastric tube drainage 3. Hyperactive bowel sounds 4. A flattened abdomen

2

The nurse is discussing the purpose of an electroencephalogram (EEG) with the family of a client with massive cerebral hemorrhage and loss of consciousness. It would be most accurate for the nurse to tell family members that the test measures which of the following conditions? 1. Extent of intracranial bleeding 2. Activity of the brain 3. Percent of functional brain tissue 4. Sites of brain injury

2

When evaluating an ABG from a client with a subdural hematoma, the nurse notes the PaCO2 is 30 mm Hg. Which of the following responses best describes this result? 1. Emergent; the client is poorly oxygenated. 2. Appropriate; lowering carbon dioxide (CO2) reduces intracranial pressure (ICP). 3. Normal 4. Significant; the client has alveolar hypoventilation.

2

Which of the following describes decerebrate posturing? 1. Supination of arms, dorsiflexion of feet 2. Back arched; rigid extension of all four extremities. 3. Back hunched over, rigid flexion of all four extremities with supination of arms and plantar flexion of the feet - Given 4. Internal rotation and adduction of arms with flexion of elbows, wrists, and fingers

2

Which of the following respiratory patterns indicate increasing ICP in the brain stem? 1. Rapid, shallow respirations 2. Slow, irregular respirations 3. Asymmetric chest expansion 4. Nasal flaring

2

The rehabilitation nurse caring for the client with an L1 SCI is developing the nursing care plan. Which intervention should the nurse implement? 1. Keep oxygen via nasal cannula on at all times. 2. Administer low-dose subcutaneous anticoagulants. 3. Perform active lower extremity ROM exercises. 4. Refer to a speech therapist for ventilator-assisted speech.

2 Deep vein thrombosis (DVT) is a potential complication of immobility,which can occur because the clientcannot move the lower extremities as a result of the L1 SCI. Low-dose anticoagulation therapy (Lovenox)helps prevent blood from coagulating,thereby preventing DVTs.

The 34-year-old male client with an SCI is sharing with the nurse that he is worried about finding employment after being discharged from the rehabilitation unit.Which intervention should the nurse implement? 1. Refer the client to the American Spinal Cord Injury Association (ASIA). 2. Refer the client to the state rehabilitation commission. 3. Ask the social worker about applying for disability. 4. Suggest that the client talk with his significant other about this concern.

2 The rehabilitation commission of each state will help evaluate and determine if the client can receive training or education for another occupation after injury.

Any client on bed rest or who is immobilized must have range of motion exercises often and very frequent position changes. Do not leave the client in any one position for longer than _________. Any position that decreases venous return is dangerous, such as __________________

2 hours sitting with dependent extremities for long periods.

When self-administering atropine (Atropisol), the nurse should instruct the client to wait how long between instilling the first drop and instilling the second drop?

2 to 3 minutes

The client with Parkinson's disease is being taught about taking carbidopalevodopa (Sinemet). What teaching points should the nurse emphasize? Select all that apply. a. Report the "on-off" effect b. Change position slowly c. Report blurred vision or a rash d. Avoid taking medication with meals e. Monitor the color of urine f. Increase fluid intake

2, 3, 6, 1, 5, 4 Rationale: Loosen the gown and turn client on his side. If on floor, place pillow under head and clear area of objects that could cause injury. Provide supplemental oxygen. Pad the side rails as soon as possible.

2. A client with a subdural hematoma becomes restless and confused, with dilation of the ipsilateral pupil. The physician orders mannitol for which of the following reasons? a. To reduce intraocular pressure b. To prevent acute tubular necrosis c. To promote osmotic diuresis to decrease ICP d. To draw water into the vascular system to increase blood pressure

2. C. Mannitol promotes osmotic diuresis by increasing the pressure gradient, drawing fluid from intracellular to intravascular spaces. Although mannitol is used for all the reasons described, the reduction of ICP in this client is a concern.

20. A nurse is planning care for a child with acute bacterial meningitis. Based on the mode of transmission of this infection, which of the following would be included in the plan of care? a. No precautions are required as long as antibiotics have been started b. Maintain enteric precautions c. Maintain respiratory isolation precautions for at least 24 hours after the initiation of antibiotics d. Maintain neutropenic precautions

20. C. A major priority of nursing care for a child suspected of having meningitis is to administer the prescribed antibiotic as soon as it is ordered. The child is also placed on respiratory isolation for at least 24 hours while culture results are obtained and the antibiotic is having an effect.

The pathophysiology of an ischemic stroke involves the ischemic cascade, which includes the following steps: 1. Change in pH 2. Blood flow decreases 3. A switch to anaerobic respiration <wbr /> 4. Membrane pumps fail 5. Cells cease to function 6. Lactic acid is generated Put these steps in order in which they occur.

236145

how long must neurological defecits persist for an "episode" to be classified as a stroke?

24 hours

The nurse has given the male client with Bell's palsy instructions on preserving muscle tone in the face and preventing denervation. The nurse determines that the client needs additional information if the client states that he or she will: a. Exposure to cold and drafts b. Massage the face with a gentle upward motion c. Perform facial exercises d. Wrinkle the forehead, blow out the cheeks, and whistle

26. Answer A. Prevention of muscle atrophy with Bell's palsy is accomplished with facial massage, facial exercises, and electrical stimulation of the nerves. Exposure to cold or drafts is avoided. Local application of heat to the face may improve blood flow and provide comfort. 27. Answer D. Guillain-Barré syndrome is a clinical syndrome of unknown origin that involves cranial and peripheral nerves. Many clients report a history of respiratory or gastrointestinal infection in the 1 to 4 weeks before the onset of neurological deficits. Occasionally, the syndrome can be triggered by vaccination or surgery. 28. Answer C. The client with Guillain-Barré syndrome experiences fear and anxiety from the ascending paralysis and sudden onset of the disorder. The nurse can alleviate these fears by providing accurate information about the client's condition, giving expert care and positive feedback to the client, and encouraging relaxation and distraction. The family can become involved with selected care activities and provide diversion for the client as well. 29. Answer D. Cranial nerve II is the optic nerve, which governs vision. The nurse can provide safety for the visually impaired client by clearing the path of obstacles when ambulating. Testing the shower water temperature would be useful if there were an impairment of peripheral nerves. Speaking loudly may help overcome a deficit of cranial nerve VIII (vestibulocochlear). Cranial nerve VII (facial) and IX (glossopharyngeal) control taste from the anterior two thirds and posterior third of the tongue, respectively. 30. Answer B. The limbic system is responsible for feelings (affect) and emotions. Calculation ability and knowledge of current events relates to function of the frontal lobe. The cerebral hemispheres, with specific regional functions, control orientation. Recall of recent events is controlled by the hippocampus.

A client has a cervical spine injury at the level of C5. Which of the following conditions would the nurse anticipate during the acute phase? 1. Decerebrate posturing 2. Absent corneal reflex 3. The need for mechanical ventilation 4. Movement of only the right or left half of the body

3

A client receiving vent-assisted mode ventilation begins to experience cluster breathing after recent intracranial occipital bleeding. Which action would be most appropriate? 1. Check deep tendon reflexes to determine the best motor response 2. Count the rate to be sure the ventilations are deep enough to be sufficient 3. Call the physician while another nurse checks the vital signs and ascertains the patient's Glasgow Coma score. 4. Call the physician to adjust the ventilator settings.

3

A client with C7 quadriplegia is flushed and anxious and complains of a pounding headache. Which of the following symptoms would also be anticipated? 1. Respiratory depression 2. Symptoms of shock 3. Hypertension and bradycardia 4. Decreased urine output or oliguria

3

A client with a subarachnoid hemorrhage is prescribed a 1,000-mg loading dose of Dilantin IV. Which consideration is most important when administering this dose? 1. Therapeutic drug levels should be maintained between 20 to 30 mg/ml. 2. Dilantin should be mixed in dextrose in water before administration. 3. Rapid dilantin administration can cause cardiac arrhythmias. 4. Dilantin should be administered through an IV catheter in the client's hand.

3

A client with head trauma develops a urine output of 300 ml/hr, dry skin, and dry mucous membranes. Which of the following nursing interventions is the most appropriate to perform initially? 1. Anticipate treatment for renal failure 2. Slow down the IV fluids and notify the physician 3. Evaluate urine specific gravity 4. Provide emollients to the skin to prevent breakdown

3

A female client admitted to an acute care facility after a car accident develops signs and symptoms of increased intracranial pressure (ICP). The client is intubated and placed on mechanical ventilation to help reduce ICP. To prevent a further rise in ICP caused by suctioning, the nurse anticipates administering which drug endotracheally before suctioning? 1. furosemide (Lasix) 2. phenytoin (Dilantin) 3. lidocaine (Xylocaine) 4. mannitol (Osmitrol)

3

A female client who was found unconscious at home is brought to the hospital by a rescue squad. In the intensive care unit, the nurse checks the client's oculocephalic (doll's eye) response by: 1. shining a bright light into the pupil. 2. introducing ice water into the external auditory canal. 3. turning the client's head suddenly while holding the eyelids open. 4. touching the cornea with a wisp of cotton.

3

A nurse assesses a client who has episodes of autonomic dysreflexia. Which of the following conditions can cause autonomic dysreflexia? 1. Lumbar spinal cord injury 2. Headache 3. Noxious stimuli 4. Neurogenic shock

3

After falling 20', a 36-year-old man sustains a C6 fracture with spinal cord transaction. Which other findings should the nurse expect? 1. Loss of bowel and bladder control 2. Paraplegia with intercostal muscle loss 3. Quadriplegia with gross arm movement and diaphragmatic breathing 4. Quadriplegia and loss of respiratory function

3

After striking his head on a tree while falling from a ladder, a young man age 18 is admitted to the emergency department. He's unconscious and his pupils are nonreactive. Which intervention would be the most dangerous for the client? 1. Give him a barbiturate. 2. Place him on mechanical ventilation. 3. Perform a lumbar puncture. 4. Elevate the head of his bed.

3

Patients with suspected stroke must receive immediate treatment within __________ hours to preserve as much brain function as possible.

3

The client with a head injury has been urinating copious amounts of dilute urine through the Foley catheter. The client's urine output for the previous shift was 3000 ml. The nurse implements a new physician order to administer: 1. Dexamethasone (Decadron) 2. Mannitol (Osmitrol) 3. Desmopressin (DDAVP, stimate) 4. Ethacrynic acid (Edecrin)

3

The nurse is planning care for the client in spinal shock. Which of the following actions would be least helpful in minimizing the effects of vasodilation below the level of the injury? 1. Applying Teds or compression stockings. 2. Monitoring vital signs before and during position changes 3. Moving the client quickly as one unit 4. Using vasopressor medications as prescribed

3

The nurse arrives at the site of a one-car motor-vehicle accident and stops to render aid. The driver of the car is unconscious. After stabilizing the client's cervical spine,which action should the nurse take next? 1. Carefully remove the driver from the car. 2. Assess the client's pupils for reaction. 3. Assess the client's airway. 4. Attempt to wake the client up by shaking him.

3 The nurse must maintain a patent air-way. Airway is the first step in resuscitation. 1. The nurse should stabilize the client's neck prior to removal from the car. 2. The nurse must stabilize the client's neck before doing any further assessment. Most nurses don't carry penlights, and the client's pupil reaction can be determined after stabilization. 4. Shaking the patient could cause further damage, possibly l

Thrombolytic therapy for the treatment of an ischemic stroke should be initiated within how many hours of the onset of symptoms to obtain the best functional outcome?

3 hours

3. A client with subdural hematoma was given mannitol to decrease intracranial pressure (ICP). Which of the following results would best show the mannitol was effective? a. Urine output increases b. Pupils are 8 mm and nonreactive c. Systolic blood pressure remains at 150 mm Hg d. BUN and creatinine levels return to normal

3. A. Mannitol promotes osmotic diuresis by increasing the pressure gradient in the renal tubes. Fixed and dilated pupils are symptoms of increased ICP or cranial nerve damage. No information is given about abnormal BUN and creatinine levels or that mannitol is being given for renal dysfunction or blood pressure maintenance.

For a client with a head injury whose neck has been stabilized, the preferred bed position is:

30-degree head elevation.

The nurse receives a physician's order to administer 1,000 ml of normal saline solution I.V. over 8 hours to a client who recently had a stroke. What should the drip rate be if the drop factor of the tubing is 15 gtt/ml?

31

The nurse receives a health care provider's order to administer 1,000 mL of intravenous (IV) normal saline solution over 8 hours to a client who recently had a stroke. What should the drip rate be if the drop factor of the tubing is 15 gtt/mL? Record your answer using a whole number.

31 Explanation: The drip rate is calculated using the following formula: Volume of infusion (in milliliters)/Time of infusion (in minutes 60 MINUTES X 8 HRS =480) × drip factor (in drops/milliliter) = drops/minute. Therefore, 1,000 mL/480 minutes × 15 drops/mL = 31 gtt/minute.

A 20-year-old client who fell approximately 30' is unresponsive and breathless. A cervical spine injury is suspected. How should the first-responder open the client's airway for rescue breathing? 1. By inserting a nasopharyngeal airway 2. By inserting a oropharyngeal airway 3. By performing the head-tilt, chin-lift maneuver 4. By performing a jaw-thrust maneuver

4

A 22-year-old client with quadriplegia is apprehensive and flushed, with a blood pressure of 210/100 and a heart rate of 50 bpm. Which of the following nursing interventions should be done first? 1. Assess patency of the indwelling urinary catheter - Given 2. Place the client flat in bed 3. Give one SL nitroglycerin tablet 4. Raise the head of the bed immediately to 90 degrees

4

A client has signs of increased ICP. Which of the following is an early indicator of deterioration in the client's condition? 1. Widening pulse pressure 2. Dilated, fixed pupil 3. Decrease in the pulse rate 4. Decrease in LOC

4

A client with a spinal cord injury is prone to experiencing autonomic dysreflexia. The nurse would avoid which of the following measures to minimize the risk of recurrence? 1. Strict adherence to a bowel retraining program 2. Preventing unnecessary pressure on the lower limbs 3. Keeping the linen wrinkle-free under the client 4. Limiting bladder catheterization to once every 12 hours

4

An 18-year-old client was hit in the head with a baseball during practice. When discharging him to the care of his mother, the nurse gives which of the following instructions? 1. "Watch him for keyhole pupil the next 24 hours." 2. "Notify the physician immediately if he has a headache." 3. "Expect profuse vomiting for 24 hours after the injury." 4. "Wake him every hour and assess his orientation to person, time, and place."

4

During an episode of autonomic dysreflexia in which the client becomes hypertensive, the nurse should perform which of the following interventions? 1. Elevate the client's legs 2. Put the client in the Trendelenburg's position 3. Put the client flat in bed 4. Put the client in the high-Fowler's position

4

The nurse is performing a mental status examination on a male client diagnosed with subdural hematoma. This test assesses which of the following? 1. Cerebellar function 2. Intellectual function 3. Sensory function 4. Cerebral function

4

Which of the following conditions indicates that spinal shock is resolving in a client with C7 quadriplegia? 1. Urinary continence 2. Spontaneous respirations 3. Absence of pain sensation in chest 4. Spasticity

4

The intensive care nurse is caring for a client with a T1 SCI. When the nurse elevates the head of the bed 30 degrees, the client complains of light headedness and dizziness. The client's vital signs are T 99.2 ̊F, P 98, R 24, and BP 84/40. Which action should the nurse implement? 1. Notify the health-care provider ASAP. 2. Calm the client down by talking therapeutically. 3. Increase the IV rate by 50 mL/hour. 4. Lower the head of the bed immediately.

4 For the first two (2) weeks after an SCI above T7, the blood pressure tends to be unstable and low; slight elevations of the head of the bed can cause profound hypotension; therefore, the nurse should lower the head of the bed immediately.

dysrthria

48-57% of stroke pts show this is a category of motor speech disorders caused by lesions in parts of the central of peripheral nervous system that mediated speech production... ...may also affect respiration, articulation, phonation, resonance....and maybe sensory feedback .......volitional and auto actions like chewing and swallowing and jaw movement are impaired resulting in slurred speech lesion can be in primary motor cortex frontal lobe, primary sensroy cortex parietal lobe, or cerebellum

A patient is admitted via ambulance to the emergency room of a stroke center at 1:30 p.m. with symptoms that the patient said began at 1:00 p.m. Within 1 hour, an ischemic stroke had been confirmed and the doctor ordered tPA. The nurse knows to give this drug no later than what time?

4:00 p.m.

A patient is admitted via ambulance to the emergency room of a stroke center at 1:30 p.m. with symptoms that the patient said began at 1:00 p.m. Within 1 hour, an ischemic stroke had been confirmed and the doctor ordered tPA. The nurse knows to give this drug no later than what time?

4:00PM

Positive emission tomography (PET)

A radioactive agent is injected, and computed tomography measures metabolic activity of the brain. Used to detect brain cancer, Alzheimer disease, epilepsy, and Parkinson disease. *Nursing Implications* Withhold food and fluids 4 hours before the exam. Explain that an IV line will be inserted. Post-test, encourage oral fluids to aid removal of radioisotope.

5. Which of the following symptoms may occur with a phenytoin level of 32 mg/dl? a. Ataxia and confusion b. Sodium depletion c. Tonic-clonic seizure d. Urinary incontinence

5. A. A therapeutic phenytoin level is 10 to 20 mg/dl. A level of 32 mg/dl indicates toxicity. Symptoms of toxicity include confusion and ataxia. Phenytoin doesn't cause hyponatremia, seizure, or urinary incontinence. Incontinence may occur during or after a seizure.

dysphagia

51% of stroke patients inability to swallow or difficulty in swallowing can be seen in hemispheric, brainstem, or pseduobulbar/suprabulbar palsy resultant of CN involvement CN V trigeminal/facial sensory, CN VII facial motor, CN IX glossopharyngeal, CN X vagus, CN XI spinal accessory big risk-aspiration-food, liquid, saliva, or gastric reflux into the airway

6. Which of the following signs and symptoms of increased ICP after head trauma would appear first? a. Bradycardia b. Large amounts of very dilute urine c. Restlessness and confusion d. Widened pulse pressure

6. C. The earliest symptom of elevated ICP is a change in mental status. Bradycardia, widened pulse pressure, and bradypnea occur later. The client may void large amounts of very dilute urine if there's damage to the posterior pituitary.

7. Problems with memory and learning would relate to which of the following lobes? a. Frontal b. Occipital c. Parietal d. Temporal

7. D. The temporal lobe functions to regulate memory and learning problems because of the integration of the hippocampus. The frontal lobe primarily functions to regulate thinking, planning, and judgment. The occipital lobe functions regulate vision. The parietal lobe primarily functions with sensory function.

The nurse is preparing to administer carbamazepine (Tegretol) oral suspension, 150 mg by mouth. The pharmacy has dispensed carbamazepine suspension 100 mg/5 ml. How many milliliters of carbamazepine should the nurse administer to the client?

7.5

A client with suspected Guillain-Barré syndrome has a lumbar puncture performed. The cerebrospinal fluid (CSF) is analyzed for protein. The nurse reviews the protein values and notes that the value that supports the diagnosis of Guillain-Barré syndrome is:

75 mg/dL

8. While cooking, your client couldn't feel the temperature of a hot oven. Which lobe could be dysfunctional? a. Frontal b. Occipital c. Parietal d. Temporal

8. C. The parietal lobe regulates sensory function, which would include the ability to sense hot or cold objects. The frontal lobe regulates thinking, planning, and judgment, and the occipital lobe is primarily responsible for vision function. The temporal lobe regulates memory.

9. The nurse is assessing the motor function of an unconscious client. The nurse would plan to use which of the following to test the client's peripheral response to pain? a. Sternal rub b. Pressure on the orbital rim c. Squeezing the sternocleidomastoid muscle d. Nail bed pressure

9. D. Motor testing on the unconscious client can be done only by testing response to painful stimuli. Nailbed pressure tests a basic peripheral response. Cerebral responses to pain are testing using sternal rub, placing upward pressure on the orbital rim, or squeezing the clavicle or sternocleidomastoid muscle.

The nurse is observing a client with cerebral edema for evidence of increasing intracranial pressure. She monitors his blood pressure for signs of widening pulse pressure. His current blood pressure is 170/80 mm Hg. What is the client's pulse pressure?

90

After falling 20', a 36-year-old man sustains a C6 fracture with spinal cord transaction. Which other findings should the nurse expect? A Quadriplegia with gross arm movement and diaphragmatic breathing B Quadriplegia and loss of respiratory function C Paraplegia with intercostal muscle loss D Loss of bowel and bladder control

A A client with a spinal cord injury at levels C5 to C6 has quadriplegia with gross arm movement and diaphragmatic breathing. Injury levels C1 to C4 leads to quadriplegia with total loss of respiratory function. Paraplegia with intercostal muscle loss occurs with injuries at T1 to L2. Injuries below L2 cause paraplegia and loss of bowel and bladder control.

Which client would the nurse identify as being most at risk for experiencing a CVA? A A 55-year-old African American male. B An 84-year-old Japanese female. C A 67-year-old Caucasian male. D A 39-year-old pregnant female

A A 55-year-old African American male. Africana Americans have twice the rate of CVA's as Caucasians; males are more likely to have strokes than females except in advanced years. Oriental's have a lower risk, possibly due to their high omega-3 fatty acids. Pregnancy is a minimal risk factor for CVA.

The client diagnosed with atrial fibrillation has experienced a transient ischemic attack (TIA). Which medication would the nurse anticipate being ordered for the client on discharge? A An oral anticoagulant medication. B A beta-blocker medication. C An anti-hyperuricemic medication. D A thrombolytic medication.

A An oral anticoagulant medication. Thrombi form secondary to atrial fibrillation, therefore, an anticoagulant would be anticipated to prevent thrombi formation; and oral (warfarin [Coumadin]) at discharge verses intravenous. Beta blockers slow the heart rate and lower the blood pressure. Anti-hyperuricemic medication is given to clients with gout. Thrombolytic medication might have been given at initial presentation but would not be a drug prescribed at discharge.

An 18-year-old client is admitted with a closed head injury sustained in a MVA. His intracranial pressure (ICP) shows an upward trend. Which intervention should the nurse perform first? A Reposition the client to avoid neck flexion B Administer 1 g Mannitol IV as ordered C Increase the ventilator's respiratory rate to 20 breaths/minute D Administer 100 mg of pentobarbital IV as ordered.

A Reposition the client to avoid neck flexion

Regular oral hygiene is an essential intervention for the client who has had a stroke. Which of the following nursing measures is inappropriate when providing oral hygiene? A Placing the client on the back with a small pillow under the head. B Keeping portable suctioning equipment at the bedside. C Opening the client's mouth with a padded tongue blade. D Cleaning the client's mouth and teeth with a toothbrush.

A Placing the client on the back with a small pillow under the head. A helpless client should be positioned on the side, not on the back. This lateral position helps secretions escape from the throat and mouth, minimizing the risk of aspiration. It may be necessary to suction, so having suction equipment at the bedside is necessary. Padded tongue blades are safe to use. A toothbrush is appropriate to use.

When assessing a patient's needs for psychologic support after the patient has been diagnosed with stage I cancer of the colon, which question by the nurse will provide the most information? A "Can you tell me what has been helpful to you in the past when coping with stressful events?" B "How long ago were you diagnosed with this cancer?" C "Are you familiar with the stages of emotional adjustment to a diagnosis like cancer of the colon?" D "How do you feel about having a possibly terminal illness?"

A "Can you tell me what has been helpful to you in the past when coping with stressful events?"

For a female client with newly diagnosed cancer, the nurse formulates a nursing diagnosis of Anxiety related to the threat of death secondary to cancer diagnosis. Which expected outcome would be appropriate for this client? A "Client verbalizes feelings of anxiety." B "Client doesn't guess at prognosis." C "Client uses any effective method to reduce tension." D "Client stops seeking information."

A "Client verbalizes feelings of anxiety."

April is diagnosed with systemic lupus erythematosus. Which instruction would be included in the teaching plan for the client? A "Wear large-brimmed hats when exposed to the sun." B "Use tanning beds instead of sunbathing outside." C "Remove all rugs, curtains, and dust-collecting items in home." D "Carry injectable epinephrine at all times in case of exacerbation."

A "Wear large-brimmed hats when exposed to the sun."

A nurse is assisting with a community screening for people at high risk for stroke. To which of the following clients would the nurse pay most attention?

A 60-year-old African-American man

The nurse is caring for a client with a T5 complete spinal cord injury. Upon assessment, the nurse notes flushed skin, diaphoresis above the T5, and a blood pressure of 162/96. The client reports a severe, pounding headache. Which of the following nursing interventions would be appropriate for this client? Select all that apply. A Elevate the HOB to 90 degrees B Loosen constrictive clothing C Use a fan to reduce diaphoresis D Assess for bladder distention and bowel impaction E Administer antihypertensive medication

A B D E The client has signs and symptoms of autonomic dysreflexia. The potentially life-threatening condition is caused by an uninhibited response from the sympathetic nervous system resulting from a lack of control over the autonomic nervous system. The nurse should immediately elevate the HOB to 90 degrees and place extremities dependently to decrease venous return to the heart and increase venous return from the brain. Because tactile stimuli can trigger autonomic dysreflexia, any constrictive clothing should be loosened. The nurse should also assess for distended bladder and bowel impaction, which may trigger autonomic dysreflexia, and correct any problems. Elevated blood pressure is the most life-threatening complication of autonomic dysreflexia because it can cause stroke, MI, or seizures. If removing the triggering event doesn't reduce the client's blood pressure, IV antihypertensives should be administered. A fan shouldn't be used because cold drafts may trigger autonomic dysreflexia

The nurse cares for a patient newly diagnosed with systemic lupus erythematosus (SLE). Which of the following is a characteristic sign of SLE? A Butterfly rash across bridge of nose and cheeks B Lesions on extremities C Pain during exposure to cold D Arthritis and joint swelling E Blood in urine

A Butterfly rash across bridge of nose and cheeks

For the first 72 hours after thyroidectomy surgery, nurse Jamie would assess the female client for Chvostek's sign and Trousseau's sign because they indicate which of the following? A Hypocalcemia B Hypercalcemia C Hypokalemia D Hyperkalemia

A Hypocalcemia

Nurse Wayne is aware that a positive Chvostek's sign indicate? A Hypocalcemia B Hyponatremia C Hypokalemia D Hypermagnesemia

A Hypocalcemia

Which clinical manifestation would cause the nurse to suspect that the client is diagnosed with systemic lupus erythematosus? A Joint edema and tenderness B Red, burning, tearing eyes C Chest tightness with wheezing on expiration D Fever and night sweats

A Joint edema and tenderness

A client is admitted to the ER for head trauma is diagnosed with an epidural hematoma. The underlying cause of epidural hematoma is usually related to which of the following conditions? A Laceration of the middle meningeal artery B Rupture of the carotid artery C Thromboembolism from a carotid artery D Venous bleeding from the arachnoid space

A Laceration of the middle meningeal artery

A female client with cancer is being evaluated for possible metastasis. Which of the following is one of the most common metastasis sites for cancer cells? A Liver B Colon C Reproductive tract D White blood cells (WBCs)

A Liver

The nurse is reviewing the lab results for a pt with cirrhosis and notes that the ammonia level is elevated. Which diet does the nurse anticipate to be presribed for this pt? A Low-protein B High-protein C Moderate-fat D High-carb

A Low-protein

When planning care for the patient with acute pancreatitis, the LPN/LVN knows that which intervention is a priority of care? A Pain control B Nutritional supplementation C Observation for mental changes D Observation for intestinal obstruction

A Pain control

What clinical manifestations does the nurse recognize when a patient has had a right hemispheric stroke?

Left visual field deficit

A client has come to the physician to seek help for a decrease in hearing. What tests are likely to be performed for this client? a. A Romberg and Sway test b. A Babinski and Stimulation test c. A Rinne and Weber test d. A Rosenbaum and LOC test

A Rinne and Weber test. Rationale: More than 10% of adults, especially those over age 75, experience hearing deficits. Two diagnostic hearing tests, Rinne and Weber, are useful to identify the extent of hearing loss. The Rosenbaum test is used for close vision reading. The Babinski is a test for reflex action of the foot testing for a CNS deficit. The Romberg also tests for CNS deficit.

seizure disorder

A Seizure disorder may be associated with birth trauma, meningitis, traumatic injury, and a variety of metabolic or developmental disorders.

Which of the following respiratory patterns indicate increasing ICP in the brain stem? A Slow, irregular respirations B Rapid, shallow respirations C Asymmetric chest expansion D Nasal flaring

A Slow irregular respirations

Which of the following lifestyle exposures can lead to a thyroid disorder? A Smoking B Nonsteroidal anti-inflammatory drugs (NSAIDs) C Diet high in oxalates D Asbestos

A Smoking

Nurse Ruth is assessing a client after a thyroidectomy. The assessment reveals muscle twitching and tingling, along with numbness in the fingers, toes, and mouth area. The nurse should suspect which complication? A Tetany B Hemorrhage C Thyroid storm D Laryngeal nerve damage

A Tetany

A nurse is providing postmortem care. Which nursing action violates the standards of caring for the body after a patient has been pronounced dead and is not scheduled for an autopsy? A The nurse places the patient in a sitting position while the family visits. B The nurse places identification tags on both the shroud and the ankle. C The nurse removes soiled dressings and tubes. D The nurse makes sure a death certificate is issued and signed.

A The nurse places the patient in a sitting position while the family visits.

Reflex

A _________ is an action or movement that is built into the nervous system and doesn't need the intervention of conscious thought to take place.

The client with a head injury has been urinating copious amounts of dilute urine through the Foley catheter. The client's urine output for the previous shift was 3000 ml. The nurse implements a new physician order to administer: 1.Desmopressin (DDAVP, stimate) 2.Dexamethasone (Decadron) 3. Ethacrynic acid (Edecrin) 4. Mannitol (Osmitrol)

A complication of a head injury is diabetes insipidus, which can occur with insult to the hypothalamus, the antidiuretic storage vesicles, or the posterior pituitary gland. Urine output that exceeds 9 L per day generally requires treatment with desmopressin. Dexamethasone, a glucocorticoid, is administered to treat cerebral edema. This medication may be ordered for the head injured patient. Ethacrynic acid and mannitol are diuretics, which would be contraindicated.

spatial relations syndrome and stroke

A contellation of impariments that have in common a difficulty in perceibing the relationship between the self and two or more objects in the envrionment: figure-ground discrimination issues form discrimination spatial relations issue position in space-cant figure it out topographical disorientation

VII Facial

Movement of facial muscles Secretions from lacrimal and salivary glands Taste in anterior two-thirds of tongue

A clinical manifestation of acute pancreatitis is epigastric pain. Your nursing intervention to facilitate relief of pain would place the patient in a: A knee-chest position B semi-Fowler's position C recumbent position D low-Fowler's position

A knee-chest position

subdural hematoma

A subdural (below the dura) hematoma is typically slow forming. It is caused by an accumulation of blood, usually from a torn vein on the brain's surface.

A patient had a carotid endarterectomy yesterday and when the nurse arrived in the room to perform an assessment, the patient states, "All of a sudden, I am having trouble moving my right side." What concern should the nurse have about this complaint?

A thrombus formation at the site of the endarterectomy

Myelin sheath

A white, fatty covering that surrounds many axons and is an excellent electrical insulator is termed what?

Which of the following are treatment options for hyperthyroidism? Please select all that apply:* A. Thyroidectomy B. Methimazole C. Liothyronine Sodium "Cytomel" D. Radioactive Iodine

A,B,D

A 57-year-old man is admitted with a diagnosis of cirrhosis. The nurse is aware that he will most likely require which intervention(s)? Select all that apply. A Diuretics B Increased fluids C Bleeding precautions D Vegetable-based proteins E Lactulose administration

A,C,D,E

A client with Bell's palsy asks the nurse what has caused this problem. The nurse's response is based on an understanding that the cause is: A. Unknown, but possibly includes ischemia, viral infection, or an autoimmune problem B. Unknown, but possibly includes long-term tissue malnutrition and cellular hypoxia C. Primary genetic in origin, triggered by exposure to meningitis D. Primarily genetic in origin, triggered by exposure to neurotoxins

A. Bell's palsy is a one-sided facial paralysis from compression of the facial nerve. The exact cause is unknown, but may include vascular ischemia, infection, exposure to viruses such as herpes zoster or herpes simplex, autoimmune disease, or a combination of these factors.

A patient is suspected of having a brain tumor. The signs and symptoms include memory defecits, visual disturbances, weakness of right upper and lower extremities, and personality changes. The nurse recognizes that the tumor is most likely located in the A. Frontal lobe B. Parietal lobe C. Occipital lobe D. Temporal lobe

A. Frontal lobe

A client is admitted with weakness, expressive aphasia, and right hemianopia. The brain MRI reveals an infarct. The nurse understands these symptoms to be suggestive of which of the following findings?

Left-sided cerebrovascular accident (CVA)

The client with a stroke has residual dysphagia. When a diet order is initiated, the nurse avoids doing which of the following? A. Giving the client thin liquids B. Assessing gag reflex before feeding the client C. Allowing plenty of time for chewing and swallowing D. Placing food on the unaffected side of the mouth

A. Giving the client thin liquids Rationale: You want to give the client thicccc liquids boi. All other interventions should be performed

The healthcare provider is assessing a patient admitted with a diagnosis of hemorrhagic stroke affecting the right cranial hemisphere. Which assessment finding is consistent with this diagnosis? A. Left sided flaccidity B. Right-sided spasticity C. Bilateral Babinski sign D. Kerning's sign

A. Left sided flaccidity Rationale: Right-sided strokes effect the LEFT side of the body, and and left-sided strokes effect the RIGHT side

During admission of a patient with a severe head injury to the emergency department, the nurse places the highest priority on assessment for A. Patency of the airway B. Presence of a neck injury C. Neurologic status with the GCS D. CSF leakage from ears or nose

A. Patency of the airway Rationale: ABCs!!

A patient tells the healthcare provider "Yesterday I felt numbness in my right hand, my vision became blurry and I felt like I was losing my balance, but after I laid down for about an hour it went away." What is the healthcare provider's best response? A. This may be a warning sign of a stroke B. Did you feel any nausea at the time? C. Call us if this happens again D. Were you drinking alcohol at the time?

A. This may be a warning sign of a stroke Rationale: This patient had a TIA, which increases the risk for a stroke

When a family member is suspected of having a stroke, which of the following should you do immediately? SELECT ALL THAT APPLY A. Write down the time that symptoms started B. Call 911 C. Ask your family member to smile and assess for any facial drooping D. Give the family member one baby aspirin

A. Write down the time that symptoms started B. Call 911 C. Ask your family member to smile and assess for any facial drooping Rationale: You want to write down what time the symptoms started so that if it is an ischemic stroke, they can administer tPA in time. You DO NOT want to give them a baby aspirin because if it's a hemorrhagic stroke, it could cause greater hemorrhagic damage

Drug therapy for MS include

ACTH (pre-cursor for cortisol, androgen, and aldosterone) Cortisone Cyclophosphamide Other immunosuppressive drugs Nursing focus: *infection prevention* Biologic response modifiers: *interferon-beta products*; effective in MS relapse

INTERNEURONS

ALINK BETWEEN MOTOR & SENSORY; CAN BE INTERCONNECTING NEURONS **ONLY FOUND IN THE CNS**

LEFT SIDE OF THE CEREBRAL HEMISPHERE

ANALYTICAL SKILLS,(READING, MATH, WORD SYMBOLS, MATH), VERBAL SKILLS, AND WALKING

A 45-year-old patient has a dysfunction of the cerebellum. The nurse will plan interventions to a. prevent falls. b. stabilize mood. c. avoid aspiration. d. improve memory

ANS: A Because functions of the cerebellum include coordination and balance, the patient with dysfunction is at risk for falls. The cerebellum does not affect memory, mood, or swallowing ability.

21. A nurse delegates care for a client with Parkinson disease to an unlicensed assistive personnel (UAP). Which statement should the nurse include when delegating this client's care? a. "Allow the client to be as independent as possible with activities." b. "Assist the client with frequent and meticulous oral care." c. "Assess the client's ability to eat and swallow before each meal." d. "Schedule appointments early in the morning to ensure rest in the afternoon."

ANS: A Clients with Parkinson disease do not move as quickly and can have functional problems. The client should be encouraged to be as independent as possible and provided time to perform activities without rushing. Although oral care is important for all clients, instructing the UAP to provide frequent and meticulous oral is not a priority for this client. This statement would be a priority if the client was immune-compromised or NPO. The nurse should assess the client's ability to eat and swallow; this should not be delegated. Appointments and activities should not be scheduled early in the morning because this may cause the client to be rushed and discourage the client from wanting to participate in activities of daily living. DIF: Applying/Application REF: 869 KEY: Parkinson disease| delegation| unlicensed assistive personnel (UAP) MSC: Integrated Process: Communication and Documentation NOT: Client Needs Category: Safe and Effective Care Environment: Management of Care

9. A nurse obtains a focused health history for a client who is suspected of having bacterial meningitis. Which question should the nurse ask? a. "Do you live in a crowded residence?" b. "When was your last tetanus vaccination?" c. "Have you had any viral infections recently?" d. "Have you traveled out of the country in the last month?"

ANS: A Meningococcal meningitis tends to occur in multiple outbreaks. It is most likely to occur in areas of high-density population, such as college dormitories, prisons, and military barracks. A tetanus vaccination would not place the client at increased risk for meningitis or protect the client from meningitis. A viral infection would not lead to bacterial meningitis but could lead to viral meningitis. Simply knowing if the client traveled out of the country does not provide enough information. The nurse should ask about travel to specific countries in which the disease is common, for example, sub-Saharan Africa. DIF: Applying/Application REF: 863 KEY: Meningitis| infection control MSC: IntegratedProcess:NursingProcess:Assessment NOT: Client Needs Category: Health Promotion and Maintenance

Which action will the nurse include in the plan of care for a patient with impaired functioning of the left glossopharyngeal nerve (CN IX) and the vagus nerve (CN X)? a. Withhold oral fluid or foods. b. Provide highly seasoned foods. c. Insert an oropharyngeal airway. d. Apply artificial tears every hour.

ANS: A The glossopharyngeal and vagus nerves innervate the pharynx and control the gag reflex. A patient with impaired function of these nerves is at risk for aspiration. An oral airway may be needed when a patient is unconscious and unable to maintain the airway, but it will not decrease aspiration risk. Taste and eye blink are controlled by the facial nerve.

To assess the functioning of the trigeminal and facial nerves (CNs V and VII), the nurse should a. shine a light into the patient's pupil. b. check for unilateral eyelid drooping. c. touch a cotton wisp strand to the cornea. d. have the patient read a magazine or book

ANS: A The trigeminal and facial nerves are responsible for the corneal reflex. The optic nerve is tested by having the patient read a Snellen chart or a newspaper. Assessment of pupil response to light and ptosis are used to check function of the oculomotor nerve.

To assess the functioning of the trigeminal and facial nerves (CN V and VII), the nurse should a. apply a cotton wisp strand to the cornea. b. have the patient read a magazine or book. c. shine a bright light into the patient's pupil. d. check for unilateral drooping of the eyelids.

ANS: A The trigeminal and facial nerves are responsible for the corneal reflex. The optic nerve is tested by having the patient read a Snellen chart or a newspaper. Assessment of pupil response to light and ptosis are used to check function of the oculomotor nerve. DIF: Cognitive Level: Comprehension REF: 1416-1417

14. A nurse witnesses a client with late-stage Alzheimer's disease eat breakfast. Afterward the client states, "I am hungry and want breakfast." How should the nurse respond? a. "I see you are still hungry. I will get you some toast." b. "You ate your breakfast 30 minutes ago." c. "It appears you are confused this morning." d. "Your family will be here soon. Let's get you dressed."

ANS: A Use of validation therapy with clients who have Alzheimer's disease involves acknowledgment of the client's feelings and concerns. This technique has proved more effective in later stages of the disease, when using reality orientation only increases agitation. Telling the client that he or she already ate breakfast may agitate the client. The other statements do not validate the client's concerns. DIF: Applying/Application REF: 876 KEY: Alzheimer's disease| patient-centered care MSC: Integrated Process: Nursing Process: Implementation NOT: Client Needs Category: Psychosocial Integrity

When admitting an acutely confused 20-year-old patient with a head injury, which action should the nurse take? a. Ask family members about the patient's health history. b. Ask leading questions to assist in obtaining health data. c. Wait until the patient is better oriented to ask questions. d. Obtain only the physiologic neurologic assessment data

ANS: A When admitting a patient who is likely to be a poor historian, the nurse should obtain health history information from others who have knowledge about the patient's health. Waiting until the patient is oriented or obtaining only physiologic data will result in incomplete assessment data, which could adversely affect decision making about treatment. Asking leading questions may result in inaccurate or incomplete information.

When admitting an acutely confused patient with a head injury, which action should the nurse take? a. Ask family members about the patient's health history. b. Ask leading questions to assist in obtaining health data. c. Wait until the patient is better oriented to ask questions. d. Obtain only the physiologic neurologic assessment data.

ANS: A When admitting a patient who is likely to be a poor historian, the nurse should obtain health history information from others who have knowledge about the patient's health. Waiting until the patient is oriented or obtaining only physiologic data will result in incomplete assessment data; this could adversely affect decision-making about treatment. Asking leading questions may result in inaccurate or incomplete information. DIF: Cognitive Level: Application REF: 1412-1413

charge nurse is observing a new staff nurse who is assessing a patient with a possible spinal cord lesion for sensation. Which action indicates a need for further teaching about neurologic assessment? a. The new nurse asks the patient, "Does this feel sharp?" b. The new nurse tests for light touch before testing for pain. c. The new nurse has the patient close the eyes during testing. d. The new nurse uses an irregular pattern to test for intact touch.

ANS: A When performing a sensory assessment, the nurse should not provide verbal clues. The other actions by the new nurse are appropriate. DIF: Cognitive Level: Application REF: 1417-1418

7. A nurse assesses clients on a medical-surgical unit. Which clients should the nurse identify as at risk for secondary seizures? (Select all that apply.) a. A 26-year-old woman with a left temporal brain tumor b. A 38-year-old male client in an alcohol withdrawal program c. A 42-year-old football player with a traumatic brain injury d. A 66-year-old female client with multiple sclerosis e. A 72-year-old man with chronic obstructive pulmonary disease

ANS: A, B, C Clients at risk for secondary seizures include those with a brain lesion from a tumor or trauma, and those who are experiencing a metabolic disorder, acute alcohol withdrawal, electrolyte disturbances, and high fever. Clients with a history of stroke, heart disease, and substance abuse are also at risk. Clients with multiple sclerosis or chronic obstructive pulmonary disease are not at risk for secondary seizures. DIF: Understanding/Comprehension REF: 858 KEY: Seizure| health screening MSC: IntegratedProcess:NursingProcess:Assessment NOT: Client Needs Category: Safe and Effective Care Environment: Management of Care

4. A nurse assesses a client who is experiencing a cluster headache. Which clinical manifestations should the nurse expect to find? (Select all that apply.) a. Ipsilateral tearing of the eye b. Miosis c. Abrupt loss of consciousness d. Neck and shoulder tenderness e. Nasal congestion f. Exophthalmos

ANS: A, B, E Cluster headache is usually accompanied by ipsilateral tearing, miosis, rhinorrhea or nasal congestion, ptosis, eyelid edema, and facial sweating. Abrupt loss of consciousness, neck and shoulder tenderness, and exophthalmos are not associated with cluster headaches. DIF: Understanding/Comprehension REF: 857 KEY: Migraine| assessment/diagnostic examination MSC: IntegratedProcess:NursingProcess:Assessment NOT: Client Needs Category: Physiological Integrity: Physiological Adaptation

9. A nurse is caring for a client with meningitis. Which laboratory values should the nurse monitor to identify potential complications of this disorder? (Select all that apply.) a. Sodium level b. Liver enzymes c. Clotting factors d. Cardiac enzymes e. Creatinine level

ANS: A, C Inflammation associated with meningitis can stimulate the hypothalamus and result in excessive production of antidiuretic hormone. The nurse should monitor sodium levels for early identification of syndrome of inappropriate antidiuretic hormone. A systemic inflammatory response (SIR) can also occur with meningitis. A SIR can result in a coagulopathy that leads to disseminated intravascular coagulation. The nurse should monitor clotting factors to identify this complication. The other laboratory values are not specific to complications of meningitis. DIF: Applying/Application REF: 864 KEY: Meningitis| assessment/diagnostic examination MSC: IntegratedProcess:NursingProcess:Assessment NOT: Client Needs Category: Physiological Integrity: Reduction of Risk Potential

3. A nurse evaluates the results of diagnostic tests on a client's cerebrospinal fluid (CSF). Which fluid results alerts the nurse to possible viral meningitis? (Select all that apply.) a. Clear b. Cloudy c. Increased protein level d. Normal glucose level e. Bacterial organisms present f. Increased white blood cells

ANS: A, C, D In viral meningitis, CSF fluid is clear, protein levels are slightly increased, and glucose levels are normal. Viral meningitis does not cause cloudiness or increased turbidity of CSF. In bacterial meningitis, the presence of bacteria and white blood cells causes the fluid to be cloudy. DIF: Applying/Application REF: 864 KEY: Meningitis| assessment/diagnostic examination MSC: Integrated Process: Nursing Process: Analysis NOT: Client Needs Category: Physiological Integrity: Reduction of Risk Potential

MULTIPLE RESPONSE 1. A nurse plans care for a client with epilepsy who is admitted to the hospital. Which interventions should the nurse include in this client's plan of care? (Select all that apply.) a. Have suction equipment at the bedside. b. Place a padded tongue blade at the bedside. c. Permit only clear oral fluids. d. Keep bed rails up at all times. e. Maintain the client on strict bedrest. f. Ensure that the client has IV access.

ANS: A, D, F Oxygen and suctioning equipment with an airway must be readily available. The bed rails should be up at all times while the client is in the bed to prevent injury from a fall if the client has a seizure. If the client does not have an IV access, insert a saline lock, especially for those clients who are at significant risk for generalized tonic-clonic seizures. The saline lock provides ready access if IV drug therapy must be given to stop the seizure. Padded tongue blades may pose a danger to the client during a seizure and should not be used. Dietary restrictions and strict bedrest are not interventions associated with epilepsy. The client should be encouraged to eat a well-balanced diet and ambulate while in the hospital. DIF: Applying/Application REF: 861 KEY: Seizure| patient safety MSC: Integrated Process: Nursing Process: Implementation NOT: Client Needs Category: Physiological Integrity: Reduction of Risk Potential

15. A nurse assesses a client after administering prescribed levetiracetam (Keppra). Which laboratory tests should the nurse monitor for potential adverse effects of this medication? a. Serum electrolyte levels b. Kidney function tests c. Complete blood cell count d. Antinuclear antibodies

ANS: B Adverse effects of levetiracetam include coordination problems and renal toxicity. The other laboratory tests are not affected by levetiracetam. DIF: Applying/Application REF: 860 KEY: Medication safety| seizure| antiepileptic MSC: IntegratedProcess:NursingProcess:Assessment NOT: Client Needs Category: Physiological Integrity: Pharmacological and Parenteral Therapies

5. A nurse witnesses a client begin to experience a tonic-clonic seizure and loss of consciousness. Which action should the nurse take? a. Start fluids via a large-bore catheter. b. Turn the client's head to the side. c. Administer IV push diazepam. d. Prepare to intubate the client.

ANS: B The nurse should turn the client's head to the side to prevent aspiration and allow drainage of secretions. Anticonvulsants are administered on a routine basis if a seizure is sustained. If the seizure is sustained (status epilepticus), the client must be intubated and should be administered oxygen, 0.9% sodium chloride, and IV push lorazepam or diazepam. DIF: Applying/Application REF: 861 KEY: Seizure| aspiration precautions MSC: Integrated Process: Nursing Process: Implementation NOT: Client Needs Category: Safe and Effective Care Environment: Management of Care

Which nursing diagnosis is expected to be appropriate for a patient who has a positive Romberg test? a. Acute pain b. Risk for falls c. Acute confusion d. Ineffective thermoregulation

ANS: B A positive Romberg test indicates that the patient has difficulty maintaining balance with the eyes closed. The Romberg does not test for orientation, thermoregulation, or discomfort.

The nurse notes in the patient's medical history that the patient has a positive Romberg test. Which nursing diagnosis is appropriate? a. Acute pain related to hyperreflexia and spasm b. Risk for falls related to dizziness or weakness c. Disturbed tactile sensory perception related to spinal cord damage d. Ineffective thermoregulation related to decreased vasomotor response

ANS: B A positive Romberg test indicates that the patient has difficulty maintaining balance with the eyes closed. The Romberg does not test for tactile perception, thermoregulation, or hyperreflexia. DIF: Cognitive Level: Application REF: 1417-1418

A nurse assesses a client after administering prescribed levetiracetam (Keppra). Which laboratory tests should the nurse monitor for potential adverse effects of this medication Serum electrolyte levels Kidney function tests Complete blood cell count Antinuclear antibodies

ANS: B Adverse effects of levetiracetam include coordination problems and renal toxicity. The other laboratory tests are not affected by levetiracetam

An unconscious male patient has just arrived in the emergency department after a head injury caused by a motorcycle crash. Which order should the nurse question? a. Obtain x-rays of the skull and spine. b. Prepare the patient for lumbar puncture. c. Send for computed tomography (CT) scan. d. Perform neurologic checks every 15 minutes

ANS: B After a head injury, the patient may be experiencing intracranial bleeding and increased intracranial pressure, which could lead to herniation of the brain if a lumbar puncture is performed. The other orders are appropriate.

The following orders are received for an unconscious patient who has just arrived in the emergency department after a head injury caused by an automobile accident. Which one should the nurse question? a. Obtain x-rays of the skull and spine. b. Prepare the patient for lumbar puncture. c. Send for computed tomography (CT) scan. d. Perform neurologic checks every 15 minutes.

ANS: B After a head injury, the patient may be experiencing intracranial bleeding and increased intracranial pressure, which could lead to herniation of the brain with lumbar puncture. The other orders are appropriate. DIF: Cognitive Level: Application REF: 1418-1419

Which information about a 76-year-old patient is most important for the admitting nurse to report to the patient's health care provider? a. Triceps reflex response graded at 1/5 b. Unintended weight loss of 20 pounds c. 10 mm Hg orthostatic drop in systolic blood pressure d. Patient complaint of chronic difficulty in falling asleep

ANS: B Although changes in appetite are normal with aging, a 20-pound weight loss requires further investigation. Orthostatic drops in blood pressure, changes in sleep patterns, and slowing of reflexes are normal changes in aging.

A nurse is planning care for a client in spinal shock. Which of the following actions would be least helpful in minimizing the effects of vasodilation below the level of the injury?

Moving the client quickly as one unit

Which information about a 71-year-old patient is most important for the admitting nurse to report to the patient's health care provider? a. Triceps reflex response graded at 1/5 b. Recent unintended weight loss of 20 pounds c. Patient complaint of chronic difficulty in falling asleep d. Orthostatic drop in systolic blood pressure of 10 mm Hg

ANS: B Although changes in appetite are normal with aging, a 20-pound weight loss requires further investigation. Orthostatic drops in blood pressure, changes in sleep patterns, and slowing of reflexes are normal changes in aging. DIF: Cognitive Level: Application REF: 1412-1413 | 1414

18. A nurse assesses a client with Huntington disease. Which motor changes should the nurse monitor for in this client? a. Shuffling gait b. Jerky hand movements c. Continuous chewing motions d. Tremors of the hands

ANS: B An imbalance between excitatory and inhibitory neurotransmitters leads to uninhibited motor movements, such as brisk, jerky, purposeless movements of the hands, face, tongue, and legs. Shuffling gait, continuous chewing motions, and tremors are associated with Parkinson disease. DIF: Remembering/Knowledge REF: 881 KEY: Huntington disease MSC: IntegratedProcess:NursingProcess:Assessment NOT: Client Needs Category: Physiological Integrity: Physiological Adaptation

. Several patients have been hospitalized for diagnosis of neurologic problems. Which patient will the nurse assess first? a. Patient with a transient ischemic attack (TIA) returning from carotid duplex studies b. Patient with a brain tumor who has just arrived on the unit after a cerebral angiogram c. Patient with a seizure disorder who has just completed an electroencephalogram (EEG) d. Patient prepared for a lumbar puncture whose health care provider is waiting for assistance

ANS: B Because cerebral angiograms require insertion of a catheter into the femoral artery, bleeding is a possible complication. The nurse will need to check the pulse, blood pressure, and the catheter insertion site in the groin as soon as the patient arrives. Carotid duplex studies and EEG are noninvasive. The nurse will need to assist with the lumbar puncture as soon as possible, but monitoring for hemorrhage after cerebral angiogram has a higher priority.

Which finding would the nurse expect when assessing the legs of a patient who has a lower motor neuron lesion? a. Spasticity b. Flaccidity c. No sensation d. Hyperactive reflexes

ANS: B Because the cell bodies of lower motor neurons are located in the spinal cord, damage to the neuron will decrease motor activity of the affected muscles. Spasticity and hyperactive reflexes are caused by upper motor neuron damage. Sensation is not impacted by motor neuron lesions

A patient has a lesion that affects lower motor neurons. During assessment of the patient's lower extremities, the nurse expects to find a. spasticity. b. flaccidity. c. loss of sensation. d. hyperactive reflexes.

ANS: B Because the cell bodies of lower motor neurons are located in the spinal cord, damage to the neuron will decrease motor activity of the affected muscles. Spasticity and hyperactive reflexes are caused by upper motor neuron damage. Sensation is not impacted by motor neuron lesions. DIF: Cognitive Level: Comprehension REF: 1408-1409

1. A nurse is teaching a client who experiences migraine headaches and is prescribed a beta blocker. Which statement should the nurse include in this client's teaching? a. "Take this drug only when you have prodromal symptoms indicating the onset of a migraine headache." b. "Take this drug as ordered, even when feeling well, to prevent vascular changes associated with migraine headaches." c. "This drug will relieve the pain during the aura phase soon after a headache has started." d. "This medication will have no effect on your heart rate or blood pressure because you are taking it for migraines."

ANS: B Beta blockers are prescribed as prophylactic treatment to prevent the vascular changes that initiate migraine headaches. Heart rate and blood pressure will also be affected, and the client should monitor these side effects. The other responses do not discuss appropriate uses of the medication. DIF: Applying/Application REF: 856 KEY: Medication safety| beta blocker| migraine MSC: IntegratedProcess:Teaching/Learning NOT: Client Needs Category: Physiological Integrity: Pharmacological and Parenteral Therapies

A patient is scheduled for a lumbar puncture. The nurse will plan to a. transfer the patient to radiology just before the procedure. b. help the patient to a side lying position before the procedure. c. place the patient on NPO status for 4 hours before the procedure. d. administer a sedative medication 30 minutes before the procedure.

ANS: B For a lumbar puncture, the patient lies in the lateral recumbent position. The procedure does not usually require a sedative, is done in the patient room, and has no risk for aspiration. DIF: Cognitive Level: Application REF: 1418-1419

6. A nurse cares for a client who is experiencing status epilepticus. Which prescribed medication should the nurse prepare to administer? a. Atenolol (Tenormin) b. Lorazepam (Ativan) c. Phenytoin (Dilantin) d. Lisinopril (Prinivil)

ANS: B Initially, intravenous lorazepam is administered to stop motor movements. This is followed by the administration of phenytoin. Atenolol, a beta blocker, and lisinopril, an angiotensin-converting enzyme inhibitor, are not administered for seizure activity. These medications are typically administered for hypertension and heart failure. DIF: Applying/Application REF: 861 KEY: Seizure| benzodiazepine| medication safety MSC: Integrated Process: Nursing Process: Implementation NOT: Client Needs Category: Physiological Integrity: Pharmacological and Parenteral Therapies

A 39-year-old patient with a suspected herniated intervertebral disc is scheduled for a myelogram. Which information is most important for the nurse to communicate to the health care provider before the procedure? a. The patient is anxious about the test. b. The patient has an allergy to shellfish. c. The patient has back pain when lying flat. d. The patient drank apple juice 4 hours earlier.

ANS: B Iodine-containing contrast medium is injected into the subarachnoid space during a myelogram. The health care provider may need to modify the postmyelogram orders to prevent back pain, but this can be done after the procedure. Clear liquids are usually considered safe up to 4 hours before a diagnostic or surgical procedure. The patient's anxiety should be addressed, but this is not as important as the iodine allergy.

A patient with suspected meningitis is scheduled for a lumbar puncture. Before the procedure, the nurse will plan to a. enforce NPO status for 4 hours. b. transfer the patient to radiology. c. administer a sedative medication. d. help the patient to a lateral position.

ANS: D For a lumbar puncture, the patient lies in the lateral recumbent position. The procedure does not usually require a sedative, is done in the patient room, and has no risk for aspiration.

A patient is scheduled for a myelogram to confirm the presence of a herniated intervertebral disk. Which information obtained when admitting the patient is most important for the nurse to communicate to the health care provider before the procedure? a. The patient is anxious about the test. b. The patient has an allergy to shellfish. c. The patient had 4 ounces of apple juice 4 hours earlier. d. The patient has back pain when lying flat for long periods.

ANS: B Iodine-containing contrast medium is injected into the subarachnoid space during a myelogram. The health care provider may need to modify the postmyelogram orders to prevent back pain, but this can be done after the procedure. Clear liquids are usually considered safe up to 4 hours before a diagnostic or surgical procedure. The patient's anxiety should be addressed, but this is not as important as the iodine allergy. DIF: Cognitive Level: Application REF: 1421-1422

4. A nurse assesses a client with a history of epilepsy who experiences stiffening of the muscles of the arms and legs, followed by an immediate loss of consciousness and jerking of all extremities. How should the nurse document this activity? a. Atonic seizure b. Tonic-clonic seizure c. Myoclonic seizure d. Absence seizure

ANS: B Seizure activity that begins with stiffening of the arms and legs, followed by loss of consciousness and jerking of all extremities, is characteristic of a tonic-clonic seizure. An atonic seizure presents as a sudden loss of muscle tone followed by postictal confusion. A myoclonic seizure presents with a brief jerking or stiffening of extremities that may occur singly or in groups. Absence seizures present with automatisms, and the client is unaware of his or her environment. DIF: Understanding/Comprehension REF: 858 KEY: Seizure MSC: Integrated Process: Communication and Documentation NOT: Client Needs Category: Physiological Integrity: Physiological Adaptation

16. A nurse cares for a client with advanced Alzheimer's disease. The client's caregiver states, "She is always wandering off. What can I do to manage this restless behavior?" How should the nurse respond? a. "This is a sign of fatigue. The client would benefit from a daily nap." b. "Engage the client in scheduled activities throughout the day." c. "It sounds like this is difficult for you. I will consult the social worker." d. "The provider can prescribe a mild sedative for restlessness."

ANS: B Several strategies may be used to cope with restlessness and wandering. One strategy is to engage the client in structured activities. Another is to take the client for frequent walks. Daily naps and a mild sedative will not be as effective in the management of restless behavior. Consulting the social worker does not address the caregiver's concern. DIF: Applying/Application REF: 878 KEY: Alzheimer's disease| patient safety MSC: Integrated Process: Nursing Process: Implementation NOT: Client Needs Category: Safe and Effective Care Environment: Safety and Infection Control

3. A nurse obtains a health history on a client prior to administering prescribed sumatriptan succinate (Imitrex) for migraine headaches. Which condition should alert the nurse to hold the medication and contact the health care provider? a. Bronchial asthma b. Prinzmetal's angina c. Diabetes mellitus d. Chronic kidney disease

ANS: B Sumatriptan succinate effectively reduces pain and other associated symptoms of migraine headache by binding to serotonin receptors and triggering cranial vasoconstriction. Vasoconstrictive effects are not confined to the cranium and can cause coronary vasospasm in clients with Prinzmetal's angina. The other conditions would not affect the client's treatment. DIF: Applying/Application REF: 856 KEY: Medication safety| migraine MSC: Integrated Process: Nursing Process: Analysis NOT: Client Needs Category: Physiological Integrity: Pharmacological and Parenteral Therapies

Neurologic testing of the patient indicates impaired functioning of the left glossopharyngeal nerve (CN IX) and the vagus nerve (CN X). Which action will the nurse include in the plan of care? a. Insert an oral airway. b. Withhold oral fluid or foods. c. Provide highly seasoned foods. d. Apply artificial tears every hour.

ANS: B The glossopharyngeal and vagus nerves innervate the pharynx and control the gag reflex; a patient with impaired function of these nerves is at risk for aspiration. An oral airway may be needed when a patient is unconscious and unable to maintain the airway, but it will not decrease aspiration risk. Taste and eye blink are controlled by the facial nerve. DIF: Cognitive Level: Application REF: 1417

Which cerebrospinal fluid analysis result will be most important for the nurse to communicate to the health care provider? a. Specific gravity 1.007 b. Protein 65 mg/dL (0.65 g/L) c. Glucose 45 mg/dL (1.7 mmol/L) d. White blood cell (WBC) count 4 cells/µL

ANS: B The protein level is high. The specific gravity, WBCs, and glucose values are normal.

Which equipment will the nurse obtain to assess vibration sense in a diabetic patient who has peripheral nerve dysfunction? a. Sharp pin b. Tuning fork c. Reflex hammer d. Calibrated compass

ANS: B Vibration sense is testing by touching the patient with a vibrating tuning fork. The other equipment is needed for testing of pain sensation, reflexes, and two-point discrimination.

Which equipment will the nurse obtain to assess vibration sense in a patient who has peripheral nerve dysfunction? a. Sharp pin b. Tuning fork c. Reflex hammer d. Calibrated compass

ANS: B Vibration sense is testing by touching the patient with a vibrating tuning fork. The other equipment is needed for testing of pain sensation, reflexes, and two-point discrimination. DIF: Cognitive Level: Comprehension REF: 1417-1419

Which assessments will the nurse make to monitor a patient's cerebellar function (select all that apply)? a. Assess for graphesthesia. b. Observe arm swing with gait. c. Perform the finger-to-nose test. d. Check ability to push against resistance. e. Determine ability to sense heat and cold.

ANS: B, C The cerebellum is responsible for coordination and is assessed by looking at the patient's gait and the finger-to-nose test. The other assessments will be used for other parts of the neurologic assessment.

Which assessments will the nurse make to test a patient's cerebellar function (select all that apply)? a. Assess for graphesthesia. b. Perform the finger-to-nose test. c. Observe arm movement with gait. d. Check ability to push against resistance. e. Determine ability to sense heat and cold.

ANS: B, C The cerebellum is responsible for coordination and is assessed by looking at the patient's gait and the finger-to-nose test. The other assessments will be used for other parts of the neurological assessment. DIF: Cognitive Level: Analysis REF: 1417-1418

10. A nurse assesses a client who has encephalitis. Which manifestations should the nurse recognize as signs of increased intracranial pressure (ICP), a complication of encephalitis? (Select all that apply.) a. Photophobia b. Dilated pupils c. Headache d. Widened pulse pressure e. Bradycardia

ANS: B, D, E Increased ICP is a complication of encephalitis. The nurse should monitor for signs of increased ICP, including dilated pupils, widened pulse pressure, bradycardia, irregular respirations, and less responsive pupils. Photophobia and headache are not related to increased ICP. DIF: Applying/Application REF: 865 KEY: Encephalitis| intracranial pressure MSC: IntegratedProcess:NursingProcess:Assessment NOT: Client Needs Category: Physiological Integrity: Reduction of Risk Potential

2. A nurse is teaching a client who has chronic headaches. Which statements about headache triggers should the nurse include in this client's plan of care? (Select all that apply.) a. "Increase your intake of caffeinated beverages." b. "Incorporate physical exercise into your daily routine." c. "Avoid all alcoholic beverages." d. "Participate in a smoking cessation program." e. "Increase your intake of fruits and vegetables."

ANS: B, D, E Triggers for headaches include caffeine, smoking, and ingestion of pickled foods, so these factors should be avoided. Clients are taught to eat a balanced diet and to get adequate exercise and rest. Alcohol does not trigger chronic headaches but can enhance headaches during the headache period. DIF: Applying/Application REF: 857 KEY: Migraine| patient education MSC: Integrated Process: Teaching/Learning NOT: Client Needs Category: Health Promotion and Maintenance

5. A nurse assesses a client who is experiencing an absence seizure. For which clinical manifestations should the nurse assess? (Select all that apply.) a. Intermittent rigidity b. Lip smacking c. Sudden loss of muscle tone d. Brief jerking of the extremities e. Picking at clothing f. Patting of the hand on the leg

ANS: B, E, F Automatisms are characteristic of absence seizures. These behaviors consist of lip smacking, picking at clothing, and patting. Rigidity of muscles is associated with the tonic phase of a seizure, and jerking of the extremities is associated with the clonic phase of a seizure. Loss of muscle tone occurs with atonic seizures. DIF: Understanding/Comprehension REF: 858 KEY: Seizure| assessment/diagnostic examination MSC: IntegratedProcess:NursingProcess:Assessment NOT: Client Needs Category: Physiological Integrity: Physiological Adaptation

20. A nurse is teaching a client with chronic migraine headaches. Which statement related to complementary therapy should the nurse include in this client's teaching? a. "Place a warm compress on your forehead at the onset of the headache." b. "Wear dark sunglasses when you are in brightly lit spaces." c. "Lie down in a darkened room when you experience a headache." d. "Set your alarm to ensure you do not sleep longer than 6 hours at one time."

ANS: C At the onset of a migraine attack, the client may be able to alleviate pain by lying down and darkening the room. He or she may want both eyes covered and a cool cloth on the forehead. If the client falls asleep, he or she should remain undisturbed until awakening. The other options are not recognized therapies for migraines. DIF: Applying/Application REF: 856 KEY: Migraine| complementary/alternative medications MSC: IntegratedProcess:Teaching/Learning NOT: Client Needs Category: Physiological Integrity: Basic Care and Comfort

13. A nurse assesses a client with Alzheimer's disease who is recently admitted to the hospital. Which psychosocial assessment should the nurse complete? a. Assess religious and spiritual needs while in the hospital. b. Identify the client's ability to perform self-care activities. c. Evaluate the client's reaction to a change of environment. d. Ask the client about relationships with family members.

ANS: C As Alzheimer's disease progresses, the client experiences changes in emotional and behavioral affect. The nurse should be alert to the client's reaction to a change in environment, such as being hospitalized, because the client may exhibit an exaggerated response, such as aggression, to the event. The other assessments should be completed but are not as important as assessing the client's reaction to environmental change. DIF: Applying/Application REF: 875 KEY: Alzheimer's disease| psychosocial response MSC: IntegratedProcess:NursingProcess:Assessment NOT: Client Needs Category: Psychosocial Integrity

Which nursing action will be included in the care for a patient who has had cerebral angiography? a. Monitor for headache and photophobia. b. Keep patient NPO until gag reflex returns. c. Check pulse and blood pressure frequently. d. Assess orientation to person, place, and time

ANS: C Because a catheter is inserted into an artery (such as the femoral artery) during cerebral angiography, the nurse should assess for bleeding after this procedure. The other nursing assessments are not necessary after angiography.

22. A nurse delegates care for a client with early-stage Alzheimer's disease to an unlicensed assistive personnel (UAP). Which statement should the nurse include when delegating this client's care? a. "If she is confused, play along and pretend that everything is okay." b. "Remove the clock from her room so that she doesn't get confused." c. "Reorient the client to the day, time, and environment with each contact." d. "Use validation therapy to recognize and acknowledge the client's concerns."

ANS: C Clients who have early-stage Alzheimer's disease should be reoriented frequently to person, place, and time. The UAP should reorient the client and not encourage the client's delusions. The room should have a clock and white board with the current date written on it. Validation therapy is used with late-stage Alzheimer's disease. DIF: Applying/Application REF: 876 KEY: Alzheimer's disease| delegation| unlicensed assistive personnel (UAP) MSC: Integrated Process: Communication and Documentation NOT: Client Needs Category: Safe and Effective Care Environment: Management of Care

17. A nurse prepares to discharge a client with Alzheimer's disease. Which statement should the nurse include in the discharge teaching for this client's caregiver? a. "Allow the client to rest most of the day." b. "Place a padded throw rug at the bedside." c. "Install deadbolt locks on all outside doors." d. "Provide a high-calorie and high-protein diet."

ANS: C Clients with Alzheimer's disease have a tendency to wander, especially at night. If possible, alarms should be installed on all outside doors to alert family members if the client leaves. At a minimum, all outside doors should have deadbolt locks installed to prevent the client from going outdoors unsupervised. The client should be allowed to exercise within his or her limits. Throw rugs are a slip and fall hazard and should be removed. The client should eat a well-balanced diet. There is no need for a high-calorie or high-protein diet. DIF: Applying/Application REF: 879 KEY: Alzheimer's disease| patient-centered care MSC: IntegratedProcess:Teaching/Learning NOT: Client Needs Category: Safe and Effective Care Environment: Safety and Infection Control

12. A nurse is teaching the daughter of a client who has Alzheimer's disease. The daughter asks, "Will the medication my mother is taking improve her dementia?" How should the nurse respond? a. "It will allow your mother to live independently for several more years." b. "It is used to halt the advancement of Alzheimer's disease but will not cure it." c. "It will not improve her dementia but can help control emotional responses." d. "It is used to improve short-term memory but will not improve problem solving."

ANS: C Drug therapy is not effective for treating dementia or halting the advancement of Alzheimer's disease. However, certain drugs may help suppress emotional disturbances and psychiatric manifestations. Medication therapy may not allow the client to safely live independently. DIF: Applying/Application REF: 877 KEY: Alzheimer's disease| safety MSC: Integrated Process: Nursing Process: Evaluation 868 KEY: Parkinson disease NOT: Client Needs Category: Physiological Integrity: Pharmacological and Parenteral Therapies

2. A nurse assesses a client who has a history of migraines. Which clinical manifestation should the nurse identify as an early sign of a migraine with aura? a. V ertigo b. Lethargy c. Visual disturbances d. Numbness of the tongue

ANS: C Early warning of impending migraine with aura usually consists of visual changes, flashing lights, or diplopia. The other manifestations are not associated with an impending migraine with aura. DIF: Understanding/Comprehension REF: 854 KEY: Migraine| assessment/diagnostic examination MSC: IntegratedProcess:NursingProcess:Assessment NOT: Client Needs Category: Physiological Integrity: Physiological Adaptation

When caring for a patient who has had cerebral angiography, which nursing action will be included in the plan of care? a. Ask about headache and photophobia. b. Keep patient NPO until gag reflex returns. c. Check pulse and blood pressure frequently. d. Assess orientation to person, place, and time.

ANS: C Since a catheter is inserted into an artery (such as the femoral artery) during cerebral angiography, the nurse should assess for bleeding after this procedure. The other nursing assessments are not necessary after angiography. DIF: Cognitive Level: Application REF: 1418-1419

8. A nurse assesses a client who is recovering from the implantation of a vagal nerve stimulation device. For which clinical manifestations should the nurse assess as common complications of this procedure? (Select all that apply.) a. Bleeding b. Infection c. Hoarseness d. Dysphagia e. Seizures

ANS: C, D Complications of surgery to implant a vagal nerve stimulation device include hoarseness (most common), dyspnea, neck pain, and dysphagia. The device is tunneled under the skin with an electrode connected to the vagus nerve to control simple or complex partial seizures. Bleeding is not a common complication of this procedure, and infection would not occur during the recovery period. DIF: Applying/Application REF: 862 KEY: Seizure MSC: IntegratedProcess:NursingProcess:Assessment NOT: Client Needs Category: Physiological Integrity: Reduction of Risk Potential

10. After teaching the wife of a client who has Parkinson disease, the nurse assesses the wife's understanding. Which statement by the client's wife indicates she correctly understands changes associated with this disease? a. "His masklike face makes it difficult to communicate, so I will use a white board." b. "He should not socialize outside of the house due to uncontrollable drooling." c. "This disease is associated with anxiety causing increased perspiration." d. "He may have trouble chewing, so I will offer bite-sized portions."

ANS: D Because chewing and swallowing can be problematic, small frequent meals and a supplement are better for meeting the client's nutritional needs. A masklike face and drooling are common in clients with Parkinson disease. The client should be encouraged to continue to socialize and communicate as normally as possible. The wife should understand that the client's masklike face can be misinterpreted and additional time may be needed for the client to communicate with her or others. Excessive perspiration is also common in clients with Parkinson disease and is associated with the autonomic nervous system's response. DIF: Applying/Application REF: MSC: IntegratedProcess:Teaching/Learning NOT: Client Needs Category: Psychosocial Integrity

7. After teaching a client who is diagnosed with new-onset status epilepticus and prescribed phenytoin (Dilantin), the nurse assesses the client's understanding. Which statement by the client indicates a correct understanding of the teaching? a. "To prevent complications, I will drink at least 2 liters of water daily." b. "This medication will stop me from getting an aura before a seizure." c. "I will not drive a motor vehicle while taking this medication." d. "Even when my seizures stop, I will continue to take this drug."

ANS: D Discontinuing antiepileptic drugs can lead to the recurrence of seizures or status epilepticus. The client does not need to drink more water and can drive while taking this medication. The medication will not stop an aura before a seizure. DIF: Applying/Application REF: 861 KEY: Medication safety| seizure| antiepileptic MSC: IntegratedProcess:Teaching/Learning NOT: Client Needs Category: Physiological Integrity: Pharmacological and Parenteral Therapies

11. A nurse plans care for a client with Parkinson disease. Which intervention should the nurse include in this client's plan of care? a. Ambulate the client in the hallway twice a day. b. Ensure a fluid intake of at least 3 liters per day. c. Teach the client pursed-lip breathing techniques. d. Keep the head of the bed at 30 degrees or greater.

ANS: D Elevation of the head of the bed will help prevent aspiration. The other options will not prevent aspiration, which is the greatest respiratory complication of Parkinson disease, nor do these interventions address any of the complications of Parkinson disease. Ambulation in the hallway is usually implemented to prevent venous thrombosis. Increased fluid intake flushes out toxins from the client's blood. Pursed-lip breathing increases exhalation of carbon dioxide. DIF: Applying/Application REF: 870 KEY: Parkinson disease| aspiration precautions MSC: Integrated Process: Nursing Process: Implementation NOT: Client Needs Category: Physiological Integrity: Reduction of Risk Potential

During the neurologic assessment, the patient cooperates with the nurse's directions to grip with the hands and to move the feet but is unable to respond orally to the nurse's questions. The nurse will suspect a. a brainstem lesion. b. a temporal lobe lesion. c. injury to the cerebellum. d. damage to the frontal lobe.

ANS: D Expressive speech is controlled by Broca's area in the frontal lobe. The temporal lobe contains Wernicke's area, which is responsible for receptive speech. The cerebellum and brainstem do not affect higher cognitive functions such as speech. DIF: Cognitive Level: Application REF: 1408-1409

The nurse will anticipate teaching a patient with a possible seizure disorder about which test? a. Cerebral angiography b. Evoked potential studies c. Electromyography (EMG) d. Electroencephalography (EEG)

ANS: D Seizure disorders are usually assessed using EEG testing. Evoked potential is used for diagnosing problems with the visual or auditory systems. Cerebral angiography is used to diagnose vascular problems. EMG is used to evaluate electrical innervation to skeletal muscle.

A patient is hospitalized with a possible seizure disorder. To determine the cause of the patient's symptoms, the nurse will anticipate the need to teach the patient about which of these tests? a. Cerebral angiography b. Evoked potential studies c. Electromyography (EMG) d. Electroencephalography (EEG)

ANS: D Seizure disorders are usually studied using EEG testing. Evoked potential is used for diagnosing problems with the visual or auditory systems. Cerebral angiography is used to diagnose vascular problems. EMG is used to evaluate electrical innervation to skeletal muscle. DIF: Cognitive Level: Comprehension REF: 1419-1423

8. After teaching a client newly diagnosed with epilepsy, the nurse assesses the client's understanding. Which statement by the client indicates a need for additional teaching? a. "I will wear my medical alert bracelet at all times." b. "While taking my epilepsy medications, I will not drink any alcoholic beverages." c. "I will tell my doctor about my prescription and over-the-counter medications." d. "If I am nauseated, I will not take my epilepsy medication."

ANS: D The nurse must emphasize that antiepileptic drugs must be taken even if the client is nauseous. Discontinuing the medication can predispose the client to seizure activity and status epilepticus. The client should not drink alcohol while taking seizure medications. The client should wear a medical alert bracelet and should make the doctor aware of all medications to prevent complications of polypharmacy. DIF: Applying/Application REF: 860 KEY: Seizure| medication safety| patient education MSC: IntegratedProcess:Teaching/Learning NOT: Client Needs Category: Health Promotion and Maintenance

The nurse performing a focused assessment of left posterior temporal lobe functions will assess the patient for a. sensation on the left side of the body. b. voluntary movements on the right side. c. reasoning and problem-solving abilities. d. understanding written and oral language.

ANS: D The posterior temporal lobe integrates the visual and auditory input for language comprehension. Reasoning and problem solving are functions of the anterior frontal lobe. Sensation on the left side of the body is located in the right postcentral gyrus. Voluntary movement on the right side is controlled in the left precentral gyrus.

When performing a focused assessment on a patient with a lesion of the left posterior temporal lobe, the nurse will assess for a. sensation on the left side of the body. b. voluntary movement on the right side. c. reasoning and problem-solving abilities. d. understanding of written and oral language.

ANS: D The posterior temporal lobe integrates the visual and auditory input for language comprehension. Reasoning and problem solving are functions of the anterior frontal lobe. Sensation on the left side of the body is located in the right postcentral gyrus. Voluntary movement on the right side is controlled in the left precentral gyrus. DIF: Cognitive Level: Application REF: 1409

19. A nurse cares for a client who has been diagnosed with the Huntington gene but has no symptoms. The client asks for options related to family planning. What is the nurse's best response? a. "Most clients with the Huntington gene do not pass on Huntington disease to their children." b. "I understand that they can diagnose this disease in embryos. Therefore, you could select a healthy embryo from your fertilized eggs for implantation to avoid passing on Huntington disease." c. "The need for family planning is limited because one of the hallmarks of Huntington disease is infertility." d. "Tell me more specifically what information you need about family planning so that I can direct you to the right information or health care provider."

ANS: D The presence of the Huntington gene means that the trait will be passed on to all offspring of the affected person. Understanding options for contraception and conception (e.g., surrogacy options) and implications for children may require the expertise of a genetic counselor or a reproductive specialist. The other statements are not accurate. DIF: Applying/Application REF: 881 KEY: Huntington disease| genetic counseling MSC: IntegratedProcess:Teaching/Learning NOT: Client Needs Category: Safe and Effective Care Environment: Management of Care

After reviewing a patient's cerebrospinal fluid analysis, which result will be most important for the nurse to communicate to the health care provider? a. Specific gravity 1.007 b. Protein 65 mg/dL (0.30 g/L) c. White blood cell (WBC) count 4/μL d. Glucose 45 mg/dL (1.7 mmol/L)

ANS: D The protein level is high. The pH, WBCs, and glucose values are normal. DIF: Cognitive Level: Comprehension REF: 1422

The priority nursing assessment for a 72-year-old patient being admitted with a brainstem infarction is a. reflex reaction time. b. pupil reaction to light. c. level of consciousness. d. respiratory rate and rhythm.

ANS: D Vital centers that control respiration are located in the medulla, and these are the priority assessments because changes in respiratory function may be life threatening. The other information will also be collected by the nurse, but it is not as urgent

A patient with a brainstem infarction is admitted to the nursing unit. The priority nursing assessment for the patient is a. reflex reaction time. b. pupil reaction to light. c. level of consciousness. d. respiratory rate and rhythm.

ANS: D Vital centers that control respiration are located in the medulla, and these are the priority assessments because changes in respiratory function may be life threatening. The other information also will be collected by the nurse, but it is not as urgent. DIF: Cognitive Level: Application REF: 1410

6. A nurse prepares to provide perineal care to a client with meningococcal meningitis. Which personal protective equipment should the nurse wear? (Select all that apply.) a. Particulate respirator b. Isolation gown c. Shoe covers d. Surgical mask e. Gloves

ANS: D, E Meningeal meningitis is spread via saliva and droplets, and Droplet Precautions are necessary. Caregivers should wear a surgical mask when within 6 feet of the client and should continue to use Standard Precautions, including gloves. A particulate respirator, an isolation gown, and shoe covers are not necessary for Droplet Precautions. DIF: Applying/Application REF: 865 KEY: Meningitis| infection control| Transmission-Based Precautions MSC: Integrated Process: Nursing Process: Implementation NOT: Client Needs Category: Safe and Effective Care Environment: Safety and Infection Control

A 48-year-old male construction worker has come to the clinic with a complaint of something in his eye. The LPN/LVN assists the physician to stain the eye. She knows the physician will use fluorescein stain. This will identify what? a. Contusion b. Laceration c. Bruising d. Abrasion.

Abrasion. Rationale: Fluorescein dye is injected onto the cornea, and the cornea is viewed with a slit lamp. The green staining allows identification of corneal ulceration or abrasion. Contusion and bruising are characteristics of injury seen during skin assessment. A laceration to t

Neurotransmitters

Acetylcholine, dopamine, and norepinephrine are the major ________________ secreted by the neuron.

The nurse is caring for a client with myasthenia gravis. When planning the client's care, what action is most important for the nurse to implement? a. Note any complaints by the client of changes in vision b. Administer medications on a strict time schedule c. Perform the client's care quickly because of tiring easily d. Monitor for facial muscle weakness

Administer medications on a strict time schedule. Outcome 6. Rationale: Clients with dysphagia may not be able to swallow the medication unless it is taken exactly on time. Doses taken too late may cause myasthenic crisis. Notes about changes in vision are not the priority; providing medication on time is. The nurse must prevent client tiring, but this is not the most important action of those listed. Monitoring for facial muscle weakness is not as important as ensuring that medications are given on time to prevent myasthenic crisis.

A 23-year-old patient with a recent history of encephalitis is admitted to the medical unit with new onset generalized tonic-clonic seizures. Which nursing activities included in the patient's care will be best to delegate to an LPN/LVN whom you are supervising? (Choose all that apply). 1.Document the onset time, nature of seizure activity, and postictal behaviors for all seizures. 2.Administer phenytoin (Dilantin) 200 mg PO daily. 3.Teach patient about the need for good oral hygiene. 4.Develop a discharge plan, including physician visits and referral to the Epilepsy Foundation.

Administration of medications is included in LPN education and scope of practice. Collection of data about the seizure activity may be accomplished by an LPN/LVN who observes initial seizure activity. An LPN/LVN would know to call the supervising RN immediately if a patient started to seize. Documentation of the seizure, patient teaching, and planning of care are complex activities that require RN level education and scope of practice.

A nurse is teaching a community group about modifiable and nonmodifiable risk factors for ischemic strokes. Which of the following is a risk factor that cannot be modified?

Advanced age

A home health nurse visits a client who's taking pilocarpine, a miotic agent, to treat glaucoma. The nurse notes that the client's pilocarpine solution is cloudy. What should the nurse do first?

Advise the client to discard the drug because it may have undergone chemical changes or become contaminated.

Blood-brain barrier

Protects from harmful agents, while allowing nutrients and gases to enter

The nurse is caring for a client who suffered a spinal cord injury 48 hours ago. The nurse monitors for GI complications by assessing for: 1.A flattened abdomen 2. Hematest positive nasogastric tube drainage 3.Hyperactive bowel sounds 4. A history of diarrhea

After spinal cord injury, the client can develop paralytic ileus, which is characterized by the absence of bowel sounds and abdominal distention. Development of a stress ulcer can be detected by hematest positive NG tube aspirate or stool. A history of diarrhea is irrelevant.

A clinic nurse is caring for a client diagnosed with migraine headaches. During the client teaching session, the client questions the nurse regarding alcohol consumption. What would the nurse be correct in telling the client about the effects of alcohol?

Alcohol causes vasodilation of the blood vessels.

A client comes into the ER after hitting his head in an MVA. He's alert and oriented. Which of the following nursing interventions should be done first? 1.Assess full ROM to determine extent of injuries 2.Call for an immediate chest x-ray 3.Immobilize the client's head and neck 4. Open the airway with the head-tilt chin-lift maneuver

All clients with a head injury are treated as if a cervical spine injury is present until x-rays confirm their absence. ROM would be contraindicated at this time. There is no indication that the client needs a chest x-ray. The airway doesn't need to be opened since the client appears alert and not in respiratory distress. In addition, the head-tilt chin-lift maneuver wouldn't be used until the cervical spine injury is ruled out.

A client is following up after a visit to the emergency department where testing indicated that the client had suffered a transient ischemic attack. To prevent the occurrence of a more serious cerebrovascular accident, which lifestyle changes would the neurologist to prescribe?

All options are correct.

A nurse is preparing a client who is scheduled to have cerebral angiography performed. The nurse should check the client for:

Allergy to iodine or shellfish

A nurse has obtained a personal and family history from a client with a neurological disorder. Which finding in the client's history will have the least amount of added risk for neurological problems?

Allergy to pollen

A client with spinal cord injury has experienced more than one episode of autonomic dysreflexia. The nurse would avoid which of the following that could trigger an episode of this complication?

Allowing the client's bladder to become distended

The nurse is caring for a client who had a hemorrhagic stroke. What assessment finding constitutes an early sign of deterioration?

Alteration in level of consciousness (LOC)

For a client with hepatic cirrhosis who has altered clotting mechanisms, which intervention would be most important? A Allowing complete independence of mobility B Applying pressure to injection sites C Administering antibiotics as prescribed D Increasing nutritional intake

B Applying pressure to injection sites

Try not to use___________; they only increase restlessness. Avoid_____________because they mask levels of responsiveness.

Restraints Narcotics

An 84-year-old client in an acute state of disorientation was brought to the emergency department by the client's daughter. The daughter states that this is the first time that the client experienced confusion. The nurse determines from this piece of information that which of the following is unlikely to be the cause of the client's disorientation?

Alzheimer's disease

The nurse overhears the term "sundowning" used to describe the behavior of a client newly admitted to the nursing unit during the previous evening shift. The nurse interprets that this client most likely has a diagnosis of:

Alzheimer's disease

The nurse is caring for an elderly client who exhibits signs of dementia. The most common cause of dementia in an elderly client is:

Alzheimer's disease.

ALS

Amyotrophic lateral sclerosis, also known as Lou Gehrig disease, is a rapidly progressive, fatal neurologic disorder resulting in destruction of motor neurons of the cortex, brain stem, and spinal cord.

Which of the following information will the nurse reinforce to the client scheduled for a lumbar puncture?

An informed consent will be required.

A client, age 21, is admitted with bacterial meningitis. Which hospital room would be the appropriate choice for this client?

An isolation room close to the nurses' station

3. A client with subdural hematoma was given mannitol to decrease intracranial pressure (ICP). Which of the following results would best show the mannitol was effective? 1. Urine output increases 2. Pupils are 8 mm and nonreactive 3. Systolic blood pressure remains at 150 mm Hg 4. BUN and creatinine levels return to normal

Answer: 1. Mannitol promotes osmotic diuresis by increasing the pressure gradient in the renal tubes. Fixed and dilated pupils are symptoms of increased ICP or cranial nerve damage. No information is given about abnormal BUN and creatinine levels or that mannitol is being given for renal dysfunction or blood pressure maintenance.

1. If a male client experienced a cerebrovascular accident (CVA) that damaged the hypothalamus, the nurse would anticipate that the client has problems with: A. body temperature control. B. balance and equilibrium. C. visual acuity. D. thinking and reasoning.

Answer A. The body's thermostat is located in the hypothalamus; therefore, injury to that area can cause problems of body temperature control. Balance and equilibrium problems are related to cerebellar damage. Visual acuity problems would occur following occipital or optic nerve injury. Thinking and reasoning problems are the result of injury to the cerebrum.

The client with a brain attack (stroke) has residual dysphagia. When a diet order is initiated, the nurse avoids doing which of the following? a. Giving the client thin liquids b. Thickening liquids to the consistency of oatmeal c. Placing food on the unaffected side of the mouth d. Allowing plenty of time for chewing and swallowing

Answer A. Before the client with dysphagia is started on a diet, the gag and swallow reflexes must have returned. The client is assisted with meals as needed and is given ample time to chew and swallow. Food is placed on the unaffected side of the mouth. Liquids are thickened to avoid aspiration.

A male client with Bell's palsy asks the nurse what has caused this problem. The nurse's response is based on an understanding that the cause is: a. Unknown, but possibly includes ischemia, viral infection, or an autoimmune problem b. Unknown, but possibly includes long-term tissue malnutrition and cellular hypoxia c. Primary genetic in origin, triggered by exposure to meningitis d. Primarily genetic in origin, triggered by exposure to neurotoxins

Answer A. Bell's palsy is a one-sided facial paralysis from compression of the facial nerve. The exact cause is unknown, but may include vascular ischemia, infection, exposure to viruses such as herpes zoster or herpes simplex, autoimmune disease, or a combination of these factors.

The nurse is working on a surgical floor. The nurse must logroll a male client following a: a. laminectomy. b. thoracotomy. c. hemorrhoidectomy. d. cystectomy.

Answer A. The client who has had spinal surgery, such as laminectomy, must be logrolled to keep the spinal column straight when turning. The client who has had a thoracotomy or cystectomy may turn himself or may be assisted into a comfortable position. Under normal circumstances, hemorrhoidectomy is an outpatient procedure, and the client may resume normal activities immediately after surgery.

A female client with Guillain-Barré syndrome has paralysis affecting the respiratory muscles and requires mechanical ventilation. When the client asks the nurse about the paralysis, how should the nurse respond? a. "You may have difficulty believing this, but the paralysis caused by this disease is temporary." b. "You'll have to accept the fact that you're permanently paralyzed. However, you won't have any sensory loss." c. "It must be hard to accept the permanency of your paralysis." d. "You'll first regain use of your legs and then your arms."

Answer A. The nurse should inform the client that the paralysis that accompanies Guillain-Barré syndrome is only temporary. Return of motor function begins proximally and extends distally in the legs.

6. A female client with a suspected brain tumor is scheduled for computed tomography (CT). What should the nurse do when preparing the client for this test? A. Immobilize the neck before the client is moved onto a stretcher. B. Determine whether the client is allergic to iodine, contrast dyes, or shellfish. C. Place a cap over the client's head. D. Administer a sedative as ordered.

Answer B. Because CT commonly involves use of a contrast agent, the nurse should determine whether the client is allergic to iodine, contrast dyes, or shellfish. Neck immobilization is necessary only if the client has a suspected spinal cord injury. Placing a cap over the client's head may lead to misinterpretation of test results; instead, the hair should be combed smoothly. The physician orders a sedative only if the client can't be expected to remain still during the CT

15. The nurse is positioning the female client with increased intracranial pressure. Which of the following positions would the nurse avoid? A. Head mildline B. Head turned to the side C. Neck in neutral position D. Head of bed elevated 30 to 45 degrees

Answer B. The head of the client with increased intracranial pressure should be positioned so the head is in a neutral midline position. The nurse should avoid flexing or extending the client's neck or turning the head side to side. The head of the bed should be raised to 30 to 45 degrees. Use of proper positions promotes venous drainage from the cranium to keep intracranial pressure down.

A female client with a suspected brain tumor is scheduled for computed tomography (CT). What should the nurse do when preparing the client for this test? a. Immobilize the neck before the client is moved onto a stretcher. b. Determine whether the client is allergic to iodine, contrast dyes, or shellfish. c. Place a cap over the client's head. d. Administer a sedative as ordered.

Answer B. Because CT commonly involves use of a contrast agent, the nurse should determine whether the client is allergic to iodine, contrast dyes, or shellfish. Neck immobilization is necessary only if the client has a suspected spinal cord injury. Placing a cap over the client's head may lead to misinterpretation of test results; instead, the hair should be combed smoothly. The physician orders a sedative only if the client can't be expected to remain still during the CT scan.

The nurse is teaching a female client with multiple sclerosis. When teaching the client how to reduce fatigue, the nurse should tell the client to: a. take a hot bath. b. rest in an air-conditioned room c. increase the dose of muscle relaxants. d. avoid naps during the day

Answer B. Fatigue is a common symptom in clients with multiple sclerosis. Lowering the body temperature by resting in an air-conditioned room may relieve fatigue; however, extreme cold should be avoided. A hot bath or shower can increase body temperature, producing fatigue. Muscle relaxants, prescribed to reduce spasticity, can cause drowsiness and fatigue. Planning for frequent rest periods and naps can relieve fatigue. Other measures to reduce fatigue in the client with multiple sclerosis include treating depression, using occupational therapy to learn energy conservation techniques, and reducing spasticity.

The nurse is assigned to care for a female client with complete right-sided hemiparesis. The nurse plans care knowing that this condition: a. The client has complete bilateral paralysis of the arms and legs. b. The client has weakness on the right side of the body, including the face and tongue. c. The client has lost the ability to move the right arm but is able to walk independently. d. The client has lost the ability to move the right arm but is able to walk independently.

Answer B. Hemiparesis is a weakness of one side of the body that may occur after a stroke. Complete hemiparesis is weakness of the face and tongue, arm, and leg on one side. Complete bilateral paralysis does not occur in this condition. The client with right-sided hemiparesis has weakness of the right arm and leg and needs assistance with feeding, bathing, and ambulating.

The nurse is assessing the motor function of an unconscious male client. The nurse would plan to use which plan to use which of the following to test the client's peripheral response to pain? a. Sternal rub b. Nail bed pressure c. Pressure on the orbital rim d. Squeezing of the sternocleidomastoid muscle

Answer B. Motor testing in the unconscious client can be done only by testing response to painful stimuli. Nail bed pressure tests a basic peripheral response. Cerebral responses to pain are tested using sternal rub, placing upward pressure on the orbital rim, or squeezing the clavicle or sternocleidomastoid muscle.

The nurse is caring for the male client who begins to experience seizure activity while in bed. Which of the following actions by the nurse would be contraindicated? a. Loosening restrictive clothing b. Restraining the client's limbs c. Removing the pillow and raising padded side rails d. Positioning the client to side, if possible, with the head flexed forward

Answer B. Nursing actions during a seizure include providing for privacy, loosening restrictive clothing, removing the pillow and raising side rails in the bed, and placing the client on one side with the head flexed forward, if possible, to allow the tongue to fall forward and facilitate drainage. The limbs are never restrained because the strong muscle contractions could cause the client harm. If the client is not in bed when seizure activity begins, the nurse lowers the client to the floor, if possible, protects the head from injury, and moves furniture that may injure the client. Other aspects of care are as described for the client who is in bed.

The nurse is positioning the female client with increased intracranial pressure. Which of the following positions would the nurse avoid? a. Head mildline b. Head turned to the side c. Neck in neutral position d. Head of bed elevated 30 to 45 degrees

Answer B. The head of the client with increased intracranial pressure should be positioned so the head is in a neutral midline position. The nurse should avoid flexing or extending the client's neck or turning the head side to side. The head of the bed should be raised to 30 to 45 degrees. Use of proper positions promotes venous drainage from the cranium to keep intracranial pressure down.

A female client has a neurological deficit involving the limbic system. Specific to this type of deficit, the nurse would document which of the following information related to the client's behavior. a. Is disoriented to person, place, and time b. Affect is flat, with periods of emotional lability c. Cannot recall what was eaten for breakfast today d. Demonstrate inability to add and subtract; does not know who is president

Answer B. The limbic system is responsible for feelings (affect) and emotions. Calculation ability and knowledge of current events relates to function of the frontal lobe. The cerebral hemispheres, with specific regional functions, control orientation. Recall of recent events is controlled by the hippocampus.

A female client with amyotrophic lateral sclerosis (ALS) tells the nurse, "Sometimes I feel so frustrated. I can't do anything without help!" This comment best supports which nursing diagnosis? a. Anxiety b. Powerlessness c. Ineffective denial d. Risk for disuse syndrome

Answer B. This comment best supports a nursing diagnosis of Powerlessness because ALS may lead to locked-in syndrome, characterized by an active and functioning mind locked in a body that can't perform even simple daily tasks. Although Anxiety and Risk for disuse syndrome may be diagnoses associated with ALS, the client's comment specifically refers to an inability to act autonomously. A diagnosis of Ineffective denial would be indicated if the client didn't seem to perceive the personal relevance of symptoms or danger.

During a routine physical examination to assess a male client's deep tendon reflexes, the nurse should make sure to: a. use the pointed end of the reflex hammer when striking the Achilles tendon. b. support the joint where the tendon is being tested. c. tap the tendon slowly and softly d. hold the reflex hammer tightly.

Answer B. To prevent the attached muscle from contracting, the nurse should support the joint where the tendon is being tested. The nurse should use the flat, not pointed, end of the reflex hammer when striking the Achilles tendon. (The pointed end is used to strike over small areas, such as the thumb placed over the biceps tendon.) Tapping the tendon slowly and softly wouldn't provoke a deep tendon reflex response. The nurse should hold the reflex hammer loosely, not tightly, between the thumb and fingers so it can swing in an arc.

1. A white female client is admitted to an acute care facility with a diagnosis of cerebrovascular accident (CVA). Her history reveals bronchial asthma, exogenous obesity, and iron deficiency anemia. Which history finding is a risk factor for CVA? A. Caucasian race B. Female sex C. Obesity D. Bronchial asthma

Answer C. Obesity is a risk factor for CVA. Other risk factors include a history of ischemic episodes, cardiovascular disease, diabetes mellitus, atherosclerosis of the cranial vessels, hypertension, polycythemia, smoking, hypercholesterolemia, oral contraceptive use, emotional stress, family history of CVA, and advancing age. The client's race, sex, and bronchial asthma aren't risk factors for CVA.

10. For a male client with suspected increased intracranial pressure (ICP), a most appropriate respiratory goal is to: A. prevent respiratory alkalosis. B. lower arterial pH. C. promote carbon dioxide elimination. D. maintain partial pressure of arterial oxygen (PaO2) above 80 mm Hg

Answer C. The goal of treatment is to prevent acidemia by eliminating carbon dioxide. That is because an acid environment in the brain causes cerebral vessels to dilate and therefore increases ICP. Preventing respiratory alkalosis and lowering arterial pH may bring about acidosis, an undesirable condition in this case. It isn't necessary to maintain a PaO2 as high as 80 mm Hg; 60 mm Hg will adequately oxygenate most clients.

Nurse Kristine is trying to communicate with a client with brain attack (stroke) and aphasia. Which of the following actions by the nurse would be least helpful to the client? a. Speaking to the client at a slower rate b. Allowing plenty of time for the client to respond c. Completing the sentences that the client cannot finish d. Looking directly at the client during attempts at speech

Answer C. Clients with aphasia after brain attack (stroke) often fatigue easily and have a short attention span. General guidelines when trying to communicate with the aphasic client include speaking more slowly and allowing adequate response time, listening to and watching attempts to communicate, and trying to put the client at ease with a caring and understanding manner. The nurse would avoid shouting (because the client is not deaf), appearing rushed for a response, and letting family members provide all the responses for the client.

Nurse Maureen witnesses a neighbor's husband sustain a fall from the roof of his house. The nurse rushes to the victim and determines the need to opens the airway in this victim by using which method? a. Flexed position b. Head tilt-chin lift c. Jaw thrust maneuver d. Modified head tilt-chin lift

Answer C. If a neck injury is suspected, the jaw thrust maneuver is used to open the airway. The head tilt-chin lift maneuver produces hyperextension of the neck and could cause complications if a neck injury is present. A flexed position is an inappropriate position for opening the airway.

A female client has experienced an episode of myasthenic crisis. The nurse would assess whether the client has precipitating factors such as: a. Getting too little exercise b. Taking excess medication c. Omitting doses of medication d. Increasing intake of fatty foods

Answer C. Myasthenic crisis often is caused by undermedication and responds to the administration of cholinergic medications, such as neostigmine (Prostigmin) and pyridostigmine (Mestinon). Cholinergic crisis (the opposite problem) is caused by excess medication and responds to withholding of medications. Too little exercise and fatty food intake are incorrect. Overexertion and overeating possibly could trigger myasthenic crisis.

A white female client is admitted to an acute care facility with a diagnosis of cerebrovascular accident (CVA). Her history reveals bronchial asthma, exogenous obesity, and iron deficiency anemia. Which history finding is a risk factor for CVA? a. Caucasian race b. Female sex c. Obesity d. Bronchial asthma

Answer C. Obesity is a risk factor for CVA. Other risk factors include a history of ischemic episodes, cardiovascular disease, diabetes mellitus, atherosclerosis of the cranial vessels, hypertension, polycythemia, smoking, hypercholesterolemia, oral contraceptive use, emotional stress, family history of CVA, and advancing age. The client's race, sex, and bronchial asthma aren't risk factors for CVA.

A female client admitted to the hospital with a neurological problem asks the nurse whether magnetic resonance imaging may be done. The nurse interprets that the client may be ineligible for this diagnostic procedure based on the client's history of: a. Hypertension b. Heart failure c. Prosthetic valve replacement d. Chronic obstructive pulmonary disorder

Answer C. The client having a magnetic resonance imaging scan has all metallic objects removed because of the magnetic field generated by the device. A careful history is obtained to determine whether any metal objects are inside the client, such as orthopedic hardware, pacemakers, artificial heart valves, aneurysm clips, or intrauterine devices. These may heat up, become dislodged, or malfunction during this procedure. The client may be ineligible if significant risk exists.

For a male client with suspected increased intracranial pressure (ICP), a most appropriate respiratory goal is to: a. prevent respiratory alkalosis. b. lower arterial pH. c. promote carbon dioxide elimination. d. maintain partial pressure of arterial oxygen (PaO2) above 80 mm Hg

Answer C. The goal of treatment is to prevent acidemia by eliminating carbon dioxide. That is because an acid environment in the brain causes cerebral vessels to dilate and therefore increases ICP. Preventing respiratory alkalosis and lowering arterial pH may bring about acidosis, an undesirable condition in this case. It isn't necessary to maintain a PaO2 as high as 80 mm Hg; 60 mm Hg will adequately oxygenate most clients.

8. A female client is admitted in a disoriented and restless state after sustaining a concussion during a car accident. Which nursing diagnosis takes highest priority in this client's plan of care? A. Disturbed sensory perception (visual) B. Self-care deficient: Dressing/grooming C. Impaired verbal communication D. Risk for injury

Answer D. Because the client is disoriented and restless, the most important nursing diagnosis is risk for injury. Although the other options may be appropriate, they're secondary because they don't immediately affect the client's health or safety.

21. The nurse is assessing the adaptation of the female client to changes in functional status after a brain attack (stroke). The nurse assesses that the client is adapting most successfully if the client: A. Gets angry with family if they interrupt a task B. Experiences bouts of depression and irritability C. Has difficulty with using modified feeding utensils D. Consistently uses adaptive equipment in dressing self

Answer D. Clients are evaluated as coping successfully with lifestyle changes after a brain attack (stroke) if they make appropriate lifestyle alterations, use the assistance of others, and have appropriate social interactions. Options A, B, and C are not adaptive behaviors.

30. During recovery from a cerebrovascular accident (CVA), a female client is given nothing by mouth, to help prevent aspiration. To determine when the client is ready for a liquid diet, the nurse assesses the client's swallowing ability once each shift. This assessment evaluates: A. cranial nerves I and II. B. cranial nerves III and V. C. cranial nerves VI and VIII. D. cranial nerves IX and X.

Answer D. Swallowing is a motor function of cranial nerves IX and X. Cranial nerves I, II, and VIII don't possess motor functions. The motor functions of cranial nerve III include extraocular eye movement, eyelid elevation, and pupil constriction. The motor function of cranial nerve V is chewing. Cranial nerve VI controls lateral eye movement.

29. The nurse is caring for a male client diagnosed with a cerebral aneurysm who reports a severe headache. Which action should the nurse perform? A. Sit with the client for a few minutes. B. Administer an analgesic. C. Inform the nurse manager. D. Call the physician immediately.

Answer D. The headache may be an indication that the aneurysm is leaking. The nurse should notify the physician immediately. Sitting with the client is appropriate but only after the physician has been notified of the change in the client's condition. The physician will decide whether or not administration of an analgesic is indicated. Informing the nurse manager isn't necessary.

A female client is admitted in a disoriented and restless state after sustaining a concussion during a car accident. Which nursing diagnosis takes highest priority in this client's plan of care? a. Disturbed sensory perception (visual) b. Self-care deficient: Dressing/grooming c. Impaired verbal communication d. Risk for injury

Answer D. Because the client is disoriented and restless, the most important nursing diagnosis is risk for injury. Although the other options may be appropriate, they're secondary because they don't immediately affect the client's health or safety.

The nurse is assessing the adaptation of the female client to changes in functional status after a brain attack (stroke). The nurse assesses that the client is adapting most successfully if the client: a. Gets angry with family if they interrupt a task b. Experiences bouts of depression and irritability c. Has difficulty with using modified feeding utensils d. Consistently uses adaptive equipment in dressing self

Answer D. Clients are evaluated as coping successfully with lifestyle changes after a brain attack (stroke) if they make appropriate lifestyle alterations, use the assistance of others, and have appropriate social interactions. Options A, B, and C are not adaptive behaviors.

The nurse is teaching the female client with myasthenia gravis about the prevention of myasthenic and cholinergic crises. The nurse tells the client that this is most effectively done by: a. Eating large, well-balanced meals b. Doing muscle-strengthening exercises c. Doing all chores early in the day while less fatigued d. Taking medications on time to maintain therapeutic blood levels

Answer D. Clients with myasthenia gravis are taught to space out activities over the day to conserve energy and restore muscle strength. Taking medications correctly to maintain blood levels that are not too low or too high is important. Muscle-strengthening exercises are not helpful and can fatigue the client. Overeating is a cause of exacerbation of symptoms, as is exposure to heat, crowds, erratic sleep habits, and emotional stress.

42. Which of the following respiratory patterns indicate increasing ICP in the brain stem? 1. Slow, irregular respirations 2. Rapid, shallow respirations 3. Asymmetric chest expansion 4. Nasal flaring

Answer: 1. Neural control of respiration takes place in the brain stem. Deterioration and pressure produce irregular respiratory patterns. Rapid, shallow respirations, asymmetric chest movements, and nasal flaring are more characteristic of respiratory distress or hypoxia.

The client diagnosed with hypothyroidism is prescribed the thyroid hormone levothyroxine. Which assessment data indicates the medication has been effective? A The client has a three (3)-pound weight gain. B The client has a decreased pulse rate. C The client's temperature increases to normal limits. D The client denies any diaphoresis.

C The client's temperature increases to normal limits.

A female client with Guillian-Barre syndrome has ascending paralysis and is intubated and receiving mechanical ventilation. Which of the following strategies would the nurse incorporate in the plan of care to help the client cope with this illness? a. Giving client full control over care decisions and restricting visitors b. Providing positive feedback and encouraging active range of motion c. Providing information, giving positive feedback, and encouraging relaxation d. Providing intravaneously administered sedatives, reducing distractions and limiting visitors

Answer D. Cranial nerve II is the optic nerve, which governs vision. The nurse can provide safety for the visually impaired client by clearing the path of obstacles when ambulating. Testing the shower water temperature would be useful if there were an impairment of peripheral nerves. Speaking loudly may help overcome a deficit of cranial nerve VIII (vestibulocochlear). Cranial nerve VII (facial) and IX (glossopharyngeal) control taste from the anterior two thirds and posterior third of the tongue, respectively

A male client has an impairment of cranial nerve II. Specific to this impairment, the nurse would plan to do which of the following to ensure client safety? a. Speak loudly to the client b. Test the temperature of the shower water c. Check the temperature of the food on the delivery tray. d. Provide a clear path for ambulation without obstacles

Answer D. Cranial nerve II is the optic nerve, which governs vision. The nurse can provide safety for the visually impaired client by clearing the path of obstacles when ambulating. Testing the shower water temperature would be useful if there were an impairment of peripheral nerves. Speaking loudly may help overcome a deficit of cranial nerve VIII (vestibulocochlear). Cranial nerve VII (facial) and IX (glossopharyngeal) control taste from the anterior two thirds and posterior third of the tongue, respectively

Female client is admitted to the hospital with a diagnosis of Guillain-Barre syndrome. The nurse inquires during the nursing admission interview if the client has history of: a. Seizures or trauma to the brain b. Meningitis during the last 5 years c. Back injury or trauma to the spinal cord d. Respiratory or gastrointestinal infection during the previous month.

Answer D. Guillain-Barré syndrome is a clinical syndrome of unknown origin that involves cranial and peripheral nerves. Many clients report a history of respiratory or gastrointestinal infection in the 1 to 4 weeks before the onset of neurological deficits. Occasionally, the syndrome can be triggered by vaccination or surgery.

A female client has clear fluid leaking from the nose following a basilar skull fracture. The nurse assesses that this is cerebrospinal fluid if the fluid: a. Is clear and tests negative for glucose b. Is grossly bloody in appearance and has a pH of 6 c. Clumps together on the dressing and has a pH of 7 d. Separates into concentric rings and test positive of glucose

Answer D. Leakage of cerebrospinal fluid (CSF) from the ears or nose may accompany basilar skull fracture. CSF can be distinguished from other body fluids because the drainage will separate into bloody and yellow concentric rings on dressing material, called a halo sign. The fluid also tests positive for glucose.

A male client is having a lumbar puncture performed. The nurse would plan to place the client in which position? a. Side-lying, with a pillow under the hip b. Prone, with a pillow under the abdomen c. Prone, in slight-Trendelenburg's position d. Side-lying, with the legs pulled up and head bent down onto chest.

Answer D. The client undergoing lumbar puncture is positioned lying on the side, with the legs pulled up to the abdomen and the head bent down onto the chest. This position helps open the spaces between the vertebrae.

A male client with a spinal cord injury is prone to experiencing automatic dysreflexia. The nurse would avoid which of the following measures to minimize the risk of recurrence? a. Strict adherence to a bowel retraining program b. Keeping the linen wrinkle-free under the client c. Preventing unnecessary pressure on the lower limbs d. Limiting bladder catheterization to once every 12 hours

Answer D. The most frequent cause of autonomic dysreflexia is a distended bladder. Straight catheterization should be done every 4 to 6 hours, and foley catheters should be checked frequently to prevent kinks in the tubing. Constipation and fecal impaction are other causes, so maintaining bowel regularity is important. Other causes include stimulation of the skin from tactile, thermal, or painful stimuli. The nurse administers care to minimize risk in these areas.

33. The client with a head injury has been urinating copious amounts of dilute urine through the Foley catheter. The client's urine output for the previous shift was 3000 ml. The nurse implements a new physician order to administer: 1. Desmopressin (DDAVP, stimate) 2. Dexamethasone (Decadron) 3. Ethacrynic acid (Edecrin) 4. Mannitol (Osmitrol)

Answer: 1. A complication of a head injury is diabetes insipidus, which can occur with insult to the hypothalamus, the antidiuretic storage vesicles, or the posterior pituitary gland. Urine output that exceeds 9 L per day generally requires treatment with desmopressin. Dexamethasone, a glucocorticoid, is administered to treat cerebral edema. This medication may be ordered for the head injured patient. Ethacrynic acid and mannitol are diuretics, which would be contraindicated.

1. Regular oral hygiene is an essential intervention for the client who has had a stroke. Which of the following nursing measures is inappropriate when providing oral hygiene? 1. Placing the client on the back with a small pillow under the head. 2. Keeping portable suctioning equipment at the bedside. 3. Opening the client's mouth with a padded tongue blade. 4. Cleaning the client's mouth and teeth with a toothbrush.

Answer: 1. A helpless client should be positioned on the side, not on the back. This lateral position helps secretions escape from the throat and mouth, minimizing the risk of aspiration. It may be necessary to suction, so having suction equipment at the bedside is necessary. Padded tongue blades are safe to use. A toothbrush is appropriate to use.

4. When evaluating an ABG from a client with a subdural hematoma, the nurse notes the PaCO2 is 30 mm Hg. Which of the following responses best describes this result? 1. Appropriate; lowering carbon dioxide (CO2) reduces intracranial pressure (ICP). 2. Emergent; the client is poorly oxygenated. 3. Normal 4. Significant; the client has alveolar hypoventilation.

Answer: 1. A normal PaCO2 value is 35 to 45 mm Hg. CO2 has vasodilating properties; therefore, lowering PaCO2 through hyperventilation will lower ICP caused by dilated cerebral vessels. Oxygenation is evaluated through PaO2 and oxygen saturation. Alveolar hypoventilation would be reflected in an increased PaCO2.

Grave's disease is: A The most common cause of hypothyroidism B The most common cause of hyperparathyroidism C The most common cause of hyperthyroidism D The most common cause of adrenal insufficiency

C The most common cause of hyperthyroidism

13. The nurse is evaluating the status of a client who had a craniotomy 3 days ago. The nurse would suspect the client is developing meningitis as a complication of surgery if the client exhibits: 1. A positive Brudzinski's sign 2. A negative Kernig's sign 3. Absence of nuchal rigidity 4. A Glascow Coma Scale score of 15

Answer: 1. Signs of meningeal irritation compatible with meningitis include nuchal rigidity, positive Brudzinski's sign, and positive Kernig's sign. Nuchal rigidity is characterized by a stiff neck and soreness, which is especially noticeable when the neck is fixed. Kernig's sign is positive when the client feels pain and spasm of the hamstring muscles when the knee and thigh are extended from a flexed-right angle position. Brudzinski's sign is positive when the client flexes the hips and knees in response to the nurse gently flexing the head and neck onto the chest. A Glascow Coma Scale of 15 is a perfect score and indicates the client is awake and alert with no neurological deficits.

1. An 18-year-old client is admitted with a closed head injury sustained in a MVA. His intracranial pressure (ICP) shows an upward trend. Which intervention should the nurse perform first? 1. Reposition the client to avoid neck flexion 2. Administer 1 g Mannitol IV as ordered 3. Increase the ventilator's respiratory rate to 20 breaths/minute 4. Administer 100 mg of pentobarbital IV as ordered.

Answer: 1. The nurse should first attempt nursing interventions, such as repositioning the client to avoid neck flexion, which increases venous return and lowers ICP. If nursing measures prove ineffective, notify the physician, who may prescribe mannitol, pentobarbital, or hyperventilation therapy.

7. The client diagnosed with atrial fibrillation has experienced a transient ischemic attack (TIA). Which medication would the nurse anticipate being ordered for the client on discharge? 1. An oral anticoagulant medication. 2. A beta-blocker medication. 3. An anti-hyperuricemic medication. 4. A thrombolytic medication.

Answer: 1. Thrombi form secondary to atrial fibrillation, therefore, an anticoagulant would be anticipated to prevent thrombi formation; and oral (warfarin [Coumadin]) at discharge verses intravenous. Beta blockers slow the heart rate and lower the blood pressure. Anti-hyperuricemic medication is given to clients with gout. Thrombolytic medication might have been given at initial presentation but would not be a drug prescribed at discharge.

3. A client with head trauma develops a urine output of 300 ml/hr, dry skin, and dry mucous membranes. Which of the following nursing interventions is the most appropriate to perform initially? 1. Evaluate urine specific gravity 2. Anticipate treatment for renal failure 3. Provide emollients to the skin to prevent breakdown 4. Slow down the IV fluids and notify the physician

Answer: 1. Urine output of 300 ml/hr may indicate diabetes insipidus, which is a failure of the pituitary to produce anti-diuretic hormone. This may occur with increased intracranial pressure and head trauma; the nurse evaluates for low urine specific gravity, increased serum osmolarity, and dehydration. There's no evidence that the client is experiencing renal failure. Providing emollients to prevent skin breakdown is important, but doesn't need to be performed immediately. Slowing the rate of IV fluid would contribute to dehydration when polyuria is present.

49. A client has been pronounced brain dead. Which findings would the nurse assess? Check all that apply. 1. Decerebrate posturing 2. Dilated nonreactive pupils 3. Deep tendon reflexes 4. Absent corneal reflex

Answer: 2, 3, 4. A client who is brain dead typically demonstrates nonreactive dilated pupils and nonreactive or absent corneal and gag reflexes. The client may still have spinal reflexes such as deep tendon and Babinski reflexes in brain death. Decerebrate or decorticate posturing would not be seen.

12. The nurse is caring for the client with increased intracranial pressure. The nurse would note which of the following trends in vital signs if the ICP is rising? 1. Increasing temperature, increasing pulse, increasing respirations, decreasing blood pressure. 2. Increasing temperature, decreasing pulse, decreasing respirations, increasing blood pressure. 3. Decreasing temperature, decreasing pulse, increasing respirations, decreasing blood pressure. 4. Decreasing temperature, increasing pulse, decreasing respirations, increasing blood pressure.

Answer: 2. A change in vital signs may be a late sign of increased intracranial pressure. Trends include increasing temperature and blood pressure and decreasing pulse and respirations. Respiratory irregularities also may arise.

16. A 23-year-old client has been hit on the head with a baseball bat. The nurse notes clear fluid draining from his ears and nose. Which of the following nursing interventions should be done first? 1. Position the client flat in bed 2. Check the fluid for dextrose with a dipstick 3. Suction the nose to maintain airway patency 4. Insert nasal and ear packing with sterile gauze

Answer: 2. Clear fluid from the nose or ear can be determined to be cerebral spinal fluid or mucous by the presence of dextrose. Placing the client flat in bed may increase ICP and promote pulmonary aspiration. The nose wouldn't be suctioned because of the risk for suctioning brain tissue through the sinuses. Nothing is inserted into the ears or nose of a client with a skull fracture because of the risk of infection.

2. A client with a subarachnoid hemorrhage is prescribed a 1,000-mg loading dose of Dilantin IV. Which consideration is most important when administering this dose? 1. Therapeutic drug levels should be maintained between 20 to 30 mg/ml. 2. Rapid Dilantin administration can cause cardiac arrhythmias. 3. Dilantin should be mixed in dextrose and water before administration. 4. Dilantin should be administered through an IV catheter in the client's hand.

Answer: 2. Dilantin IV shouldn't be given at a rate exceeding 50 mg/minute. Rapid administration can depress the myocardium, causing arrhythmias. Therapeutic drug levels range from 10 to 20 mg/ml. Dilantin shouldn't be mixed in solution for administration. However, because it's compatible with normal saline solution, it can be injected through an IV line containing normal saline. When given through an IV catheter hand, Dilantin may cause purple glove syndrome.

5. A client who had a transsphenoidal hypophysectomy should be watched carefully for hemorrhage, which may be shown by which of the following signs? 1. Bloody drainage from the ears 2. Frequent swallowing 3. Guaiac-positive stools 4. Hematuria

Answer: 2. Frequent swallowing after brain surgery may indicate fluid or blood leaking from the sinuses into the oropharynx. Blood or fluid draining from the ear may indicate a basilar skull fracture.

Anesthesia (sensory system)

absence of sensation lesions in spinal cord, thalamus, sensory cortex, or peripheral sensory nerves specific medications

5. What is a priority nursing assessment in the first 24 hours after admission of the client with a thrombotic stroke? 1. Cholesterol level 2. Pupil size and pupillary response 3. Bowel sounds 4. Echocardiogram

Answer: 2. It is crucial to monitor the pupil size and pupillary response to indicate changes around the cranial nerves. Cholesterol level is an assessment to be addressed for long-term healthy lifestyle rehabilitation. Bowel sounds need to be assessed because an ileus or constipation can develop, but is not a priority in the first 24 hours. An echocardiogram is not needed for the client with a thrombotic stroke.

2. A 78 year old client is admitted to the emergency department with numbness and weakness of the left arm and slurred speech. Which nursing intervention is priority? 1. Prepare to administer recombinant tissue plasminogen activator (rt-PA). 2. Discuss the precipitating factors that caused the symptoms. 3. Schedule for A STAT computer tomography (CT) scan of the head. 4. Notify the speech pathologist for an emergency consult.

Answer: 3. A CT scan will determine if the client is having a stroke or has a brain tumor or another neurological disorder. This would also determine if it is a hemorrhagic or ischemic accident and guide the treatment, because only an ischemic stroke can use rt-PA. This would make (1) not the priority since if a stroke was determined to be hemorrhagic, rt-PA is contraindicated. Discuss the precipitating factors for teaching would not be a priority and slurred speech would as indicate interference for teaching. Referring the client for speech therapy would be an intervention after the CVA emergency treatment is administered according to protocol.

7. A client comes into the ER after hitting his head in an MVA. He's alert and oriented. Which of the following nursing interventions should be done first? 1. Assess full ROM to determine extent of injuries 2. Call for an immediate chest x-ray 3. Immobilize the client's head and neck 4. Open the airway with the head-tilt chin-lift maneuver

Answer: 3. All clients with a head injury are treated as if a cervical spine injury is present until x-rays confirm their absence. ROM would be contraindicated at this time. There is no indication that the client needs a chest x-ray. The airway doesn't need to be opened since the client appears alert and not in respiratory distress. In addition, the head-tilt chin-lift maneuver wouldn't be used until the cervical spine injury is ruled out.

26. An 18-year-old client was hit in the head with a baseball during practice. When discharging him to the care of his mother, the nurse gives which of the following instructions? 1. "Watch him for keyhole pupil the next 24 hours." 2. "Expect profuse vomiting for 24 hours after the injury." 3. "Wake him every hour and assess his orientation to person, time, and place." 4. "Notify the physician immediately if he has a headache."

Answer: 3. Changes in LOC may indicate expanding lesions such as subdural hematoma; orientation and LOC are assessed frequently for 24 hours. A keyhole pupil is found after iridectomy. Profuse or projectile vomiting is a symptom of increased ICP and should be reported immediately. A slight headache may last for several days after concussion; severe or worsening headaches should be reported.

4. During the first 24 hours after thrombolytic therapy for ischemic stroke, the primary goal is to control the client's: 1. Pulse 2. Respirations 3. Blood pressure 4. Temperature

Answer: 3. Controlling the blood pressure is critical because an intracerebral hemorrhage is the major adverse effect of thrombolytic therapy. Blood pressure should be maintained according to physician and is specific to the client's ischemic tissue needs and risks of bleeding from treatment. Other vital signs are monitored, but the priority is blood pressure

2. A client with a subdural hematoma becomes restless and confused, with dilation of the ipsilateral pupil. The physician orders mannitol for which of the following reasons? 1. To reduce intraocular pressure 2. To prevent acute tubular necrosis 3. To promote osmotic diuresis to decrease ICP 4. To draw water into the vascular system to increase blood pressure

Answer: 3. Mannitol promotes osmotic diuresis by increasing the pressure gradient, drawing fluid from intracellular to intravascular spaces. Although mannitol is used for all the reasons described, the reduction of ICP in this client is a concern.

43. Which of the following nursing interventions is appropriate for a client with an ICP of 20 mm Hg? 1. Give the client a warming blanket 2. Administer low-dose barbiturate 3. Encourage the client to hyperventilate 4. Restrict fluids

Answer: 3. Normal ICP is 15 mm Hg or less. Hyperventilation causes vasoconstriction, which reduces CSF and blood volume, two important factors for reducing a sustained ICP of 20 mm Hg. A cooling blanket is used to control the elevation of temperature because a fever increases the metabolic rate, which in turn increases ICP. High doses of barbiturates may be used to reduce the increased cellular metabolic demands. Fluid volume and inotropic drugs are used to maintain cerebral perfusion by supporting the cardiac output and keeping the cerebral perfusion pressure greater than 80 mm Hg.

6. Which of the following signs and symptoms of increased ICP after head trauma would appear first? 1. Bradycardia 2. Large amounts of very dilute urine 3. Restlessness and confusion 4. Widened pulse pressure

Answer: 3. The earliest symptom of elevated ICP is a change in mental status. Bradycardia, widened pulse pressure, and bradypnea occur later. The client may void large amounts of very dilute urine if there's damage to the posterior pituitary.

3. A client arrives in the emergency department with an ischemic stroke and receives tissue plasminogen activator (t-PA) administration. Which is the priority nursing assessment? 1. Current medications. 2. Complete physical and history. 3. Time of onset of current stroke. 4. Upcoming surgical procedures.

Answer: 3. The time of onset of a stroke to t-PA administration is critical. Administration within 3 hours has better outcomes. A complete history is not possible in emergency care. Upcoming surgical procedures will need to be delay if t-PA is administered. Current medications are relevant, but onset of current stroke takes priority.

6. What is the expected outcome of thrombolytic drug therapy? 1. Increased vascular permeability. 2. Vasoconstriction. 3. Dissolved emboli. 4. Prevention of hemorrhage

Answer: 3. Thrombolytic therapy is use to dissolve emboli and reestablish cerebral perfusion.

A 23-year-old client has been hit on the head with a baseball bat. The nurse notes clear fluid draining from his ears and nose. Which of the following nursing interventions should be done first? A Position the client flat in bed B Check the fluid for dextrose with a dipstick C Suction the nose to maintain airway patency D Insert nasal and ear packing with sterile gauze

B Check the fluid for dextrose with a dipstick

46. Which of the following describes decerebrate posturing? 1. Internal rotation and adduction of arms with flexion of elbows, wrists, and fingers 2. Back hunched over, rigid flexion of all four extremities with supination of arms and plantar flexion of the feet 3. Supination of arms, dorsiflexion of feet 4. Back arched; rigid extension of all four extremities.

Answer: 4. Decerebrate posturing occurs in patients with damage to the upper brain stem, midbrain, or pons and is demonstrated clinically by arching of the back, rigid extension of the extremities, pronation of the arms, and plantar flexion of the feet. Internal rotation and adduction of arms with flexion of the elbows, wrists, and fingers described decorticate posturing, which indicates damage to corticospinal tracts and cerebral hemispheres.

44. A client has signs of increased ICP. Which of the following is an early indicator of deterioration in the client's condition? 1. Widening pulse pressure 2. Decrease in the pulse rate 3. Dilated, fixed pupil 4. Decrease in LOC

Answer: 4. A decrease in the client's LOC is an early indicator of deterioration of the client's neurological status. Changes in LOC, such as restlessness and irritability, may be subtle. Widening of the pulse pressure, decrease in the pulse rate, and dilated, fixed pupils occur later if the increased ICP is not treated.

6. After a hypophysectomy, vasopressin is given IM for which of the following reasons? 1. To treat growth failure 2. To prevent syndrome of inappropriate antidiuretic hormone (SIADH) 3. To reduce cerebral edema and lower intracranial pressure 4. To replace antidiuretic hormone (ADH) normally secreted by the pituitary.

Answer: 4. After hypophysectomy, or removal of the pituitary gland, the body can't synthesize ADH. Somatropin or growth hormone, not vasopressin is used to treat growth failure. SIADH results from excessive ADH secretion. Mannitol or corticosteroids are used to decrease cerebral edema.

26. The nurse has given the male client with Bell's palsy instructions on preserving muscle tone in the face and preventing denervation. The nurse determines that the client needs additional information if the client states that he or she will: A. Exposure to cold and drafts B. Massage the face with a gentle upward motion C. Perform facial exercises D. Wrinkle the forehead, blow out the cheeks, and whistle

Answer: A. Exposure to cold and drafts Exposure to cold or drafts is avoided. Local application of heat to the face may improve blood flow and provide comfort.

20. The client with a brain attack (stroke) has residual dysphagia. When a diet order is initiated, the nurse avoids doing which of the following? A. Giving the client thin liquids B. Thickening liquids to the consistency of oatmeal C. Placing food on the unaffected side of the mouth D. Allowing plenty of time for chewing and swallowing

Answer: A. Giving the client thin liquids Before the client with dysphagia is started on a diet, the gag and swallow reflexes must have returned.

4. When obtaining the health history from a male client with retinal detachment, the nurse expects the client to report: A. Light flashes and floaters in front of the eye. B. A recent driving accident while changing lanes. C. Headaches, nausea, and redness of the eyes. D. Frequent episodes of double vision.

Answer: A. Light flashes and floaters in front of the eye. The sudden appearance of light flashes and floaters in front of the affected eye is characteristic of retinal detachment.

6. To encourage adequate nutritional intake for a female client with Alzheimer's disease, the nurse should: A. Stay with the client and encourage him to eat. B. Help the client fill out his menu. C. Give the client privacy during meals. D. Fill out the menu for the client.

Answer: A. Stay with the client and encourage him to eat. Staying with the client and encouraging him to feed himself will ensure adequate food intake. A client with Alzheimer's disease can forget how to eat.

21. The nurse is monitoring a male client for adverse reactions to atropine sulfate (Atropine Care) eyedrops. Systemic absorption of atropine sulfate through the conjunctiva can cause which adverse reaction? A. Tachycardia B. Increased salivation C. Hypotension D. Apnea

Answer: A. Tachycardia Systemic absorption of atropine sulfate can cause tachycardia, palpitations, flushing, dry skin, ataxia, and confusion. To minimize systemic absorption, the client should apply digital pressure over the punctum at the inner canthus for 2 to 3 minutes after instilling the drops.

11. An auto mechanic accidentally has battery acid splashed in his eyes. His coworkers irrigate his eyes with water for 20 minutes, and then take him to the emergency department of a nearby hospital, where he receives emergency care for the corneal injury. The physician prescribes dexamethasone (Maxidex Ophthalmic Suspension), two drops of 0.1% solution to be instilled initially into the conjunctival sacs of both eyes every hour; and polymyxin B sulfate (Neosporin Ophthalmic), 0.5% ointment to be placed in the conjunctival sacs of both eyes every 3 hours. Dexamethasone exerts its therapeutic effect by: A. Increasing the exudative reaction of ocular tissue. B. Decreasing leukocyte infiltration at the site of ocular inflammation. C. Inhibiting the action of carbonic anhydrase. D. Producing a miotic reaction by stimulating and contracting the sphincter muscles of the iris.

Answer: B. Decreasing leukocyte infiltration at the site of ocular inflammation. Dexamethasone exerts its therapeutic effect by decreasing leukocyte infiltration at the site of ocular inflammation. This reduces the exudative reaction of diseased tissue, lessening edema, redness, and scarring.

6. A female client with a suspected brain tumor is scheduled for computed tomography (CT). What should the nurse do when preparing the client for this test? A. Immobilize the neck before the client is moved onto a stretcher. B. Determine whether the client is allergic to iodine, contrast dyes, or shellfish. C. Place a cap on the client's head. D. Administer a sedative as ordered.

Answer: B. Determine whether the client is allergic to iodine, contrast dyes, or shellfish. Because CT commonly involves the use of a contrast agent, the nurse should determine whether the client is allergic to iodine, contrast dyes, or shellfish.

15. The nurse is positioning the female client with increased intracranial pressure. Which of the following positions would the nurse avoid? A. Head midline B. Head turned to the side C. Neck in neutral position D. Head of bed elevated 30 to 45 degrees

Answer: B. Head turned to the side The head of the client with increased intracranial pressure should be positioned so the head is in a neutral midline position. The nurse should avoid flexing or extending the client's neck or turning the head side to side. The head of the bed should be raised to 30 to 45 degrees. Use of proper positions promotes venous drainage from the cranium to keep intracranial pressure down.

8. Shortly after admission to an acute care facility, a male client with a seizure disorder develops status epilepticus. The physician orders diazepam (Valium) 10 mg I.V. stat. How soon can the nurse administer the second dose of diazepam, if needed and prescribed? A. In 30 to 45 seconds B. In 10 to 15 minutes C. In 30 to 45 minutes D. In 1 to 2 hours

Answer: B. In 10 to 15 minutes When used to treat status epilepticus, diazepam may be given every 10 to 15 minutes, as needed, to a maximum dose of 30 mg. The nurse can repeat the regimen in 2 to 4 hours, if necessary, but the total dose shouldn't exceed 100 mg in 24 hours.

5. Which nursing diagnosis takes highest priority for a client with Parkinson's crisis? A. Imbalanced nutrition: Less than body requirements B. Ineffective airway clearance C. Impaired urinary elimination D. Risk for injury

Answer: B. Ineffective airway clearance In Parkinson's crisis, dopamine-related symptoms are severely exacerbated, virtually immobilizing the client. A client confined to bed during such a crisis is at risk for aspiration and pneumonia. Also, excessive drooling increases the risk of airway obstruction. Because of these concerns, the nursing diagnosis of Ineffective airway clearance takes highest priority. Although the other options also are appropriate, they aren't immediately life-threatening.

22. A male client is admitted with a cervical spine injury sustained during a diving accident. When planning this client's care, the nurse should assign the highest priority to which nursing diagnosis? A. Impaired physical mobility B. Ineffective breathing pattern C. Disturbed sensory perception (tactile) D. Self-care deficit: Dressing/grooming

Answer: B. Ineffective breathing pattern Because a cervical spine injury can cause respiratory distress, the nurse should take immediate action to maintain a patent airway and provide adequate oxygenation.

18. The nurse is caring for the male client who begins to experience seizure activity while in beD. Which of the following actions by the nurse would be contraindicated? A. Loosening restrictive clothing B. Restraining the client's limbs C. Removing the pillow and raising padded side rails D. Positioning the client to side, if possible, with the head flexed forward

Answer: B. Restraining the client's limbs The limbs are never restrained because the strong muscle contractions could cause the client harm. If the client is not in bed when seizure activity begins, the nurse lowers the client to the floor, if possible, protects the head from injury, and moves furniture that may injure the client. Other aspects of care are as described for the client who is in bed.

7. During a routine physical examination to assess a male client's deep tendon reflexes, the nurse should make sure to: A. Use the pointed end of the reflex hammer when striking the Achilles' tendon. B. Support the joint where the tendon is being tested. C. Tap the tendon slowly and softly D. Hold the reflex hammer tightly.

Answer: B. Support the joint where the tendon is being tested. To prevent the attached muscle from contracting, the nurse should support the joint where the tendon is being tested.

19. The nurse is assigned to care for a female client with complete right-sided hemiparesis. The nurse plans care knowing that this condition: A. The client has complete bilateral paralysis of the arms and legs. B. The client has weakness on the right side of the body, including the face and tongue. C. The client has lost the ability to move the right arm but can walk independently. D. The client has lost the ability to move the right arm but can walk independently.

Answer: B. The client has weakness on the right side of the body, including the face and tongue. Hemiparesis is a weakness of one side of the body that may occur after a stroke. Complete hemiparesis is a weakness of the face and tongue, arm, and leg on one side. Complete bilateral paralysis does not occur in this condition.

14. A female client who's paralyzed on the left side has been receiving physical therapy and attending teaching sessions about safety. Which behavior indicates that the client accurately understands safety measures related to paralysis? A. The client leaves the side rails down. B. The client uses a mirror to inspect the skin. C. The client repositions only after being reminded to do so. D. The client hangs the left arm over the side of the wheelchair.

Answer: B. The client uses a mirror to inspect the skin. Using a mirror enables the client to inspect all areas of the skin for signs of breakdown without the help of staff or family members.

2. A female client admitted to an acute care facility after a car accident develops signs and symptoms of increased intracranial pressure (ICP). The client is intubated and placed on mechanical ventilation to help reduce ICP. To prevent a further rise in ICP caused by suctioning, the nurse anticipates administering which drug endotracheally before suctioning? A. Phenytoin (Dilantin) B. Mannitol (Osmitrol) C. Lidocaine (Xylocaine) D. Furosemide (Lasix)

Answer: C. Lidocaine (Xylocaine) Administering lidocaine via an endotracheal tube may minimize elevations in ICP caused by suctioning.

27. A female client who was trapped inside a car for hours after a head-on collision is rushed to the emergency department with multiple injuries. During the neurologic examination, the client responds to painful stimuli with decerebrate posturing. This finding indicates damage to which part of the brain? A. Diencephalon B. Medulla C. Midbrain D. Cortex

Answer: C. Midbrain Decerebrate posturing, characterized by abnormal extension in response to painful stimuli, indicates damage to the midbrain.

20. Nurse Marty is monitoring a client for adverse reactions to dantrolene (Dantrium). Which adverse reaction is most common? A. Excessive tearing B. Urine retention C. Muscle weakness D. Slurred speech

Answer: C. Muscle weakness The most common adverse reaction to dantrolene is muscle weakness. The drug also may depress liver function or cause idiosyncratic hepatitis.

13. A female client admitted to the hospital with a neurological problem asks the nurse whether magnetic resonance imaging may be done. The nurse interprets that the client may be ineligible for this diagnostic procedure based on the client's history of: A. Hypertension B. Heart failure C. Prosthetic valve replacement D. Chronic obstructive pulmonary disorder

Answer: C. Prosthetic valve replacement The client having a magnetic resonance imaging scan has all metallic objects removed because of the magnetic field generated by the device. A careful history is obtained to determine whether any metal objects are inside the client, such as orthopedic hardware, pacemakers, artificial heart valves, aneurysm clips, or intrauterine devices. These may heat up, become dislodged, or malfunction during this procedure. The client may be ineligible if a significant risk exists.

28. A female client with Guillain-Barre syndrome has ascending paralysis and is intubated and receiving mechanical ventilation. Which of the following strategies would the nurse incorporate in the plan of care to help the client cope with this illness? A. Giving client full control over care decisions and restricting visitors B. Providing positive feedback and encouraging active range of motion C. Providing information, giving positive feedback and encouraging relaxation D. Providing intravenously administered sedatives, reducing distractions and limiting visitors

Answer: C. Providing information, giving positive feedback, and encouraging relaxation The client with Guillain-Barré syndrome experiences fear and anxiety from the ascending paralysis and sudden onset of the disorder. The nurse can alleviate these fears by providing accurate information about the client's condition, giving expert care and positive feedback to the client, and encouraging relaxation and distraction. The family can become involved with selected care activities and provide diversion for the client as well.

24. A female client who was found unconscious at home is brought to the hospital by a rescue squaD. In the intensive care unit, the nurse checks the client's oculocephalic (doll's eye) response by: A. Introducing ice water into the external auditory canal. B. Touching the cornea with a wisp of cotton. C. Turning the client's head suddenly while holding the eyelids open. D. Shining a bright light into the pupil.

Answer: C. Turning the client's head suddenly while holding the eyelids open. To elicit the oculocephalic response, which detects cranial nerve compression, the nurse turns the client's head suddenly while holding the eyelids open. Normally, the eyes move from side to side when the head is turned; in an abnormal response, the eyes remain fixed.

12. Nurse Amber is caring for a client who underwent a lumbar laminectomy two (2) days ago. Which of the following findings should the nurse consider abnormal? A. More back pain than the first postoperative day B. Paresthesia in the dermatomes near the wounds C. Urine retention or incontinence D. Temperature of 99.2° F (37.3° C)

Answer: C. Urine retention or incontinence Urine retention or incontinence may indicate cauda equina syndrome, which requires immediate surgery.

29. The nurse is caring for a male client diagnosed with a cerebral aneurysm who reports a severe headache. Which action should the nurse perform? A. Sit with the client for a few minutes. B. Administer an analgesic. C. Inform the nurse manager. D. Call the physician immediately.

Answer: D. Call the physician immediately. A headache may be an indication that an aneurysm is leaking. The nurse should notify the physician immediately.

9. A female client complains of periorbital aching, tearing, blurred vision, and photophobia in her right eye. Ophthalmologic examination reveals a small, irregular, nonreactive pupil — a condition resulting from acute iris inflammation (iritis). As part of the client's therapeutic regimen, the physician prescribes atropine sulfate (Atropisol), two drops of 0.5% solution in the right eye twice daily. Atropine sulfate belongs to which drug classification? A. Parasympathomimetic agent B. Sympatholytic agent C. Adrenergic blocker D. Cholinergic blocker

Answer: D. Cholinergic blocker Atropine sulfate is a cholinergic blocker. It isn't a parasympathomimetic agent, a sympatholytic agent, or an adrenergic blocker.

21. The nurse is assessing the adaptation of the female client to changes in functional status after a brain attack (stroke). The nurse assesses that the client is adapting most successfully if the client: A. Gets angry with family if they interrupt a task B. Experiences bouts of depression and irritability C. Has difficulty with using modified feeding utensils D. Consistently uses adaptive equipment in dressing self

Answer: D. Consistently uses adaptive equipment in dressing self Clients are evaluated as coping successfully with lifestyle changes after a brain attack (stroke) if they make appropriate lifestyle alterations, use the assistance of others, and have appropriate social interactions.

30. During recovery from a cerebrovascular accident (CVA), a female client is given nothing by mouth, to help prevent aspiration. To determine when the client is ready for a liquid diet, the nurse assesses the client's swallowing ability once each shift. This assessment evaluates: A. Cranial nerves I and II. B. Cranial nerves III and V. C. Cranial nerves VI and VIII. D. Cranial nerves IX and X.

Answer: D. Cranial nerves IX and X. Swallowing is a motor function of cranial nerves IX and X.

17. A male client with a spinal cord injury is prone to experiencing automatic dysreflexia. The nurse would avoid which of the following measures to minimize the risk of recurrence? A. Strict adherence to a bowel retraining program B. Keeping the linen wrinkle-free under the client C. Preventing unnecessary pressure on the lower limbs D. Limiting bladder catheterization to once every 12 hours

Answer: D. Limiting bladder catheterization to once every 12 hours The most frequent cause of autonomic dysreflexia is a distended bladder. Straight catheterization should be done every four (4) to six (6) hours, and foley catheters should be checked frequently to prevent kinks in the tubing. Other causes include stimulation of the skin from tactile, thermal, or painful stimuli. The nurse administers care to minimize risk in these areas.

25. While reviewing a client's chart, the nurse notices that the female client has myasthenia gravis. Which of the following statements about neuromuscular blocking agents is true for a client with this condition? A. The client may be less sensitive to the effects of a neuromuscular blocking agent. B. Succinylcholine shouldn't be used; pancuronium may be used in a lower dosage. C. Pancuronium shouldn't be used; succinylcholine may be used in a lower dosage. D. Pancuronium and succinylcholine both require cautious administration.

Answer: D. Pancuronium and succinylcholine both require cautious administration. The nurse must cautiously administer pancuronium, succinylcholine, and any other neuromuscular blocking agent to a client with myasthenia gravis.

29. A male client has an impairment of cranial nerve II. Specific to this impairment, the nurse would plan to do which of the following to ensure client to ensure client safety? A. Speak loudly to the client B. Test the temperature of the shower water C. Check the temperature of the food on the delivery tray. D. Provide a clear path for ambulation without obstacles

Answer: D. Provide a clear path for ambulation without obstacles Cranial nerve II is the optic nerve, which governs vision. The nurse can provide safety for the visually impaired client by clearing the path of obstacles when ambulating.

For a patient who has chronic pancreatitis, which nursing intervention would be most helpful? A Allowing liberalized fluid intake B Counseling to stop alcohol consumption C Encouraging daily exercise D Modifying dietary protein

B Counseling to stop alcohol consumption

27. A female client is admitted to the hospital with a diagnosis of Guillain-Barre syndrome. The nurse inquires during the nursing admission interview if the client has a history of: A. Seizures or trauma to the brain B. Meningitis during the last five (5 years C. Back injury or trauma to the spinal cord D. Respiratory or gastrointestinal infection during the previous month.

Answer: D. Respiratory or gastrointestinal infection during the previous month. Guillain-Barré syndrome is a clinical syndrome of unknown origin that involves cranial and peripheral nerves. Many clients report a history of respiratory or gastrointestinal infection in the 1 to 4 weeks before the onset of neurological deficits. Occasionally, the syndrome can be triggered by vaccination or surgery.

8. A female client is admitted in a disoriented and restless state after sustaining a concussion during a car accident. Which nursing diagnosis takes highest priority for this client's plan of care? A. Disturbed sensory perception (visual) B. Self-care deficient: Dressing/grooming C. Impaired verbal communication D. Risk for injury

Answer: D. Risk for injury Because the client is disoriented and restless, the most important nursing diagnosis is risk for injury.

16. A female client has clear fluid leaking from the nose following a basilar skull fracture. The nurse assesses that this is cerebrospinal fluid if the fluid: A. Is clear and tests negative for glucose B. Is grossly bloody in appearance and has a pH of 6 C. Clumps together on the dressing and has a pH of 7 D. Separates into concentric rings and test positive of glucose

Answer: D. Separates into concentric rings and test positive of glucose Leakage of cerebrospinal fluid (CSF) from the ears or nose may accompany basilar skull fracture. CSF can be distinguished from other body fluids because the drainage will separate into bloody and yellow concentric rings on dressing material, called a halo sign. The fluid also tests positive for glucose.

14. A male client is having a lumbar puncture performed. The nurse would plan to place the client in which position? A. Side-lying, with a pillow under the hip B. Prone, with a pillow under the abdomen C. Prone, in slight-Trendelenburg's position D. Side-lying, with the legs, pulled up and head bent down onto the chest.

Answer: D. Side-lying, with the legs, pulled up and head bent down onto the chest. The client undergoing lumbar puncture is positioned lying on the side, with the legs pulled up to the abdomen and the head bent down onto the chest. This position helps open the spaces between the vertebrae.

3. A male client is having tonic-clonic seizures. What should the nurse do first? A. Elevate the head of the bed. B. Restrain the client's arms and legs. C. Place a tongue blade in the client's mouth. D. Take measures to prevent injury.

Answer: D. Take measures to prevent injury. Protecting the client from injury is the immediate priority during a seizure.

24. The nurse is teaching the female client with myasthenia gravis about the prevention of myasthenic and cholinergic crises. The nurse tells the client that this is most effectively done by: A. Eating large, well-balanced meals B. Doing muscle-strengthening exercises C. Doing all chores early in the day while less fatigued D. Taking medications on time to maintain therapeutic blood levels

Answer: D. Taking medications on time to maintain therapeutic blood levels Taking medications correctly to maintain blood levels that are not too low or too high is important.

23. A male client has a history of painful, continuous muscle spasms. He has taken several skeletal muscle relaxants without experiencing relief. His physician prescribes diazepam (Valium), two (2) mg P.O. twice daily. In addition to being used to relieve painful muscle spasms, Diazepam also is recommended for: A. long-term treatment of epilepsy. B. postoperative pain management of laminectomy clients. C. postoperative pain management of diskectomy clients D. treatment of spasticity associated with spinal cord lesions.

Answer: D. Treatment of spasticity associated with spinal cord lesions. In addition to relieving painful muscle spasms, Diazepam also is recommended for treatment of spasticity associated with spinal cord lesions. Diazepam's use is limited by its central nervous system effects and the tolerance that develops with prolonged use.

Cranial nerve XI

accessory nerve

Following an eye examination, a client is given the following prescription: "pilocarpine ophthalmic solution, 0.25%, 1 gtt both eyes qid." Which statements indicate the correct interpretation and procedure for administering the medication? Select all that apply.

Apply pressure to the inner corner of eye for 1 to 2 minutes after application to prevent systemic absorption. Instill one drop of pilocarpine solution into each eye every 4 hours each day. Wash hands before and after instilling eye drops and do not allow dropper to touch the eye or any other surfaces.

The nurse is taking care of a client with a headache. In addition to administering medications, the nurse takes which measure to assist the client in reducing the pain associated with the headache?

Apply warm or cool cloths to the forehead or back of the neck.

The nurse is taking care of a client with a headache. In addition to administering medications, the nurse takes measures to assist the client in reducing the pain associated with his headache. Which of the following appropriate nursing interventions may be provided by the nurse to assist this client in reducing or eliminating his pain?

Apply warm or cool cloths to the forehead or back of the neck. Warmth promotes vasodilation; cool stimuli reduce blood flow.

A nurse is monitoring a client with a spinal cord injury for signs of spinal shock. Which of the following is indicative of this complication of a spinal cord injury?

Areflexia below the level of injury

A nurse is caring for a group of clients on the neurologic unit. Which task should the nurse perform first?

Arrange an escort for a client who needs to go to the physical therapy department.

The nurse is reviewing the medication administration record of a female client who possesses numerous risk factors for stroke. Which of the woman's medications carries the greatest potential for reducing her risk of stroke?

Aspirin 81 mg PO o.d.

The nurse is caring for a client who is known to be at risk for cardiogenic embolic strokes. What dysrhythmia does this client most likely have?

Atrial fibrillation

What laboratory finding is the primary diagnostic indicator for pancreatitis? A Elevated blood urea nitrogen (BUN) B Elevated serum lipase C Elevated aspartate aminotransferase (AST) D Increased lactate dehydrogenase (LD)

B Elevated serum lipase

A client with a history of epilepsy is admitted to the medical-surgical unit. While assisting the client from the bathroom, the nurse observes the start of a tonic-clonic seizure. Which nursing interventions are appropriate for this client? Select all that apply.

Assist the client to the floor. Turn the client to the side. Place a pillow under the client's head.

Which of the following clients on the rehab unit is most likely to develop autonomic dysreflexia? 1.A client with a brain injury 2.A client with a herniated nucleus pulposus 3.A client with a high cervical spine injury 4.A client with a stroke

Autonomic dysreflexia refers to uninhibited sympathetic outflow in clients with spinal cord injuries about the level of T10. The other clients aren't prone to dysreflexia.

For a client who has had a stroke, which nursing intervention can help prevent contractures in the lower legs?

Attaching braces or splints to each foot and leg

On the 5th postoperative day, a client who underwent spinal fusion begins to complain of nausea and has an episode of vomiting. How should a nurse intervene?

Auscultate the abdomen for bowel sounds.

autonomic dysreflexia

Autonomic dysreflexia (also known as autonomic hyperreflexia), an exaggerated response to stimuli below the level of the lesion in clients with lesions at or above T6, is a medical emergency that requires prompt treatment.

Which of the following interventions describes an appropriate bladder program for a client in rehabilitation for spinal cord injury? A Insert an indwelling urinary catheter to straight drainage B Schedule intermittent catheterization every 2 to 4 hours C Perform a straight catheterization every 8 hours while awake D Perform Crede's maneuver to the lower abdomen before the client voids.

B Intermittent catherization should begin every 2 to 4 hours early in the treatment. When residual volume is less than 400 ml, the schedule may advance to every 4 to 6 hours. Indwelling catheters may predispose the client to infection and are removed as soon as possible. Crede's maneuver is not used on people with spinal cord injury.

Which of the following conditions indicates that spinal shock is resolving in a client with C7 quadriplegia? A Absence of pain sensation in chest B Spasticity C Spontaneous respirations D Urinary continence

B Spasticity, the return of reflexes, is a sign of resolving shock. Spinal or neurogenic shock is characterized by hypotension, bradycardia, dry skin, flaccid paralysis, or the absence of reflexes below the level of injury. The absence of pain sensation in the chest doesn't apply to spinal shock. Spinal shock descends from the injury, and respiratory difficulties occur at C4 and above.

A client with a spinal cord injury is prone to experiencing autonomic dysreflexia. The nurse would avoid which of the following measures to minimize the risk of recurrence? A Strict adherence to a bowel retraining program B Limiting bladder catheterization to once every 12 hours C Keeping the linen wrinkle-free under the client D Preventing unnecessary pressure on the lower limbs

B The most frequent cause of autonomic dysreflexia is a distended bladder. Straight catherization should be done every 4 to 6 hours, and Foley catheters should be checked frequently to prevent kinks in the tubing. Constipation and fecal impaction are other causes, so maintaining bowel regularity is important. Other causes include stimulation of the skin from tactile, thermal, or painful stimuli. The nurse administers care to minimize risk in these areas.

What is a priority nursing assessment in the first 24 hours after admission of the client with a thrombotic stroke? A Cholesterol level B Pupil size and pupillary response C Bowel sounds D Echocardiogram

B Pupil size and pupillary response It is crucial to monitor the pupil size and pupillary response to indicate changes around the cranial nerves. Cholesterol level is an assessment to be addressed for long-term healthy lifestyle rehabilitation. Bowel sounds need to be assessed because an ileus or constipation can develop, but is not a priority in the first 24 hours. An echocardiogram is not needed for the client with a thrombotic stroke.

Following instruction for a patient with newly diagnosed systemic lupus erythematosus (SLE), the nurse determines that teaching about the disease has been effective when the patient says: A "I should expect to have a low fever all the time with this disease." B "I need to restrict my exposure to sunlight to prevent an acute onset of symptoms." C "I should try to ignore my symptoms as much as possible and have a positive outlook." D "I can expect a temporary improvement in my symptoms if I become pregnant."

B "I need to restrict my exposure to sunlight to prevent an acute onset of symptoms."

A nurse is visiting a male patient with pancreatic cancer who is dying at home. During the visit, he breaks down and cries and tells the nurse that it is unfair that he should have to die now when he's finally made peace with his family and wants to live. Which response by the nurse would be most appropriate? A "You can't be feeling this way. You know you are going to die." B "It does seem unfair. Tell me more about how you are feeling." C "You'll be all right; who knows how much time any of us has" D "Tell me about your pain. Did it keep you awake last night?"

B "It does seem unfair. Tell me more about how you are feeling."

At what age should a patient without a family history of breast cancer begin screenings? A 25-35 B 45-55 C 35-45 D 55-65

B 45-55

The PCP orders furosemide and spironolactone for a client. Prior to administering the medication, the LPN determines that the client's potassium is 3.2 mEq/L. In addition to notifying the supervising RN, the LPN should anticipate taking which of the following actions? A Hold either the furosemide or spironolactone. B Administer the spironolactone only. C Administer the furosemide only. D Administer the furosemide and spironolactone.

B Administer the spironolactone only.

A patient presents to the ER after falling from the roof of his house. He was found unconscious at the time of injury, and is now experiencing lucid intervals and a change in LOC. The patient states he has a severe headache and you notice dilation of the pupil on one side. What type of injury has he sustained? A Subdural hematoma B Epidural hematoma C Concussion without bleeding D Mild Contusion

B Epidural hematoma

Which signs/symptoms should make the nurse suspect the client is experiencing a thyroid storm? A Obstipation and hypoactive bowel sounds. B High fever and extreme tachycardia. C Hypotension and bradycardia. D Decreased respirations and hypoxia.

B High fever and extreme tachycardia.

Management of SLE depends on disease severity and manifestations. However, which of the following medications is considered the cornerstone of SLE management? A Nonsteroidal anti-inflammatory drugs (NSAIDs) B Hydroxychloroquine C Methotrexate D Belimumab

B Hydroxychloroquine

Spironolactone (Aldactone) is prescribed for a client with chronic cirrhosis and ascites. The nurse should monitor the client for which of the following medication-related side effects? A Jaundice B Hyperkalemia C Tachycardia D Constipation

B Hyperkalemia

A patient who is near death has the following nursing diagnosis—Impaired gas exchange related to fluid in the lungs. Which intervention is most appropriate? A Suction the patient as needed. B Place the patient in the Fowler position. C Teach the patient how to do pursed-lip breathing. D Encourage the patient to cough and breathe deeply at least every 3 hours.

B Place the patient in the Fowler position.

Nurse Oliver should expect a client with hypothyroidism to report which health concerns? A Increased appetite and weight loss B Puffiness of the face and hands C Nervousness and tremors D Thyroid gland swelling

B Puffiness of the face and hands

A middle-aged female client has just underwent a thyroidectomy for treatment of Graves' disease. The nurse caring for this client is aware she may need to monitor the client for what possible complication? A Bone pain B Tetany C Goiter D Oliguria E Tetanus

B Tetany

Early this morning, a female client had a subtotal thyroidectomy. During evening rounds, nurse Tina assesses the client, who now has nausea, a temperature of 105° F (40.5° C), tachycardia, and extreme restlessness. What is the most likely cause of these signs? A Diabetic ketoacidosis B Thyroid crisis C Hypoglycemia D Tetany

B Thyroid crisis

A client who is regaining consciousness after a craniotomy becomes restless and attempts to pull out her IV line. Which nursing intervention protects the client without increasing her ICP? A Place her in a jacket restraint B Wrap her hands in soft "mitten" restraints C Tuck her arms and hands under the draw sheet D Apply a wrist restraint to each arm

B Wrap her hands in soft "mitten" restraints

The nurse has identified the nursing diagnosis of imbalanced nutrition: less than body requirements related to altered taste sensation in a patient with lung cancer who has had a 10% loss in weight. An appropriate nursing intervention that addresses the etiology of this problem is to A provide foods that are highly spiced to stimulate the taste buds. B avoid presenting foods for which the patient has a strong dislike. C add strained baby meats to foods such as soups and casseroles. D teach the patient to eat whatever is nutritious since food is tasteless.

B avoid presenting foods for which the patient has a strong dislike.

When evaluating a male client for complications of acute pancreatitis, the nurse would observe for: A increased intracranial pressure B decreased urine output C bradycardia D hypertension

B decreased urine output

In teaching about cancer prevention to a community group, the nurse stresses promotion of exercise, normal body weight, and low-fat diet because A most people are willing to make these changes to avoid cancer. B dietary fat and obesity promote growth of many types of cancer. C people who exercise and eat healthy will make other lifestyle changes. D obesity and lack of exercise cause cancer in susceptible people.

B dietary fat and obesity promote growth of many types of cancer.

While being prepared for a biopsy of a lump in the right breast, the patient asks the nurse what the difference is between a benign tumor and a malignant tumor. The nurse explains that a benign tumor differs from a malignant tumor in that benign tumors A do not cause damage to adjacent tissue. B do not spread to other tissues and organs. C are simply an overgrowth of normal cells. D frequently recur in the same site

B do not spread to other tissues and organs.

Nurse Amy is speaking to a group of women about early detection of breast cancer. The average age of the women in the group is 47. Following the American Cancer Society guidelines, the nurse should recommend that the women: A perform breast self-examination annually. B have a mammogram annually. C have a hormonal receptor assay annually. D have a physician conduct a clinical examination every 2 years.

B have a mammogram annually.

Vasogenic cerebral edema increases intracranial pressure by A. Shifting fluid in the gray matter B. Altering the endothelial lining of cerebral capillaries C. Leading molecules from the intracellular fluid to the capillaires D. Altering the osmotic gradient flow into the intravascular component

B. Altering the endothelial lining of cerebral capillaries

A patient has been diagnosed with an embolic stroke. Which of the following in the patient history would you expect to see? A. History of acute MI B. Aspirin in the medication list to treat atrial fibrillation C. LDL level at 84 mg/dL D. History of COPD

B. Aspirin in the medication list to treat atrial fibrillation Rationale: An embolic stroke is commonly caused by cardiac problems, such as a-fib

A client with a suspected brain tumor is scheduled for computed tomography (CT). What should the nurse do when preparing the client for this test? A. Immobilize the neck before the client is moved onto a stretcher. B. Determine whether the client is allergic to iodine, contrast dyes, or shellfish. C.Place a cap over the client's head. D. Administer a sedative as prescribed by doc.

B. Because CT commonly involves use of a contrast agent, the nurse should determine whether the client is allergic to iodine, contrast dyes, or shellfish. Neck immobilization is necessary only if the client has a suspected spinal cord injury. Placing a cap over the client's head may lead to misinterpretation of test results; instead, the hair should be combed smoothly. The physician orders a sedative only if the client can't be expected to remain still during the CT scan.

A patient in the ED comes in with a change in LOC. Which of the following is of most concern? A. Heart rate of 95 beats per minute B. Blood pressure increased from 150/84 to 165/68 C. Oxygen saturations at 93% D. Pupils measuring 2mm bilaterally

B. Blood pressure increased from 150/84 to 165/68 Rationale: A widening pulse pressure is indicative of Cushing's triad which is a medical emergency! If the SBP increases and the DBP decreases, that is not good!

A client with a spinal cord injury (SCI) reports a sudden severe throbbing headache that started a short time ago. Assessment of the client reveals increased blood pressure (168/94) and decreased heart rate (48 bpm), diaphoresis and flushing of the face and neck. What action should you take first? A. Administer the ordered acetaminophen B. Check the Foley tubing for kinks or obstruction C. Adjust the temperature in the clients room D. Notify the physician about the change in status

B. Check the Foley tubing for kinks or obstruction Rationale: This is autonomic reflexia, which is most commonly caused by bladder distention. The first action should be to check the foley catheter for any kinks or obstruction. If there is no foley catheter, insert a straight catheter to empty the bladder. Remove all restrictive clothing, elevate HOB to 45, check for fecal impaction, and notify the HCP!

A nursing measure that is indicated to reduce the potential with bacterial meningitis is A. Administering codeine for relief of head and neck pain B. Controlling fever with prescribed drugs and cooling techniques C. Keeping the room dark and quiet to minimize environmental stimulation D. Maintaining the patient on strict bed rest with the head of the bed slightly elevated

B. Controlling fever with prescribed drugs and cooling techniques

After experiencing a stroke, a patient is admitted to the ICU. Which of the following assessments would alert the healthcare provider that the patient is experiencing increased ICP? SELECT ALL THAT APPLY A. Oxygen saturation of 94% B. Decreased pulse rate C. Behavioral changes D. Decreased temperature E. Widened pulse pressure F. Irregular respirations

B. Decreased pulse rate C. Behavioral changes E. Widened pulse pressure F. Irregular respirations Rationale: These are all manifestations of ICP

A nurse plans care for the patient with increased intracranial pressure with the knowledge that the best way to position the patient is to A. Keep the head of the bed flat B. Elevate the head of the bed to 30 degrees C. Maintain patient on the left side with the head supported on a pillow D. Use a continuous-rotation bed to continuously change the patient position

B. Elevate the head of the bed to 30 degrees

The patient with sudden pain in the left upper quadrant radiating to the back and vomiting was diagnosed with acute pancreatitis. What intervention(s) should the nurse expect to include in the patient's plan of care? A. Immediately start enteral feeding to prevent malnutrition. B. Insert an NG tube and maintain NPO status to allow the pancreas to rest. C. Initiate early prophylactic antibiotic therapy to prevent infection. D. Administer acetaminophen (Tylenol) every 4 hours for pain relief.

B. Insert an NG tube and maintain NPO status to allow the pancreas to rest.

The nurse is assessing the motor function of an unconscious male client. The nurse would plan to use which plan to use which of the following to test the client's peripheral response to pain? A. Sternal rub B. Nail bed pressure C. Pressure on the orbital rim D. Squeezing of the sternocleidomastoid muscle

B. Nailbed pressure Motor testing in the unconscious client can be done only by testing response to painful stimuli. Nail bed pressure tests a basic peripheral response. Cerebral responses to pain are tested using sternal rub, placing upward pressure on the orbital rim, or squeezing the clavicle or sternocleidomastoid muscle.

Your patient has been diagnosed with increased intracranial pressure (ICP). Which of the following orders would you question? A. Administering Mannitol B. Performing a lumbar puncture C. Keeping the HOB elevated to 30 degrees with neck in midline position D. Hypertonic IV solution rate at 100 mL/hr

B. Performing a lumbar puncture Rationale: Performing a lumbar puncture can actually increase ICP because it could cause brain herniation. Mannitol and hypertonic IV solution will be part of the care plan.

A 73-year-old client is visiting the neurologist. The client reports light-headedness, speech disturbance, and left-sided weakness that have lasted for several hours. In the examination, an abnormal sound is auscultated in an artery leading to the brain. What is the term for the auscultated discovery?

Bruit abnormal sound caused by blood flowing over a rough surface within one or both carotid arteries

A patient arrives at the ED with slurred speech, right facial droop, and right arm weakness. Which of these actions by the healthcare provider is the priority? A. Transfer the patient to the neurological care unit B. Prepare the patient for a CT scan of the head C. Call the speech pathologist to the ED D. Prepare to administer a thrombolytic medication

B. Prepare the patient for a CT scan of the head Rationale: The first, priority action would be to get a CT scan of the head in order to determine the kind of stroke this patient is having. This will allow the HCP to create an appropriate plan of care

A client has a neurological deficit involving the limbic system. Specific to this type of deficit, the nurse would document which of the following information related to the client's behavior. A. Is disoriented to person, place, and time B. Affect is flat with periods of emotional lability C. Cannot recall what was eaten for breakfast today D.Demonstrate inability to add and subtract; does not know who is president

B.The limbic system is responsible for feelings (affect) and emotions.

A male patient who is diagnosed with acute pancreatitis is under the care of Nurse Bryan. Which intervention should the nurse include in the care plan for the client? A Administration of vasopressin and insertion of a balloon tamponade B Preparation for a paracentesis and administration of diuretics C Maintenance of NPO status and insertion of NG tube with low intermittent suction D Dietary plan of a low-fat diet and increased fluid intake to 2,000 ml/day

C Maintenance of NPO status and insertion of NG tube with low intermittent suction

A patient with an acute exacerbation of systemic lupus erythematosus (SLE) is hospitalized with incapacitating fatigue, acute hand and wrist pain, and proteinuria. The health care provider prescribes prednisone (Deltasone) 40 mg twice daily. Which nursing action should be included in the plan of care? A Institute seizure precautions. B Reorient to time and place PRN. C Monitor intake and output. D Place on cardiac monitor.

C Monitor intake and output.

A client experienced a stroke that damaged the hypothalamus and was admitted to an acute unit. Which body function would the nurse anticipate that the client has problems with and assess as needed?

Body temperature control

osteoporosis, fracture risk and stroke

Bone disease characterized by a loss of bone mass per unit volume: Common in elderly and results rom decreased physical activity, changes in protein nutrition, hormonal deficiency(estrogen), and calcium deficiency. Pts with stroke who are immobilized & restricted in WB demonstrate increased risk of osteoporosis and disuse atrophy. High Risk of Falls also.

What is External Otitis? What're its characteristics and how do you treat it?

Inflammation/infection of skin of external ear Itching or pain-movement of tragus or pinna Drainage Hearing loss Apply heat & topical antibiotics/steroid

A nurse is monitoring a client with a C5 spinal cord injury for spinal shock. Which of the following findings would be associated with spinal shock in this client? Select all that apply.

Bowel sounds are absent. The client's abdomen is distended. Respiratory excursion is diminished. Accessory muscles of respiration are areflexic.

A 78 year old client is admitted to the emergency department with numbness and weakness of the left arm and slurred speech. Which nursing intervention is priority? A Prepare to administer recombinant tissue plasminogen activator (rt-PA). B Discuss the precipitating factors that caused the symptoms. C Schedule for A STAT computer tomography (CT) scan of the head. D Notify the speech pathologist for an emergency consult.

C Schedule for A STAT computer tomography (CT) scan of the head. A CT scan will determine if the client is having a stroke or has a brain tumor or another neurological disorder. This would also determine if it is a hemorrhagic or ischemic accident and guide the treatment, because only an ischemic stroke can use rt-PA. This would make (1) not the priority since if a stroke was determined to be hemorrhagic, rt-PA is contraindicated. Discuss the precipitating factors for teaching would not be a priority and slurred speech would as indicate interference for teaching. Referring the client for speech therapy would be an intervention after the CVA emergency treatment is administered according to protocol.

Mentally incompetent

You should avoid talking to an aphasic person as if he/she were _________ __________.

Which of the following clients on the rehab unit is most likely to develop autonomic dysreflexia? A A client with a brain injury B A client with a herniated nucleus pulposus C A client with a high cervical spine injury DA client with a stroke

C Autonomic dysreflexia refers to uninhibited sympathetic outflow in clients with spinal cord injuries about the level of T10. The other clients aren't prone to dysreflexia.

A client is admitted with a spinal cord injury at the level of T12. He has limited movement of his upper extremities. Which of the following medications would be used to control edema of the spinal cord? A Acetazolamide (Diamox) B Furosemide (Lasix) C Methylprednisolone (Solu-Medrol) D Sodium Bicarbonate

C High doses of Solu-Medrol are used within 24 hours of spinal injury to reduce cord swelling and limit neurological deficit. The other drugs aren't indicated in this circumstance.

A 20-year-old client who fell approximately 30' is unresponsive and breathless. A cervical spine injury is suspected. How should the first-responder open the client's airway for rescue breathing? A By inserting a nasopharyngeal airway B By inserting a oropharyngeal airway C By performing a jaw-thrust maneuver D By performing the head-tilt, chin-lift maneuver

C If the client has a suspected cervical spine injury, a jaw-thrust maneuver should be used to open the airway. If the tongue or relaxed throat muscles are obstructing the airway, a nasopharyngeal or oropharyngeal airway can be inserted; however, the client must have spontaneous respirations when the airway is open. The head-tilt, chin-lift maneuver requires neck hyperextension, which can worsen the cervical spine injury.

A client with a T1 spinal cord injury arrives at the emergency department with a BP of 82/40, pulse 34, dry skin, and flaccid paralysis of the lower extremities. Which of the following conditions would most likely be suspected? A Autonomic dysreflexia B Hypervolemia C Neurogenic shock D Sepsis

C Loss of sympathetic control and unopposed vagal stimulation below the level of injury typically cause hypotension, bradycardia, pallor, flaccid paralysis, and warm, dry skin in the client in neurogenic shock. Hypervolemia is indicated by rapid and bounding pulse and edema. Autonomic dysreflexia occurs after neurogenic shock abates. Signs of sepsis would include elevated temperature, increased heart rate, and increased respiratory rate.

The nurse is planning care for the client in spinal shock. Which of the following actions would be least helpful in minimizing the effects of vasodilation below the level of the injury? A Monitoring vital signs before and during position changes B Using vasopressor medications as prescribed C Moving the client quickly as one unit D Applying Teds or compression stockings.

C Reflex vasodilation below the level of the spinal cord injury places the client at risk for orthostatic hypotension, which may be profound. Measures to minimize this include measuring vital signs before and during position changes, use of a tilt-table with early mobilization, and changing the client's position slowly. Venous pooling can be reduced by using Teds (compression stockings) or pneumatic boots. Vasopressor medications are administered per protocol.

The nurse is evaluating neurological signs of the male client in spinal shock following spinal cord injury. Which of the following observations by the nurse indicates that spinal shock persists? A Positive reflexes B Hyperreflexia C Inability to elicit a Babinski's reflex D Reflex emptying of the bladder

C Resolution of spinal shock is occurring when there is a return of reflexes (especially flexors to noxious cutaneous stimuli), a state of hyperreflexia rather than flaccidity, reflex emptying of the bladder, and a positive Babinski's reflex.

Which assessment data would indicate to the nurse that the client would be at risk for a hemorrhagic stroke? A A blood glucose level of 480 mg/dl. B A right-sided carotid bruit. C A blood pressure of 220/120 mmHg. D The presence of bronchogenic carcinoma

C A blood pressure of 220/120 mmHg. Uncontrolled hypertension is a risk factor for hemorrhagic stroke, which is a rupture blood vessel in the cranium. A bruit in the carotid artery would predispose a client to an embolic or ischemic stroke. High blood glucose levels could predispose a patient to ischemic stroke, but not hemorrhagic. Cancer is not a precursor to stroke.

During the first 24 hours after thrombolytic therapy for ischemic stroke, the primary goal is to control the client's: A Pulse B Respirations C Blood pressure D Temperature

C Blood pressure Controlling the blood pressure is critical because an intracerebral hemorrhage is the major adverse effect of thrombolytic therapy. Blood pressure should be maintained according to physician and is specific to the client's ischemic tissue needs and risks of bleeding from treatment. Other vital signs are monitored, but the priority is blood pressure.

The nurse and unlicensed assistive personnel (UAP) are caring for a client with right-sided paralysis. Which action by the UAP requires the nurse to intervene? A The assistant places a gait belt around the client's waist prior to ambulating. B The assistant places the client on the back with the client's head to the side. C The assistant places her hand under the client's right axilla to help him/her move up in bed. D The assistant praises the client for attempting to perform ADL's independently.

C The assistant places her hand under the client's right axilla to help him/her move up in bed. This action is inappropriate and would require intervention by the nurse because pulling on a flaccid shoulder joint could cause shoulder dislocation; as always use a lift sheet for the client and nurse safety. All the other actions are appropriate.

A client arrives in the emergency department with an ischemic stroke and receives tissue plasminogen activator (t-PA) administration. Which is the priority nursing assessment? A Current medications. B Complete physical and history. C Time of onset of current stroke. D Upcoming surgical procedures.

C Time of onset of current stroke. The time of onset of a stroke to t-PA administration is critical. Administration within 3 hours has better outcomes. A complete history is not possible in emergency care. Upcoming surgical procedures will need to be delay if t-PA is administered. Current medications are relevant, but onset of current stroke takes priority.

A 30-year-old was admitted to the progressive care unit with a C5 fracture from a motorcycle accident. Which of the following assessments would take priority? A Bladder distension B Neurological deficit C pulse ox readings D The client's feelings about the injury

C After a spinal cord injury, ascending cord edema may cause a higher level of injury. The diaphragm is innervated at the level of C4, so assessment of adequate oxygenation and ventilation is necessary. Although the other options would be necessary at a later time, observation for respiratory failure is the priority.

While in the ER, a client with C8 tetraplegia develops a blood pressure of 80/40, pulse 48, and RR of 18. The nurse suspects which of the following conditions? A Autonomic dysreflexia B Hemorrhagic shock C Neurogenic shock D Pulmonary embolism

C Symptoms of neurogenic shock include hypotension, bradycardia, and warm, dry skin due to the loss of adrenergic stimulation below the level of the lesion. Hypertension, bradycardia, flushing, and sweating of the skin are seen with autonomic dysreflexia. Hemorrhagic shock presents with anxiety, tachycardia, and hypotension; this wouldn't be suspected without an injury. Pulmonary embolism presents with chest pain, hypotension, hypoxemia, tachycardia, and hemoptysis; this may be a later complication of spinal cord injury due to immobility.

The LPN is caring for a client immediately after a paracentesis. It is most important for the LPN to ask which of the following questions? A "Do your clothes feel tight?" B "Do you need to void?" C "Are you feeling dizzy?" D "Do you have any pain?"

C "Are you feeling dizzy?"

An 18-year-old client was hit in the head with a baseball during practice. When discharging him to the care of his mother, the nurse gives which of the following instructions? A "Watch him for keyhole pupil the next 24 hours." B "Expect profuse vomiting for 24 hours after the injury." C "Wake him every hour and assess his orientation to person, time, and place." D "Notify the physician immediately if he has a headache."

C "Wake him every hour and assess his orientation to person, time, and place."

A client is to undergo a breast biopsy. The client tells the LPN, "If I lose my breast, I know my husband will no longer find me attractive." Which of the following responses by the LPN would be most appropriate? A "You don't know if you are going to lose your breast. They are just doing the biopsy now." B "You should focus on your children right now. They are young and they need you." C "You seem to be concerned that your relationship with your husband might change." D "Why don't you wait and see what your husband's reaction is before you get upset."

C "You seem to be concerned that your relationship with your husband might change."

The nurse is assigned to care for a client with systemic lupus erythematosus (SLE). The nurse plans care knowing that this disorder is: A A local rash that occurs as a result of allergy B A disease caused by overexposure to sunlight C An inflammatory disease of the collagen contained in connective tissue D A disease caused by the continuous release of histamine in the body

C An inflammatory disease of the collagen contained in connective tissue

When discharging a client from the ER after a head trauma, the nurse teaches the guardian to observe for a lucid interval. Which of the following statements best described a lucid interval? A An interval when the client's speech is garbled. B An interval when the client is alert but can't recall recent events. C An interval when the client is oriented but then becomes somnolent (sleepy, groggy,drowsy) D An interval when the client has a "warning" symptom, such as an odor or visual disturbance.

C An interval when the client is oriented but then becomes somnolent (sleepy, groggy, drowsy)

A patient has recently been diagnosed with cancer. What is the best initial nursing intervention for this patient? A Provide literature about the specifics of the cancer B Introduce the patient to a recovering cancer patient C Encourage the patient to verbalize feelings and fears D Reassure the patient that "everything will work out fine"

C Encourage the patient to verbalize feelings and fears

A client arrives at the ER after slipping on a patch of ice and hitting her head. A CT scan of the head shows a collection of blood between the skull and dura mater. Which type of head injury does this finding suggest? A Subdural hematoma B Subarachnoid hemorrhage C Epidural hematoma D Contusion

C Epidural hematoma

You are taking care of a 66-year old female who complains of weight gain, lethargy, dry skin, hair loss, constipation, and increased cold sensitivity. Based upon this assessment, you feel that the most likely diagnosis is which of the following? A Thyroid storm B Hyperthyroidism C Hypothyroidism D Diabetes insipidus E Diabetic ketoacidosis

C Hypothyroidism

A client comes into the ER after hitting his head in an MVA. He's alert and oriented. Which of the following nursing interventions should be done first? A Assess full ROM to determine extent of injuries B Call for an immediate chest x-ray C Immobilize the client's head and neck D Open the airway with the head-tilt chin-lift maneuver

C Immobilize the client's head and neck

A client was admitted with injury to the thalamus. Which manifestation would the nurse observe during data collection?

aching sensation over half of the body

The nurse is trying to communicate with a client with brain attack (stroke) and aphasia. Which of the following actions by the nurse would be least helpful to the client? A. Speaking to the client at a slower rate B. Allowing plenty of time for the client to respond C. Completing the sentences that the client cannot finis D. Looking directly at the client during attempts at speech

C. Clients with aphasia after brain attack (stroke) often fatigue easily and have a short attention span. General guidelines when trying to communicate with the aphasic client include speaking more slowly and allowing adequate response time, listening to and watching attempts to communicate, and trying to put the client at ease with a caring and understanding manner. The nurse would avoid shouting (because the client is not deaf), appearing rushed for a response, and letting family members provide all the responses for the client.

You are providing care for a client with an acute hemorrhagic stroke. The client's spouse tells you that he has been reading a lot about strokes and asks why his wife has not received TPA. What is your best response? A. "Your wife was not admitted within the time frame that TPA is administered." B. "This drug is used primarily for clients who experience an acute heart attack." C. "TPA dissolves clots and may cause more bleeding into your wife's brain." D. "Your wife had gallbladder surgery just 6 months ago, and this prevents the use of TPA."

C. "TPA dissolves clots and may cause more bleeding into your wife's brain." Rationale: tPA is only used for ischemic strokes!!

Which patient below is at MOST risk for CHRONIC pancreatitis?* A. A 25 year old female with a family history of gallstones. B. A 35 year old male who reports social drinking of alcohol. C. A 15 year old female with cystic fibrosis. D. A 66 year old female with stomach cancer.

C. A 15 year old female with cystic fibrosis.

Nursing management of a patient with a brain tumor includes SELECT ALL THAT APPLY A. Discussing with the patient methods to control inappropriate behavior B. Using diversion techniques to keep the patient stimulated and motivated C. Assisting and supporting the family in understanding any changes in behavior D. Limiting self-care activities until the patient has regained maximum physical functioning E. Planning for seizure precautions and teaching the patient and the caregiver about antiseizure drugs

C. Assisting and supporting the family in understanding any changes in behavior E. Planning for seizure precautions and teaching the patient and the caregiver about antiseizure drugs

The factor related to cerebral blood flow that most often determines the extent of cerebral damage from a stroke is the A. Amount of CO B. O2 content of the blood C. Degree of collateral circulation D. Level of CO2 in the blood

C. Degree of collateral circulation

The nurse is alerted to a possible acute subdural hematoma in the patient who A. Has a linear skull fracture cross a major artery B. Has focal symptoms of brain damage with no recollection of a head injury C. Develops decreased LOC and a headache within 48 hours of a head injury D. Has in immediate loss of consciousness with a brief lucid interval followed by a decreased LOC

C. Develops decreased LOC and a headache within 48 hours of a head injury

A health care provider is providing community education on signs and symptoms of stroke. Which of the following best describe the signs and symptoms of a stroke? A. Diaphoresis and jaw pain B. Weakness and edema C. Facial droop and slurred speech D. Indigestion and shortness of breath

C. Facial droop and slurred speech Rationale: FAST. Facial drooping, arm weakness, slurred speech, and time!

A patient is admitted with complaints of palpations, excessive sweating, and unable to tolerate heat. In addition, the patient voices concern about how her appearance has changed over the past year. The patient presents with protruding eyeballs and pretibial myxedema on the legs and feet. Which of the following is the likely cause of the patient's signs and symptoms?* A. Thyroiditis B. Deficiency of iodine consumption C. Grave's Disease D. Hypothyroidism

C. Grave's Disease

A male client with a spinal cord injury is prone to expiring autonomic dysreflexia. The nurse would avoid which of the following measures to minimize the risk of reoccurrence? A. Strict adherence to a bowel retraining program B. Keeping the linen wrinkle free beneath him C. Limiting bladder catheterization to once every 12 hours D. Discouraging tight fitting clothing

C. Limiting bladder catheterization to once every 12 hours Rationale: Bladder catheterization should be done every 4-6 hours. All other options are interventions that should be done

A white female client is admitted to an acute care facility with a diagnosis of cerebrovascular accident (CVA). Her history reveals bronchial asthma, exogenous obesity, and iron deficiency anemia. Which history finding is a risk factor for CVA? A. Caucasian race B. Female sex C. Obesity D. Asthma

C. Obesity Rationale: Obesity is a modifiable risk factor that increases your risk for stroke. African-American ethnicity and male gender also increase your risk. Asthma is not related.

A white female client is admitted to an acute care facility with a diagnosis of cerebrovascular accident (CVA). Her history reveals bronchial asthma, exogenous obesity, and iron deficiency anemia. Which history finding is a risk factor for CVA? A. Caucasian race B. Female C. Obesity D. Bronchial asthma

C. Obesity Obesity is a risk factor for CVA. Other risk factors include a history of ischemic episodes, cardiovascular disease, diabetes mellitus, atherosclerosis of the cranial vessels, hypertension, polycythemia, smoking, hypercholesterolemia, oral contraceptive use, emotional stress, family history of CVA, and advancing age.

A female client has experienced an episode of myasthenic crisis. The nurse would assess whether the client has precipitating factors such as: A. Getting too little exercise B. Taking excess medication C. Omitting doses of medication D. Increasing intake of fatty foods

C. Omitting doses of meds. Myasthenic crisis often is caused by undermedication and responds to the administration of cholinergic medications, such as neostigmine (Prostigmin) and pyridostigmine (Mestinon). Cholinergic crisis (the opposite problem) is caused by excess medication and responds to withholding of medications.

The nurse on the clinical unit is assigned to four patients. Which patient should she assess first? A. Patient with a skull fracture whose nose is bleeding B. Older patient with a stroke who is confused and whose daughter is present C. Patient with meningitis who is suddenly agitated and reporting a headache of 10 on a 0-to-10 scale D. Patient who had a craniotomy for a brain tumor and who is now 3 days post-op and has had continued vomiting

C. Patient with meningitis who is suddenly agitated and reporting a headache of 10 on a 0-to-10 scale

The client diagnosed with acute pancreatitis is in pain. Which position should the nurse assist the client into to help decrease the pain? A. Recommend lying in the prone position with legs extended B. Maintain a tripod position over the bedside table C. Place in side-lying position with knees flexed D. Encourage a supine position with a pillow under the knees

C. Place in side-lying position with knees flexed

The Glasgow Coma Scale (GCS) measures all of the following except: A. Verbal response B. Motor response C. Sensation to extremities D. Eye opening

C. Sensation to extremities Rationale: The GCS includes opening eyes when a verbal or painful stimulus is applies, speaking, and obeying commands

A patient is being discharged home for treatment of hypothyroidism. Which medication is most commonly prescribed for this condition?* A. Tapazole B. PTU (Propylthiouracil) C. Synthroid D. Inderal

C. Synthroid

A client injures his or her spinal cord in a diving accident. Which cerebral vertebral level would the nurse associates the injury site if the client is unable to breathe spontaneously?

C4

A client injures his spinal cord in a diving accident. The nurse knows that the client will be unable to breathe spontaneously if the injury site is above which vertebral level?

C4

A client is thrown from an automobile during a collision. The nurse knows that the client will be able to maintain gross arm movements and diaphragmatic breathing if the injury occurs at what vertebral level?

C5

NERVE

CARRIES NERVE IMPULSE WITHIN THE BODY. MACRO CORDLIKE STRUCTURE CONTAINS INDIVIDUAL NERVE FIBERS

CNS

CENTRAL NERVOUS SYSTEM

FRONTAL LOBE

CEREBRAL LOBE THAT ALLOWS FOR HIGHER LEVELS OF MENTAL FUNCTIONING INCLUDING JUDGEMENT, COMMUNICATION, & BODY MOVEMENT. **CONTROLS AREAS FOR WRITTEN & MOTOR SPEECH

TEMPORAL LOBE

CEREBRAL LOBE THAT CONTROLS SENSATIONS OF: HEARING, AUDITORY INTERPRETATION, & SMELL

OCCIPITAL LOPE

CEREBRAL LOBE THAT HOUSES THE VISUAL AREAS, DIRECTS VISUAL EXPERIENCES. VISUAL TRANSITIONS AND INTERPETATION

Pt had a TURP and 24 hrs later there's bright red blood in urine. What should the nurse do?

Call the health care provider, as this could be an arterial bleed. Even though bleeding is common after, it should be darker. Bright red may indicate clotting/hemorrhage.

A nurse is caring for a client diagnosed with a cerebral aneurysm who reports a severe headache. Which action should the nurse perform?

Call the physician immediately.

The nurse is caring for a client diagnosed with a cerebral aneurysm, who reports a severe headache. Which action should the nurse perform first?

Call the physician immediately.

A female client is receiving chemotherapy to treat breast cancer. Which assessment finding indicates a fluid and electrolyte imbalance induced by chemotherapy? A Urine output of 400 ml in 8 hours B Serum potassium level of 3.6 mEq/L C Blood pressure of 120/64 to 130/72 mm Hg D Dry oral mucous membranes and cracked lips

D Dry oral mucous membranes and cracked lips.

An older client is at risk for falls. When developing an individualized plan of care for this client, the nurse recalls that which concept is least relevant to maintenance of balance for the older client?

Older clients cannot think quickly enough to respond to emergencies.

A client with myasthenia gravis is experiencing prolonged periods of weakness. The health care provider prescribes a test dose of edrophonium (Enlon) and the client becomes weaker. The nurse interprets this outcome as:

Cholinergic crisis

A client was accidentally struck in the head by a baseball bat. Which condition should the nurse anticipate as a serious complication? a. Clear fluid draining from the ears b. Large hematoma at the impact site c. Headache d. Complaints of dizziness

Clear fluid draining from the ears. Rationale: Blunt trauma may cause a contusion of the brain. If the skull is fractured, there may be leakage of cerebrospinal fluid into the ears or nose. Infection is a serious complication of brain injury. Hematomas commonly develop from blood leaking underneath the skin at the site of injury; the serious complication would be leakage of CSF from the ears. Headaches are commonly seen after a blow to the head and are not a serious complication like leakage of CSF from the ear. Dizziness is commonly seen after a blow to the head and is not a serious complication.

The nurse is collecting neurological data on an unconscious client. On application of a central noxious stimulus, the nurse observes this response (refer to figure). How should the nurse document this response on the client's record?

Client demonstrated decerebrate posturing.

A client who experienced a stroke that left her with residual right-sided weakness was just discharged to go home. The client lives in a two-story house in which the bathroom is located on the second floor. A home health care nurse is visiting the client for the first time. Which issue should the nurse address during this visit?

Client's ability to climb the stairs while using a walker

A nursing instructor asks a nursing student about the points to document if the client has had a seizure. The instructor determines that the student needs to read about seizures and related documentation points if the student stated that it is important to document:

Client's diet in the 2 hours preceding seizure activity

The nurse is caring for the client in the ER following a head injury. The client momentarily lost consciousness at the time of the injury and then regained it. The client now has lost consciousness again. The nurse takes quick action, knowing this is compatible with: A Skull fracture B Concussion C Subdural hematoma D Epidural hematoma

D Epidural hematoma

A client has a halo vest that was applied following a C6 spinal cord injury. The nurse performs which of the following to determine whether the client is ready to begin sitting up?

Compares the client's pulse and blood pressure when both flat and sitting

A nurse is trying to communicate with a brain attack (stroke) client with aphasia. Which action by the nurse would be least helpful to the client?

Completing the sentences that the client cannot finish

What is the most common cause of hyperthyroidism? A Thyroid storm B Toxic nodular goiter C Hashimoto's thyroiditis D Graves' disease

D Graves' Disease

A client has experienced an episode of myasthenic crisis. The nurse collects data to determine whether the client has precipitating factors such as:

Omitted doses of medication

What is the difference between conductive and sensorineural hearing loss?

Conductive Difficulty external or middle ear Sound waves blocked Inflammation or obstruction Sensorineural Difficulty inner ear or acoustic nerve Sensory nerve fibers damaged Prolonged loud noises, medications Presbycusis - result of aging

The nurse is reviewing the medical record for a client in a long-term care facility. The nurse notes an entry by the primary care physician indicating the client is colorblind. The nurse understands this condition results from a problem with which structure(s) of the eye?

Cones

A nurse caring for a client following a craniotomy monitors for signs of increased intracranial pressure (ICP). Which of the following indicates an early sign of increased ICP?

Confusion

A community health nurse is conducting a workshop for unlicensed care providers who work in a chain of long-term care facilities. The nurse is teaching the participants about the signs and symptoms of stroke. What signs and symptoms should the nurse identify? Select all that apply.

Confusion Sudden numbness Visual disturbances

A nurse is collecting data on a client with Parkinson's disease. Which finding indicates a serious complication of this disorder?

Congested cough and coarse rhonchi heard on auscultation

pseudobulbar affect

Lesion affecting frontal lob, hypothalamus, and limbic system PBA aka emotional lability or emotional dysregulation syndrome 18% of stroke pts emotional outbursts of uncontrolled or exaggerated laughing or crying that are inconsistent with mood pt quickly changes mood

A nurse is completing the admission assessment of a client who has acute pancreatitis. Which finding is the first priority? A History of cholelithiasis B Elevated serum amylase levels C Decrease in bowel sounds upon auscultation D Hand spasms occur when blood pressure is checked

D Hand spasms occur when blood pressure is checked

When the client has difficulty maintaining balance, what cranial nerve is involved? a. Cranial nerve III b. Cranial nerve V c. Cranial nerve VIII d. Cranial nerve XI

Cranial nerve VIII Rationale: Cranial nerve VIII (vestibulocochlear) is responsible for hearing and equilibrium. Cranial nerve III (oculomotor) controls pupil constriction and eye movement; V (trigeminal) controls chewing and scalp sensations; and XI (accessory) controls neck and shoulder movement.

The neurologic unit has identified a 30% occurrence of pressure ulcers in clients admitted with the diagnosis of stroke. Which of the following actions should be included in the unit's performance improvement plan?

Creating a spreadsheet on which nursing staff should document repositioning of clients admitted with a stroke

A 40-year-old paraplegic must perform intermittent catheterization of the bladder. Which of the following instructions should be given? A "Clean the meatus from back to front." B "Measure the quantity of urine." C "Gently rotate the catheter during removal." D "Clean the meatus with soap and water."

D Intermittent catheterization may be performed chronically with clean technique, using soap and water to clean the urinary meatus. The meatus is always cleaned from front to back in a woman, or in expanding circles working outward from the meatus in a man. It isn't necessary to measure the urine. The catheter doesn't need to be rotated during removal.

During an episode of autonomic dysreflexia in which the client becomes hypertensive, the nurse should perform which of the following interventions? A Elevate the client's legs B Put the client flat in bed C Put the client in the Trendelenburg's position D Put the client in the high-Fowler's position

D Putting the client in the high-Fowler's position will decrease cerebral blood flow, decreasing hypertension. Elevating the client's legs, putting the client flat in bed, or putting the bed in the Trendelenburg's position places the client in positions that improve cerebral blood flow, worsening hypertension.

The nurse is caring for a client admitted with spinal cord injury. The nurse minimizes the risk of compounding the injury most effectively by: A Keeping the client on a stretcher B Logrolling the client on a firm mattress C Logrolling the client on a soft mattress D Placing the client on a Stryker frame

D Spinal immobilization is necessary after spinal cord injury to prevent further damage and insult to the spinal cord. Whenever possible, the client is placed on a Stryker frame, which allows the nurse to turn the client to prevent complications of immobility, while maintaining alignment of the spine. If a Stryker frame is not available, a firm mattress with a bed board should be used.

A client with a C6 spinal injury would most likely have which of the following symptoms? A: Aphasia B: Hemiparesis C:Paraplegia D: Tetraplegia

D Tetraplegia occurs as a result of cervical spine injuries. Paraplegia occurs as a result of injury to the thoracic cord and below.

A client has a cervical spine injury at the level of C5. Which of the following conditions would the nurse anticipate during the acute phase? A Absent corneal reflex B Decerebrate posturing C Movement of only the right or left half of the body D The need for mechanical ventilation

D The diaphragm is stimulated by nerves at the level of C4. Initially, this client may need mechanical ventilation due to cord edema. This may resolve in time. Absent corneal reflexes, decerebrate posturing, and hemiplegia occur with brain injuries, not spinal cord injuries.

A 22-year-old client with quadriplegia is apprehensive and flushed, with a blood pressure of 210/100 and a heart rate of 50 bpm. Which of the following nursing interventions should be done first? A Place the client flat in bed B Assess patency of the indwelling urinary catheter C Give one SL nitroglycerin tablet D raise the head of the bed immediately to 90 degrees

D Anxiety, flushing above the level of the lesion, piloerection, hypertension, and bradycardia are symptoms of autonomic dysreflexia, typically caused by such noxious stimuli such as a full bladder, fecal impaction, or decubitus ulcer. Putting the client flat will cause the blood pressure to increase even more. The indwelling urinary catheter should be assessed immediately after the HOB is raised. Nitroglycerin is given to reduce chest pain and reduce preload; it isn't used for hypertension or dysreflexia.

Which client is at the highest risk for systemic lupus erythematous (SLE)? A An Asian male B A white female C An African-American male D An African-American female

D An African-American female

A client has signs of increased ICP. Which of the following is an early indicator of deterioration in the client's condition? A Widening pulse pressure B Decrease in the pulse rate C Dilated, fixed pupil D Decrease in LOC

D Decrease in LOC

A client, age 41, visits the gynecologist. After examining her, the physician suspects cervical cancer. The nurse reviews the client's history for risk factors for this disease. Which history finding is a risk factor for cervical cancer? A Onset of sporadic sexual activity at age 17 B Spontaneous abortion at age 19 C Pregnancy complicated with eclampsia at age 27 D Human papillomavirus infection at age 32

D Human papillomavirus infection at age 32

Which nursing diagnosis takes highest priority for a female client with hyperthyroidism? A Risk for imbalanced nutrition: More than body requirements related to thyroid hormone excess B Risk for impaired skin integrity related to edema, skin fragility, and poor wound healing C Body image disturbance related to weight gain and edema D Imbalanced nutrition: Less than body requirements related to thyroid hormone excess

D Imbalanced nutrition: Less than body requirements related to thyroid hormone excess

A family member asks the nurse to explain the purpose of hospice care. Which of the following is the best response? A Is appropriate when the patient desires to intentionally end his life B Focuses on minimizing the disease process as rapidly as possible C Focuses on symptom management for patients not responding to treatment D Is holistic care for patients that are dying or debilitated and are not expected to improve

D Is holistic care for patients that are dying or debilitated and are not expected to improve

The nurse is interviewing a male client about his past medical history. Which preexisting condition may lead the nurse to suspect that a client has colorectal cancer? A Duodenal ulcers B Hemorrhoids C Weight gain D Polyps

D Polyps

During a routine health examination, a 30-year-old patient tells the nurse about a family history of colon cancer. The nurse will plan to A teach the patient about the need for a colonoscopy at age 50. B ask the patient to bring in a stool specimen to test for occult blood. C schedule a sigmoidoscopy to provide baseline data about the patient. D have the patient ask the doctor about specific tests for colon cancer.

D have the patient ask the doctor about specific tests for colon cancer.

You're caring for a 45 year old patient who is admitted with suspected acute pancreatitis. The patient reports having extreme mid-epigastric pain that radiates to the back. The patient states the pain started last night after eating fast food. As the nurse, you know the two most common causes of acute pancreatitis are:* A. High cholesterol and alcohol abuse B. History of diabetes and smoking C. Pancreatic cancer and obesity D. Gallstones and alcohol abuse

D. Gallstones and alcohol abuse

The nurse is positioning the female client with increase ICP. Which of the following position would the nurse avoid? A. Head of bed elevated 30 to 45 degrees B. Neck in the neutral position C. Head midline D. Head turned to the side

D. Head turned to the side Rationale: Any other positions beside a neutral neck position, head midline, and head of bed elevated to 30-45 degrees could actually increase ICP

A patient reports they do not eat enough iodine in their diet. What condition are they most susceptible to?* A. Pheochromocytoma B. Hyperthyroidism C. Thyroid Storm D. Hypothyroidism

D. Hypothyroidism

A health care provider is conducting a community education class on stroke prevention. The teaching plan for this class will include which of these instructions? A. There is no known link between risk of stroke and regular exercise B. There is no correlation between diabetes and risk of stroke C. Foods high in saturated fats can be consumed in moderate amounts. D. It is important to maintain a healthy weight and control your BP.

D. It is important to maintain a healthy weight and control your BP. Rationale: This one is literally just common sense y'all

A patient is 6 hours post-opt from a thyroidectomy. The surgical site is clean, dry and intact with no excessive swelling noted. What position is best for this patient to be in?* A. Fowler's B. Prone C. Trendelenburg D. Semi-Fowler's

D. Semi-Fowler's

Information provided by the patient that would help differentiate a hemorrhagic stroke from a thrombotic stroke includes A. Sensory disturbances B. A history of HTN C. Presence of motor weakness D. Sudden onset of severe headache

D. Sudden onset of severe headache

A female client is admitted in a disoriented and restless state after sustaining a concussion during a car accident. Which nursing diagnosis takes highest priority in this client's plan of care? A. Disturbed sensory perception (visual) B. Self-care deficient: Dressing/grooming C. Impaired verbal communication D. Risk for injury

D:Because the client is disoriented and restless, the most important nursing diagnosis is risk for injury.

A young man was running along an ocean pier, tripped on an elevated area of the decking, and struck his head on the pier railing. According to his friends, "He was unconscious briefly and then he became alert and behaved as though nothing had happened." Shortly afterward, he began complaining of a headache and asked to be taken to the emergency department. If the client's intracranial pressure (ICP) is increasing, the nurse would expect to observe which sign first?

Declining level of consciousness

Acetazolamide is prescribed for a client with a diagnosis of a supratentorial lesion. A nurse monitors the client for effectiveness of this medication, knowing that its primary action is to:

Decrease cerebrospinal fluid production.

The nurse is collecting data on a geriatric client with senile dementia. When reviewing this client's file, which neurotransmitter condition is the nurse likely to find in the client's history as a contributory factor to his or her cognitive changes?

Decreased acetylcholine level

The nurse is collecting data on a geriatric client with senile dementia. Which neurotransmitter condition is likely to contribute to this client's cognitive changes?

Decreased acetylcholine level

Which of these physiologic results would occur from the stimulation of the sympathetic nervous system? Select all that apply. a. Decreased peristalsis b. Slower heart rate c. Dilation of skin blood vessels d. Increased blood glucose levels e. Dilation of the pupils f. Constriction of the bronchi

Decreased peristalsis. Increased blood glucose levels. Dilation of the pupils. Rationale: The sympathetic nervous system decreases peristalsis, increases blood glucose levels, and dilates the pupils. Slowing of the heart rate is an action of the parasympathetic nervous system. Dilation of skin blood vessels is an action of the parasympathetic nervous system.

A nurse is caring for the client with a head injury secondary to a motor vehicle accident. The nurse observes the client's status regularly, monitoring closely for which changes in vital signs?

Decreasing pulse, decreasing respirations, increasing BP

A client who sustained a closed head injury has a new onset of copious urinary output. Urine output for the previous 8-hour shift was 3300 mL, and 2800 mL for the shift before that. The findings have been reported to the health care provider, and the nurse anticipates a prescription for which of the following medications?

Desmopressin (DDAVP)

A client with a suspected brain tumor is scheduled for computed tomography (CT). What should the nurse do when preparing the client for this test?

Determine whether the client is allergic to iodine, contrast dyes, or shellfish.

A nurse is preparing for the admission of a client with a suspected diagnosis of Guillain-Barré syndrome. Which clinical manifestation is considered a primary symptom of this syndrome?

Development of muscle weakness

Dysphagia (cranial nerve)

Difficulty ion swallowing Lesions involving motor pathways of CNs IX, X (including lower brainstem)

A client with a subarachnoid hemorrhage is prescribed a 1,000-mg loading dose of Dilantin IV. Which consideration is most important when administering this dose? 1.Therapeutic drug levels should be maintained between 20 to 30 mg/ml. 2.Rapid dilantin administration can cause cardiac arrhythmias. 3.Dilantin should be mixed in dextrose in water before administration. 4.Dilantin should be administered through an IV catheter in the client's hand.

Dilantin IV shouldn't be given at a rate exceeding 50 mg/minute. Rapid administration can depress the myocardium, causing arrhythmias. Therapeutic drug levels range from 10 to 20 mg/ml. Dilantin shouldn't be mixed in solution for administration. However, because it's compatible with normal saline solution, it can be injected through an IV line containing normal saline. When given through an IV catheter hand, dilantin may cause purple glove syndrome

A client with Parkinson's disease is embarrassed about the symptoms of the disorder and is bored and lonely. The nurse would plan which approach as therapeutic in assisting the client to cope with the disease?

Encourage and praise perseverance in exercising and performing ADL.

A nursing student is writing a care plan for a newly admitted client who has been diagnosed with a stroke. What major nursing diagnosis should most likely be included in the client's plan of care?

Disturbed sensory perception

A client is admitted into a medical unit confused and agitated. Which nursing measure should the nurse implement first to keep the client safe?

Encourage family, friends, or a sitter to stay with the client.

A nurse is planning care for a client who displays confusion secondary to a neurological problem. Which approach by the nurse would be least helpful in assisting this client?

Encouraging multiple visitors at one time

diplopia

Double vision (diplopia)

A client has just undergone computed tomography (CT) scanning with a contrast medium. The nurse determines that the client understands postprocedure care if the client verbalizes that he or she will:

Drink extra fluids for the day.

The nurse is caring for a client who has just had a myelogram (water-soluble dye was used). What teaching point should the nurse emphasize to the client? a. Avoid coughing or sneezing b. Stay flat in bed for 8 to 12 hours c. Drink plenty of fluids d. Avoid eating for 6 hours

Drink plenty of fluids. Rationale: The client should have a fluid intake of 2,400 to 3,000 mL in 24 hours to reduce the possibility of headache. There is no need to avoid coughing or sneezing after a myelogram. The client does not need to stay flat in bed for 8-12 hours after a myelogram. The client does not need to avoid eating after a myelogram.

A nurse has provided discharge instructions to a client with an application of a halo device. The nurse determines that the client needs further clarification of the instructions if the client states that he or she will:

Drive only during the daytime.

A nurse is caring for a client with an intracranial aneurysm who was previously alert. Which finding would be an early indication that the level of consciousness (LOC) is deteriorating?

Drowsiness

Stages of Alzheimer's Disease

Early, Middle, Late and Terminal

A nurse is caring for a client scheduled for magnetic resonance imaging (MRI). Which instruction does the nurse reinforce to the client?

Earplugs can be worn if the noise from the machine is uncomfortable.

Ultrasound

Echoes from high-frequency sound waves are used to study blood flow within a vessel.t

Increased ICP is related to

Edema, hemorrhage, impaired cerebral auto regulation, and hydrocephalus

A client arrives at the emergency department complaining of extreme muscle weakness after minimal effort. The physician suspects myasthenia gravis. Which drug will be used to test for this disease?

Edrophonium (Tensilon)

Myasthenic crisis is associated with a positive ________, whereas a cholinergic crisis is associated with ___________.

Edrophonium (Tensilon) test A negative tezt

A client with right leg hemiplegia is experiencing difficulty with mobility. The nurse determines that the family needs reinforcement of teaching if the nurse observes which of the following being done by the family?

Encouraging the client to stand unassisted on the leg

A nurse is admitting a client with Guillain-Barré syndrome to the nursing unit. The client has an ascending paralysis to the level of the waist. Knowing the complications of the disorder, the nurse brings which of the following items into the client's room?

Electrocardiographic monitoring electrodes and intubation tray

Electroencephalogram (EEG)

Electrodes are placed on the scalp to record brain electrical activity. Done to diagnose epilepsy, brain tumor, abscess, or hematoma, and brain death. *Nursing Implications* Noninvasive procedure that does not cause electric shock. Assist the patient to wash electrode paste out of hair.

A nurse reinforces what information to a client who is scheduled for an electromyogram (EMG)?

Electrodes will be injected into the skeletal muscles.

When caring for the client with a herniated lumbar disk, which intervention should the nurse implement? a. Encourage the client to sit in a comfortable chair b. Elevate the client's head and place a small pillow under the knees c. Place the client in a low Fowlers position d. Turn the client once a shift

Elevate the client's head and place a small pillow under the knees. Outcome 5. Rationale: Elevate the client's head and place a small pillow under the knees for a herniated lumbar disk to decrease intervertebral pressure. Sitting in a chair would increase intervertebralpressure. Movement is not contradicted, but elevating the client's head and placing a small pillow under the knees for a herniated lumbar disk will decrease intervertebral pressure. Clients need to be turned more often than once in 8 hours.

A nurse is caring for a client with a T5 complete spinal cord injury. Upon assessment, the nurse notes flushed skin, diaphoresis above T5, and blood pressure of 162/96 mm Hg. The client reports a severe, pounding headache. Which nursing intervention would be appropriate for this client? Select all that apply.

Elevate the head of the bed to 90°. Loosen constrictive clothing. Assess for bladder distention and bowel impaction. Administer antihypertensive medication.

A client undergoes a craniotomy with supratentorial surgery to remove a brain tumor. On the first postoperative day, the nurse notes the absence of a bone flap at the operative site. How should the nurse position the client's head?

Elevated 30 degrees

While bathing a client who sustained a stroke, a nurse is asked by a coworker to assist with repositioning another client. What should the nurse do?

Ensure the client's privacy, put up the side rail, and explain that she'll return shortly.

A client with an altered state of consciousness is fed via __________ because the likelihood of aspiration. The presence of 100ml of residual in an adult usually indicates ___________, and the feeding should be withheld; however, the residual should be returned as it is partially digested

Enteral routes Poor gastric emptying

After a plane crash, a client is brought to the emergency department with severe burns and respiratory difficulty. The nurse helps to secure a patent airway, attends to the client's immediate needs, and then prepares to perform a neurologic assessment. Because the client is unstable and in critical condition, the examination must be brief but will include which nursing intervention?

Evaluation of the corneal reflex response

A client is hospitalized with Guillain-Barré syndrome. Which data collection finding is most significant?

Even, unlabored respirations

A client is brought by ambulance to the ED after suffering what the family thinks is a stroke. The nurse caring for this client is aware that an absolute contraindication for thrombolytic therapy is what?

Evidence of hemorrhagic stroke

A nurse is caring for a client diagnosed with Bell's palsy 1 week ago. Which of the following data would indicate a potential complication associated with Bell's palsy?

Excessive tearing

A nurse is assisting in caring for a client who sustained a traumatic head injury following a motor vehicle accident. The nurse documents that the client is exhibiting decerebrate posturing. The nurse bases this documentation on which observation?

Extension of the extremities and pronation of the arms

After a motor vehicle accident, a client is admitted to the medical-surgical unit with a cervical collar in place. The cervical spinal X-rays haven't been read, so the nurse doesn't know whether the client has a cervical spinal injury. Until such an injury is ruled out, the nurse should restrict this client to which position?

Flat, except for logrolling as needed

A client has been diagnosed as having global aphasia. The nurse recognizes that the client will be unable to perform which action?

Form words that are understandable or comprehend spoken words

Astrocytes

Found primarily in gray matter and provide structural support to neurons

A nurse suspects neurogenic shock in a client with complete transection of the spinal cord at the T3 (thoracic 3) level if which of the following clinical manifestations are observed?

Hypotension and bradycardia

For a client with suspected increased intracranial pressure (ICP), the most appropriate respiratory goal is to:

promote carbon dioxide elimination.

A nurse is caring for an unconscious client who is experiencing persistent hyperthermia with no signs and symptoms of infection. The nurse understands that there may be damage to the client's thermoregulatory center in the:

Hypothalamus

The nurse formulates a nursing diagnosis of Risk for imbalanced body temperature for a client who suffers a stroke after surgery. The expected outcomes incorporate assessment of the client's temperature to detect abnormalities. The thermoregulatory centers are located in which part of the brain?

Hypothalamus

A client with Guillain-Barré syndrome has been asking many questions about the condition, and the nursing staff feels that the client is very discouraged about her condition. It is important for the nurse to include which of the following information in discussions with the client?

Generally, a vast number of people recover from this condition.

A client with a brain attack (stroke) has residual dysphagia. When a diet prescription is initiated, the nurse avoids doing which of the following?

Giving the client thin liquids

What's Guillain-Barre syndrome? What does it involve?

Guillain-Barré syndrome is a clinical syndrome of unknown origin that involves cranial and peripheral nerves. Many clients report a history of respiratory or gastrointestinal infection in the 1 to 4 weeks before the onset of neurological deficits. Occasionally, the syndrome can be triggered by vaccination or surgery.

31 PAIRS CERVICAL 8 PAIRS THORACIC 12 PAIRS LUMBAR 5 PAIRS SACRAL 5 PAIRS COXCYGGAL 1 PAIR

HOW MANY PAIRS OF SPINAL NERVES ARE THERE?

The nurse practitioner advises a patient who is at high risk for a stroke to be vigilant in his medication regime, to maintain a healthy weight, and to adopt a reasonable exercise program. This advice is based on research data that shows the most important risk factor for stroke is:

HTN

A nurse notices that a client with trigeminal neuralgia has been withdrawn, is having frequent episodes of crying, and is sleeping excessively. The best way for the nurse to explore issues with the client regarding this behavior is to:

Have the client express the feelings in writing.

The LPN/LVN knows that an elderly client that is homebound is at risk for safety issues and impaired self-care. This is due to: a. loss of their spouse b. hearing and vision impairment c. lack of motivation due to aging d. inability to socialize

Hearing and vision impairment. Rationale: As clients age, hearing and vision impairment increases the risk for injury and safety issues. Loss of a spouse can be devastating but usually does not increase safety issues as long as the widow/widower is still able to perform ADLs. Lack of motivation and social isolation can be indicators of depression.

What three problems does a pt with Meniere's disease have?

Hearing loss, tinnitus and vertigo

A client in the emergency department has a suspected neurologic disorder. To assess gait, the nurse asks the client to take a few steps; with each step, the client's feet make a half circle. To document the client's gait, the nurse should use which term?

Helicopod

A client is experiencing dysphagia following a stroke. Which measure may be taken by the nurse to ensure that the client's diet allows for easy swallowing?

Help the client sit upright when eating and feed slowly.

What's the difference between hemiparesis and complete hemiparesis?

Hemiparesis is a weakness of one side of the body that may occur after a stroke. Complete hemiparesis is weakness of the face and tongue, arm, and leg on one side.

A client undergoes cerebral angiography for evaluation after an intracranial computed tomography scan revealed a subarachnoid hemorrhage. Afterward, the nurse checks frequently for signs and symptoms of complications associated with this procedure. Which findings indicate spasm or occlusion of a cerebral vessel by a clot?

Hemiplegia Seizures decreased LOC aphasia hemiparesis increased focal symptoms

A client undergoes cerebral angiography to evaluate for neurologic deficits. Afterward, the nurse checks frequently for signs and symptoms of complications associated with this procedure. Which findings indicate spasm or occlusion of a cerebral vessel by a clot?

Hemiplegia, seizures, and decreased level of consciousness (LOC)

A client undergoes cerebral angiography to evaluate for neurologic deficits. Afterward, the nurse checks frequently for signs and symptoms of complications associated with this procedure. Which findings should the nurse notify the physician of because they indicate spasm or occlusion of a cerebral vessel by a clot?

Hemiplegia, seizures, and decreased level of consciousness (LOC)

A nurse is collecting neurological data on a post-stroke adult client. Which of the following techniques will the nurse perform to adequately check proprioception?

Hold the sides of the client's great toe and, while moving it, ask what position it is in.

A client seeking treatment for an episode of hyperthermia is being discharged to home. The nurse determines that the client needs clarification of discharge instructions if the client stated that he or she will:

Resume full activity level immediately.

A nurse is providing education to a community group about ischemic strokes. One group member asks if there are ways to reduce the risk for stroke. Which of the following is a risk factor that can be modified?

Hypertension

Hemorrhagic Stroke reasons for urgent treatment

ICP is increased by space-occupying bleed ruptured intracranial aneurysm must quickly be repaired a ruptured arteriovenous malformation will cause deficits until it is stopped

GLUCOSE, WHICH IS WHAT IS PRESENT IN CSF(CEREBREAL SPINAL FLUID), AND NOTIFY HP IMMEADIATLEY CAUSE IT IS LIFE THREATENING

IF A CLIENT SUSTAINS HEAD TRAUMA AND FLUID IS NOTED LEAKING FROM THE EARS OR NOSE, THIS FLUID WOULD NEED TO BE TESTED FOR WHAT?

A client in a nursing home is diagnosed with Alzheimer's disease. He exhibits the following symptoms: difficulty with recent and remote memory, irritability, depression, restlessness, difficulty swallowing, and occasional incontinence. This client is in what stage of Alzheimer's disease?

II

PERMANENT PARALYSIS OCCURS BELOW THE LEVEL OF THE INJURY

INJURY TO SPINAL CORD WHERE THE CORD IS TRANSECTED WHAT WOULD BE THE RESULT ?

VISCERAL

INVOLUNTARY EFFECTOR. CANNOT BE CONTROLLED BY VOLUNTARY ACTIONS, MAY INCLUDE THE PERISTALTIC MOVEMENT OF FOOD THROUGH THE DIGESTIVE SYSTEM

Keratitis or corneal ulceration

If a patient cannot shut his/her eyes, you must provide care to prevent what?

Parotid gland

If a patient is unconscious, the mouth must be kept clean to avoid infection of the ________ _______.

A client comes to the emergency department after hitting his or her head in a motor vehicle collision. The client is alert and oriented. Which nursing intervention should be done first?

Immobilize the client's head and neck.

A client is receiving an I.V. infusion of mannitol (Osmitrol) after undergoing intracranial surgery to remove a brain tumor. To determine whether this drug is producing its therapeutic effect, the nurse should consider which finding most significant?

Increased Urine Output (therapeutic level of mannitol is diuresis)

When obtaining the vital signs of a client with multiple traumatic injuries, the nurse detects bradycardia, bradypnea, and systolic hypertension. The nurse must notify the physician immediately because these findings may reflect which complication?

Increased intracranial pressure (ICP)

A client is receiving an I.V. infusion of mannitol (Osmitrol) after undergoing intracranial surgery to remove a brain tumor. To determine whether this drug is producing its therapeutic effect, the nurse should consider which finding the most significant?

Increased urine output

A nurse is caring for a client with increased intracranial pressure (ICP). The nurse should monitor for which of the following trends in vital signs that would occur if ICP is rising?

Increasing temperature, decreasing pulse, decreasing respirations, increasing blood pressure (BP)

A nurse notes documentation that a postcraniotomy client is having difficulty with body image. The nurse determines that the client is still working on the postoperative outcome criteria if the client:

Indicates that facial puffiness will be a permanent problem

The nurse is caring for a client who has sustained a spinal cord injury above the sixth cervical vertebrae. Which of the following stimuli could trigger autonomic dysreflexia? a. Bladder distention b. A headache c. Sneezing d. Cold feet

Ineffective Breathing Pattern Rationale: SCI at C4 have a high risk for breathing problems that must be addressed first. Impaired Urinary Elimination will be addressed later, but is not the highest priority on admission. Low Self-Esteem may become an issue later and will be addressed. Risk for Injury: Stress Ulcers will be addressed later, but is not the highest priority on admission.

A client with epilepsy is having a seizure. During the active seizure phase, the nurse should:

place the client on his side, remove dangerous objects, and protect his head.

A nurse is planning discharge for a client who experienced right-sided weakness caused by a stroke. During his hospitalization, the client has been receiving physical therapy, occupational therapy, and speech therapy daily. The family voices concern about rehabilitation after discharge. How should the nurse intervene?

Inform the case manager of the family's concern and provide information about the client's current clinical status so appropriate resources can be provided after discharge.

A client complains of vertigo. The nurse anticipates that the client may have a problem with which portion of the ear?

Inner ear

The nurse is presenting a lecture on vertigo at a community setting. The nurse will include that the client with vertigo may have a problem with which portion of the ear?

Inner ear

A nurse is planning care for a client with Bell's palsy. Which measure should be included in the plan?

Instill artificial tears and wear a patch over the affected eye at night.

Which of the following interventions describes an appropriate bladder program for a client in rehabilitation for spinal cord injury? 1.Insert an indwelling urinary catheter to straight drainage 2.Schedule intermittent catheterization every 2 to 4 hours 3. Perform a straight catheterization every 8 hours while awake 4. Perform Crede's maneuver to the lower abdomen before the client voids.

Intermittent catherization should begin every 2 to 4 hours early in the treatment. When residual volume is less than 400 ml, the schedule may advance to every 4 to 6 hours. Indwelling catheters may predispose the client to infection and are removed as soon as possible. Crede's maneuver is not used on people with spinal cord injury.

A physician diagnoses a client with myasthenia gravis and prescribes pyridostigmine (Mestinon), 60 mg by mouth every 3 hours. Before administering this anticholinesterase agent, the nurse reviews the client's history. Which preexisting condition would contraindicate the use of pyridostigmine?

Intestinal obstruction

intracranial pressure

Intracranial pressure is the pressure that the brain, blood, and CSF exert inside the cerebrospinal cavity.

Cerebral angiogram

Invasive procedure that combines x-ray and fluoroscopy (a radiographic image displayed on a screen) with injection of contrast medium into the vessel to illuminate blood flow through the vessel and evaluate its patency. Used to detect an aneurysm, brain tumor, and stroke. *Nursing Implications* Withhold fluids and food for 8 hours before test. Explain that the patient will have hot flush of head and neck when contrast medium is injected. *clinicalAlErT* Closely monitor neurologic and vital signs; maintain pressure dressing and ice to the injection site. Immediately report bleeding or swelling to the charge nurse and physician.

A client experienced a blow to the right frontal region of the head. If the client begins to develop increased intracranial pressure, which manifestation should the nurse see first? a. Decreased heart rate b. Sluggish response by pupils to light c. Irritability d. Projectile vomiting

Irritability. Rationale: The earliest sign of increased intracranial pressure is level of consciousness. Irritability, personality changes, restlessness, and disorientation are early manifestations of ICP. Decreased heart rate is seen in a later stage of IICP. Slow pupil response is seen in a late, not early, stage of IICP. Projectile vomiting is seen during later, not early, stages of IICP.

What is the function of cerebrospinal fluid (CSF)?

It cushions the brain and spinal cord.

The physician prescribes diazepam (Valium), 10 mg I.V., for a client experiencing status epilepticus. Which statement about I.V. diazepam is true?

It should be administered no faster than 5 mg/minute in an adult.

If a pt presents with cerumen & foreign bodies, what part of the ear is that and what are the s/s?

It's the external ear. S/s: Pain, itching, dizziness, bleeding, +/- hearing loss

Nystagmus (motor system)

Jerking or bobbing of eyes as they track moving objects lesions in cerebellum, brainstem, vestibular system, Anti-seizure drugs, sedatives, hypnotic drug toxicity (including alcohol)

A nurse is assisting in admitting a client who experienced seizure activity in the emergency department. The nurse avoids doing which of the following when managing this client's environment?

Keeping the bed position raised to the nurse's waist level

SPINAL CORD

LONG MASS OF NERVE CELLS & FIBERS EXTENDING THROUGH A CENTRAL CANAL FROM THE BRAINS MEDULLA SPINAL NERVES TO THE APPROX LEVEL OF 1ST OR 2ND LUMBAR VERTEBRAE

Which of the following antiseizure medication has been found to be effective for post-stroke pain?

Lamotrigine (Lamictal)

A nurse is assisting to care for a client who has sustained a nasal fracture. The nurse monitors for which priority finding specifically related to this injury?

Leakage of clear fluid from the nose.

VI Abducens

Lateral movement of the eyeball

A client with meningitis is being prepared for a lumbar puncture. In which position should the nurse place the client for this procedure?

Laterally, with knees drawn up to the abdomen and chin touching the chest

How should the nurse position a client for a lumbar puncture?

Laterally, with knees drawn up to the abdomen and chin touching the chest

A client admitted to an acute care facility after a car accident develops signs and symptoms of increased intracranial pressure (ICP). The client is intubated and placed on mechanical ventilation to help reduce ICP. To prevent a further rise in ICP caused by suctioning, the nurse anticipates administering which drug endotracheally before suctioning?

Lidocaine (Xylocaine)

A client with spinal cord injury is prone to experiencing autonomic dysreflexia. The nurse should avoid which measure to minimize the risk of recurrence?

Limiting bladder catheterization to once every 12 hours

Ependymal cells

Line the brain ventricles and aid in the secretion of CSF

A patient who has suffered a stroke begins having complications regarding spasticity in the lower extremity. What ordered medication does the nurse administer to help alleviate this problem?

Lioresal (Baclofen)

A patient who has suffered a stroke begins having complications regarding spasticity in the lower extremity. What ordered medication does the nurse administer to help alleviate this problem?

Lioresal (Baclofen) Botulinum toxin type A IM into wrist and finger

Magnetic resonance imaging (MRI)

MRI uses a super magnet and radiofrequency signals to elicit a response from hydrogen nuclei. Used to identify stroke, tumor, trauma, multiple sclerosis, and seizures. *Nursing Implications* The patient is not exposed to radiation during an MRI. Assess for metal implants such as pacemaker, shrapnel, or body piercing. Provide teaching, as the experience can be frightening

Which of the following is accurate regarding a hemorrhagic stroke?

Main presenting symptom is an "exploding headache." Not: One of the main presenting symptoms is numbness or weakness of the face. It is caused by a large-artery thrombosis.

Altered level of consciousness; nursing interventions

Maintain airway, respirations , and oxygenation: ❈ position patient in *semi-prone/ three-quarters prone* to prevent tongue from obstructing airway, slightly to one side with arm away from chest wall ❈ insert airway if tongue is obstructing or if patient paralyzed ❈ Prepare for Cuffed ET tube insertion/ attach tube Fluid and electrolyte support ❈ Keep patient NPO until responsive, provide mouth care Q 4 hrs. ❈ Maintain calorie count, monitor I & O Prevent complications also. w/ immobility and contracture ❈ Passive ROM to lower extremity Q 4 hrs, turn Q 2 hrs. ❈ Apply SCD ❈ Apply splints or other assistive device to prevent foot drop, wrist drop, and other improper alignment Urinary Calculi ❈ Increase fluid intake PO or via gastric tube or IV Injury prevention and safety promotion ❈ Approach in a calm manner ❈ Pad side rails, keep side rails up, and place bed in low position, restraint pt if indicated ❈ inform patient about intervention ❈ avoid oversedation (narcotics and sedatives depress responsiveness and affect pupillary reaction) Observe for bladder elimination and record bowel movement, report abnormal pattern ❈ insert indwelling catheter if needed (remove as soon as possible) ❈ initiate bowel regimen (rapid infusion lead to diarrhea, lack of fiber cause constipation) Prevent corneal injury and drying ❈ irrigate eyes, ophthalmic ointment in each eye ❈ help close eyelids if blink reflex is absent

70

Many changes occur with aging, and after age ______ the brain atrophies somewhat.

A nurse observes that a client with Parkinson's disease has very little facial expression. The nurse attributes this piece of data to which of the following?

Mask-like facies is a component of Parkinson's disease.

Name one complication of Otitis Media

Mastoiditis.

A client with a spinal cord injury expresses little interest in food, and is very particular about the choice of meals that are actually eaten. The nurse interprets that:

Meal choices represent an area of client control and should be encouraged as much as is nutritionally reasonable.

A client comes to the emergency department complaining of headache, malaise, chills, fever, and a stiff neck. Vital sign assessment reveals a temperature elevation, increased heart and respiratory rates, and normal blood pressure. On physical examination, the nurse notes confusion, a petechial rash, nuchal rigidity, Brudzinski's sign, and Kernig's sign. What does Brudzinski's sign indicate?

Meningeal irritation

A client was admitted with an injury to the occipital lobe. Which nursing action should the nurse perform?

Monitor client for visual disturbances.

A client is admitted with myasthenia gravis. Which nursing intervention should be priority?

Monitor respiratory status.

A thymectomy via a median sternotomy approach is performed on a client with a diagnosis of myasthenia gravis. The nurse has assisted in developing a plan of care for the client and includes which of the following in the plan?

Monitor the chest tube drainage.

A nurse develops a plan of care for a client following a lumbar puncture. Which interventions should be included in the plan? Select all that apply.

Monitor the client's ability to void. Maintain the client in a flat position. Monitor the client's ability to move the extremities. Inspect the puncture site for swelling, redness, and drainage.

After a lumbar puncture, which nursing action should the nurse implement? a. Remind the client not to move legs after the procedure b. Monitor the puncture site for CSF leakage c. Have the client empty his bladder d. Limit the client's fluid intake 4. What teaching point should the nurse emphasize to a client taking phenytoin (Dilantin) on a daily basis? a. Check urine for brownish color b. Report fatigue and weakness c. Do not take with food d. Brush and floss daily

Monitor the puncture site for CSF leakage. Rationale: It is most important to monitor for CSF leak and hematoma formation. Client is kept flat afterward for 4 to 24 hours but can move the legs. Fluids are increased, not limited. Client should empty bladder before the procedure; not necessary afterward.

A nurse is preparing a plan of care to monitor for complications in a client who will be returning from the operating room following transsphenoidal resection of a pituitary adenoma. Which of the following does the nurse document in the plan as the priority nursing intervention for this client?

Monitor urine output.

Which nursing intervention can prevent a client from experiencing autonomic dysreflexia?

Monitoring the patency of an indwelling urinary catheter

MS

Multiple sclerosis is one of the most common nerve disorders in the United States, typically affecting young adults and people living in northern temperature climates.With recurrent episodes, increasingly severe symptoms, such as paralysis, dysphagia (difficulty in swallowing), and bladder and bowel dysfunctions develop.

A quadriplegic client is prescribed baclofen (Lioresal), 5 mg by mouth three times daily. What is the principal indication for baclofen?

Muscle spasms with paraplegia or quadriplegia from spinal cord lesions

The nurse is monitoring a client for adverse reactions to dantrolene (Dantrium). Which adverse reaction is most common?

Muscle weakness

MEDULLA OBLONGATE, PONS, MIDBRAIN

NAME THREE PARTS OF THE BRAIN STEM

* REFLEX CENTER BY SENDING & RECEVING MESSAGES THROUGH NERVE FIBERS (SENSORY TO BRAIN/MOTOR AWAY FROM BRAIN) *CONDUCT IMPLULESES TO & FROM THE BRAIN

NAME TWO FUNCTIONS OF SPINAL CORD?

DENDRITE

NERVE BRANCH THAT CONDUCTS IMPULSES TOWARD THE CELL BODY

SPINAL NERVES

NERVES THAT ORIGINATE IN THE SPINAL CORD

EFFECTORS

NEURONS THAT CARRY OUT ACTIVITY IN RESPONSE TO MESSAGES RELAYED BY SENSORY NEURONS

scale screening tool used acutely following stroke

NIHSS national institutes of health stroke scale stuff like level of conscioiusness, best gaze,visual fields, facial palsy, limb ataxia, snesory, best language, dysarthria, extinction, and inattention takes 5-8min to complete

XI Accessory

Neck and shoulder movement

The nurse is caring for a patient with dysphagia. Which of the following interventions would be contraindicated while caring for this patient?

placing food on the affected side of mouth

Electromyogram (EMG)

Needles are inserted into skeletal muscles (as on the legs) to record electrical activity. Used to diagnose such disorders as multiple sclerosis, myasthenia gravis, and spinal cord injury or disease. *Nursing Implications* Explain that there is slight discomfort when the needles are inserted. Tell the patient to avoid caffeine or nicotine for 3 hours before the test.

A nurse is preparing to care for a client with a diagnosis of brain attack (stroke). The nurse notes in the client's record that the client has anosognosia. The nurse plans care, knowing that the client will:

Neglect the affected side.

Brain Tumor

Neoplasm occurring in the brain ❈ Primary tumors can arise in any tissue of the brain ❈Secondary tumors: result of metastasis from other areas (most commonly lungs, followed by breast) ❈ *Benign as well as malignant tumors lead to death* without treatment

The elderly client has had a recent stroke causing drooling and the inability to raise the eyebrow on one side. The nurse knows that which two cranial nerves have been affected? a. Nerves I and IV b. Nerves V and VIII c. Nerves III and VII d. Nerves II and IX

Nerves III and VII. Rationale: Cranial nerve III is the Oculomotor nerve controlling pupil constriction, eyeball movement and raising of upper eyelid, while cranial nerve VII is the Facial nerve controlling movement of facial muscles, secretions from lacrimal and salivary glands and taste in anterior two-thirds of tongue,

The LPN/LVN must add a thickener to all liquids given to a client with dysphagia. The nurse knows that what cranial nerves are involved? a. Nerves IX and X b. Nerves IV and II c. Nerves I and VI d. Nerves V and VI

Nerves IX and X Rationale: Cranial nerve IX is the Glossopharyngeal controlling swallowing, gag reflex, sensation of pharynx and tongue, taste in posterior one-third of tongue, and secretions of parotid gland while Cranial nerve X is the Vagus nerve which controls swallowing.

A patient is brought to the emergency department with a possible stroke. What initial diagnostic test for a stroke, usually performed in the emergency department, would the nurse prepare the patient for?

Noncontrast computed tomogram

A client is suspected of having had a stroke. Which is the initial diagnostic test for a stroke?

Noncontrast computed tomography

Normal cerebrospinal fluid

Normal is clear colorless odorless free of red blood cells contains little protein

A client who experienced a severe stroke develops a fever and a cough that produces thick, yellow sputum. A nurse observes sediment in the client's urine in the indwelling urinary catheter tubing. Based on these findings, which action should the nurse take?

Notify a physician of the findings.

A nurse is caring for a client with a closed head injury. What is the appropriate action by the nurse?

Notify the health care provider of a blood pressure change from 147/72 mm Hg to 176/70 mm Hg.

During the course of a busy shift, a nurse fails to document that a client's ventricular drain had an output of 150 ml. Assuming that the drain was no longer draining cerebrospinal fluid, the physician removes the drain. When the nurse arrives for work the next morning, she learns that the client became agitated during the night and his blood pressure became elevated. What action should the nurse take?

Notify the physician of the documentation omission.

A client who recently experienced a stroke tells the nurse that he has double vision. Which nursing intervention is the most appropriate?

Notify the physician.

An adult client was admitted with myasthenia gravis. While reviewing the client's chart, the licensed practical nurse (LPN)/licensed vocational nurse (LVN) noticed the medication administration record (MAR). Based on the information, what should the nurse do next?

Notify the registered nurse and question the morphine sulfate.

A client recovering from a craniotomy complains of a "runny nose." Based on the interpretation of the client's complaint, the best nursing action is to:

Notify the registered nurse.

A white female client is admitted to an acute care facility with a diagnosis of stroke. Her history reveals bronchial asthma, exogenous obesity, and iron deficiency anemia. Which history finding is a risk factor for stroke?

Obesity

A nurse is collecting data on a client with a diagnosis of meningitis and notes that the client is assuming this posture. (Refer to figure.) The nurse contacts the health care provider and reports that the client is exhibiting:

Opisthotonos

opisthotonos

Opisthotonos, an acute spasm in which the body is bowed forward, with the head and heels bent backward, is often present. Children have tense or bulging fontanels and a high-pitched cry.

The physician determines that a client's chronic, progressive hearing loss results from excess bone formation around the oval window, which impedes normal stapes movement and prevents sound transmission. What is the clinical term for this correctable middle ear disorder?

Otosclerosis

CEREBELLUM

PART OF THE BRAIN LOCATED ON THE BACK OF THE BRAIN STEM. HAS 3 LOBES (1 MEDIAN"VERMIS"), 2 LATERAL HEMISPHERES

Paraplegia

Paralysis of lower extremities spinal cord transection or mass lesion (thoracolumbar region)

edema

Paralyzed extremities are susceptible to _________ and should be elevated when the patient is at rest.

paraplegia

Paraplegia means paralysis of the legs and lower body; it usually results from injury to the cord below the first thoracic vertebra.

parkinsonism

Parkinson disease, also called parkinsonism, is second only to Alzheimer disease as the most common neurologic disease in older adults.

A clinic nurse is reviewing the medical record of a client scheduled to be seen in the clinic. The nurse notes that the client is prescribed selegiline hydrochloride (Eldepryl). The nurse understands that this medication is prescribed for which diagnosis?

Parkinson's disease

After striking his head on a tree while falling from a ladder, a young man is admitted to the emergency department. He's unconscious and his pupils are nonreactive. Which intervention would be the most dangerous for the client?

Perform a lumbar puncture.

Parasympathetic nervous system

Preganglion cell bodies are located in brainstem and sacral spinal segments

A client has had an ischemic stroke and has been admitted to the medical unit. What action should the nurse perform to best prevent joint deformities?

Place a pillow in the axilla when there is limited external rotation.

After a craniotomy for removal of a brain tumor located in the occipital lobe, what nursing intervention should the nurse implement? a. Place the client in a side-lying position b. Monitor for signs of cranial nerve impairment c. Give morphine for headache d. Monitor vital signs every four hours

Place in a side-lying position. Rationale: Removing a tumor from the occipital lobe requires the client to be placed in a sidelying position. Assessing for cranial nerve impairment is important after carotid artery surgery. Morphine is never given after a craniotomy, as it could mask signs of neurologic deterioration. Only Tylenol or codeine is given for headache. Vital signs must be monitored every 1-2 hours.

A client is diagnosed with a right-sided stroke. The client is now experiencing hemianopsia. How might the nurse help the client manage her potential sensory and perceptional difficulties?

Place the client's extremities where she can see them.

The nurse is caring for a client with dysphagia. Which intervention would be contraindicated while caring for this client?

Placing food on the affected side of the mouth

When the nurse taps at the level of the client's facial nerve, the following response is noted (refer to figure). How should the nurse document this finding on the client record?

Positive Chvostek's sign

A client complains of pain in the lower back and pain and spasms in the hamstrings when the nurse attempts to extend the client's leg. (Refer to figure.) How should the nurse record this finding on the client's medical record?

Positive Kernig's sign

A nurse is collecting admission data on a client with Parkinson's disease. The nurse asks the client to stand with the feet together and the arms at the side and then to close the eyes. The nurse notes that the client begins to fall when the eyes are closed. Based on this finding, the nurse documents which of the following in the client's record?

Positive Romberg's test

A client with amyotrophic lateral sclerosis (ALS) tells the nurse, "Sometimes I feel so frustrated. I can't do anything without help!" This comment best supports which nursing diagnosis?

Powerlessness

A client is scheduled for electroconvulsive therapy (ECT). Before ECT begins, the nurse reviews the client's chart and prepares to administer which neuromuscular blocking agent?

Succinylcholine

A nurse is preparing to care for a client following a lumbar puncture. The nurse plans to place the client in which position immediately after the procedure?

Prone with a pillow under the abdomen

A client has just undergone lumbar puncture (LP). The nurse assists the client into which most optimal position if tolerated by the client?

Prone, with a pillow under the abdomen

A client admitted to the hospital with a neurological problem indicates to the nurse that magnetic resonance imaging (MRI) may be done. The nurse interprets that the client may be ineligible for this diagnostic procedure based on the client's history of:

Prosthetic valve replacement

What's the immediate priority for a nurse to do when a patient is experiencing a tonic-clonic seizure?

Protecting the client from injury is the immediate priority during a seizure.

The client has been diagnosed with aphasia after suffering a stroke. What can the nurse do to best make the client's atmosphere more conducive to communication?

Provide a board of commonly used needs and phrases.

A client has an impairment of cranial nerve II. Specific to this impairment, the nurse would plan to do which of the following to ensure client safety?

Provide a clear path for ambulation without obstacles.

A client who suffered a cervical spine injury had Crutchfield tongs applied in the emergency department. The nurse would avoid which of the following actions in the care of the client?

Removing the weights when repositioning the client

A nurse is monitoring a client with a spinal cord injury who is experiencing spinal shock. Which of the following will provide the nurse with the best information about recovery from the spinal shock?

Reflexes

A nurse has instructed the family of a brain attack (stroke) client who has homonymous hemianopsia about measures to help the client overcome the deficit. The nurse determines that the family understands the measures to use if they state that they will:

Remind the client to turn the head to scan the lost visual field.

A nurse is caring for a client who begins to experience seizure activity while in bed. Which action by the nurse would be contraindicated?

Restrain the client's limbs.

A nurse in the ICU is providing care for a client who has been admitted with a hemorrhagic stroke. The nurse is performing frequent neurologic assessments and observes that the client is becoming progressively more drowsy over the course of the day. What is the nurse's best response to this assessment finding?

Report this to the health care provider as a possible sign of clinical deterioration.

An emergency department nurse is assigned to assist in caring for a client who has suffered a head injury following a motor vehicle accident. The nurse understands that the initial data collection should focus on which of the following?

Respiratory status

tonic phase

Rigid contraction of body muscles (tonic phase). The tonic-clonic or grand mal seizure is perhaps the most life-threatening type of seizure.

When caring for a client with a head injury, the nurse must stay alert for signs and symptoms of increased intracranial pressure (ICP). Which cardiovascular findings are late indicators of increased ICP?

Rising blood pressure and bradycardia

Which nursing diagnosis has the highest priority for a client with status epilepticus? a. Anxiety b. Risk for Injury c. Risk for Ineffective Airway Clearance d. Risk for Ineffective Cerebral Tissue Perfusion

Risk for Ineffective Airway Clearance.

A client admitted with a cerebral contusion is confused, disoriented, and restless. Which nursing diagnosis takes highest priority?

Risk for injury related to neurologic deficit

A nurse has given instructions to the client with Parkinson's disease about maintaining mobility. The nurse determines that the client understands the directions if the client states that he or she will:

Rock back and forth to start movement with bradykinesia.

ACTION POTENTIAL

STATE THAT RESULTS WHEN A STIMULUS CAUSES AN ORGANIZED, RAPID EXCHANGE OF SODIUM & POTASSIUM IONS ACROSS A CELL MEMBRANE. WHICH SPREADS LIKE AN ELECTRIC CURRENT ALONG THE MEMBRANE

NEUROGLIA

SUPPORTING STRUCTURE OF NERVE TISSUE

SNS

SYMPATHETIC NERVOUS SYSTEM

A client is scheduled for electroconvulsive therapy (ECT). Before ECT begins, the nurse expects which neuromuscular blocking agent to be administered?

Succinylcholine (Anectine)

The nurse is caring for a patient having a hemorrhagic stroke. What position in the bed will the nurse maintain this patient?

Semi-Fowler's

A nurse is assisting with caring for a client after a craniotomy. The nurse plans to position the client in a:

Semi-Fowler's position

A client is about to undergo a lumbar puncture (LP). The nurse tells the client that which of the following positions will be used during the procedure?

Side-lying with the legs pulled up and the head bent down onto the chesT

A client is having a lumbar puncture (LP) performed. The nurse would place the client in which position for the procedure?

Side-lying, with legs pulled up and head bent down onto the chest

nuchal rigidity

Signs and symptoms include fever, chills, severe headache, nausea and vomiting, nuchal rigidity (stiff neck), and irritability. A change in LOC is present. Two neurologic signs are present: positive Kernig sign and positive Brudzinski.

focal point

Surgery may be performed in certain circumstances in which the seizure's focal point can be clearly identified in the brain.

status epilepticus

Status epilepticus refers to the occurrences of a single, unremitting seizure that lasts longer than 5 minutes or frequent clinical seizures without an interictal return to baseline.

A pt presents with (uroliathiasis) sudden onset of severe right flank pain. What's the nurse's highest priority?

Strain all urine through several layers of gauze to collect any stones that might have been in the urine.

Altered consciousness

Stuporous mute diminished response to verbal cues or pain Intracranial lesions metabolic disorder psychiatric disorders

When assisting with the education of the family of a client with C4 quadriplegia on how to perform tracheostomy suctioning, which instruction should the nurse be sure to include?

Suction for 10 to 15 seconds at a time.

A nurse is preparing for the admission of a client with a prescription for seizure precautions. Which supplies will the nurse make available to this client? Select all that apply.

Suction machine Oxygen administration Padding for the side rails Prescribed diazepam (Valium)

A client with respiratory complications of multiple sclerosis (MS) is admitted to the medical-surgical unit. Which equipment is most important for the nurse to keep at the client's bedside?

Suction machine with catheters

craniotomy

Surgical entry into the skull (cranium) is called a craniotomy. This invasive procedure is performed for many reasons; one of the most common being a brain tumor.

IX Glossopharyngeal

Swallowing Gag reflex Sensation of pharynx and tongue Taste in posterior one-third of tongue Secretions of parotid gland

The nurse is preparing discharge teaching for a patient who is being discharged after hospitalization for a hemorrhagic stroke. What should be included in the discharge teaching for this patient?

Take antihypertensive medication as ordered.

vertigo

Symptoms include headache, anxiety, fatigue, or vertigo (a sensation of rotation of self or one's surroundings; not true dizziness).

The nurse is preparing health education for a client who is being discharged after hospitalization for a hemorrhagic stroke. What content should the nurse include in this education?

Take antihypertensive medication as prescribed.

CORPUS COLLOSUM

THE CEREBRAL HEMISPHERES ARE CONNECTED BY _________________________? WHICH ALLOWS THE HEMISPHERES TO SHARE INFO WITH EACH OTHER

THALMUS AND HYPOTHALMUS

THE DIENCEPHALON PORTION OF THE FOREBRAIN CONSIST OF ____________________& ________________________?

SYNAPSE

THE FUNCTIONAL JUNCTION BETWEEN 2 NEURONS(NERVE CELLS) AT WHICH POINT THE IMPULSE IS TRANSMITTED

CRANIAL NERVES SPINAL NERVES

THE PERIPHERAL NERVOUS SYSTEM CONSIST OF THE ANS, AND, _____________________&_____________________

A client is hospitalized when they present to the Emergency Department with right-sided weakness. Within 6 hours of being admitted, the neurologic deficits had resolved and the client was back to their presymptomatic state. The nurse caring for the client knows that the probable cause of the neurologic deficit was what?

TIA

HYPOTHALMUS

TINY BUT COMPLEX PORTION OF THE BRAIN BELIEVED TO BE THE "MASTER CONTROLLER" OF THE HORMONES

True or False: the nurse should establish the pt's (with Alzhemier's and urinary incontinence) voiding pattern before implementing a plan for urinary training ?

TRUE. It's best to monitor and document for 3 days to determine the pt's voiding pattern.

The nurse is monitoring a client for adverse reactions to atropine sulfate (Atropine Care) eyedrops. Systemic absorption of atropine sulfate through the conjunctiva can cause which adverse reaction?

Tachycardia

A nursing student is collecting data on a client recently diagnosed with meningitis. The student expects to note which of the following signs and symptoms? Select all that apply.

Tachycardia Photophobia Red, macular rash Positive Kernig's sign

A nurse is providing a list of instructions to a client who is scheduled to have an electroencephalogram (EEG). Choose the instructions that the nurse places on the list. Select all that apply.

Tea and coffee are restricted on the day of the test. The test will take between 45 minutes and 2 hours. The hair should be washed the evening before the test.

A client is admitted with a spinal cord injury at C4. Which of the following nursing diagnoses is the highest priority for this client? a. Ineffective Breathing Pattern b. Impaired Urinary Elimination c. Low Self-Esteem d. Risk for Injury: Stress Ulcers

Teach client how to reduce frequency of attacks Rationale: Client education is a priority because most headaches are treatable at home. Helping a client understand what causes the migraines may help reduce the number of attacks. Diaries may be useful, but the priority is teaching how to reduce frequency. Comfort measures are important, but the priority is teaching how to reduce frequency of attacks. Stress reduction classes are also important, but the priority is teaching how to reduce frequency of attacks.

The parent of a child with a ventriculoperitoneal shunt calls the nurse saying that the child has a temperature of 101.2° F (38.4° C), a blood pressure of 108/68 mm Hg, and a pulse of 100 beats/minute. The child is lethargic and vomited the night before. Other children in the family have had similar symptoms. Which nursing intervention is most appropriate?

Tell the parent to bring the child to the primary health care provider's office.

The nurse is discussing the purpose of an electroencephalogram with the family of a client who has massive cerebral hemorrhage and loss of consciousness. Which response by the nurse would be the most accurate in describing what the test measures?

activity of the brain

tetraplegia

Tetraplegia (also referred to as quadriplegia) means paralysis to the cervical segments of nerves from C1 to C8. resulting from impaired function of the upper extremities, trunk (including respiratory involvement), pelvic organs, and lower extremities.

Contusion

actual bruising and bleeding within the cranial vault into the brain tissue

A client who's paralyzed on the left side has been receiving physical therapy and attending teaching sessions about safety. Which behavior indicates that the client accurately understands safety measures related to paralysis?

The client uses a mirror to inspect his skin.

A nurse is caring for a client with a head injury and is monitoring the client for signs of increased intracranial pressure (ICP). Which sign if noted in the client would the nurse report immediately?

The client vomits.

PNS

The ______ is composed of the sensory organs, 12 pairs of cranial nerves, and 31 pairs of spinal nerves and ganglia.

Parasympathetic

The ___________ nerves conserve and restore energy used to maintain homeostasis.

A nurse is caring for a client that is comatose and notes in the client's chart that the client is exhibiting decerebrate posturing. The nurse understands that decerebrate posturing is characterized by:

The extension of the extremities and pronation of the arms

HTN and stroke risk

ppl with htn 140/90 the cutoff twice the lifetime risk of stroke

A client with a T4 spinal cord injury is to be monitored for autonomic dysreflexia (hyperreflexia). Which finding is indicative of this complication?

The client complains of a headache and the blood pressure is elevated.

The nurse is caring for a client with a T5 complete spinal cord injury. Upon assessment, the nurse notes flushed skin, diaphoresis above the T5, and a blood pressure of 162/96. The client reports a severe, pounding headache. Which of the following nursing interventions would be appropriate for this client? Select all that apply. 1.Elevate the HOB to 90 degrees 2.Loosen constrictive clothing 3.Use a fan to reduce diaphoresis 4. Assess for bladder distention and bowel impaction 5. Administer antihypertensive medication

The client has signs and symptoms of autonomic dysreflexia. The potentially life-threatening condition is caused by an uninhibited response from the sympathetic nervous system resulting from a lack of control over the autonomic nervous system. The nurse should immediately elevate the HOB to 90 degrees and place extremities dependently to decrease venous return to the heart and increase venous return from the brain. Because tactile stimuli can trigger autonomic dysreflexia, any constrictive clothing should be loosened. The nurse should also assess for distended bladder and bowel impaction, which may trigger autonomic dysreflexia, and correct any problems. Elevated blood pressure is the most life-threatening complication of autonomic dysreflexia because it can cause stroke, MI, or seizures. If removing the triggering event doesn't reduce the client's blood pressure, IV antihypertensives should be administered. A fan shouldn't be used because cold drafts may trigger autonomic dysreflexia.

A client with quadriplegia complains bitterly about the nurse's slow response to the call light and the rigidity of the therapy schedule. Which interpretation of this behavior would serve as a basis for planning nursing care?

The client is reacting to loss of control.

A client who suffered an ischemic stroke now has disturbed sensory perception. What principle should guide the nurse's care of this client?

The client should be approached on the side where visual perception is intact.

A client recovering from a stroke has severe shoulder pain from subluxation of the shoulder. To prevent further injury and pain, the nurse caring for this client is aware of what principle of care?

The client should be taught to interlace fingers, place palms together, and slowly bring scapulae forward to avoid excessive force to shoulder.

factors affecting stroke survival rates

age, HTN, heart disease, diabetes LOC at onset of stroke, lesion size, persistance of severe hemiplegia, hx of previous stroke, multiple # of neurological defecits

what term refers to the failure to recognize familiar objects perceived by the senses?

agnosia

what term refers to disturbances in writing intelligible words

agraphia

aura

The person may have sensory warnings or premonitions (aura) that a headache will occur. Various auras include mood changes, anorexia, numbness of a body part, or visual symptoms, such as flashing lights or floating spots.

restorative interventions

aimed at improving impairments, activity limitations, and participation restrictions

preventative interventions

aimed at minimizinf potential complications and indirect impairments

compensatory interventions

aimed at modifying tasks or activities to improve function

primary cerebral hemorrhage

aka nontraumatic spontaneous hemorrhage typically occurs in small blood vessels weakened by atherosclerosis produces aneurysm

ipsilateral pushing

aka pusher syndrome pusher love girl, contraversive pushing unusual motor behavior characterized by active pushing with the stronger extremities toward the hemiparetic side with a lateral postural imbalance

flaccidity

The seizure may commence with the person falling down because of brief loss of muscle tone (flaccidity).

Treatment for Tic Douloureux

alcohol or phenol injection directly into the nerve causing localized temporary paralysis

Cerebral Vascular Accident (CVA)

also called a "stroke," a disruption in the cerebral blood flow, either ischemic or hemorrhagic, resulting in brain injury and tissue death

clonic phase

alternates with rhythmic jerking movements (clonic phase), follows.

bradykinesia

This progressive brain disease is characterized by bradykinesia (slowness of movement) and fine, rhythmic tremors of the hands, arms, legs, jaw, and face. The limbs and trunk become rigid and stiff.

A nurse is gathering neurological data from a client. To determine if a Babinski reflex is present, indicate the point where the nurse places the tongue blade to begin stroking the foot.

To test for a Babinski reflex, use a tongue blade to slowly stroke the side of the sole of the foot. Start at the heel and move toward the great toe. The normal response in an adult is plantar flexion of the toes. Upward movement of the great toe and fanning of the little toes—Babinski's reflex—is abnormal in clients over the age of 12 to 18 months.

Is the following statement true or false? When caring for a client with cardiovascular accident (CVA) and hemiplegia, the client should be encouraged to use the trapeze bar on the bed.

True Frequent position changes help to prevent disorders caused by immobility-disuse disorders. Use of trapeze bar allows the client to assist with position changes and to reduce the shearing forces that can lead to skin breakdown. Mobility will prevent respiratory complications and will help the client to maintain self-esteem. Splints or high-top sneakers aid in maintaining normal anatomical alignment of the joints.

A common cause of death after spinal cord injury is ________________. Bacteria grow best in alkaline media, so keeping urine *diluted and acidic* is prophylactic against infection. ___________ assists in avoiding bacterial growth in urine that has stagnated in the bladder

Urinary tract infection Keeping the bladder emptied

Tensilon Test

Used to diagnose MG and to differentiate between myasthenic crisis and cholinergic crisis. Tensilon acts systemically to increase muscle strength; it lasts several minutes. Tensilon produces a brief increase in muscle strength; with a negative response the client will demonstrate no change in symptoms. Tensilon may intensify muscle weakness in a cholinergic crisis. Tensilon does not cause lasting effects. Tensilon acts systemically on all muscles, rather than selectively on the eyelids.

Radiography

Used to identify neurologic abnormalities such as lesions and tumors; when combined with use of an injected contrast medium, used to study blood flow through vessels. *Nursing Implications* If contrast medium is used, ask about allergies to iodine and seafood before the exam; ensure good hydration before and after the exam to reduce the risk of kidney damage. Although noninvasive, radiography exposes the patient to potentially damaging radiation. Ask women of childbearing age about possible pregnancy before the exam.

A nurse is assisting with the development of a teaching plan for a client who will undergo a stapedectomy for the treatment of otosclerosis. Which instruction should the plan include?

Vertigo and dizziness are common after surgery.

Evoked potentials

Visual or auditory stimulus evokes electrical activity related to nerve conduction along sensory pathways. Electrodes on the scalp and skin record the activity level. Done to diagnose multiple sclerosis, acoustic neuroma, Parkinson disease, spinal cord disease, or blindness. *Nursing Implications* Instruct the patient to wash hair before the exam.

A 58-year-old client complaining of difficulty driving at night states that the "lights bother my eyes." The client wears corrective glasses. The nurse would expect the physician to have the client increase his or her intake of which vitamin because the client is experiencing a deficiency in this vitamin?

Vitamin A

EXTEROCEPTORS (SENSORY) -EXTERNAL STIMUILI, (TOUCH, PAIN, HEAT, SMELL VISION, HEARING ETC) PROPRIOCEPTORS (MOTOR)- POSITION, BAL, MOVMENT INTEROCEPTORS - INTERNAL STIMULI (VISERAL PAIN, HUNGER, OR THIRST

WHAT ARE THE 3 CLASSIFICATION OF RECEPTORS

DURA MATER, ARACHNOID, PIA MATER

WHAT ARE THE 3 MENINGES?

MENINGES, CSF, VENTRICLES

WHAT ARE THE ACCESSORY STRUCTURES TO THE CNS?

DECREASED NUMBER OF NEURONS SLOWER RESPONSE TIME AND VOLUNTARY MOVEMENTS MORE ACCIDENTS SLOWED REFLEXES & DECISION MAKING SLOWED THERMOREGUALTION SHORT TERM MEMORY LOSS

WHAT ARE THE AFFECTS ON THE NERVOUS SYSTEM WITH AGING?

* HELP TO FORM THE BLOOD-BRAIN BARRIER, THE CSF, AND THE MYELIN SHEATH *OBTAIN NUTRIENTS FOR THE NEURONS *SUPPORTING AND PROTECTING THE CNS & PNS

WHAT ARE THE FUNCTIONS OF NEUROGLIA " GLIAL CELLS"

CONNECTS THE CEREBRAL HEMISPHERES WITH THE SPINAL CORD CONTAINS THE CENTERS FOR MANY VITAL BODY FUNCTIONS (HR, BP, RR) & REFLEXES(SWALLOWING, COUGHING AND SNEEZING)

WHAT ARE THE FUNCTIONS OF THE BRAIN STEM?

BONE, MENINGES & FLUID

WHAT ARE THE PROTECTIVE COVERINGS OF THE BRAIN?

SYMPATHETIC DIVISION PARASYMPATHETIC DIVISON

WHAT ARE THE TWO DIVISIONS OF THE ANS(AUTONOMIC NERVOUS SYSTEM?

* HIPPOCAMPUS (LEARNING&LONGTERM MEMORY) * RAS(RETICULAR FORMATION SYSTEM) (YOUR WAKEFULNESS & SLEEP)

WHAT ARE TWO PARTS OF THE LIMBIC SYSTEM?

VAGUS

WHAT CRANIAL NERVE IS X (10)? (MIXED NERVE) SENSORY & MOTOR CARRIES MOTOR IMPULSES TO AND SENSORY IMPULES FROM MOST ORGANS IN THORACIC CAVITIES

III, IV, VI(3, 4, &6)

WHAT CRANIAL NERVES MOVE THE EYES?

EEG (ELECTROENCEPHALOGRAM)

WHAT DIAGNOSTISTIC TEST WOULD YOU USE TO DIAGNOSIS NEUROLOGICAL PROBLEMS(EX. SEIZURES)

CRAINIAL /SPINAL NERVES, AND ANS(AUTONOMIC NERVOUS SYSTEM)

WHAT DOES PNS(PERIPHERIAL NERVOUS SYSTEM) CONSIST OF?

OXYGEN

WHAT DOES THE BRIAN REQUIRE AND SHOULD NOT EXCEED 10 SECS OR CAN RESULT IN UNCONSCIOUSNESS?

HYPOTHALMUS

WHAT IN DIENCEPHALON REGULATES VISCERAL ACTIVITY, INCREASES OR DECREASES BODY FUNCTIONS; RELEASE OF HORMONES FROM THE PITUITARY GLAND, EMOTIONS, DIGEST, WATER BALANCE, THIRST AND HUNGER ,

THALMUS

WHAT IN THE DIENCEPHALON SERVES AS A RELAY STATION BETWEEN THE SKIN RECEPTORS AND THE CEREBRAL CORTEX?

OLFACTORY

WHAT IS CRANIAL NERVE I(1), RESPONSIBLE FOR SMELL, SENSORY NERVE

OCULOMOTOR

WHAT IS CRANIAL NERVE III(3)? RESPONSIBLE FOR MOVEMENT OF THE EYE AND UPPER EYELID, MOTOR NERVE

GLOSSOPHARYNGEAL

WHAT IS CRANIAL NERVE IX(9)? (MIXED NERVE) SENSORY&MOTOR M: SWALLOWING AND SPEECH, SALIVARY GLANDS S: ALL SENSATIONS INCLUDING TASTE ON BACK 1/3 OF TONGUE

ABDUCENS

WHAT IS CRANIAL NERVE VI(6)? MOTOR NERVE, MOVES THE EYES SIDE TO SIDE EX: IF THIS NERVE WAS CLIPPED EYE IS STUCK LOOKING INWARD.

FACIAL

WHAT IS CRANIAL NERVE VII(7)? MIXED NERVE(SENSORY&MOTOR) S: TASTE FROM ANTERIOR 2/3 OF TONGUE M: MOVEMENT OF FACE, SCALP, NECK MUSCLES, TEAR AND SALIVARY GLANDS

(SPINAL) ACCESSORY

WHAT IS CRANIAL NERVE XI(11) ? MOTOR NERVE HEAD & SHOULDER MOVEMENTS

HYPOGLOSSAL

WHAT IS CRANIAL NERVE XII(12)? MOTOR NERVE MOVEMENT OF TONGUE DURING SWALLOWING AND SPEECH

BETWEEN THE CEREBRUM AND THE INNER BRAIN

WHERE IS YOUR LIMBIC SYSTEM LOCATED?

PIA MATER(INNER MOST LAYER)

WHICH MENINGES&LAYER PROVIDES NUTRIENTS TO THE BRAIN?

Which one of the following self-care strategies should the nurse teach to the client with trigeminal neuralgia? a. Increase fluid intake b. Chew on the affected side c. Monitor calorie intake d. Ways to avoid trigger points

Ways to avoid trigger points. Outcome 5. Rationale: Clients must understand that stimulating trigger points can initiate an attack. They are taught to chew on the unaffected side unless they cannot eat; then, a high-calorie, high-protein diet is needed. Clients with trigeminal neuralgia would not be taught to monitor calories but to ingest high-protein, high-calorie foods. Fluids do not need to be increased.

A client with diplopia has been taught to use an eye patch to promote better vision and prevent injury. The nurse determines that the client understands how to use the patch if the client states that he or she will:

Wear the patch continuously, alternating eyes each day.

Ascending tracts

carry specific sensory information to higher levels of CNS sensory receptors - skin - muscle joints viscera blood vessels deep pressure vibration position sense kinesthesia

Subarachnoid space

What is located between the pia mater and the arachnoid membrane and is where CSF circulates?

15

What is the optimal GCS (Glasgow Coma Scale) score?

GCS (Glasgow Coma Scale)

What is usually used to determine a patient's LOC?

Narcotic analgesics

When a patient has suffered a head injury or is experiencing increasing ICP, ________ _________ may not be given, because they mask the signs of rising ICP.

A client reports light-headedness, speech disturbance, and left-sided weakness lasting for several hours. The neurologist diagnosed a transient ischemic attack, which caused the client great concern. What would the nurse include during client education?

When symptoms cease, the client will return to presymptomatic state.

An older gentleman is brought to the emergency department by a neighbor who heard him talking and wandering in the street at 3 AM. The nurse should first determine which of the following about the client?

Whether this is a change in his usual level of orientation

EEG (Electroencephalography)

Which diagnostic test might be used to detect abnormal brain wave patterns that are indicative of specific diseases, such as a seizure disorder?

Lumbar puncture

Which diagnostic test might be used to determine whether CSF pressure is elevated?

Thalamus

Which division of the brain functions as the relay center between the spinal cord and cerebrum?

Cerebellum

Which division of the brain is responsible for coordination of movement, posture, and muscle tones that are the mechanisms of balance?

Pia mater

Which layer of the meninges covers the brain?

Arachnoid

Which layer of the meninges encases the entire CNS?

Pons

Which part of the brainstem links connecting various parts of the brain and helps regulate respiration?

Midbrain

Which part of the brainstem mediates visual and auditory reflexes and controls cranial nerves III, IV, and certain eye movements?

A nurse is planning care for the client with hemiparesis of the right arm and leg. The nurse incorporates in the care plan placement of objects:

Within the client's reach, on the left side

Myelogram

X-ray of spinal cord and canal after injection of contrast medium. Identifies spinal cord tumors, herniated intervertebral disks, and arthritic bone spurs. *Nursing Implications* Closely monitor neurologic and vital signs. Immediately report leakage or bleeding from lumbar puncture site to the charge nurse and physician.

X-ray and CT scan

X-rays and CT scan are used to evaluate the auditory canal for diagnosing Ménière disease. *Nursing Implications* Explain procedure to the patient.

Facial x-rays and CT scan

X-rays and CT scan are used to identify orbital fractures or the presence of foreign bodies in the eye. *Nursing Implications* Explain procedure to the patient.

Should the dialysate be warmed to body temp before admin to pt via peritoneal dialysis?

Yes, it should be warmed to body temp and weigh pt before and after.

Neurons

_________ are the functional units of the nervous system and consist of a cell body, dendrites, and an axon.

Parallel

_________ talk describes for the patient what he/she is doing while he/she is performing some activity.

Calculi

__________ are less likely to develop when there is a high fluid intake and frequent, complete emptying of the bladder.

Ganglia

___________ are collections of nerve cell bodies outside the CNS.

Depression

____________ often occurs with chronic pain and lack of sleep.

CVA Diagnosis begins with

a complete history and physical examination. The time of the onset of stroke manifestations is an essential part of the assessment.

Spinal nerves

a pair of afferent sensory nerve fibers or roots and efferent motor fibers or roots

A client with a tentative diagnosis of myasthenia gravis is admitted for a diagnostic workup. Myasthenia gravis is confirmed by:

a positive edrophonium (Tensilon) test.

conjunctiva,

a thin mucous membrane that covers the anterior surface of the eye, also lines the inner surfaces of the eyelids.

Spinal Shock

a transient response occurring 30-60 minutes and lasting weeks to months after a spinal cord injury resulting in muscle flaccidity below the level of injury

Simple-Partial Seizures

a type of seizure involving only one part of the brain which may manifest with motor, somatosensory or autonomic symptoms, concsciousness is not impaired

Complex-Partial Seizures

a type of seizure which does not involve the whole brain and is characterized by impaired consciousness and repetitive and bizarre behaviors

Which of these is a key component of a neurological assessment? a. Signs of increased intracranial pressure and eye signs b. Signs of heightened hearing c. Loss of the sensation of touch d. Loss of appetite e. Loss of hearing

a. Signs of increased intracranial pressure and eye signs Other key components of a neurological assessment include: -Neurologic nursing history, including history given by family and Neurologic status -Speech patterns, Level of consciousness (LOC), changes in LOC -Gross evaluation of muscle tone/strength -Overview of balance, coordination, and protective reflexes and of sensory function -Function of selected cranial nerves

Neuron Influence

ability to impact other neurons, muscle cells, and glandular cells by transmitting nerve impulses

Decorticate Posturing

abnormal flexion indicating damage to the cerebral cortex

Bladder dysfunction (spinal cord) Atonic (autonomous)

absence of muscle tone and contractility, enlargement of capacity, no sensation of discomfort, overflow with large residual, inability to voluntary empty. Early stage of spinal cord injury

blood-brain barrier i

astrocytes that are joined by tight junctions. This decreases permeability so that harmful substances in the blood can- not enter the brain. It allows only the passage of lipids, glu- cose, some amino acids, carbon dioxide, oxygen, anSubstances such as urea, creatinine, some toxins, and most antibiotics cannot pass this barrier. However, brain injury or infection may cause a local breakdown of the barrier.

marked elevations of hematocrit

at increased risk for stroke due to general reduction of cerebral blood flow

A client recovering from a spinal cord injury has a great deal of spasticity. What medication administered by the nurse may be used to control spasticity?

baclofen

what is a lacunar stroke

caused by a small vessel disease deep in cerebral white matter (penetrating artery disease) strongly associated with hypertensive hemorrhage and diabetic microvascular disease

focused physical examination for the patient with a neurologic problem

begins by assessing the patient's pos- ture, movement, and appearance and identifying orienta- tion, mental state, and emotional state. Identify whether the patient can see, hear, and feel your touch. Assess dress, hygiene, and grooming, as well as gait and posture. Observe the patient's actions and affect. Note the LOC; content and quality of speech; mood swings; person- ality changes; and orientation to time, place, and person. Assess for memory or perceptual deficits. Obtain blood pressure in both arms (unless contraindi- cated), pulse, respiratory rate, and temperature. Note any abnormal breathing patterns such as increased or decreased rate and depth of breathing. Check pupil response to light by using a penlight. Normal response should be PERRLA (pupils equally round and reactive to light and accommodation). Observe for ptosis (drooping eyelids) and nystagmus (involuntary eye movement). Assess the ability to swallow a small drink of water, noting the presence of dysphagia (difficulty swallowing). Determine the patient's ability to shrug the shoulders and stick out the tongue. Note that the tongue should be midline. Assess for facial droop. Have the patient turn head side to side against resistance. Perform the Romberg test. Assess upper and lower extremities for weakness, atrophy, and tremors, as well as identifying decreased muscle tone (flaccidity) or increased muscle tone (spasticity). Ask the pa- tient to squeeze your hands, push feet against the resistance of your hands, and raise both legs off the bed. Note if thereis any difference between left and right side. Assess gait and ability to stand on one foot and walk heel to toe. ataxia is a lack of coordination and an unbalanced gait. Assess DTRs. Abnormal DTRs require further medical evaluation. Assess distant vision by using the Snellen chart (see Figure 5-4) and asking the patient to cover one eye at a time to read the chart and then repeat the test for the other eye. Assess near vision in the same manner, using a Rosenbaum chart (Figure 37-11▪). To test extraocular movements, ask the patient to follow a pen or your finger while keeping the head stationary. Inspect the eyelids for unusual redness or discharge. Note abnormal wideness of the lids, which may be due to ex- ophthalmos. Inspect the cornea and iris for cloudiness or irregu- larities and the sclera for redness or yellow discoloration. Assess for hearing loss by performing the whisper test and the Rinne and Weber tests (see Box 40-9). Inspect the auricle for redness, drainage, scales, or skin lesions. Palpate the auricles and over the mastoid process for tenderness, swelling, or nodules.

head/neck postural deviations associtated with stroke

protraction with lateral trunk flexion lateral flexion of the head with rotation away from more affected side

A client reports light-headedness, speech disturbance, and left-sided weakness that have lasted for several hours. In the examination, an abnormal sound is auscultated in an artery leading to the brain. What is the term for the auscultated discovery?

bruit

cerebral edema

byproduct of ischemic stroke fluids accumulate within the brain w/in minutes of insult....reach max @ 3-4 days result of tissue necrosis and widespread rupture of cell membranes with movement of fluid from the blood into brain tissues swelling gradually subsides, typically gone by 2-3 wks

Which of these medications are given to a client with Parkinson's disease? a. Neostigmine methylsulfate b. Zolmitriptan c. MAO B inhibitors d. Beta-blockers e. Phenytoin

c. MAO B inhibitors Medications given to a client with Parkinson's disease include: -Levodopa (L-dopa, Dopar, Larodopa) -Levodopa plus carbidopa (Sinemet) -Dopamine agonists -MAO B inhibitors -Anticholinergics -Glutamate (NMDA) blocking drugs

vertebrobasilar artery syndrome

can produce a wide variety of symptoms with both ipsilateral and contralateral signs because some of the tracts in brainstem will have crosses and others will have not locked in wallenburgs horners any pontine syndrome any medullary syndrome

A nurse is reading a journal article about stroke and the underlying causes associated with this condition. The nurse demonstrates understanding of the information when identifying which subtype of stroke as being due to atrial fibrillation?

cardio embolic

A client is sitting in a chair and begins having a tonic-clonic seizure. The most appropriate nursing response is to:

carefully move him to a flat surface and turn him on his side.

contributing cofactors in cause of migraine

changes in serotonin receptors which promote dilation of cerebral blood vessels and pain intensification from neurochemicals released from trigeminal nerve. medications, chemicals in certain foods, reproductive hormones

Guillain-Barre Syndrome

characterized by abrupt symmetrical paresis that progresses to paralysis, thought to be autoimmune and usually resolves in a few weeks to a few months

Audiometry confirms a client's chronic progressive hearing loss. Further investigation reveals ankylosis of the stapes in the oval window, a condition that prevents sound transmission. This type of hearing loss is called a:

conductive hearing loss.

A client is color blind. The nurse understands that this client has a problem with:

cones.

what is a major contributing factor to the formation of aneurysm

congenital defects produce that weakness in blood vessel wall

reflex arc

consists of a receptor, an afferent sensory neuron, the response center in the spinal cord or brain, an efferent motor neuron, and an effector muscle or gland

The client asks the nurse, "How does ergotamine (Ergostat) relieve migraine headaches?" The nurse should respond that it:

constricts cerebral blood vessels.

white matter

contains the axons of the ascending sensory and descending motor fiber

perseveration

continued repition of words, thoughts, or acts not related to current context.... pt is stuck repeating shit results from lesions in the premotor and/or prefrontal cortex The premotor cortex is an area of motor cortex lying within the frontal lobe of the brain just anterior to the primary motor cortex.

Spinal cord

continuous with brain-stem and exist from cranial cavity through foreman magnum

MOST COMMON characteristics of ACA syndrome

contralateral hemiparesis and sensory loss with greater LE involvement

Triggers of Autonomic Dysreflexia

distended bladder or plugged catheter, constipation or impaction, pressure sores, enemas, tight clothing, uterine contractions and ejaculation

MCA clinical manifestations

contralateral hemiparesis involving mainly the UEs and face-affecting primary motor cortex and interal capsule contralateral hemisensory loss involving main the the UE and face-involving primary sensory cortex and internal capsule Motor speech impairment-broca's or nonfluent aphasia...impaired auditory comprehension, limited vocab, slow, hesitant speech-affecting broca's cortial are in dominant hemisphere (typically left) receptive speech impairment-wernicke's, or fluent aphasia---impaired auditory comprehension...and fluent speech with norma, rate and melody global aphasia-nonfluent speech with poor comprehension-affecting both the third frontal convolution and posterior portion of the superior temporal gyrus unilateral neglect, poor depth perception, agnosia, poor spatial relations-affeting parietal sensory association cortex in nondominant hemishere (usually right) limb-kinetic apraxia-affecting the premotor or parietal cortex contralateral homonymous hemianopsia-affecting optic radiation in internal capsule loss of conjugate gaze to oppo sie-affectiing frontal eye fields...A conjugate gaze palsy is inability to move both eyes in a single horizontal (most commonly) or vertical direction. ataxia of contralateral limb or limbs-affecting parietal lobe pure motor hemiplegia aka lacunar stroke-affecting upper portion of posterior limb of internal capsule

ACA syndrome clincal manifestions

contralateral hemiplegia mainly in LEs, UEs more spared-affecting primary motor area, medial aspect or cortex, internal capsule contralateral hemisensory loss mainly in LEs-affecting primary sensory area, medial aspect of cortex urinary incontinence-affection posteromedial aspect of superior frontal gyrus problems imitating, apraxia, issues with bimanual tasks-affecting corpus callosum abulia (mutism) slowness, delay, motor inaction contralateral grasp relfex, sucking reflex

clincal manifestation of PCA syndrome

contralateral homonymous hemianopsia bilateral homonymous hemianopsia visual agnosia prosopagnosia-difficulty naming ppl on sight dyslexia memory defects topographic disorientation central post-stroke thalamic pain, spontaneous pain involuntary movements like choreoathetosis, intention tremor, hemiballisimus contralateral hemiplegia weber's syndrome- unilateral oculomotor nerve palsy and contralateral hemiplegia paresis of vertical eye movemets

MOST COMMON characteristics of MCA syndrome

contralateral spastic hemiparesis sensory loss of face, UE, LE...more and UE more involved lesion of parieto-occiptal cortex of dominant hemispehre typically produce aphasia lesions of right parietal lobe of nondominant hemipshere typically produce perceptual defecits like unilateral neglect, apraxia,, anosognosia, spatial disorganization

Stretch reflexes

control muscle tone and help maintain posture.

Frontal lobe

controls higher cognitive function, memory retention, voluntary eye movements, voluntary motor movement, and speech in Broca's area

The right hemisphere

controls sensation and movement of the left side of the body, whereas the left hemi- sphere controls sensation and movement of the right side of the body. controls visual-spatial information such as art, music, and the surrounding physical environment.

Diplopia (eyes)

couble vision Lesions affecting nerves of extra-ocular muscles, cerebellar damage

During recovery from a stroke, a client is given nothing by mouth, to help prevent aspiration. To determine when the client is ready for a liquid diet, the nurse assesses the client's swallowing ability once each shift. This assessment evaluates:

cranial nerves IX and X.

An auto mechanic accidentally has battery acid splashed in his eyes. His coworkers irrigate his eyes with water for 20 minutes, and then take him to the emergency department of a nearby health care facility. He receives emergency care for corneal injury. The physician prescribes dexamethasone (Maxidex Ophthalmic Suspension), two drops of 0.1% solution to be instilled initially into the conjunctival sacs of both eyes every hour; and polymyxin B sulfate (Neosporin Ophthalmic), 0.5% ointment to be placed in the conjunctival sacs of both eyes every 3 hours. Dexamethasone exerts its therapeutic effect by:

decreasing leukocyte infiltration at the site of ocular inflammation.

A client has just been diagnosed with early glaucoma. During a teaching session, the nurse should:

demonstrate eyedrop instillation.

Dysarthria:

difficulty in speech caused by paralysis of the muscles that control speech.

The nurse is caring for a patient with a history of transient ischemic attacks (TIAs) and moderate carotid stenosis who has undergone a carotid endarterectomy. Which of the following postoperative findings would cause the nurse the most concern?

difficulty swallowing, hoarseness, other signs of cranial nervedysfunction

Care of Seizure Patients

don not restrain, position on side if possible, protect/cradle the head, move away from objects which could cause harm, do not insert anything into the mouth, oxygen and suction if indicated, record and monitor closely, do not leave patient

Diagnostic tests - blood glucose level

done to identify the presence of hypoglycemia.

sleep apnea and stroke

doubles risk of stroke or death takeaway from all chronic diseases----control to reduce stroke risk!

trunk-postural deviations associated with stroke

due to sacral sitting-flattened lumbar curve with exaggerated thoracic curve and forward head posutre lateral flexion with trunk shortening on more affected side

The nurse observes that a comatose client's response to painful stimuli is decerebrate posturing. The client exhibits extended and pronated arms, flexed wrists with palms facing backward, and rigid legs extended with plantar flexion. Decerebrate posturing as a response to pain indicates:

dysfunction in the brain stem.

how do they differentiate btw atherothrombotic stroke and hemorrhagic stroke?

early CT scan if stroke is atherothrombotic, tPA clot buster can be used. tPA must be given w/ in 3 hrs of onset of symptoms, cant be given with hemorrhagic stroke could worsen bleeding in that case pts who get tPA within 3 hrs are at least 33% more likely to recover from their stroke with little or no disability after 3 months compared to ppl who didnt get it fewer than half of ppl experiencing stroke get to ER within 2 hrs of symptoms. women less likely than men to arrive in time

stroke risk factors specific to women

early menopause (42 or younger') have twice the risk of ischemic stroke as women with later menopause use of estrogen inc risk of ischemic stroke 45-55% pregnancy, birth, 1t 6 wks post partum carry increased stroke risk, esp older pregnant women and african americnas preeclampsia----a pregnancy complication characterized by high blood pressure and signs of damage to another organ system, often the kidneys. Preeclampsia usually begins after 20 weeks of pregnancy in a woman whose blood pressure had been normal...inc risk of stroke

significant cerebral edema can do what

elevate intracranial pressures leads to intracranial hypertension and neurological deterioration associated with contralateral wand caudal shifts of brain structures...this is known as brainstem herniation

When caring for a client with the nursing diagnosis Impaired swallowing related to neuromuscular impairment, the nurse should:

elevate the head of the bed 90 degrees during meals.

TIA causes

emboli occlusive episodes reduced cerebral perfusion---arrythmia, dec CO, hypotension, too much HTN meds, whatever the hell subclavian steal syndrome is) cerebrovascular spasm approx 15% of all strokes are preceded by a TIA, greatest risk of occurence is withing 90 days

spasticity and stroke

emerges in about 90% of cases occurs on side of body oppo the lesion frequently strong in scapular retractors, shoulder adductors, depressors, and internal rotators LE spasticity often strong in pelvic retractors, hip adductors, hip int rot, hip and knee extensors, pfers, toe flexors severity of spasticity-graded by the modified ashworth scale

After a plane crash, a client is brought to the emergency department with severe burns and respiratory difficulty. The nurse helps to secure a patent airway, attends to the client's immediate needs, and then prepares to perform a neurologic assessment. Because the client is unstable and in critical condition, this examination must be brief but should include:

evaluation of the corneal reflex response.

Infratentorial herniation

expansion of mass lesions in cerebrum forces brain through opening created by the brainstem

what is the term for when patient is unable to form words that are understandable

expressive aphasia

cerebral vascular anatomy over view

extracranial blood supply to brain is provided by the rt and lt internal carotid arteries and rt/lt vertebral arteries internal carotid artery begins at the bifuction of the common carotid artery and ascends in the deep portions of neck to carotid canal.. ..then it turns "rostromedially" and ascends into cranial cavity ...then it pierces the dura mater, giving off opthalmalamic and anterior choroidal arteries before bifurcating into the the MIDDLE and ANTERIOR cerebral arteries, gives rise to rostral portion of circle of willis

A nurse is communicating with a client who has aphasia after having a stroke. Which action should the nurse take?

face the client and establish eye contact; use short sentences; give the client time between phrases to understand; keep extraneous and background noise to a minimum; not necessary to speak in louder or softer voice than normal

The nurse is caring for a patient with a history of transient ischemic attacks (TIAs) and moderate carotid stenosis who has undergone a carotid endarterectomy. The nurse focuses on assessment of which cranial nerves? (4)

facial (VII) vagus (X) spinal accessory(XI) hypoglossal(XII)

what is the anterior cerebral artery

first and smaller of two terminal branches of the internal carotid artery supplies the medial aspect of the cerebral hemisphere (frontal and parietal lobes)...and subcortical structures including basal ganglia, anterior fornix, and anterior 4/5 of corpus callosum .........bc anterior COMMUNICATING artery allows perfusion of proximal ACA from both sides, occlusion proximal to that pt results in min. defecits...more distal lesion, more defecits

nonfluent aphasia aka broca's

flow of speech is slow and hesitant, limited vocab, syntax impaired speech production is labored or lost completely, but comprehension is good when this happens the lesion is located in the premotor are of the left frontal lobe

aphasia

general term used to describe an acquired communication disorder caused by brain and is characterized by an impairment of language comprehension, fomulation, use seen in 30-36% of pts with stroke

Absence Seizures

generalized seizures characterized by an abrupt loss of consciousness and amnesia lasting only a few seconds and resolving without a postictal state

Tonic Seizures

generalized seizures consisting of brief but sudden stiffening of the muscles and loss of consciousness

Clonic Seizures

generalized seizures involving rhythmic contraction and relaxation of muscles and a loss of consciousness

Myoclonic Seizures

generalized seizures more common in older children, involve "lightening jerks" occurring from seconds to minutes

Concussion

head trauma leading to a brief loss of consciousness followed by confusion and possible memory loss, there may not always be a loss of consciousness

What deficits would you expect from damage to the motor neurons?

hemiparesis, hemiplegia, change in reflexes

chronic HTN is linked to what....ischemic stroke or hemorrhagic stroke

hemorrhagic

The health care provider prescribes several drugs for a client with hemorrhagic stroke. Which drug order should the nurse question prior to administration?

heparin sodium

LE extension synergy componets

hip ext, add, int rot knee ext ankle pf, inversion toe pf

LE flexion synergy components

hip flexion, abduction, ext rot knee flexion ankle DF, inversion toe DF

The nurse is teaching a client with a T4 spinal cord injury and paralysis of the lower extremities how to transfer from the bed to a wheelchair. The nurse should instruct the client to move:

his upper body to the wheelchair first.

A client injured in a train derailment is admitted to an acute care facility with a suspected dysfunction of the lower brain stem. The nurse should monitor this client closely for:

hypoxia.

Positron emission tomography (PET)

identifies the amount of tissue damage after a CVA.

A client with a spinal cord injury and subsequent urine retention receives intermittent catheterization every 4 hours. The average catheterized urine volume has been 550 ml. The nurse should plan to:

increase the frequency of the catheterizations.

A client with an inflammatory ophthalmic disorder has been receiving a ½-inch ribbon of corticosteroid ointment in the lower conjunctival sac four times per day as directed. The client reports a headache and blurred vision. The nurse suspects that these symptoms represent:

increased intraocular pressure.

Cushing's Triad

increased systolic blood pressure, decreased pulse and deceased respirations

Bell's Palsy

inflammation of the 7th cranial nerve resulting in unilateral paralysis of the face usually resolving in 1-8 weeks

Meningitis

inflammation of the meningeal covering of the brain and spinal cord, may be bacterial or viral

After an eye examination, a client is diagnosed with open-angle glaucoma. The physician's prescription says "pilocarpine ophthalmic solution (Pilocar), 0.25% 1 gtt both eyes q.i.d." Based on this prescription, the nurse should teach the client or a family member to administer the drug by:

instilling one drop of pilocarpine 0.25% into both eyes four times daily.

Nerve impulses

involves generation of an action potential

Cerebrospinal fluid (CSF)

is found in the subarachnoid space between the arachnoid and the pia mater.

Lumbar puncture

is used to obtain CSF for examination. Blood in the CSF indicates a hemorrhagic CVA

Dysarthria (speech)

lack of coordination in articulating speech cerebellar or cranial nerve lesion Anti-seizure drugs, sedatives, hypnotic drug toxicity (including alcohol)

Ataxia (motor system)

lack of coordination of movement Lesions of sensory or motor pathways, cerebellum - Antiseizure drugs - sedatives - hypnotic drug toxicity (including alcohol)

The nurse is working on a surgical floor. The nurse must logroll a client following a:

laminectomy.

A client has experienced an ischemic stroke that has damaged the frontal lobe of his brain. What deficits does the nurse expect to observe during assessment?

limited attention span, difficulties in comprehension, forgetfulness, lack of motivation

A client reports intermittent cluster headaches over the last year. The nurse reviews with the client interventions to alleviate the symptoms associated with headaches. The plan of care would include all except:

listening to a relaxation tape.

Myasthenia Gravis

literally translated "grave muscle weakness"

Aphasia (speech)

loss of or impaired language faculty (comprehension, expression, or both) Left cerebral cortex lesion

Analgesia (motor system)

loss of pain sensation Lesion in spinothalamic tract or thalamus, specific medication

homonymous hemianopsia (eyes)

loss pf vision in one side of visual field lesions in the contralateral occipital lobe

An older adult client admitted to the hospital with chest pain has difficulty hearing. Which method should the nurse use when collecting data from this client?

lower voice pitch while facing the client

right hemisphere lesion behaviors

lt side hemiplegia or hemiparesis lt side sensory loss left side unilateral neglect agnosias visuospatial disorders disturbances of body image and body scheme difficulty processing VCs impulsive, quick behaviors poor judgement and unrealistic inability to self-correct poor insight, poor awareness of impairment, deny disability big safety risk more than left side difficulties with abstract reasoning, problem solving difficulty synthesizing info-grasping whole idea of a task thoughts are rigid memory impairments are typically related to spatial-perceptual info difficulty perceving emotions difficulty with expression of negative fluctuations in performance of tasks

The nurse is caring for a patient diagnosed with a hemorrhagic stroke. The nurse recognizes that which of the following interventions is most important?

maintaining a patent airway

atherosclerosis and its role in stroke

major contributing factor quick and dirty: plaque formation on arterial walls leading to progressive narrowing of blood vessels blood flow interruption tyically occurs are certain sites-bifurcations, constrictions, dilations, angulations of arteries... ...most common sites: origin of common carotid artery, @ transition of common carotid to middle cerebral artery, junction of vertebral arteries w/ basilar artery

Amyotrophic Lateral Sclerosis

progressive and degenerative condition affecting motor neurons within the brain and spinal cord resulting in the loss of conductivity leading to muscular atrophy and eventually death

a caution to take in a patient with hemiplegia (right lesion)

may behave in way that minimize their disabilities easy to overestimate the pts' perceptual abilities

Diagnostic tests - Serum sodium and osmolarity

measured because low sodium levels, or increased or decreased osmolarity, may cause an altered LOC.

The nurse is making assignments for the day. Which of the following tasks can be safely assigned to unlicensed assistive personnel (UAP)?

measuring the intake of a client with multiple sclerosis

Status Epilepticus

medical emergency in which an individual has a continuous or rapid succession of seizures depriving the brain of its oxygen and glucose by overutilization

Autonomic Dysreflexia

medical emergency, an exaggerated response by the ANS to noxious stimuli causing sudden hypertension, flushing and diaphoresis and severe anxiety

name of a test than can examine higher cortical function

mini-mental status exam: provides a valid and reliable quick screen of cognitive function

apraxia

more evident in left hemipshere damage pt demonstates difficulty planning and executing purposeful movements that cannot be accounted for by any other reason (like impiared strength, coordination, etc)

Alzheimer's Disease

most common cause of dementia

Lumbar puncture

most common method of sampling CSF

ischemic stroke

most common type of stroke affects about 80% of ppl with stroke results from a clot blocking or impairing blood flow....deprives the brain of o2 and nutrients

CT scan and stroke...disadvantages/advantages

most commonly used, most readily available imaging technique allows identification of large arteries and veins, but poor sensitivity for detecting small infarcts

stroke and cardio/pulmonary dysfunction

most strokes caused by vascular disease they're deconditioned everything is reduced may demonstrate impaired CO, rhythym disoders you can figure this out do energy conservation

A patient is brought to the emergency department with a possible stroke. What initial diagnostic test for a stroke, usually performed in the emergency department, would the nurse prepare the patient for?

noncontrast computed tomogram (CT Scan) should be performed within 25 minutes of time pt presents in ED to determine if event is ischemic or hemorrhagic

In caring for a child immediately after a head injury, the nurse notes a blood pressure of 110/60, a heart rate of 78 beats/minute, dilated and nonreactive pupils, minimal response to pain, and slow response to name. Which symptom would cause the nurse the most concern?

nonreactive pupils

Obtunded

not yet comatose but does not respond appropriately and present with extreme lethargy or minimal responsiveness

Withdrawal reflexes

occur when a person expects pain and automatically withdraws the threatened body part.

locked in syndrome

occurs with basilar artery thrombosis and bil. infarction of the ventral pons. catastrophic event with suddent onset communicayion only thru vertical eye movements

Cranial nerve III

oculomotor; helps move the eyes

A client is diagnosed with a conductive hearing loss. When performing Weber's test, the nurse expects that this client will hear sound:

on the affected side by bone conduction.

stroke disability stats

once again leading cause of disability in USA for ischemic stroke survivors 65 or older-disabillities at 6 months out are below: hemiparesis-50% cant walk w/o assist-30% dependent in ADLs-26% aphasia-19% depression-35% 26 % of pts w/ stroke end up in a long term care facility

A nurse is administering intravenous tissue plasminogen activator (tPA) to a client having a stroke. Which criteria must be met before administering the agent? Select all that apply.

onset of stroke within 3 hours of initiation of therapy normal prothrombin (PT) and partial thromboplastin (PTT) times

central post-stroke pain

pain arising as a direct consequence of a lesion or disease affecting the central somatosensory system. occurs in about 10% of strokes thalamus is somehow involved in this

Tetraplegia (quadriplegia)

paralysis of all extremities spinal cord transection or mass lesion (cervical region)

Hemiplegia (motor system)

paralysis on one side stroke and other lesions involving motor cortex

A client at the eye clinic is newly diagnosed with glaucoma. What should the nurse inform the client might occur if administration of the medication is not closely adhered to?

permanent vision loss

gli/o

pertaining to neuroglia cells or a gluey substance

The nurse is caring for a patient diagnosed with a subarachnoid hemorrhage resulting from a leaking aneurysm. The patient is awaiting surgery. What nursing interventions would be appropriate for the nurse to implement? (5)

provide dimly light environment administer Colace per order (no enemas) elevate HOB 15-30 degrees absolute bed rest restrict visitors to family

The nurse is providing diet-related advice to a male patient following a cerebrovascular accident (CVA). The patient wants to minimize the volume of food and yet meet all nutritional elements. Which of the following suggestions should the nurse give to the patient about controlling the volume of food intake?

provide thickened commercial beverages and fortified cooked cereals, scrambled eggs avoid high-fat foods (pt should lose weight) avoid peanut butter, bread, tart foods, dry or crisp foods, chewy meats because they cause choking

Cerebrospinal fluid

provides cushioning for brain and spinal cord

Magnetic resonance Imaging

provides improved detail intracranial structures

obligatory synergies following stroke

pt is unable to perform an isolated movment of a single limb segment without producing movments in the remainder of the limb ex----tries to flex elbow, shoulder flexion happens too certrain muscles are not involved in either the flexion or extension synergy---they are very hard to activate while pt is exhitibing the synergies.... ....muscles are: lats, teres major, serratus, finger extensors, ankle evertors what's the big deal? these synergies are often incompataible with normal ADL and functional mobility skills

ideomotor apraxia

pt unable to produce a movement on command but is able to move automaticallty pt can perform habitual tasks when not commanded to do so, often perserverates

deteriorating stroke-definition

pt's neurological status deteriorates after admission to hospital change in status due to---stuff like cerebral edema, progressiong thrombi_

How often should neurologic assessments and vital signs be taken initially for the patient receiving tissue plasminogen activator (tPA)?

q15min

A client who has been severely beaten is admitted to the emergency department. The nurse suspects a basilar skull fracture after assessing:

raccoon eyes and Battle's sign.

computed tomography scan

rapid means of obtaining radio-graphic images of the brain

A male patient with cerebrovascular accident (CVA) is prescribed medication to treat the disorder. The patient wants to know what other measures may help reduce CVA. Which of the following is an accurate suggestion for the patient?

reduce HTN and high blood cholesterol levels

The nervous system

regulates and integrates all body functions, muscle movements, senses, mental abilities, and emotions. Alterations in the nervous system can affect human functions such as cardiac and respiratory function, activity, comfort, and elimination. consists of the brain, spinal cord, and peripheral nerves.

A physician has ordered home health and physical therapy for an older adult who will be discharged home following an acute stroke. The nurse's discharge teaching should include instructions about:

reporting specific signs and symptoms to the physician, discharge medications, and dietary concerns.

The nurse is teaching a client with multiple sclerosis. When teaching the client how to reduce fatigue, the nurse should tell the client to:

rest in a room set at a comfortable temperature.

Superficial reflexes

result from gently stimulating the skin. For example, the plantar reflex is elic- ited by stroking the sole of the foot. The normal response is to curl the toes downward.

what causes ischemic stroke

result of thrombus, embolism, or conditions that produce low systemic perfusion pressures, results in decreased cerebral blood flow lack of blood flow deprives the brain of needed o2, glucose, disrupts cellular metabolism, leads to tissue death

multi-infarct dementia aka vasuclar dementia

results from multiple small infarcts of the brain. seen in 6-32% of stroke pts. more common if you're over 60 exhibits impairments of memory of cognition that fluctuate...get worse, then better, then worse, whatever that's how its different from alzheimer's which is more of a gradual, steady progression

thrombus defined

results from platelet adhesion and aggregation on plaques thrombi lead to ischemia aka occlusion of an artery with a resulting cerebral infarction or tissue (ABI) thrombi can also become dislodged and travel to a more distal site in the form of an intra-artery embolus

left hemipshere lesion behaviors

rt hemiparesis or hemiplegia rt sensory loss if left is dominant hemisphere (typically is)-broca's, wernicke's, or global aphasia difficulty processing Verbal Cues/Commands slow, cautious disorganized typically very aware of impairments or extent of disability disorganized problem solving difficult initiating tasks, delay is processing highly distractible memory impairements related to language perseveration difficult expressing positive emotions ideational or ideomotor apraxia

A client with Parkinson disease is receiving selegiline. Which foods should the nurse instruct the client to avoid while taking this medication? Select all that apply.

salami aged cheese soy sauce sauerkraut

UE extension synergy componenets

scapular protraction shoulder add, int rot elbow ext forearm pronation wrist and finger flexion

UE flexion synergy components

scapular retraction/elevatiom or hyperext shoulder abd, ext rot elbow flexion forearm supination wrist and finger flexion

Karen Nichols, a 32-year-old concert promoter, has a 12-year history of migraine headaches. As she discusses her most recent headaches, she voices frustration over how her life is at the mercy of these headaches. As she describes the characteristics of her head pain, you begin to discuss the potential triggers of her migraines. Which of the following is not a potential trigger to migraines?

seasonal changes Are triggers: specific food chemicals reproductive hormone fluctuations medications

what is the MCA middle cerebral artery

second of twio main branches of the internal carotid artery suplllies the entire lateral aspect of the cerebral hemisphere (frontal, temporal, and parietal lobes) occlusion of proximal MCA produces a lot of neurological damage with signifcant cerebral edema most occomon stie of occlusion in stroke

delirium aka acute confusional state

seen more commonly in acute care setting why does it happen? o2 deprovation to brain, metabolic imbalance, adverse drug reaction clouding of consciousness or dulling of cognitive processes and impaired alterness. pt is inattentive, incoherent, fluctuating levels of consciousness night time is particularly problematic

post-stroke muscle weakness

selective loss of type II fast-twitch fibers decrease in number of functional motor units, decrease in their firing rate abnormal recruitment of motor units of motor units with altered timing...pts demonstate inefficient patterns of muscle activation and higher levels of co-contraction

global aphasia

severe aphasia characterized by marked impairments of both production and comprehension of language often an indication of extensive brain damage big time impairment change for rehab

A client admitted to the emergency department is being evaluated for the possibility of a stroke. Which assessment finding would lead the nurse to suspect that the client is experiencing a hemorrhagic stroke?

severe exploding headache

While providing information to a community group, the nurse tells them the primary initial symptoms of a hemorrhagic stroke are (4)

severe headache, early change in level of consciousness, vomiting, seizures

short-term memory loss vs long-term memory loss

short term is associated with lesions of the limbic system, limbic association cortex, or temporal lobes long term is associated with lesions of the hippocampus of the limbic system LTM typically remains intact for stroke pts STM loss-36% of stroke pts experience

LE postural deviations associated with stroke

sitting-more ffected le typically hel in hip abd and ext rot with hip and knee flex (flexion synergy) standing-more affected LE typically help in hip and knee ext with adduction and int rot (scissoring), , ankle PF unequal WB on feet like pelvis in sitting

modifiable stroke risk factors

smokes inactivity obesity diet current smokers have 2-4x inc risk compared to non smokers who have quit for more than 10 yrs mod to vigorous exercise reduces stroke risk 35%, light exercise like walking does not have the same effect sooo lose weight, quit smoking, exercise more, nail diet for cholesterol and tryglycerides, control the bp

fluent aphasia aka wernicke's

speech flows smoothly with a good variety of gramatical constructions and a preserved melody of speech. but....auditory comprehension is is impaired. so pt shows difficult in comprehending spoken launguage, following commands when this happens...lesion is located in auditory association cortex in left lateral temportal lobe

sequential motor recovery stages following stroke

stage 1-recovery from hemiplegia occurs in stereotyped sequence of events that begins with a period of flaccidity immediately following the acute episode-no movement of the limbs can be elicited stage2-basic limb synergies or some of their components may appear as associated reactions, or minimal voluntary movement responses may be present. spasticity begins to develop stage 3-pt gains voluntary control of movement synergies, but full range of all synergy components does not nescessarily develop. increased spasticity, may be severe stage 4-some movement combos that do not follow the paths of either synergy are mastered....first its hard, then easier...spasticity begins to decline stage 5-if they reach here-more difficult movement combos are learned as the basic limb synergies lose dominance over motor acts stage 6-disappearnce of spasticity...individual jt movements become possible and coordination approaches normal. recovery process may plateau at ansy stage!

stroke gait deviations: hip

stance phase: poor hip position due to poor proprioception. trendelenburg limp due to weak abductors. scissoring due to spastic adductors. swing phase: indadequate hip flexion due to--weak hip flexotrs, poor proprioception, spastic quads, ab weakness (hiking of hip), hip abd weak of oppo site...leads to all sort of shit like circumduction, back leaning of trunk/dragging toes, uncontrolled swing

stroke gait deviations: trunk/pelvis

stance phase: unaware of affected side, poor proprioception. forward trunk due to weak hip extension or a flexion contracture swing phase: insufficient forward pelvic retraction due to weak ab muscles leaning to sound side for foot clearance-weakness of flexor muscles

strokes and coordination defecits

strokes affecting the cerebellum typically produce cerebellr ataxia-----basilar artery, pontine, lateral medullary syndromes---) basal ganglia involvement (this is PCA) may lead to slowed movements, bradykinesia, choreathetosis, hemiballisiumus in general proprioceptive losses can result in sensory ataxia

clinical signs of elevating intracranial pressure

stupor, coma widened pulse pressure inc heart irregular respirations (cheyne stokes) vomiting unreacting pupils (CN III dysfunction?) papilledema cerebral edema is most frequent cause of death in acute stroke ....characteristic of large infarcts involving middle cerebral artery and internal carotid artery

carotid endarterectomy

surgical procedure used to remove fatty deposits from the carotid artery useful in preventing recurrent strokes or preventing the development of stroke in ppl with TIAs

transient ischemic attack

temporary interruption of blood supply to the brain symps of FOCAL neurological defecit last for few mins-several hours, but do not last longer than 24 hrs...after attack is over there is no evidence of residual brain damage or permanent neurological dysfunction

A client reports frequent headaches and is seeing the physician to determine their cause. In client education, which type of headache does the nurse indicate is most common?

tension

In the older adult

the brain atrophies, causing slower move- ment and reflexes, as well as a degree of forgetfulness. How- ever, significant short- and long-term memory loss, mental status changes, altered coordination, loss of motor skills, and altered speech signal a need for further assessment. The patient may have difficulty changing position or standing up without assistance. This may be accompanied by dizziness, which is the result of anemia, ear infection, eye problems, stroke, or drug toxicity. These changes are often related to common chronic diseases that develop in older adults. Decreased corneal sensation and tear secretion Constriction of the pupil Decreased elasticity and increased density of the lens Loss of rods at the periphery of the retina Loss of fat and subcutaneous tissue around the eyes

confabulations

the filling of memory gaps with inapporprate words orfabricated stories

Apraxia:

the inability to carry out a familiar routine (e.g., brushing teeth or combing hair), even when paralysis is not present.

Agnosia:

the inability to recognize a familiar object such as a toothbrush.

acute phase rehab for stroke

therapist assisnt in ongoing monitoring of recovvery (vitals and whatnot) (Low intentsity rehabilitation; medically stablized, within 72 hrs;in the ICU) early mobilization prevents or minimizes the harmful effects of bed rest and deconditioning functional reorganization is promoted thru early stimulation and use of the hemiparetic side....minimize learned nonuse interventions: bed mobility, sitting, TFs, ADL training, ROM, splinting, positioning

meds commonly used to treat pts with stroke

thrombolytics-activase of tPA-degrades fibrin present in clots, dissolves clots and reestablishes blood flow anticoagulants-used to reduced risk of blood clots and prevent existing clots from getting bigger,,,,by thinning the blood, antiplatelet therapy-ex aspirin, plavix-prevents platelets from sticking together....long term, low dose (baby aspirin) use to decrease the risk of thrombosis and recurrent stroke. antihypertensive agents-ace inhibitiors, beta blockers, etc angiotensin II receptor antagonists-blocks angiotensin II which triggers muscle contraction around blood vessels causing them to narrow.....the blocker enlarges blood vessels and reduces BP anticholesterol agents/statins-inhibit enztme in blood that produces cholestrol in the liver antispasmodics-used to relax skeletal muscle and decrease muscle spasm antispastics anti-convulstants antidepressants

The physician suspects myasthenia gravis in a client with chronic fatigue, muscle weakness, and ptosis. Myasthenia gravis is associated with:

thymus gland hyperplasia.

The nurse is gathering data from a client with Ménière disease. Which symptom does the nurse relate to the disease process?

tinnitus

A client with new-onset seizures of unknown cause is started on phenytoin, 750 mg IV now and 100 mg P.O. t.i.d. Which statement best describes the purpose of the loading dose?

to more quickly attain therapeutic levels

A client has a history of painful, continuous muscle spasms. He has taken several skeletal muscle relaxants without experiencing relief. His physician prescribes diazepam (Valium), 2 mg by mouth twice daily. In addition to being used to relieve painful muscle spasms, diazepam also is recommended for:

treatment of spasticity associated with spinal cord lesions.

what is the PCA

two of them arise as terminal branches of the basilar artery each supplies its corresponding occipital lobe and medial and inferior temporal lobe aslo supplies upper brainstem, midbrain, posterior diencephalon, most of thalamus occusion proximal to posterior communicating artery typically results in min defeficts (collateral circ)

MRA magnetic resonance Angiography

type of MRI that uses special software to crease an image of the arteries in the brain used to identify vascular abnormalties like stenosis

what are associated reactions

typically present in pts with stroke who exhibit strong spasticity and synergies unintentional movments of hemiparetic limb caused by voluntary action of another limb or by stuff like yawning, sneezing, or coughing

ICA syndrome

typically produces massive infarction in the region of the brain supplied by the MCA ICA supplies both MCA and ACA...if collateral circ to ACA is absent, infarction can occur at both ACA and MCA significant edema is common....possible uncal herniation, coma, death

A client with a subdural hematoma was given mannitol. Which result would best show the effectiveness of the medication?

urine output of 65 ml/hour

When communicating with a client who has sensory (receptive) aphasia, the nurse should:

use short, simple sentences.

Tensilon Test

used as a diagnostic test for myasthenia gravis

Three Types of Headaches

vascular, tension and traction (inflammatory)

ventricul/o

ventricle of the heart or brain

A client is admitted to the facility for investigation of balance and coordination problems, including possible Ménière's disease. When assessing this client, the nurse expects to note:

vertigo, tinnitus, and hearing loss.

What deficit would you expect from damage to the occipital lobe?

visual agnosia

A 78-year-old Alzheimer's client is being treated for malnutrition and dehydration. The nurse decides to place him closer to the nurses' station because of his tendency to:

wander.

A client was diagnosed with having right subarachnoid hemorrhage. The nurse should plan to place the client in which position?

with the head of the bed elevated

sudden cessation of cerebral blood flow and o2, glucose deprivation and the physiology

within minutes neurons die within the ischemic core tissue cell survival depends on the severity and duration of ischemic episode without reperfusion in timely manner neuronal cells die in surrounding areas and neuronal activity ceases...infarct expands

more stroke stats that may be on the test for some reason

women have lower age-adjusted stroke incidence than men... ...but that incidence whatever reverses in older age--women over 85 have elevated risk compared to men african americans have twice the risk of first ever stroke compared to white folks...rates are also higher in mexican americans, american indians, alaska natives....first donald trump, now this!

A client with a conductive hearing disorder caused by ankylosis of the stapes in the oval window undergoes a stapedectomy to remove the stapes and replace the impaired bone with a prosthesis. After the stapedectomy, the nurse should provide which client instruction?

• "Don't fly in an airplane, climb to high altitudes, make sudden movements, or expose yourself to loud sounds for 30 days."

The nurse is preparing to administer carbamazepine oral suspension, 150 mg by mouth. The pharmacy has dispensed carbamazepine suspension 100 mg/5 mL. How many milliliters of carbamazepine should the nurse administer to the client? Record your answer using one decimal place.

• 7.5 To calculate the dose, the nurse should use this equation: 100 mg/5 mL = 150 mg/X 100X = 750 X = 7.5 mL

A client is admitted into a medical unit confused and agitated. Which nursing measure should the nurse implement first to keep the client safe?

• Encourage family, friends, or a sitter to stay with the client.

The nurse is scheduled to administer an otic medication. Which action should the nurse perform first?

• Check and verify the proper client's name. When giving medications, a nurse follows the 10 rights of medication administration: right client, right drug, right dose, right route, right time, right documentation, right assessment, right to refuse, right evaluation, and right client education or information. Put the client in the lateral position, not semi-Fowler's position, for 5 minutes to prevent the drops from draining out. The drops may be warmed to prevent pain or dizziness, but this is not the first action. An emesis basin would be used for irrigation of the ear.

A client undergoes a craniotomy with supratentorial surgery to remove a brain tumor. On the first postoperative day, the nurse notes the absence of a bone flap at the operative site. How should the nurse position the client's head?.

• Elevated 30 degrees

The nurse is caring for a client with stroke in evolution. Which nursing intervention is priority?

• Have tracheal suction available at all times.

A physician diagnoses a client with myasthenia gravis and prescribes pyridostigmine, 60 mg by mouth every 3 hours. Before administering this anticholinesterase agent, the nurse reviews the client's history. Which preexisting condition would contraindicate the use of pyridostigmine?

• Intestinal obstruction

What is the function of cerebrospinal fluid (CSF)?

• It cushions the brain and spinal cord.

A client comes to the emergency department complaining of headache, malaise, chills, fever, and a stiff neck. Vital sign assessment reveals a temperature elevation, increased heart and respiratory rates, and normal blood pressure. On physical examination, the nurse notes confusion, a petechial rash, nuchal rigidity, Brudzinski's sign, and Kernig's sign. What does Brudzinski's sign indicate?

• Meningeal irritation

A client was hit in the head with a baseball during practice. Which discharge instructions should the nurse reinforce?

• Wake client every hour and check orientation to person, time, and place.

A client is admitted with cardioembolic stroke. Which vital piece of information in the client's history is strongly associated with this type of stroke?

• atrial fibrillation

The nurse is teaching a client who has facial muscle weakness and has recently been diagnosed with myasthenia gravis. The nurse should teach the client that myasthenia gravis is caused by:

• destruction of acetylcholine receptors.

A client had cataract surgery. Which sign or symptom should the nurse tell the client to report immediately to the health care provider?

• eye pain

A client arrives at the emergency department after falling on ice outside of the senior citizens' housing facility and sustaining a right hip fracture. Which finding would be most important for the nurse to evaluate?

• neurovascular compromise

The nurse is caring for a client who underwent a stapedectomy. Which position would have the greatest benefit for prevention of complications and promotion of comfort?

• on the unaffected side

A client who has been severely beaten is admitted to the emergency department. The nurse suspects a basilar skull fracture after assessing:

• raccoon eyes and Battle's sign.

Which nursing data should be given the highest priority for a child with clinical findings related to tubercular meningitis?

• signs of increased intracranial pressure (ICP)

A nurse is caring for a client who had a lumbar laminectomy 2 days ago. Which finding should the nurse report to the health care provider?

• urine retention or incontinence

Drug therapy to reduce ICP

❈ *Hyperosmotic agents and diuretics* to dehydrate brain and reduce cerebral edema (Mannitol, Urea) ❈ Steroids (Dexamethasone, Methylprednisoline sodium succinctness to reduce brain edema) ❈ Barbiturates (reduce brain metabolism and systemic BP) ❈ Passive hyperventilation (lead to Resp. alkalosis) causes cerebral vasoconstriction and decreased cerebral blood flow, therefore decreasing ICP) ❈ Prophylactic anti-seizure (Phenytoin a.k.a Dilantin)

Head injury, nursing assessment

❈ *Unconsciousness or disturbances in consciousness* (most important indicator of increased ICP) ❈ Vertigo ❈ Confusion, delirium, disorientation ❈ Cushing's triad ( slowed and irregular respirations; increased temperature; widening pulse pressure) ❈ Headache ❈ Seizures ❈ Ataxia, abnormal posturing ❈ Projectile vomiting ❈ pupillary changes reflecting pressure on optic/oculomotor nerve (papiledema, lack of conjugate eye movement) ❈ CSF leakage ❈ Hematoma

Mannitol

❈ Acts on renal tubules by osmosis to prevent water re-absorption ❈ In bloodstream, draws fluid from extra-vascular spaces into the plasma Side effects: disorientation, confusion, headache, nausea and vomiting, convulsion and anaphylactic reactions Nursing implications: *Short-term use only*, do not give in patient with anuria (accumulation lead to pulmonary edema ad water intoxication) or cerebral hemorrhage

Multiple Scelerosis; Nursing interventions

❈ Allow hospitalized client to keep own routine (encourage self-care and rest period) ❈ Orient to environment, and teach strategies to maximize vision ❈ Teach about muscle spasticity, *stretch-hold-relax*exercises are helpful, as are riding a stationary bicycle and swimming; *take precaution against fall* ❈ Initially work with client on a voiding schedule, teach client that as incontinence worsens, female patients need to learn to *clean self-catheterization* and male patients need a *condom catheter* ❈ Encourage adequate fluid intake, high-fiber foods, and a bowel regimen for constipation problems ❈Encourage the client and the family to verbalize their concerns about ongoing care issues ❈ Administer steroid therapy ad chemotherapy and chemotherapeutic drugs for acute exacerbation to shorten length of attack

Spinal Cord injury; Nursing assessment

❈ Assess breathing pattern, auscultate lungs ❈ Check neurological vital signs, sensory and motor function, as well as cardiovascular status ❈ Assess abdomen: girth, bowel sounds, lower abdomen (for bladder distension) ❈ Assess temperature, *hyperthermia is common* ❈ Assess psychosoical status ❈ *Hypotension and bradycardia* occur with any injury above *T6*, where sympathetic outflow is affected

Craniotomy, Preoperative medications

❈ Corticosteroid: reduce swelling ❈ Agents and osmotic diuretics: reduce secretions ❈ Agents to reduce seizures ❈ Prophylactic antibiotics

Spinal cord injury; Nursing Interventions (Rehabilitative)

❈ Encourage deep-breathing exercises ❈Administer chest physiotherapy ❈ Kinetic ed to promote blood flow to extremities ❈ Apply SCDs ❈ Facilitate ROM exercises and turn patient frequently ❈ Mobilize client to chair as soon as possible ❈ Maintain hygienic practices and prevent skin breakdown ❈Teach bladder-emptying techniques and begin bowel-training program ❈ Monitor I&O, encourage to drink fluid that promote acidic urine, including *cranberry juice, prune juice, bouillon, tomato juice, and water*

Brain tumor; Nursing assessment

❈ Headache (more severe when awakening) ❈ Vomiting *not associated with nausea* ❈ Papiledema with visual changes ❈ Behavioral and personality changes ❈ Seizures ❈ Aphasia, hemiplegia, ataxia ❈ Cranial nerve dysfunction ❈ Abnormal CT/MRI, Pet scan (*looking for metastasis*)

Head injury, nursing interventions

❈ Maintain adequate ventilation ❈ Keep head of bed elevated at around 30 degree to aid venous return form neck and to decrease cerebral volume ❈Maintain continuous record of GCS, obtain neurological vital sign ❈ Avoid activities that increases ICP: Change in bed position; Hip flexion, Endotracheal suctioning; compression of jugular veins; coughing, vomiting, straining; increased temperatures ❈ Intracranial monitoring system: Licox catheter placed in lateral ventricle, sensor placed in dura and screw into sub-arachnoid space attached to a pressure transducer; *Elevation of ICP>20 mmHG should be reported stat* ❈ Maintain seizure precaution ❈ Education on post-traumatic syndrome (headache, vertigo, emotional instability, inability to concentrate, impaired memory), post-traumatic epilepsy, post-traumatic neurosis

Spinal cord injury; Nursing Interventions (Acute)

❈ Maintain client in a extended position with cervical collar, stabilize when transfer ❈ Maintain a patent airway ❈ Skeletal contraction maintained by *use of skull tongs* or *halo ring* ❈ *High dose of corticosteroids * are usually given to help *control edema* during the first 8-24 hours ❈ Kinetic therapy treatment table provides side to side motion ❈ Use firm mattress with board underneath ❈ Assess for resp. failure, especially in clients with *high cervical injuries* ❈ Progressive, further loss of sensory and motor function below injury can indicate *additional damage to cord due to edema*, needs to be reported immediately ❈ Evaluate presence of *Spinal shock* (complete loss of all reflexes; motor, sensory, and autonomic activity below the lesion), hypotension, bradycardia, complete paralysis ad lack of sensation below lesion, bladder ad bowel distension ❈ Evaluate for *autonomic dysreflexia*: usually occurs after the period of spinal shock and triggered by *a noxious stimulus* such as bowel/bladder distension, also might be triggered by vaginal examination: *rapidly and highly elevated BP; pounding headache, sweating, nasal congestion, goose bumps bradycardia* ❈ Watch for acute *paralytic ileus*, lack of gastric activity: assess bowel sounds frequently, initiate gastric suction to reduce distension, prevent vomiting and aspiration ❈ Suction with caution to prevent *vagus nerve stimulation*, which can cause *cardiac arrest*

Myasthenia gravis, nursing interventions

❈ Tracheostomy kit available ❈ Teach importance of MedicAlert bracelet ❈ Schedule nursing activities to help reserve energy; plan activities during high-energy times (early morning) ❈Instruct to avoid situations that produce fatigue, physical/emotional stress ❈Encourage coughing and deep breathing q4-6 hrs. (muscle weakness limits ability to cough up secretions, promotes URI)

Brain tumor; Nursing interventions

❈Elevate head of bed 30-40 degree, maintain head in *neutral position* ❈ Facilitate radiation therapy: skin care with no-oiled based soap and water; avoid putting alcohol, powder, or oils; explain temporary alopecia, instruct client to not wash off the lines drawn by the radiologist ❈Administer chemotherapy ❈Facilitate surgical removal (craniotomy) ✂ Preoperative: shave head ✂ Postoperative: Perform frequent neurological and vital sign assessment, position elient with head of bed *elevated for supratentorial lesions* and *flat for infratentorial lesion* Position client *on the opposite side of the operative site* Monitor dressing for signs of drainage (excess amount of CSF) Monitor respiratory status to prevent hypo-ventilation Avoid activities that cause increased ICP Monitor for seizure activity


Related study sets

Buyer Behavior CH11 part 1 (ch09) 26-50

View Set

Midterm: Ch 27, 32, 35, 38, 40, 68

View Set

NCIV Acid Base and Mechanical Ventilator CoursePoint Quiz - Exam 1

View Set

Business Networks Chapter 10 edit

View Set